Download as pdf or txt
Download as pdf or txt
You are on page 1of 378

[Study Package - Physics - Class XI]

STUDY PACKAGE

PHYSICS
CLASS XI

Brilliant
STUDY CENTRE
PALA
Mutholy Campus, Ph: 04822 - 206100, 206800
Arunapuram Campus, Ph: 04822 - 212415, 210949, 216975
Ernakulam - Ph: 0484 - 2665080, 2665090

www.brilliantpala.org., email: brilliantstudycentre@gmail.com

1
Brilliant STUDY CENTRE

BLANK

2
[Study Package - Physics - Class XI]

CONTENTS

1. Units and Measurements ------------------------------------------------------------------- 05

2. Motion in a Straight Line-------------------------------------------------------------------- 28

3. Motion in a Plane ----------------------------------------------------------------------------- 47

4. Laws of Motion -------------------------------------------------------------------------------- 76

5. Work, Energy and Power ----------------------------------------------------------------- 105

6. Systems of Particles and Rotational Motion --------------------------------------- 131

7. Gravitation ------------------------------------------------------------------------------------ 163

8. Mechanical Properties of Solids ------------------------------------------------------- 195

9. Mechanical Properties of Fluids ------------------------------------------------------- 218

10. Thermal Properties of Matter ------------------------------------------------------------ 246

11. Thermodynamics --------------------------------------------------------------------------- 274

12. Kinetic Theory ------------------------------------------------------------------------------- 300

13. Oscillations ----------------------------------------------------------------------------------- 321

14. Waves ------------------------------------------------------------------------------------------ 349

3
Brilliant STUDY CENTRE

BLANK

4
[Study Package - Physics - Class XI]

CHAPTER - 01
UNITS AND MEASUREMENTS

Physical Quantities:
The quantities which can be measured by an instrument and by means of which we can describe the
laws of physics are called physical quantities. eg. length, velocity, acceleration, force, time, pressure,
mass, density etc.

Physical quantities are of three types

Fundamental Derived Supplementary


or Quantities Quantities
Basic quantities

1. Fundamental and Derived Units


The physical units which can neither be derived from one another, nor they can be further resolved
into more simpler units are called fundamental units. e.g., metre, kg, sec
All other physical untis which can be expressed in terms of fundamental untis are called derived units.

e.g., ms 1 , kg ms 2  N 
2. Supplimentary Units
a. Plane angle - Unit - radian (rad)
It is the angle subtended at the centre of a circle by an arc of length equal to its radius.
b. Solid angle - Unit - Steradian (sr)
It is the solid angle subtended at the centre of a sphere by a portion of its surface area equal to a
square with sides of length equal to the radius of the sphere.
UNITS :
To measure a physical quantity and express its measurement we need a ‘standard’ of the quantity.
This chosen standard is called unit of the physical quantity.
The magnitude of a physical quantity is given by M  nU
where n = numerical value of the measured quantity. U = unit of the quantity.

1
* Smaller the unit, greater will be the numerical value. i.e., Numerical value 
unit

5
Brilliant STUDY CENTRE

Classification of units

4. System of Units- FPS, CGS, MKS and SI are used


m CGS system: In this system length, mass and time are expressed in cm, gram, and second respectively.
m FPS system: In this system length, mass and time are measured in foot, pound and second respectively
m MKS system: In this system length, mass and time are expressed in metre, kilogram and second
respectively.
1. SI Units of Basic Quantities :

Base Quantity Name Symbol


Length metre m
Mass kilogram kg
Time second s
Electric Current ampere A
Thermodynamic
kelvin K
Temperature
Amount of Substance mole mol
Luminous Intensity candela cd

DIMENSIONS
Dimension of a physical quantity are the powers to which the base quantities are raised to represent
the quantity.
1. Dimensional formula: The dimensional formula of any physical quantity is that expression which
represents how and which of the base quantities are included in that quantity.
Dimensional formula of mass is [M1L0T0] and that of speed(= distance/time) is [M0L1T–1]
2. Dimensional equation: The equation obtained by equating a physical quantity with its dimensional
formula is called a dimensional equation.
e. g. [v] = [M0L1T–1]
[F] = [MLT–2] is a dimensional equation, [MLT–2] is the dimensional formula of the force and the dimensions
of force are 1 in mass, 1 in length and –2 in time.
Examples

 mass M
• [Density] = =   M1L3 
 volume L3 

1 2 2
work  work  M L T
• Power = ; [Power] =  time    [M1L2T 3 ]
time   T

Force Force  M1L1T 2


• Pressure  ;  Pressure     2
 M1L1T 2
Area  Area  L

6
[Study Package - Physics - Class XI]

Arc Arc L
• (Angular displacement)   ;       M 0 L0 T 0  (Dimensionless)
radius radius L

    1  M 0 L0T 1 
• Angular velocity ( )    
t t T  

0 0 1
d  d  M L T 
• Angular acceleration (  )       M 0 L0 T 2 
dt dt  T

• Torque = Force  Arm length

•  Torque   Force  Arm length   [M1L1T 2 ]   L   [M1L2T 2 ]


• If light of frequency  is falling, energy of a photon is given byE = h 
Here h = Planck’s constant
[E] = [h] [  ]
1 1
 = frequency = Time Period     
 T 

so M1L2 T 2   h   T 1  ; [h]  M1L2 T 1

3. Some special features of dimensions:


Principle of homogeneity of dimensions
Suppose in any formula, (L+ ) term is coming (where L is length). As length can be added only with
a length, so  should also be a kind of length.

So     L

Similarly consider a term (F- ) where F is force. A force can be added/subtracted with a force only
and give rises to a third force. So  should be a kind of force and its result (F- ) should also be a kind
of force.
Uses of dimensions:
1. To check the correctness of the formula :
If the dimensions of the L.H.S and R.H.S are same, then we can say that this eqn. is atleast
dimensionally correct. So this equation may be correct. (Homogeneity of dimensions)
But if dimensions of L.H.S. and R.H.S is not same then the equation is not even dimensionally correct.
So it cannot be correct.

mv 2
e.g. centrifugal force, Fe =
r
(where m = mass, v = velocity, r = radius)
we have to check whether it is correct or not.

7
Brilliant STUDY CENTRE

Dimension of L.H.S is, [F] = [M1L1T-2]


2
 m   V 2   M  LT 1 
Dimension of R.H.S is,   M1L1T  2
r   L
So this eqn. is atleast dimensionally correct.
Thus we can say that this equation may be correct.
2. To convert a physical quantity from one system of units to another .
Convert SI unit of force - (Newton), to CGS unit (dyne)
F = M1L1T–2
1 1 2 1 1 2
 M1   L1   T1   Kg   m   s 
n=             = 1000 × 100 × 1 = 105
 M 2   L 2   T2   g   cm   s 
1N = 105 dynes
100
• The acceleration due to gravity is 9.8m/s2 = 9.8 × cms–2 = 980cms–2
1
3. To derive the relationship between different physical quantities
Homogeneity of dimensions enable us to derive relations between physical quantities, if the dependent
quantities are known
The period of the satellite may depend on gravitational constant G, mass of the earth m and orbital
radius r
o o 1  a 3a 2a b c
[T] = [G]a[M]b[R]c ;  M L T    M L T  [M ]  L  ; M–a+b L3a+c T–2a

1 1 3
1 = –2a, a =  ; 0=b–a ,b=  ; 0 = 3a + c c= 
2 2 2

1 1 3 R3
  T  2
TG M R ; 2 2 2
GM
LIMITATIONS OF DIMENSIONAL ANALYSIS:
• Dimensional analysis doesn’t give information about the “Some Number”: “The dimensionless constant”.
• This method is useful only when a physical quantity depends on other quantities by multiplication and
power relations. (i.e., f = xa yb zc)
It fails if a physical quantity depends on sum or difference of two quantities (i.e.f = x + y – z)

1 2
i.e., we cannot get the relation, S = ut + at from dimensional analysis
2
• This method will not work if a quantity depends on another quantity as sine, cosine, logarithmic or
exponential relation. The method works only if the dependence is by power functions.
• We equate the powers of M, L and T hence we get only three equations. So we can have only three
variables (only three dependent quantities)
So dimensional analysis will work only if the quantity depends only on three parameters, not more
than that.

8
[Study Package - Physics - Class XI]

ERRORS IN MEASUREMENT
1. LEAST COUNT
We have studied that no measurement is perfect. Every instrument can measure upto a certain
accuracy; called least count.
Least count : The smallest quantity an instrument can measure

mm. scale Vernier Screw gauge stop Thermometer


L.C. = 1 mm L.C. = 0.1 mm L.C. = 0.01 mm watch o
L.C. = 1 C
L.C. = 0.1 sec
2. PERMISSIBLE ERROR
Error in measurement due to the limitation (least count) of the instrument, is called permissible error.
From mm scale  we can measure upto 1 mm accuracy (least count = 1mm). From this we will get
measurement like t = 34mm


Max uncertainty can be 1 mm
Max permissible error (Dl) = 1mm
But if from any other instrument we get l = 34.6 mm then max permissible error(Dl) = 0.1mm
and if from a more accurate instrument, we get l = 34.527 mm then max permissible error (Dl) =
0.001mm = place value of last digit.
Maximum permissible error in a measured quantity is the least count of the measuring instrument and
if nothing is given about least count, then maximum permissible error is the place value of the last
digit.
3. Max. Permissible Error in the result due to error in each measurable quantity:
Let result f(x, y) contains two measurable quantities x and y

Let error in x is = x i.e. x  (x  x, x  x)

error in y is = y i.e. y  (y  y, y  y)


Case:(I) If f(x, y) = x + y, df = dx + dy
error in f = Df =  Dx +  Dy

max possible error in f =  f  max = max of  x  y     x  y 

 f max  x  y
Case: (II)

If f = x – y; df = dx – dy;  f   x  y
max possible error in f =  f  max  max of  x  y    f max    x  y 

For getting maximum permissible error, sign should be adjusted, so that errors get added up to give
maximum effect

9
Brilliant STUDY CENTRE

Case (III) If f(x, y, z) = (constant) xaybzc


to scatter all the terms, let’s take log on both sides
log f = log (constant) + a log x + b log y + c log z

 Differentiating both sides

df dx dy dz
 0a b c
f x y z

f x y z
= a b c
f x y z

 f   x y z 
   max of   a b c 
 f max  x y z 
4. Errors in averaging:
Suppose to measure some quantity, we take several observations, a1, a2, a3 ----- an. To find the absolute
error in each measurement and percentage error, we have to follow these steps

 a1  a 2      a n 
a) First of all mean of all the observations is calculated : a mean  . The mean of
n
these values is taken as the best possible value of the quantity under the given conditions of
measurements.
b) Absolute Error:
The magnitude of the difference between the best possible or mean value of the quantity and the
individual measurement value is called the absolute error of the measurement. The absolute error in
an individual meaured value is:

a n  a mean  a n

The arithmetic mean of all the absolute errors is taken as the final or mean absolute error.

 n 
 a  a 2  a 3         a n ;   a i 
  i 1 
1
a mean  a mean
n n

we can say amean – Damean  a  amean + a mean

c) Relative and Percentage Error


Relative error is the ratio of the mean absolute error to the arithmetic mean,

a mean
Relative error =
a mean

When the relative error is expressed in percentage, it is called the percentage error

10
[Study Package - Physics - Class XI]

a mean
Thus Percentage error =
a mean × 100%

5. SIGNIFICANT FIGURES
The reliable digits plus the first uncertain digit are known as significant figures.
1. Common rules of counting significant figures:
Rule 1 :
All non-zero digits are significant
Rule 2 :
All zeros occuring between two non-zero digits are significant
Rule 3 :
Zeros to the right of the decimal are significant if there is a non zero digit before the decimal
point. If there is no non zero digit before the decimal point ,the zeros adjacent to the right of the
decimal point are not significant.
Rule 4:
In the number less than one, all zeros after decimal point and to the left of first non-zero digit are
insignificant (arises only due to change of unit)
Rule 5:
The terminal or trailing zeros in a number without a decimal point are not significant.(Also arises only
due to change of unit)
Rule 6:
The power of 10 is irrevelant to the determination of significant figure.
Examples significant figures
3.0800 - 5
0.00418 - 3
7.09 × 105 - 3
91600 - 3
0.003005 - 4
3.200 × 109 - 4
250 - 2
78000,0000 - 2
0.0101 - 3
0.000800 - 3
11
Brilliant STUDY CENTRE

ALGEBRAIC OPERATIONS WITH SIGNIFICANT FIGURES

Now let us see how to do arithmetic operations ie, addition, subtraction, multiplication and division
according to significant figures

In addition and subtraction, the final result should retain as many decimal places as there in the number
with the least decimal places.

eg: 24.36 + 0.0623 + 256.2 = 280.62223

It is to be rounded off to 1 decimal place.

 The result is 280.6


In multiplication and division, the final result should retain as many significant figures as there in the
original number with least significant digits.

eg: 2.80 × 4.5039 = 12.61092

It is to be rounded off to 3 significant figures.

 the result is 12.6.


ORDER OF MAGNITUDE

When physical quantities vary over a wide range it is difficult to get a comparison about their magnitudes.
For a quick comparison among them we can use powers of ten method. In this method each number is
b
expressed as a  10 where 1  a  10 and b is a positive or negative integer. eg. diameter of sun is 1.39 ×
109m . diameter of hydrogen atom is 1.06 × 10–10m. To get an approximate idea of the number one may
round the number a to 1 if it is less than 5 and to 10 if it is greater than 5. Thus the diameter of the sun is of
the order of 109m and that of hydrogen atom is 10–10m . The exponent of 10 is the order of the magnitude of
the quantity.

12
[Study Package - Physics - Class XI]

QUESTIONS
LEVEL - I
1. The density of a material in CGS system of units is 4 g cm–3. In a system of units in which unit of length
is 10cm and unit of mass is 100 g, the value of density of material will be
1) 0.04 2) 0.4 3) 40 4) 400

aBC
2. A wave is represented by y = a sin  At  Bx  C  , where A, B, C are constants. The dimensions of
A
are same as those of
1) length 2) mass 3) time 4) none of these

3.
2
 
The time dependence of a physical quantity is given by P  P0 exp t [Where  is a constant and
t is time.] The constant 
1) is dimensionless 2) Has dimensions [T–2] 3) Has dimensions [T2] 4) Has dimensions of P

a a  t2
4. The dimensions of in the equation P  where P is the pressure, x is the distance and t is the
b bx
time are

1) MT 2 2) M 2 LT 3 3) ML3T 1 4) LT 3

5. Which two of the following five physical parameters have the same dimensions?
1. energy density 2. refractive index 3. dielectric constant 4. Young’s modulus 5. magnetic field
1) 1 and 4 2) 1 and 5 3) 2 and 4 4) 3 and 5
6. The mass of the liquid flowing per second per unit area of cross section of the tube is proportional to px
and vy where p is the pressure difference and v is the velocity then the relation between x and y is
1) x = y 2) x = –y 3) y2 =x 4) y = –x2
7. Turpentine oil is flowing through a tube of length l and radius r. The pressure difference between the
P  r2  x2 
two ends of the tube is P; the viscosity of the oil is given by   where v is the velocity of oil
4vl
at a distance x from the axis of the tube. From this relation, the dimensions of viscosity  are

0 0 0 1 2 2 1 1
1)  M L T  2)  MLT  3)  ML T  4)  ML T 

8. A small steel ball of radius r is allowed to fall under gravity through a column of a viscous liquid of
coefficient of viscosity  . After sometime the velocity of the ball attains a constant value known as
terminal velocity, v T . The terminal velocity depends on (i) the mass of the ball m (ii)  (iii) r (iv)
acceleration due to gravity g. Which of the following relations is dimensionally correct?

mg r mgr
1) v T  2) v T  3) v T  rmg 4) v T 
r mg 

13
Brilliant STUDY CENTRE

9. Match List I with List II.

List I List II
1 1 –3 –1
A) Capacitance, C (i) MLT A
–1 –3 4 2
B) Permittivity of free space, e0 (ii) M L TA
–1 –2 4 2
C) Permeability of free space, m0 (iii) M L TA
1 1 –2 –2
D) Electric field, E (iv) MLT A

Choose the correct answer from the options given below.


1) (A) - (iv), (B) - (ii), (C) - (iii), (D) - (i) 2) (A) - (iii), (B) - (iv), (C) - (ii), (D) - (i)
3) (A) - (iii), (B) - (ii), (C) - (iv), (D) - (i) 4) (A) - (iv), (B) - (iii), (C) - (ii), (D) - (i)
10. The physical quantities not having same dimensions are
1) torque and work 2) momentum and Planck’s constant

1/ 2
3) stress and Young’s modulus 4) speed and  m 0 e 0 

11. Surface tension has the same dimensions as that of


1) frequency 2) impulse 3) momentum 4) spring constant

L
12. If L and R are respectively the inductance and resistance, then the dimensions of will be
R
1) M0L0T–1 2) M0LT0 3) M0L0T
4) Cannot be represented in terms of M, L and T

x y z
13. If dimensions of critical velocity vc of a liquid flowing through a tube are expressed as    r  , where
,  and r are the coefficient of viscosity of liquid, density of liquid and radius of the tube respectively,,
then the values of x, y and z are given by
1) –1, –1, 1 2) –1, –1, –1 3) 1, 1, 1 4) 1, –1, –1
14. If P, Q and R are physical quantities, having different dimensions, which of the following combinations
can never be a meaningful quanitty?

1)
PQ
2) PQ – R 3)
PQ
4)
 PR  Q  2

R R R
15. If the units of mass, length and time are doubled, unit of angular momentum will be
1) Doubled 2) Tripled 3) Quadrupled 4) 8 times the original value

16. The displacement of a particle moving along x-axis with respect to time t is x  at  bt 2  ct 3 . The
dimensions of c are

1) T3 2) LT 2 3) LT 3 4) LT3
14
[Study Package - Physics - Class XI]

B
17. Suppose refractive index m is given as m  A  where A and B are constants and  is wavelength,
2
then dimensions of B are same as that of
1) wavelength 2) volume 3) pressure 4) area
18. If velocity (v), acceleration (a) and force (F) are taken as fundamental quantities, the dimensions of
Youngs modulus (Y) would be

2 2 2 3
1)  Fa v  2)  Fa v 

2 4 2 5
3)  Fa v  4)  Fa v 

19. A vernier calliper has its main scale of 10cm equally divided into 200 equal parts. Its vernier scale of 25
divisions coincides with 12mm on the main scale. The least count of the instrument is
1) 0.020cm 2) 0.002cm 3) 0.010cm 4) 0.001cm
20. If X = a + b, the maximum percentage error in the measurement of X will be

 a b   a b 
1)     100% 2)     100%
 a b  ab ab

 a b   a b 
3)     100% 4)     100%
ab ab  a b 

21. The mean time period of second’s pendulum is 2.00s and mean absolute error in the time period is
0.05s. To express maximum estimate of error, the time period should be written as

1)  2.00  0.01 s 2)  2.00  0.025  s

3)  2.00  0.05  s 4)  2.00  0.10  s

22. In an experiment, the percentage of error occurred in the measurement of physical quantities A, B, C
and D are 1%, 2%, 3% and 4% respectively. Then the maximum percentage of error in the measurement
A 2 B1/ 2
X, where X  , will be
C1/3D3

1) 10% 2) (3/13)% 3) 16% 4) –10%


23. If the error in measuring the radius of the sphere is 2% and that in measuring its mass is 3%, then the
error in measuring the density of material of the sphere is
1) 5% 2) 7% 3) 9% 4) 11%
24. The percentage errors in the measurement of mass and speed are 2% and 3% respectively. How
much will be the maximum error in the estimation of the kinetic energy obtained by measuring mass
and speed.
1) 11% 2) 8% 3) 5% 4) 1%

15
Brilliant STUDY CENTRE

25. If the time period of oscillation of a pendulum is measured as 2.5 second using a stop watch with the
1
least count second, then the permissible error in the measurement is
2
1) 10% 2) 30% 3) 15% 4) 20%

26. A physical quantity is given by X  M a Lb T c . The percentage error in measurement of M, L and T are
,  and  respectively. Then maximum percentage error in the quantity X is

1) a  b  c 2) a  b  c

a b c
3)   4) None of these
  

27. The volume of a sphere is 1.76 cm3. The volume of 25 such spheres taking into account the significant
figures is
1) 0.44 × 102 cm3 2) 44.0 cm3 3) 44 cm3 4) 44.00 cm3

28. Number of significant figure in 0.0006702 is

1) 4 2) 8 3) 2 4) 7
29. Which of the following statements is incorrect regarding significant figures?

1) All the non-zero digits are significant

2) All the zeros between two non-zero digits are significant


3) Zeros to the right of the decimal are significant if there is a non zero digit before the decimal.

4) The power of 10 is counted while counting the number of significant figures

LEVEL - II
1. If E and H represents the intensity of electric field and magnetising field respectively, then the unit of E/
H will be

1) joule 2) ohm 3) newton 4) mho

2. In C.G.S. system the magnitude of the force is 100 dyne. In another system where the fundamental
physical quantities are kilogram, metre and minute, the magnitude of the force is
1) 0.036 2) 0.36 3) 3.6 4) 36

3. The density of a material in SI units is 128 kg m–3. In certain units in which the unit of length is 25 cm
and the unit of mass is 50 g, the numerical value of density of the material is

1) 410 2) 40 3) 640 4) 16
4. ‘Pascal second’ is a unit of

1) Energy 2) Young’s modulus

3) Stress 4) Viscosity
16
[Study Package - Physics - Class XI]

5. Which of the following ratios has the dimension of mass?


1) Volume / Density
2) Surface tension / (Angular velocity)2
3) Linear momentum / Force
4) Pressure / Power

6. If the energy, E  G p h q cr where G is the universal gravitational constant, h is the Planck’s constant
and c is the velocity of light, then the value of values of p, q and r are, respectively
1) –1/2, 1/2 and 5/2 2) 1/2, –1/2 and –5/2
3) –1/2, 1/2 and 3/2 4) 1/2, –1/2 and –3/2

 a 
  V  b   cT , where P, V, T are pressure, volume
2
7. The equation of state of a gas is given by  P 
 V3 
and temperature respectively, and a, b, c are constants. The dimensions of a and b are respectively
8 2 3/ 2 5 2 3
1)  ML T  and  L  2)  ML T  and  L 

5 2 6 6 2 3/ 2
3)  ML T  and  L  4)  ML T  and  L 
   
8. The dimensions of the quantity E  B , where E represents the electric field B the magnetic field may
be given as
3 2 5 2
1)  MT  2)  M LT A 

2 3 1 2 2
3)  M LT A  4)  MLT A 

9. If P represents radiation pressure, c represents speed of light and Q represents radiation striking unit
area per second, then non-zero integers x, y and z such that Px Qy cz is dimensionless, are
1) x = 1, y = 1, z = – 1 2) x = 1, y = –1, z = 1 3) x = –1, y = 1, z = 1 4) x = 1, y = 1, z = 1
10. If C be the capacitance and V be the electric potential, then the dimensional formula of CV2 is
1 2 2 0 1 1 2 1
1)  M L T A  2)  M L T A 

0 1 2 0 1 3 1 1
3)  M L T A  4)  M L T A 

11. If C (velocity of light), h (Planck’s constant) and G (Universal gravitational constant) are taken as
fundamental quantities, then the dimensional formula of mass is
1 1 1 1 1 1 1 1 1 1 1
1) h  2 G 2 C0 2) h 2 C 2 G  2 3) h  2 C 2 G  2 4) h  2 C  2 G  2

12. The time period T of a small drop of liquid due to surface tension depends on density  , radius r and
surface tension S. The relation is

1
1) T  r 3 / S 2 2) T = rS 3) T = r / S 4) T = S / 
 

17
Brilliant STUDY CENTRE

e2
13. The dimensions of , where e, e0 , h and c are electronic charge, electric permittivity, Planck’s
s
4e0 hc
constant and velocity of light in vacuum respectively, are

0 0 0 0 0
1)  M L T  2)  ML T 

0 0 0 0 1
3)  M LT  4)  M L T 

14. If frequency F, velocity v, and density D are considered fundamental units, the dimensional formula for
momentum will be
1) DvF2 2) Dv2F–1 3) D2v2F2 4) DV4F–3

15. The speed (v) of ripples on water surface depends on surface tension(  ) density(  ) and wavelength
(  ). The square of speed (v) is prop. to

   
1) 2) 3) 4)
   

16. The length l, breadth b, and thickness t of a block of wood were measured with the help of a measuring
scale. The results with premissible errors (in cm) are
l = 15.12  0.01, b = 10.15  0.01, and t = 5.28  0.01

The percentage error in volume up to proper significant figures is


1) 0.28% 2) 0.35% 3) 0.48% 4) 0.64%
17. A body accelerates from rest with a uniform acceleration a for a time t. The uncertainty in ‘a’ is 8% and
the uncertainty in ‘t’ is 4%. The uncertainty in the speed is
1) 32% 2) 12% 3) 8% 4) 2%

18. A physical quantity P is described by the relation P  a1/ 2 b 2 c3 d 4 . If the relative errors in the measurement
of a, b, c and d respectively, are 2%, 1%, 3% and 5%, then the relative error in P will be
1) 25% 2) 12% 3) 8% 4) 32%

19. A thin copper wire of length  metre increases in length by 2% when heated through 10oC. What is the
percentage increase in area when a square copper sheet of length  metre is heated through 10oC?

1) 4% 2) 8%
3) 16% 4) None of above

2k 32
20. A physical quantity X is given by X  . The percentage error in the measurements of k,  , m and
m n
n are 1%, 2%, 3% and 4% respectively. The value of X is uncertain by
1) 8% 2) 10% 3) 12% 4) None of these

18
[Study Package - Physics - Class XI]

21. When a current of  2.5  0.5  A flows through a wire, it develops a potential difference of  20  1 V ,
the resistance of the wire is

1)  8  2   2)  8  1.6  

3)  8  1.5   4)  8  3 

22. Which of the following has the highest number of significant figures?
1) 0.007 m2 2) 2.64 × 1024 kg
3) 0.0006032 m3 4) 6.3200 J
23. The mass of a box measured by a grocer’s balance is 2.3 kg. Two gold pieces of masses 20.15 g and
20.17g are added to the box. What is the total mass of the box and the difference in the masses of the
pieces to correct significant figures?

1) 2.34 kg, 0 g 2) 2.3 kg, 0.02 g 3) 2.34 kg, 0.02 g 4) 2.3 kg, 0 g
24. A student measured the diameter of a small steel ball using a screw gauge of least count 0.001 cm.
The main scale reading is 5 mm and zero of circular scale division coincides with 25 divisions above
the reference level. If screw gauge has a zero error of –0.004 cm, the correct diameter of the ball is
1) 0.521 cm 2) 0.525 cm 3) 0.053 cm 4) 0.529 cm
25. Time intervals measured by a clock give the following readings
1.25 s, 1.24 s, 1.27 s, 1.21 s and 1.28 s. What is the percentage relative error of the observation?
1) 2% 2) 4% 3) 16% 4) 1.6%
26. In an experiment four quantities a, b, c and d are measured with percentage error 1%, 2%, 3% and 4%
a 3b2
respectively. Quantity P is calculated as follows P  % , Error in P is
cd
1) 14% 2) 10% 3) 7% 4) 4%

27. In the density measurement of a cube, the mass and edge length are measured as 10.00  0.10  kg

and  0.10  0.01 m , respectively. The error in the measurement of density is

1) 0.07 kg / m3 2) 0.10 kg / m3 3) 0.01 kg / m3 4) 0.31 kg / m3

19
Brilliant STUDY CENTRE

KEY WITH HINTS


LEVEL - I

g 100g
1. 3
4
As n1u1  n 2 u 2 ; cm 3
 n 2 3
 n 2  40
10cm 
2. 3 y  a sin  At  Bx  C  ; As sin  is dimensionless, therefore, a = y = [L]

1 1 1 1 aBC L
Also, A =  ; B =  , C is dimensionless     T   time
t T x L A 1
L 
T

3. 2 P  P0 exp  t 2  ; To find the constant  ;

0 0 0
All the quantities inside the exp ( ) will be dimensionless  M L T  .

So      T    M L T  ;      M 0L0T 2 
2 0 0 0 0 0 2 2
 the dimension are  M L T  or  T  .

a  t2 a 1 t 2
4. 1 P  . 
bx b x bx

a 1   t2 
[Pressure] =  .  . Also [Pressure] =  ;
b x   b.x 

a   1  a 
 ML1T 2            MT 2 
b L b
2
 Work done  MLT .L
5. 1 [Energy density]     3
  ML1T 2 
 Volume  L

 Force  l  MLT L
2

[Young’s modulus] = [Y]     2


.   ML1T 2 
 Area  l L L
The dimensions of 1 and 4 are the same.

M x y
6. 2  p x v y ; ML2 T 1   ML1T 2   LT 1  ; x = 1, –x + y = –2, –2x – y = –1; y = –1, x = –y
At    

P.  r 2  x 2  [pressure] [L2 ]
7. 4  ; Viscosity  ; 
4vl [LT 1 ] [L]

 ML1T 2   L2 
       ML1T 1 
 LT 1   L ;

20
[Study Package - Physics - Class XI]

mg mg
8. 1 mg  F  6rv T ; v T  ; vT 
6r r
1 2 4 2
9. 3 The dimensions of capacitance C   M L T A 

1 3 4 2
The dimensions of permittivity of free space, e 0   M L T A 

2 2
The dimensions of permeability of free space, m 0   MLT A 

3 1
The dimensions of electric field, E   MLT A 

1
10. 2 [Momentum]   MLT 

2 1
[Planck’s constant’ =  ML T 

Momentum and Planck’s constant do not have same dimensions.

 MLT 2  0 2
 MLT 2 
11. 4 Surface tension    ML T  ; Spring constant    ML0 T 2 
 L  L
Magnetic flux / current ML2 T 2 A 1
12. 3  T
Potential difference / current ML2 T 3 A 1

x z y
13. 4 Vc  x y r z ; LT 1   ML1T 1   ML3   L ; x = 1; y = –1; z = –1

P  Q
14. 1
R
Since P and Q have different dimensions, therefore, their subtraction is not possible.
2 1
15. 3 Angular momentum; L  mrv  mass   length    sec 

2 1
New angular momentum   2M  2L   2T    4  ML2T 1   4L
  

16. 3 ct 3  x   c   T 3    L  ;   c    LT 3 

velocity of light in vacuum


17. 4 As m 
velocity of light in medium
Hence, m is dimensionless. Thus each term on the RHS of given equation should be
dimensionless.

B
 is dimensionless, i.e., B should have dimensions of  2 , i.e., m 2 , i.e., area
2

21
Brilliant STUDY CENTRE

18. 3 Let Y  kv x a v Fz ; where k is a dimensionless constant.


x y z
  ML1T 2    LT 1   LT 2   MLT 2    M z Lx  y  z T  x  2y  2z 

Equating the powers of M, L and T, we get x  1, x  y  z  1,  x  2y  2z  2

Solving, we get, x  4, y  2, z  1 ;   Y    v a F    Fa v 


4 2 1 2 4

10
19. 2 I MSD = cm  0.050 cm
200

1.2
1 VSD = cm  0.048cm ; LC= 1MSD – 1VSD = 0.05 – 0.048 = 0.002 cm
25

x  a  b   a b 
20. 3 x  a  b , x  100   a  b   100   a  b  a  b   100
   

21. 3 T  0.05 sec , T = 2.00 sec; T  T  T   2.00  0.05  sec.

A 2 B1/ 2  dX 
22. 3 X 1/3 3 ; Maximum percentage error in X ,    100
C D  X 

 dA 1 dB 1 dC dD  1 1
 2   3  100  2  1   2   3  3  4  16%
 A 2 B 3 C D  2 3

M 3 M
mass   3
4
23. 3 As density 
volume
; R 3 4 R ;  The percentage error in density is
3

  M R 
 100%   3   100%  3%  3  2%   3%  6%  9%
  M R 

1  KE   m v   KE 
24. 2 KE  mv 2 ;   %   2 % ;    %  2  2  3  8%
2  KE   m v   KE 

1
25. 4 T = 2.5 sec, T  sec  0.5 sec
2

T 0.5
The permissible error  100   100  20%
T 2.5

 x   M L T 
26. 1 X  M a LbT c ;   x   100   a m  b L  c T   100   a  b  c  %

27. 2 The final result should be having 3 significant digits. Just multiply 1.76 × 25 and then scale the
result to 3 significant digits.
22
[Study Package - Physics - Class XI]

28. 1
29. 4 The power of 10 is irrelevant to the determination of significant figures.
LEVEL - II

volt
1. 2 The unit of electric field intensity is E 
metre

ampere
The unit of magnetising field intensity is H 
metre

E volt metre volt


   
H metre ampere ampere
Volt is the unit of voltage and ampere is the unit of current.

V
Therefore, the ratio of E and H becomes which is equal to resistance according to the
I
E
Ohm’s law. Thus, has units of resistance i.e, Ohm (  )
H
1 1 2
 M1   L1   T1 
2. 3 In C.G.S. Force = 100 dyne; n 2  n1      
 M 2   L2   T2 

1 1 2
 g   cm   sec  100  3600
 100   100cm   60sec    3.6
1000g  100000

M 3 3
3. 2     
3 ; n1u1 = n2u2;  128  M1L1   n 2  M 2 L 2 
L
3 3
 M  L  1000   25  1
 n 2  128   1   2   128       128  20   40
 M 2   L1   50  100  64

4. 4 Pascal × second = Pressure × time  ML1T 2  T   ML1T 1 

dV MLT 2 1
again, F  A .   2
. 1   ML1T 1 
dx L T

Volume  M 0 L3 T0  surface tension  ML0 T 2 


   M 1L6  ;   M
5. 2 2 2
Density 3 0
 ML T   Angular velocity   M 0 L0 T 1 

1 1 2
Linear momentum  MLT  Pressure  ML T 
  T ;    L3 T 
Force  MLT 2  Power  ML2 T 3  

23
Brilliant STUDY CENTRE

1 2 2 1 3 2 p 2 1 1 q r
 p  q 3p  q  r 2p  q  r
6. 1 E  G p h q c r ------(i);  M L T    M L T   ML T   LT    M L T 

Applying principle of homogeneity of dimensions, we get – p + q = 1 ------(ii)


3p + 2q + r = 2 --------(iii); – 2p – q – r = – 2 ------(iv)
Adding (iii) and (iv), we get p + q = 0 -------(v)

1 1 1
Adding (ii) and (v), we get q  ; From (ii), we get p  q  1   1  
2 2 2
3 5
Put in (iii), we get   1  r  2, r 
2 2

 a  a
3 
7. 1 Given,  P  V  b 2   cT ; Dimension of 3 = Dimensions of P
 V  V
 Dimensions of a = dimensions of PV3
2
F F   MLT   3  3  8 2 
 a    V 3  
 P     L2    L    ML T 
A   A  
1/ 2 1/ 2
Dimensions of b2 = dimensions of V;   b   V   L3  or  b    L3/ 2 

2
F  MLT 
8. 2 Electric field, E     MLT 3 A 1 
q  AT 

F  MLT 2 
Magnetic field, B     MT 2 A 1 
current  length  A  L
  3 1 2 1 2 5 2
 The dimensions of quantity E  B =  MLT A   MT A    M LT A 

9. 2  P  M1L1T 2 ;  c  M 0 L1T 1 ;
M1L2T 2
 Q  2 1  M1T 3
LT
x y z
Mo LoT o   ML1T 2   MT 3   LT 1 
PxQycz is dimensionless if x + y = 0 and – x + z = 0
x  y
ie, x = 1, y = –1, z = 1
xz
PC
 is a constant
Q

10. 1 Dimensional formulas of CV 2   M 1L2 T 4 A 2  ML2 T 3A 1 2   ML2 T 2 

24
[Study Package - Physics - Class XI]

11. 2 Let, M  C a h b G c

a b c
ML0 T 0   LT 1   ML2 T 1   M 1L3 T 2  .... (i)

Energy  ML2 T 2 
Where, h   1
  ML2 T 1 
Frequency  T 

Metre
C   LT 1 
Second

Force  (distance) 2  MLT 2   L2 


G  2
  M 1L3 T 2 
(mass) 2  M 

Comparing the coefficients M, L, T, of both sides we get


b – c = 1 ........ (ii)
a + 2b + 3c = 0 ....... (iii)

  a  b  2c   0 ....... (iv)
Solve the equations (ii), (iii) and (iv), we get

1 1 1
a  ,b  ,c  
2 2 2
1 1 1
So, M  h 2 C 2 G  2

a c
12. 1 T  a r bSc ; T   ML3  Lb  MT 2   Ma  c L3a  b T 2c
a + c = 0; 3a  b  0 ; 2c  1
1/2
1 3 1  r 3 
Solving a  , b  and c    T k 
2 2 2  S 

e2
13. 1 The dimensions of y  ; Putting the dimensions of
4e0 hc

 e   Q   AT  ;  e0    M 1L3T 4 A 2  , h   ML2 T 1  , c   LT 1 
 A 2 T 2 
y y   M 0 L0 T 0 
 M 1L3 T 4 A 2   ML2 T 1   LT 1  ;

1 x y z 1 1 3 x x y
y y z
z 3z
14. 4 P  mv  F x v y D z ; MLT   F v D   T   LT   ML   T   L T   M L 

Equating the powers we get, z = 1; y = 4 ; x=–3  P  F3 v 4 D1


25
Brilliant STUDY CENTRE

15. 1 v  σ a ρb λc ; T–1 = (MT–2)a (ML–3)bLc;


LT a+b=0 –3b + c = 1 –2a = –1

1 1 1 
b c a ; v
2 2 2 

 0.01 0.01 0.01 


16. 2 percentage error in volume is     100  0.35%
 15.12 10.15 5.28 
17. 2 Using kinematic eqn, v = u + at = at  u  0
v a t a t v
For error calculation,   ; Here,  8%,  4% ; So,  8  4  12%
v a t a t v
18. 4 Here, P  a1/ 2 b 2 c3 d 4

P 1 a b c d
 2 3 4
P 2 a b c d

 P   1 a b c d 
or   100  %   2 3 4   100%
 P  2 a b c d 

1 
 Relative error in P   2  2  2  1  3  3  4  5  %  32%
 

    A    
19. 1    100  2% , area (A)  2 ;     100  2  100   4%
    A    

2k 32 X  k  m 1 n 
20. 3 Given: X  ; Percentage error in X;  100   3 2     100
m n X  k  m 2 n 

1
= 3 × 1% + 2 × 2% + 3% +  4% ; Hence, the value of X is uncertain by 12%
2

V 20V
21. 1 Current, I   2.5  0.5  A ; V   20  1 V ; R    8 ;
I 2.5A

R  V I   1 0.5 
         0.05  0.2     0.25 
R  V I   20 2.5 

R    0.25  8    2 ;  Resistance of the wire is  8  2  


22. 4 According to the rules of significant figures 0.007 m2 has one significant figure.
2.64 × 1024 kg has three significant figures. 0.0006032 m2 has four significant figures.
6.3200 J has five significant figures.
23. 2 Total mass = 2.3 kg + 0.02015 kg + 0.02017 kg = 2.34032 kg = 2.3 kg
Difference of mass of the gold pieces = 20.17 g – 20.15 g = 0.02 g
26
[Study Package - Physics - Class XI]

24. 4 Diameter of the ball = MSR + CSR × (Least count) – Zero error
= 5 mm + 25 × 0.001 cm – (–0.004) cm = 0.5 cm + 25 × 0.001 cm + 0.004 cm = 0.529 cm
1.25  1.24  1.27  1.21  1.28
25. 4 Mean time interval T 
5
6.25
   1.25s
5
| T1 |  | T2 |  | T3 |  | T4 |  | T2 |
Mean absolute error, T 
5
|1.25  1.25 |  |1.25  1.24 |  |1.25  1.27 |  |1.25  1.21|  |1.25  1.28 |
 
5
0  0.01  0.02  0.04  0.03 0.1
    0.02 s
5 5
T 0.02
 Percentage relative error  100  100  1.6%
T 1.25
a 3b2 P  3a 2b c d 
26. 1 Here, P  ;  100        100
cd P  a b c d 
a b c d
3  100  2  100   100   100
a b c d
 3 1  2  2  3  4  3  4  3  4  14%

m d dm 3da d 0.1  0.01  d d


27. 4  ;     3 ;  0.01  0.3   0.31 Kg / m3
a3  m a  10  0.1   

27
Brilliant STUDY CENTRE

CHAPTER - 02
MOTION IN A STRAIGHT LINE

ONE DIMENSION MOTION


It is that motion in which a particle or a body moves in one particular direction with respect to a point of
reference.
Objects in Rest and motion
Rest: An object is said to be at rest if its position does not change with time with respect to its
surroundings
Motion: An object is said to be in motion if it changes its position with time, with respect to its
surroundings.
1. Distance and Displacement
Distance is the actual path length covered by a moving particle or a body in a given interval of time,
while displacement is the change in position vector. ie., a vector joining initial to final position.
 
Consider a particle at A moving to C along a path ABC. Its initial and final position vectors are r1 & r2 .
  
The distance travelled is the actual path ABC while the displacement is  r  r2  r1

C B
r2 r

A
r1

 Displacement  distance travelled.


 Displacement is a vector quantity and distance travelled is a scalar quantity.
 The value of distance travelled can never be zero or negative. The value of displacement can be
positive, negative or zero.
2. Speed and Velocity
The distance covered by a particle per unit time is known as speed. The rate of change of position is
known as velocity.
Average speed and Average velocity
The average speed of a particle in a given time interval is defined as the ratio of total distance travelled
total distance
to the total time taken. average speed =
total time

28
[Study Package - Physics - Class XI]

Average velocity is defined as the ratio of total displacement to the total time taken

total displacement r r2  r1
average velocity = vav = ; 
total time t t
 When particle moves the first half of a distance at a speed of v1 and second half of the distance at
speed v2 then

2v1v 2
v av 
v1  v 2

Instantaneous Speed and Instantaneous Velocity


The magnitude of speed at any instant of time is known as instantaneous speed.

s ds
Instantaneous speed = lim = where s is the distance travelled in time t.
t0 t dt

t
Instantaneous velocity of a particle is the velocity of the particle at an instant v av 
t
 
 r dr
Instantaneous velocity v = lim
t0

t dt

 
dr dr ds
Magnitude of the instantaneous velocity is v =   . ie, It is equal to the instantaneous
dt dt dt
speed at time t
Uniform Velocity
If the velocity of a particle remains constant as time passes, it is moving with uniform velocity and the
body is in uniform motion. If the velocity changes with time the body is accelerated.
3. Acceleration
It is the rate of change of velocity. Acceleration is a vector.
Average Acceleration is the ratio of change in velocity in a certain time interval to the time interval.
 
v 2  v1
av 
t 2  t1

Instantaneous Acceleration is the acceleration of a body at a certain instant of time.

 
 v dv
a  lim 
t  0  t dt

As t  0 , this average acceleration become instantaneous acceleration.

29
Brilliant STUDY CENTRE

Graphical analysis of 1-D motion

I. Position - time graph.


Position-time graph is plotted by taking time t along horizontal axis and position x on vertical axis.

 Slope of position-time graph at any instant of time gives instantaneous velocity..


(i) When particle is at rest.

v = slope = 0

(ii) When the particle is moving with uniform velocity


Graph : Straight line having non-zero slope

dx
v
dt
(iii) When the particle is moving with uniform acceleration.
x – t graph will be parabolic

v increases with time v decreases with time


acceleration is positive acceleration is negative

II. Velocity - time graph

v-t graph is plotted by taking time on horizontal axis and velocity on vertical axis.

 Slope of v-t graph gives acceleration.


 Area under v-t graph gives displacement
30
[Study Package - Physics - Class XI]

(i)

Motion in positive direction with positive acceleration.

(ii)

Motion in positive direction with –ve acceleration

(iii)

Motion of an object with negative acceleration. In between time 0 to t1 it moves in +ve x direction and
after t1 it starts moving in opposite direction.

(iv)

Motion in –ve direction with –ve acceleration

31
Brilliant STUDY CENTRE

III. Acceleration - time graph

a-t graph is plotted by taking acceleration along vertical axis and time along horizontal axis. Area under
a-t gives “change” in velocity.

Kinematic equations for uniformly accelerated motions

Suppose the body is having a constant acceleration a and at t = 0, the velocity is u and at t = t, the
velocity is v

(i) v = u + at

1 2
(ii) s = ut + at
2

(iii) v 2 = u 2 + 2as

 1 1
(iv) Sn = u + a  n -  u+ a (2n –1)
 2 2

Retardation or Deceleration

When the velocity and acceleration of a body are along opposite direction, the speed of the body
decreases. Then the body is said to be decelerated.

Stopping time and stopping distance

u
Stopping time, t 
a

u2
Stopping distance, s 
2a

Motion of a body under gravity.

When a body moves under the action of earth’s gravitational field alone, the body is said to be freely
falling.

A body is dropped from some height with initial velocity zero and a = g, then equations of motion :

32
[Study Package - Physics - Class XI]

v  gt (i)
1 2
h gt (ii)
2
v 2  2gh (iii)
g
h n   2n  1 (iv)
2
 If a body starts from rest and moves with uniform acceleration that distance covered by the body in t
second is proportional to t2 as h  t 2 , ratio of distance covered in 1 second, 2 seconds and 3 seconds
is 12 : 22 : 32 or 1 : 4 : 9
 If a body starts from rest and move with uniform acceleration then distance covered by the body in nth
second is proportional to (2n – 1). Ratio of distance covered in 1 second, 2 second, 3 second is 1:3:5.
 The motion is independent of the mass of the body, as in any equation of motion, mass is not involved.
That is why a heavy and light body when released from the same height, reach the ground simultaneously
and with same velocity i.e., t   2h / g  and v  2gh

 In case of motion under gravity, time taken to go up is equal to the time taken to fall through the same
u
distance. Time of descent (t1) = Time of ascent (t2) 
g

2t
 Total time of flight T  t1  t 2  g

 A ball is dropped from a building of height h and it reaches after t seconds on earth. From the same
building it two ball are thrown (one upwards and other downwards) with the same velocity u and they
reach the earth surface after t1 and t2 seconds respectivley, then t  t1t 2
RELATIVE VELOCITY
Relative velocity is defined as the time rate of change of relative position of one object with respect to
another.
If the velocity of two particles A and B are VA and VB, then the velocity of A relative to B.
v AB  v A  v B

v BA  v B  v A

v AB   v BA
(i) When the two objects are moving along parallel straight lines in the same direction i.e., angle
between them is 00
Since v AB , v A and v B all are in the same direction, we can write

v AB  v A  v B
(ii) When the two objects are moving along parallel straight lines in opposite directions
i.e., angle between them is 1800.
  
v AB  v A  v B

33
Brilliant STUDY CENTRE

QUESTIONS
LEVEL - I
1. If displacement of a particle is zero, the distance covered:
1) must be zero 2) may or may not be zero
3) cannot be zero 4) depends upon the particle
2. The three initial and final positions of a man on the x-axis are given as
(i) (–3 m, 7 m) (ii) (7 m, –3 m) (iii) (–7 m, 3m)
Which pair gives the negative displacement?
1) (i) 2) (ii) 3) (iii) 4) (i) and (iii)
3. When a particle returns to its initial point, its
1) displacement is zero 2) instantaneous speed is zero
3) distance is zero 4) average speed is zero
4. The numerical value of the ratio of velocity to speed is:
1) always less than one 2) always equal to one 3) always more than one4) equal to or less than one
5. The magnitude of the displacement is equal to the distance covered in a given interval of time if the
particle:
1) moves with constant acceleration 2) moves with constant speed
3) moves with constant velocity 4) none of the above
6. Consider the acceleration, velocity and displacement of a tennis ball as it falls to the ground and
bounces back. Directions of which of these changes in the process?
1) Velocity only 2) Displacement and velocity
3) Acceleration, velocity and displacement 4) Displacement and acceleration
7. A truck travels a distance A to B at a speed of 40 km/h and returns to A at a speed of 50 km/h, then what
is the average velocity of the whole journey?
1) 34.5 km/h 2) Zero 3) 35 km/hr 4) 40 km/hr
8. The magnitude of average velocity is equal to the average speed when a particle moves:
1) on a curved path 2) in the same direction
3) with constant acceleration 4) with constant retardation
9. A car moves a distance of 200 m. It covers the first half of the distance at speed 40 km/h and the
second half of distance at speed v. The average speed is 48 km/h. Find the value of v.
1) 56 km/h 2) 60 km/h 3) 50 km/h 4) 48 km/h
10. The motion of a particle is described by the equation x = a + bt2, where a = 10 cm, b = 15 cm–2. Its
instantaneous velocity at t = 3 second will be?
1) 10 cms–1 2) 20 cms–1 3) 60 cms–1 4) 90 cms–1
11. A car accelerated from initial position and then returned at initial point, then:
1) velocity is zero but speed increases 2) speed is zero but velocity increases
3) both speed and velocity increase 4) both speed and velocity decrease

34
[Study Package - Physics - Class XI]

12. The acceleration a of the body starting from rest varies with time following the equation a = 8t + 5. The
velocity of the body at time t = 2 sec will be
1) 22 m/s 2) 26 m/s 3) 28 m/s 4) 30 m/s
13. A particle moves along a straight line such that its displacement at any time t is given by:

s   t 3  3t 2  2  m The displacement when the acceleration becomes zero is


1) 0 m 2) 2 m 3) 3 m 4) –2 m
14. The distance travelled by a particle is directly proportional to t1/2, where t is time elapsed. The motion
is one with:
1) increasing acceleration 2) decreasing acceleration
3) increasing retardation 4) decreasing retardation
15. The slope of the tangent to the v-t curve gives the value of:
1) instantaneous acceleration 2) Instantaneous velocity
3) Average acceleration 4) None of these
16. An object is moving in negative direction with a negative acceleration. The velocity-time graph with
constant acceleration which represents the above situation is:

17. A particle starts from rest at t = 0 and moves in a straight line with an acceleration shown below. The
velocity of the particle at t = 3 s is

1) 5 m/s2 2) 6 m/s2 3) 10 m/s2 4) 15 m/s2


18. Which of the following cannot be speed-time graph?

1) (B) and (D) 2) (C) and (E) 3) (D) only 4) (A) only

35
Brilliant STUDY CENTRE

19. Which of the following graphs cannot possible represent one dimensional motion of a particle?

Speed
Position Velocity
Total distance
covered

Time

Time Time Time


I II III IV

1) I and II 2) II and III 3) II and IV 4) All four


20. A ball thrown vertically upward with a a speed of 19.6 m/s from the top of a tower returns to the earth in
6 second. What is the height of the tower?
1) 40 m 2) 58.8 m 3) 50 m 4) 70 m
21. A body is travelling east with a speed of 9 m/s and with an acceleration of 2 m/s2 acting west on it. The
displacement of the body during the 5th second of its motion is
1) 0.25 m 2) 0.5 m 3) 0.75 m 4) zero
22. A car moving with a velocity of 10 m/s can be stopped by the application of a constant force F in a
distance of 20 m. If the velocity of the car is 30 m/s it can be stopped by this force in
1) 100 m 2) 120 m 3) 180 m 4) 160 m
23. Two balls of different masses ma and mb are dropped from two different heights a and b. The ratio of
the time taken by the two to cover these distances are

1) 1 2) a/b 3) b : a 4) a : b

24. From a building two balls A and B are thrown such that A is thrown upwards and B downwards with
same velocity. VA and VB are the velocities on reaching the ground then:
1) VB > VA 2) VA = VB 3) VA > VB 4) Velocity depends upon
mass
25. A body is dropped from a balloon moving up with a velocity 4 ms–1 when the balloon is at a height of
120.5m from the ground; the height of the body after five seconds from the ground is:
1) 8 m 2) 12 m 3) 18 m 4) 24 m
26. A ball is thrown vertically downward with a velocity of 20 m/s from the top of a tower. It hits the ground
after sometime with a velocity of 80 m/s. The height of the tower is (g =10 m/s2)
1) 340 m 2) 320 m 3) 300 m 4) 360 m
27. A boy standing at the top of a tower of 20 m height drops a stone. Assuming g = 10 ms–2, the velocity
with which it hits the ground is
1) 20 m/s 2) 40 m/s 3) 5 m/s 4) 10 m/s
28. A very large number of balls are thrown vertically upwards in quick succession in such a way that the
next ball is thrown when the previous one is at the maximum height. If the maximum height is 5 m, the
number of balls thrown per minute is (take g = 10 ms–2)
1) 40 2) 60 3) 80 4) 120

36
[Study Package - Physics - Class XI]

29. A train of 200 m long travelling at 50 m/s overtakes another train 130 m long travelling at 30 m/s. The
time taken by the first train to pass the second train is
1) 15 second 2) 17 second
3) 16.5 second 4) 18 second

LEVEL - II
1. When a particle moves with uniform velocity, which of the following relations are correct?
(I) Average speed = average velocity
(II) Instantaneous speed = instantaneous velocity
(III) Distance covered = magnitude of displacement
1) I, II, III 2) I, II 3) II, III 4) I, III
2. The x and y coordinates of the particle at any time are x = 5t – 2t2 and y = 10 t respectively, where x and
y are in meters and t in seconds. The acceleration of the particle at t = 2 s is
1) 5 m/s2 2) –4 m/s2
3) –8 m/s2 4) 0

3. The position x of a particle values with time, t, as x  at 2  bt 3 . The acceleration of the particle will be
zero at time t equal to

a 2a
1) 2)
b 3b

a
3) 4) zero
3b
1/2
4. If the velocity of a car is given by v  150  10x  m / s . If car retards their motion by applying brakes
then what will be the acceleration?
1) 1 m/s2 2) 2 m/s2
3) 5 m/s2 4) –5 m/s2
5. The velocity time graph of a body is shown in the figure. The ratio of the average acceleration during
the intervals OA and AB is

v(m/s)
D C

30o 60o t(s)


O A B

1) 1 2) 1/2 3) 1/3 4) 3

37
Brilliant STUDY CENTRE

6. Match the terms in Column I with items (position - time graph) in column II and choose the correct
option from the codes given below:

1) A -1, B - 2, C -3 2) A - 1, B - 3, C - 2
3) A - 2, B - 1, C - 3 4) A - 3, B - 2, C - 1
7. The v-t curve shown above is a straight line parallel to time-axis. The displacement in the time interval
t = 0 and t = T is equal to

1) T 2) Area of the rectangle of height  and base T


3) Both 1 and 2 4) Slope of the curve
8. The variation of quantity A with quantity B, plotted in figure. Describe the motion of a particle in a straight
line.

a) Quantity B may represent time


b) Quantity A is velocity if motion is uniform
c) Quantity A is displacement if motion is uniform
d) Quantity A is velocity if motion is uniformly accelerated
1) ab 2) abc 3) acd 4) bcd

38
[Study Package - Physics - Class XI]

9. Velocity-time graph corresponding to displacement-time graph shown in adjoining figure is:

10. A particle starts from rest. Its acceleration at time t = 0 is 5 m/s2 which varies with time as shown in
the figure. The maximum speed of the particle will be
a
5 m/s2

0 t
6s
1) 7.5 m/s 2) 15 m/s 3) 30 m/s 4) 37.5 m/s
11. A particle moving with uniform acceleration has velocity 6 m/s at a distance of 5 m from the initial
position. After moving another 7 m, the velocity becomes 8 m/sec. The initial velocity and acceleration
of the particle are:
1) 2 m/s, 4 m/s2 2) 4 m/s , 2 m/s2 3) 4 m/s, 4 m/s2 4) 6 m/s, 1 m/s2
12. A particle moves in a straight line with a constant acceleration. It changes its velocity from 10 ms–1 to
20 ms–1 while passing through a distance 135 m in t second. The value of t is
1) 12 s 2) 9 s 3) 10 s 4) 1.8 s
13. An elevator, in which a man is standing, is moving upward with a constant acceleration of 2 m/s2. At
some instant when speed of elevator is 10 m/s, the man drops a coin from a height of 1.5 m. Find the
time taken by the coin to reach the floor

1 1 1
1) sec 2) sec 3) sec 4) 1 sec
3 2 2
14. A body starting from rest moves with constant acceleration. The ratio of distance covered by the body
during the 5th second to that covered in 5 second is

9 3 25 1
1) 2) 3) 4)
25 25 9 25
15. Speeds of two identical cars are u and 4u at a specific instant. If the same deceleration is applied on
both the cars, the ratio of the respective distances in which the two cars are stopped from the instant
is
1) 1 : 1 2) 1 : 4 3) 1 : 8 4) 1 : 16

39
Brilliant STUDY CENTRE

16. A car travelling at a speed of 30 km h–1 is brought to a halt in 8 m by applying brakes. If the same car
is travelling at 60 km h–1, it can be brought to a halt with the same braking power in
1) 32 m 2) 24 m 3) 16 m 4) 8 cm
17. A stone starts to fall under gravity. The distances covered by it in first, second and third second are in
ratio.
1) 1 : 3 : 5 2) 1 : 2 : 3 3) 1 : 4 : 9 4) 1 : 5 : 9
18. A stone is dropped from the top of a tower of height h. After 1 second another stone is dropped from the
balcony 20 m below the top. Both reach the bottom simultaneously. What is the value of h? (Take g =
10 ms–2)
1) 3125 m 2) 312.5 m 3) 31.25 m 4) 25.31 m
19. A ball is dropped from a high rise platform at t = 0 starting from rest. After 6 s another ball is thrown
downwards from the same platform with a speed v. The two balls meet at t = 18 s. What is the value
of v? (takge g = 10 ms–2)
1) 74 ms–1 2) 55 ms–1 3) 40 ms–1 4) 60 ms–1
20. Two balls A and B, mass of A is m and that of B is 5 m are dropped from the towers of height 36 m and
64m respectively. The ratio of the time taken by them to reach the ground is:

1 3 2 5
1) 2) 3) 4)
2 4 5 2
21. When a ball be thrown up vertically with velocity v0, it reaches a maximum height of h. If one wishes to
triple the maximum height then the ball should be thrown with velocity

3v0
1) 3 v0 2) 3v0 3) qv 0 4)
2
22. From balloon rising vertically upward at 6 m/s a stone is thrown up at 16 m/s relative to the balloon. Its
velocity with respect to the ground after 2 second is
1) 10 m/s 2) 4 m/s 3) 6 m/s 4) 2 m/s
23. A thief is running away on a straight road in a jeep moving with a speed of 9 ms–1. A policeman chases
him on a motor cycle moving at a speed of 10 ms–1. If the instantaneous separation of the jeep from
the motor cycle is 100 m., how long will it take for the policeman to catch the thief?
1) 1 s 2) 19 s 3) 90 s 4) 100 s
24. A train of 150 m length is going towards north direction at a speed of 10 m/s. A bird flies at a speed of
5 m/s towards south direction parallel to the railway track. The time taken by the bird to cross the train
is equal to
1) 20 s 2) 16 s 3) 12 s 4) 10 s
25. A bus is moving with a speed of 10 ms–1 on a straight road. A scooterist wishes to overtake the bus in
100 s. If the bus is at a distance of 1 km from the scooterist, with what speed should the scooterist
chase the bus?
1) 40 ms–1 2) 25 ms–1 3) 10 ms–1 4) 20 ms–1

40
[Study Package - Physics - Class XI]

KEY WITH HINTS


LEVEL - I
1. 2
2. 2 Displacement = Final position – Initial position

x  x 2  x1

Then, (i) x  7m   3m   10 m (ii) x  3m  7m  10 m

(iii) x  3m   7m   10m


3. 1 When a particle returns to its intial position its displacement is zero.
4. 4
5. 3 To cover the distance equal to magnitude of displacement, the particle has to move with constant
velocity.
6. 2
7. 2 Total displacement = 0

Total displacement
Average velocity  0
Time
8. 2

200 1 1 2 1
9. 2 48  or   
100 / 40   100 / v  40 v 48 24

1 1 1 2 1
or     ; or v = 60 km/hr
v 24 40 120 60

dx d  a  bt 2 
10. 4 x = a + bt2;   V  0  2bt  90cms 1
dt dt
11. 1 Velocity is zero as total displacement is zero but speed increases as it is being accelerated.

12. 2 a  8t  5  dv  8t  5  dt
v t
8t 2
 dv   8t  5  dt  v 
0 0 2
 5t

v  4t 2  5t  26 m / s

ds
2 d 2s
13. 1 s  t  3t  2 ;  dt  3t  6t ; and acceleration 2  6t  6
3 2
dt

s  t 3  3t 2  2  1  3  2  0
41
Brilliant STUDY CENTRE

dx 1 dx
14. 4 Given that x  t ;   or  t 1/2
dt 2 t dt

d2 x 1 3/ 2
Further, 2   t or a  t 3/ 2
dt 2
 a is retardation and as t increases retardation decreases.
15. 1
16. 4
17. 1 Velocity = area under acceleration time graph.
Velocity = 5 – 5 + 5 = 5 m/s2
18. 4 A particle can have only one-speed at one time and it cannot have two speeds at one time
19. 4 I is not possible because total distance covered by a particle increases with time.
II is not possible because at a particular time, position cannot have two values.
III is not possible because at a particular time, velocity cannot have two values.
IV is not possible because speed can never be negative.

1
20. 2 S  ut  at 2 ; S  19.66  4.9  36 ; S = 58.8 m
2
21. 4 Taking west to east as positive, then u = 9 m/s and a = –2 m/s2

a W 2
As Sn th  u   2n  1 +ve
E  S5th  9   2  5  1  9  9  0
2 2
22. 3 v 2  u 2  2as
After applying the brakes car will come to rest v = 0

2 2
s u2 u   30 
u s;
2  1  12  s 2   2   s1 ; s 2     20  9  20  180 m
s2 u 2  u1   10 

1 2 ta a
23. 2 h gt (u = 0); h  t 2 ;  t  h ; Hence, 
2 tb b

24. 2 VA  VB . Since, the motion is uniform. The velocity at each point is same so this velocity of A
after coming back is also VA, just the direction is reversed. Thus VA = VB.

1
25. 3 s  ut  gt 2  102.5 m
2
This shows that the body is 102.5 m below the initial position.
h = H – S = 120.5 – 102.5 = 18 m.
height of the body = 18 m.

42
[Study Package - Physics - Class XI]

26. 3 From kinematic equation of motion, v 2  u 2  2gh

v2  u 2
h  300 m
2g

27. 1 g = 10 ms–2 and h = 20 m; We have v  2gh  20 ms1

2s 2 5
28. 2 Time taken by ball to go upto the highest point   1 s
g 10
As next ball has been thrown when the previous one is highest point, hence one ball is thrown
per second. So number of balls thrown per minute = 1 × 60 = 60.
29. 3 Relative velocity of 1st train w.r.t 2nd = 20 m/s
Total distance = 330
Time taken = 16.5 seconds
LEVEL - II
1. 1

2. 2 x  5t  2t 2 ; y = 10 t;

dv x
 v x  5  4t ; v y  10 ; a x   4 m / s 2
dt

dx dv
3. 3 x  at 2  bt 3 ; v   2at  3bt 2 ; Acceleration, a   2a  6bt
dt dt

2a a
If a = 0 then 2a – 6bt = 0; or t  
6b 3b
1/ 2
1/ 2 dv d 150  10x  dx
4. 4 v  150  10x  ;  
dt dx dt

a
1  10  dx

5 1/ 2
 150  10x  ;
1/ 2 ; 1/ 2
2 150  10x  dt 150  10x 

a  5 m / s 2
5. 3 Average acceleration = slope of velocity time graph.

a1 tan 30o 1 / 3 1
   
a 2 tan 60o 3 3

6. 2 In position-time curve, upward direction for positive acceleration and downward for negative
acceleration and it is straight line for zero acceleration.

43
Brilliant STUDY CENTRE

7. 3 Area under the v-t curve = Displacement during t = 0 and t = T    T  T

8. 3

A=x A=v

x = displacement v = velocity
I II

dx
In case I velocity =  = slope = constant.
dt
Here motion is uniform and A represent displacement.

dv
In case II a  = slope = constant.
dt
Here motion is acceleration and A represent velocity.
9. 2 In the portion OA, slope (= velocity) of the curve is +ve; at the point A, slope of the curve is zero;
while in the portion AB, slope of the curve is –ve. Hence, (v - t) curve will be as shown in option
2
10. 2 Area under a-t graph gives the change in velocity during given time interval.

1
 Change in velocity   5  6  15 m / s ; Since initial velocity = 0
2
 Maximum speed of the particle = 15 m/s
11. 2 v 2  u 2  2as ; and 82  u 2  2a  12
solving, a = 2 m/s2 and u = 4 m/s

400  100 10
12. 2 a   m / s2
2  135 9

vu
v  u  at or t   9s
a

1 1 1 1
13. 2 d rel  a rel t 2  1.5  10  2  t 2 1.5   12  t 2  t  sec
2 2 2 2

a 9a
14. 1 Distance covered in 5th second is D5  0   2  5  1 
2 2

1 2 25a D5 9
Distance covered in 5 seconds is s 5  0  a5  ;  
2 2 s5 25

44
[Study Package - Physics - Class XI]

15. 4 Both are given the same deceleration simultaneously and both finally stop.
Formula relevant to motion : u 2 = 2as

2
u2

 4u  
16u 2

s1 1

 For first car, s1  ; For second car, s 2
2a 2a 2a s 2 16

2 2
v 2  u 2 0   25 / 3 a    25   1 50 1
16. 1 Retardation, a   ;   ; For u = 60 km h–1  ms
2s 28  3  16 3

2
0   50 / 3
s 2
 32 m
2     25 / 3  1 / 16 
 

g g g
17. 1 Sn th  u   2n  1 ; S1  0   2  1 
2 2 2

g 3 g 5g
S2  0   2  2  1  g ; S3  09   2  3  1 
2 2 2 2

g 3g 5g
S1 : S2 : S3  : :  1: 3 : 5
2 2 2

1 2 1 2
18. 3 h gt -------- (i) ; and h  20  g  t  1 -------(ii)
2 2
Solving eqn. (i) and (ii) t = 2.5 s and h = 31.25 m

1 2 1 2
19. 1 For first ball, u = s1  gt1   g 18 
2 2
For second ball, initial velocity = v

1 1 2
s 2  vt 2  gt 2 ; t 2  18  6  12s ;  s 2  v  12  g 12 
2 2

Here, s1  s 2 ;  v  74 ms 1

2h tA 3
20. 2 Time period is independent of mass of body; t  
g tB 4

2
21. 1 As v 2  u 2  2as ; So 0  v 0  2  g  h ; or v 0  h ; when h  3h

v0 3h
then v0  h  3h ;    3 ; or v  3 v
v0 h 0 0

45
Brilliant STUDY CENTRE

22. 4 Initial velocity of the balloon w.r.t. the ground u = 6 + 16 = 22 m/s

Velocity after 2 second  v  u  gt  2 m / s

23. 4 Relative velocity of policeman w.r.t the thief is 10 ms–1 –9 ms–1 = 1 ms–1. Since, the separation
between them is 100 m, hence the time taken will be 100 s.

24. 4 Relative velocity of bird with respect to train  5   10   15 m/s towards south

distance to be travelled 150


Time taken t= = =10 s
relative velocity 15

25. 4 Let vs be the velocity of the scooter, the distance between the scooter and the bus = 1000 m.
The velocity of the bus = 10 ms–1
Time taken to overtake = 100 s
Relative velocity of the scooter with respect to the bus = (vs – 10)

1000
  100 s or v  20 ms 1
 vs  10  s

46
[Study Package - Physics - Class XI]

CHAPTER - 03
MOTION IN A PLANE

A. VECTORS
Physical quantities are of two types:
¨ Scalars
¨ Vectors
Scalar Quantities or Scalars
The physical quantities which have only magnitude but no direction, are called scalar quantities or
scalars. Mass, length, time, distance covered, speed, temperature, work etc, are a few examples of scalars.
Vector quantities or vectors
The physical quantities which have magnitude as well as direction are called vector quantities or vectors.
Displacement, velocity, acceleration, force, torque, gravitational intensity etc. are a few examples of vectors.
Vectors in a plane (or in two dimensions)
Consider the motion of an object in the X-Y plane with origin at O. Let at time t, the object be at point A.

If we draw an arrow with its tail at point O and head at point A, as shown in Fig., then OA is called position
vector of the object at point A and is generally represented by r .
y

r
r1
B
r2
x
O

The magnitude of the vector is called the modulus of the vector. The modulus of a vector A is represented
 
by A or A. e.g., For a force F  5 N acting towards North, the magnitude of the force vector is 5N

* Negative vector : A vector is said to be negative of a given vector if its magnitude is the same as that
of the given vector but direction is opposite to that of it.

47
Brilliant STUDY CENTRE

* Equal vectors : Two vectors are said to be equal if they have same magnitude and direction regardless
of the position of their initial points.
* Collinear vectors : Vectors which are either along the same line or along parallel lines irrespective of
their magnitudes and directions are collinear vectors.
* Coplanar vectors : Vectors which are parallel to the same plane or lying in the same plane are coplanar
vectors.
* Unit vector : A unit vector of the given vector is a vector of unit magnitude and has the same direction
as that of the given vector. A unit vector in a given direction is also defined as a vector in that direction
divided by the magnitude of the given vector. It is a unitless and dimensionless vector and represents
direction only.

 a
Unit vector of a is written as â and is given by â  
|a|

a x ˆi  a y ˆj  a z kˆ
If a x ˆi  a y ˆj  a z kˆ then â 
a 2x  a 2y  a 2z

In Cartesian coordinates, ˆi, ˆj, kˆ are the unit vectors along x-axis, y-axis and z-axis respectively..

* Null vector or zero vector : A vector of zero magnitude is known as zero or null vector. Its direction is

not defined. It is denoted by 0
Multiplication of a Vector by a Real Number
 
The multiplication of a vector A by a real number n becomes another vector nA .
A A

2A 2A
(a) (b)
Its magnitude becomes n times the magnitude of the given vector. Its direction is the same or opposite
 
as that of A , according as n is a positive or a negative real number. The unit of nA , is the same as the unit

of vector A .
Multiplication of a Vector by a Scalar
 
When a vector A is multiplied by a scalar S, it becomes a vector SA , whose magnitude is S times the
   
magnitude of A and it acts along the direction of A . The unit of SA is different from the unit of vector A .
Resultant vector
The resultant vector of two or more vectors is defined as that, single vector which produces the same
effect as is produced by individual vectors together. It is to be noted that the nature of the resultant vector is
the same as that of the given vectors.
LAWS OF VECTOR ADDITION
(1) Vectors of the same nature alone can be added eg. a force vector cannot be added to velocity
vector, but can be added to force vector only.
48
[Study Package - Physics - Class XI]

(2) Vector addition is commutative. It states that the sum of the vectors remains the same in whatever
order they may be added ie.,
   
AB  BA
(3) Vector Addition is Associative : It states that the sum of the vectors remains the same in whatever,
grouping they are added i.e.,
     
 A  B  C  A   B  C 
(i) Triangle law of vectors
It states that if two vectors acting on a particle at the same time are represented in magnitude and
direction by the two sides of a triangle taken in an order, their resultant vector is represented in magnitude
and direction by the third side of the triangle taken in the opposite order.
Magnitude and direction of R
 
Let the two vector A and B , inclined at an angle  , act on a particle at the same time. Let them be
 
represented in magnitude and direction by the two sides OP and PQ of triangle OPQ, taken in the same
 
order, Then, according to triangle law of vector addition, the resultant R is represented by the third side OQ
of triangle, taken in opposite order.

B sin 
i.e., R  A 2  B2  2AB cos  ; tan  
A  B cos 
(ii) Parallelogram law of vector addition
 
Let the two vectors A and B , inclined at angle  be acting on a particle at the same time.

 
Let they be represented in magnitude and direction by two adjoining sides OP and OS of parallelogram

OPQS, drawn from a point O. According to parallelogram law of vectors, their resultant vector R will be

represented by the diagonal OQ of the parallelogram.

Bsin 
R  A 2  B2  2AB cos  and tan  
A  Bcos 
49
Brilliant STUDY CENTRE

Lamis’ theorem
A


 C

B
Lami’s theorem states that if three forces acting at a point are in equilibrium, then each force is
proportional to the sine of the angle between the other two forces. Let A, B and C be three forces acting
at a point on a body making angles ,  and  with each other,,

A B C
Then  
sin  sin  sin 
Conditions for Zero Resultant Vector
If three vectors acting on a point object at the same time are represented in magnitude and direction by
the three sides of a triangle taken in the same order, their resultant is zero. Now the object is in equilibrium.
Then
Q

A  B  C B
OP PQ QO
O P
A

Consider three vectors A, B and C acting on an object at the same time. Let they be represented in
magnitude and direction by the various sides of a triangle taken in the same order, their resultant vector is
zero.
Polygon law of vector addition

Let the number of vectors A, B, C and D etc. be acting in different directions as shown in the Fig..

S
D
D C
C
T A  B  C  Q

B R A  B B

A O A P
(a) (b)
This method of finding the resultant is called polygon law of vectors.

50
[Study Package - Physics - Class XI]

Thus polygon law of vectors states that if a number of vectors, acting on a particle at the same time are
represented in magnitude and direction by various sides of an open polygon taken in the same order, their
resultant is represented in magnitude and direction by the closing side of the polygon taken in opposite
order.
Polygon law of vectors is an extension of the triangle law of vectors. According to polygon law of vectors.
    
A  B C  D  R
SUBTRACTION OF VECTORS
   
Subtraction of vector B from a vector A is defined as the addition of vector  B (negative of vector B )

to vector A . Thus
   
A  B  A  (  B).
   o

If the angle between A and B is  , then the angle between A and  B is 180   . 
 
A  B  A 2  B2  2AB cos 

RESOLUTION OF A VECTOR
The process of splitting up a vector into two or more vectors is known as resolution of a vector. The
vectors into which a given vector is split are known as component vectors. When a vector is splitted into
two component vectors at right angles to each other, the component vectors are called rectangular
components of a vector.
Rectangular components of a vector in a plane :


Let vector A makes an angle  with x-axis as shown in the figure. Then,

A  Ax ˆi  Ayˆj

Here, A x  A cos  and A y  A sin 

A 2x  A 2y  A 2 (cos 2   sin 2 )

1
Ay

or A  A  A
2
x y
2 2
and tan  
Ax
.

51
Brilliant STUDY CENTRE

Rectangular components of a vector in a space :


Let ,  and  are the angles between vector A and the x, y and z-axes, respectively as shown in the
figure. Then

A  A x ˆi  A y ˆj  A z kˆ . The magnitude of vector A is A  A 2x  A 2y  A z2
PRODUCT OF TWO VECTORS
The Product of two vectors can be of two types

(A) Scalar product or dot product of two vectors .


(B) Vector or cross product of two vectors
SCALAR PRODUCT OR DOT PRODUCT
   
The dot product or scalar product of two vectors A and B given by A.B  AB cos 

Properties of scalar product


   
i) A. B  B.A
      
ii)  
A. B  C  A.B  A.C

    1
iii) A. A  AA cos 0o  A 2 or A  A. A   2

ˆi.iˆ  ˆj. ˆj  k.k


ˆ ˆ 1

Also ˆi . ˆj  0, ˆj.kˆ  0, kˆ .iˆ  0

CROSS PRODUCT OR VECTOR PRODUCT


 
The cross product or vector product of two vectors A and B is given by
   
A  B  A B s in  nˆ where n̂ is a unit vector indicating the direction of A × B .

52
[Study Package - Physics - Class XI]

Properties of vector product


   
i) A  B  B  A
      
ii) 
A B C  AB AC 
 
iii) o
A  B  ABsin 90 nˆ  ABnˆ

iv) ˆi  ˆj  kˆ , ˆj  kˆ  ˆi, kˆ  ˆi  ˆj Similarly, ˆj  iˆ   k,


ˆ kˆ  ˆj   ˆi, ˆi  kˆ   ˆj

ˆi  ˆi  0, ˆj ˆj  kˆ  kˆ  0
Vector product of two parallel vectors is zero.
  
A  B  AB sin0o nˆ  0
This leads us to the following condition of parallelism of two vectors.
“Two nonzero vectors are parallel if and only if the magnitude of their cross product is zero.”
Vector product of two vectors in Cartesian coordinates

Let A  A x ˆi  A y ˆj  A z kˆ

and B  Bx ˆi  By ˆj  Bz kˆ

ˆi ˆj kˆ
 
Then A  B  A x A y A z  ˆi  A y Bz  A z By   ˆj  A z B x  A x B z   kˆ  A x B y  A y B x 
Bx By Bz

  
Note: If A  B  C  0
     
then A  B  B  C  C  A
RELATIVE VELOCITY

Let  be the angle between the directions of motion of the objects A and B, moving with velocities v A

and v B as shown in Fig.

Q
vA

q
O P
–vB

 
 
Where, v A  OQ and v B  OP .  
53
Brilliant STUDY CENTRE

In magnitude, the relative velocity of A with respect to B is given by

v AB  v2A  v 2B  2vA v B cos 


Applications of relative velocity
Crossing River
When a man crosses a river, the direction of the movement of the man will be in the direction of the
resultant velocity of the man and the river.
For shortest time
The man should travel perpendicular to the direction of flow of river.

When he crosses the river velocity along the river = v x  VR

Velocity perpendicular to the river  v f  VmR

2
The net velocity v m  VmR  VR2

mR V
The angle  is given by tan  
VR (down stream with river flow)

d
Time taken to cross the river t 
VmR

d
The drift along the river = x = Vxt , x  VR
VmR

B x

d VR
VmR Vm

x
A
Shortest path
For this the swimmer should swim upsteram making an angle  with AB such that the resultant
velocity Vm is perpendicular to the river flow. i.e., Along AB.

2
The net speed Vm  VmR  VR2 along AB

.y
B

d
VR
Vm
VmR
A
54
[Study Package - Physics - Class XI]

d d
Time to cross the river
 
Vy V2
mR  VR 2

 V 
  sin 1  R 
 VmR 

 1  Vm 
To swim along shortest path the man should swim at an angle of 2  sin  V R  upstream from the
 m 
direction of river flow.
Rain-man problem


vm
 
v rm vr


v rm -velocity of rain w.r.t. man

v r - velocity of rain w.r.t ground.

v m - velocity of man w.r.t ground
  
v r  v rm  v m
B. TWO DIMENSION MOTION
Projectile Motion
When a body is thrown at angle with horizontal, then its motion is governed by gravitational acceleration.
(neglect air resistance)
The body is known as projectile and its motion is known as projectile motion.

Horizontal motion Vertical motion


u x  u cos  u y  u sin 
ax  0 ay  g
1 1
x  uxt  axt2 y  u yt  a yt 2
2 2

55
Brilliant STUDY CENTRE

Trajectory of projectile
1
x  u cos t; y  u sin t  gt 2
2
x
Eliminating t, using t 
u cos 
2
 x  1  x 
 y  u sin    g 
 u cos   2  u cos  

gx 2
y  x tan   2
2u cos2 
The trajectory of the projectile is a parabola.
vy
 At any instant t, during projectile motion, the velocity is v  v 2x  v 2y and tan  
vx
Time of Flight (FT)
at t = T, y = 0
1
y  uyt  a yt2
2
1
0   u sin   T   g  T 2
2
1 2
gT   u sin   T
2
2u sin 
T
g
Range (R)
at t  T, x  R
1
x  ux t  a  t 2
2
R   u cos    T  0
2u sin 
R  u cos 
g

u 2 sin 2
R
g
 Range is maximum when   45o

u2
 R max 
g
56
[Study Package - Physics - Class XI]

Maximum Height (H)


When y = H, then vy = 0

v 2y  u 2y  2a y y

0  u 2 sin 2   2  g  H

2gH  u 2 sin 2 

u 2 sin 2 
H
2g

u 2 sin 2 45o u 2 R max


 When range is maximum the height H attained by the projectile H   
2g 4g 4

u 2 R max
H
 When range is maximum the maximum height attained by the projectile max  
2g 2
Energy of projectile.
When a projectile moves upwards its KE decreases and potential energy increases. But the total
1
energy remains constant. Let a body is projected with K.E. = mv 2 at an angle  with the horizontal.
2
1
Then at the highest point into KE is mu 2 cos 2 
2

u cos 
u

k' = kcos2 

mgu 2 sin 2  1
Potential energy  mgH   mu 2 sin 2 
2g 2

1 1 1
Total energy  mu 2 cos 2   mu 2 sin 2   mu 2
2 2 2
i.e., the energy at the point of projection.
Relative motion in projectile
If one of the projectiles is observed from the other projectile, then path of the projectile becomes
straight line. This is because acceleration of one of the projectiles with respect to the other projectile is zero
and so relative velocity vector becomes constant.

57
Brilliant STUDY CENTRE

C. CIRCULAR MOTION
When a particle moves in a plane such that its distance from a fixed (or moving) point remains con-
stant then its motion is called as the circular motion with respect to that fixed (or moving) point.
Variables of Motion
(a) Angular Position
The angle made by the position vector with given line (reference line) is called the angular position.

(b) Angular Displacement


Angle rotated by the position vector of the moving particle with some reference line is called angular
displacement.
(c) Angular Velocity w

Rate of change of angular displacement is angular velocity,  
t
(i) Average Angular Velocity
Total angle of rotation  2  1 
av = ; av  
Total time taken t 2  t1 t where q1 and q2 are angular positions of the particle
at times t1 and t2
(ii) Instantaneous Angular Velocity
The rate at which the position vector of a particle with respect to the centre rotates, is called as
instantaneous angular velocity with respect to the centre.
 d
  lim  -
t 0 t dt
Relation Between Linear Speed and Angular Velocity is v  r ;
  
 v   r
(d) Angular Acceleration a

Rate of change of angular velocity in angular acceleration  
t
(i) Average Angular Acceleration
Let 1 and 2 be the instantaneous angular speeds at times t1 and t2 respectively, then the average

2  1 
angular acceleration  av is defined as  av  
t 2  t1 t
(ii) Instantaneous Angular Acceleration
It is the limit of average angular acceleration as Dt approaches zero, i.e,
 d d d d
  lim   
t  0  t dt dt d d
58
[Study Package - Physics - Class XI]

If a = 0, circular motion is said to be uniform

d d d 2
As  = ,  
dt dt dt 2
i.e, second derivative of angular displacement with respect to time gives angular acceleration.

Tangential acceleration  α T 
Tangential acceleration is in the direction of motion or opposite to motion, and this acceleration is
responsible for change in speed of the particle. Its magnitude is rate of change of speed of the particle.

dv d|v| d  r
aT  or a T  or a T  ; a T  r
dt dt dt
Centripetal acceleration (ac)
In circular motion, direction of motion of the body changes due to centripetal force and acceleration
produced by this force is known as centripetal acceleration. Its direction is always towards centre of
the body magnitude of centripetal acceleration is given by

v2
ac  or a c  2 R
R
Types of circular motion
1) Uniform circular motion
If a particle moves with constant speed in a circle then that circular motion is known as uniform circular
motion.
 In uniform circular motion
tangential acceleration = 0

v2
Centripetal acceleration 
R
2) Non-uniform circular motion
If a particle moves with variable speed in circle, then that motion is known as non uniform circular
motin. In this motion, there will be both centripetal acceleration and tangential acceleration

a c  0 a T  0 ; then net acceleration is given by a net  a C3  a T2

2
dv 2  v v2
at  = rate of change of speed and ac = r  r     v
dt r r
Here, the two components are mutually perpendicular. Therefore, net acceleration of the particle will
be:

2 2 2
2 2  dv   v 2   dv  2 2 2
2
a  ar  at  2
 r         
 dt 
 r    r 
 r   dt 

59
Brilliant STUDY CENTRE

QUESTIONS
LEVEL - I

1. The vector sum of two forces is perpendicular to their vector differences. In that case, the forces:
1) cannot be predicted
2) are perpendicular to each other
3) are equal to each other in magnitude
4) are not equal to each other in magnitude
2. Two forces have magnitudes in the ratio 3 : 5 and the angle between their directions is 60 o. If their
resultant is 42N, find their magnitudes
1) 15 N, 25 N 2) 18 N, 30 N 3) 5 N, 15 N 4) 21 N, 35 N

3. The resultant of two vectors of magnitudes 2A and 2 A acting at an angle  is 10 A . Find the value
of 
1) 35o 2) 15o 3) 60o 4) 45o
     
4. Two vectors A and B are such that | A  B || A  B | then the angle between the two vectors
 
A and B will be

  
1) 0o 2) 3) 4)
3 6 2

5. The position vector of a particle is r   a cos t  ˆi   a sin t  ˆj .The velocity vector of the particle is

1) Parallel to position vector 2) Perpendicular to position vector


3) Directed towards the origin 4) Directed away from the origin

6.    
A particle starts with initial velocity of 2iˆ  ˆj m/s. Uniform acceleration  ˆi  3jˆ m / s 2 . What is
y component of velocity at the instant when x component of velocity becomes zero.
1) 5 m/s 2) 7 m/s 3) 6 m/s 4) 10 m/s
7. A particle moves towards east with velocity 5 m/s. After 10 seconds its direction changes towards
north with the same velocity. The average acceleration of the particle is

1
1) Zero 2) m / s2 N  W
2

1 1
3) m / s2 N  E 4) m / s 2S  W
2 2
8. A particle is executing a circular motion of radius R with a uniform speed v. After completing half the
cirlce, the change in velocity and in speed will be respectively
1) zero, zero 2) 2v, zero 3) 2v, 2v 4) zero, 2v

60
[Study Package - Physics - Class XI]

9. Rain is falling vertically with a speed of 4 ms–1. After sometime, wind starts blowing with a speed of 3
ms–1 in the north to south direction. In order to protect himself from rain, a man standing on the ground
should hold his umbrella at an angle  is given by

1 3 1 3
1)   tan   with the vertical towards south 2)   tan   with the vertical towards north
4 4

1 3 1 3
3)   cot   with the vertical towards south 4)   cot   with the vertical towards north
4 4
10. A river is flowing due east with a speed of 3 ms–1. A swimmer can swim in still water at a speed of
4 ms–1. If the swimmer starts swimming due north, then the resultant velocity of the swimmer is:

1) 3ms 1 2) 5ms 1 3) 7ms 1 4) 2ms 1


11.
2
 2
  
The position vector of a particle is given as r  t  4t  6 iˆ  t ˆj . Find the time after which the
velocity vector and acceleration vector becomes perpendicular to each other.
1) t = 1 sec 2) t = 2 sec 3) t = 3 sec 4) t = 4 sec

 
12. The velocity of a projectile at the initial point A is 2iˆ  3jˆ m/s. Its velocity (in m/s) at ponit B is

1) 2iˆ  3jˆ 2) 2iˆ  3jˆ 3) 2iˆ  3jˆ 4) 2iˆ  3jˆ

   
13. A particle has initial velocity 2iˆ  3jˆ and acceleration 0.3iˆ  0.2ˆj . The magnitude of velocity after
10 s will be

1) 9 2 units 2) 5 2 units 3) 5 units 4) 9 units

14. One fielder throws a ball and in 4 secs it reaches to another player. So, the maximum height reached
by the ball above the point of projection would be about
1) 10 m 2) 7.5 m 3) 5 m 4) 20 m
15. A projectile is projected with a kinetic energy K. Its range is R. It will have the minimum kinetic energy,
after covering a horizontal distance equal to
1) 0.25 R 2) 0.5 R 3) 0.75 R 4) R

gx 2
16. The equation of the projectile is y  3 x  . The angle of projection is given by
2

1 
1) tan   2) tan   3 3) 4) zero
3 2

61
Brilliant STUDY CENTRE

17. A bomber is flying horizontally with a constant speed of 150 m/s at a height of 78.4 m. The pilot has to
drop a bomb at the enemy target. At what horizontal distance from the target should he release the
bomb?
1) 0 m 2) 300 m 3) 600 m 4) 1000 m
18. The equations of motion of a projectile are given by x = 36 t metre and 2y = 96t – 9.8 t2 metre. The
angle of projection is

1 4 1 3 1 4 1 3


1) sin   2) sin   3) sin   4) sin  
5 5 3 4
19. Kinetic energy of a body projected at an angle of 30o is E. The kinetic energy at the topmost point of the
journey is
3E E E
1) E 2) 3) 4)
4 2 4
20. A body is projected horizontally from a point above the ground and motion of the body is described by
the equation x = 2t, y = 5t2 where x, and y are horizontal and vertical coordinates in metre after time t.
The initial velocity of the body will be
1) 29 m/s horizontal 2) 5 m/s horizontal
3) 2 m/s vertical 4) 2 m/s horizontal
21. An aircraft executes a horizontal loop of radius 1.0 km with a steady speed of 900 km h–1. The ratio of
centrepetal acceleration to the acceleration due to gravity is
1) 5.6 2) 4.6 3) 6.4 4) 8.1
22. A stone tied to the end of a string 1 m long is whirled in a horizontal circle with a constant speed. If the
stone makes 22 revolution in 44 second, what is the magnitude and direction of acceleration of the
stone?
1) 2 ms 2 and direction along the tangent to the circle
2) 2 ms 2 and direction along the radius towards the centre

2
3) ms 2 and direction along the radius towards the centre
4
4) 2 ms 2 and direction along the radius away from the centre
23. A particle describes a uniform circular motion. By what angle does its angular velocity change when it
completes half of the circular path?
1) 2  radian 2)  radian 3)  /2 radian 4) zero
2
24. In the given figure, a = 15 m/s represents the total acceleration of a particle moving in the clockwise
direction in a circle of radius R = 2.5 m at a given instant of time. The speed of the particle is

1) 4.5 m/s 2) 5.0 m/s 3) 5.7 m/s 4) 6.2 m/s

62
[Study Package - Physics - Class XI]

LEVEL - II
   
1. A  4iˆ  4ˆj  4kˆ and B  3iˆ  ˆj  4kˆ then angle between vectors A and B is
1) 180o 2) 90o 3) 45o 4) 0o
  
2. The resultant of two vectors  and B is perpendicular to the vector A and its magnitude is equal to
  
half of the magnitude of vector B . The angle between A and B is
1) 120o 2) 30o 3) 135o 4) 150o
3. The sum of the magnitudes of two forces acting at a point is 18 and the magnitude of their resultant is
12. If the resultant is at 90o with the force of smaller magnitude, what are the magnitudes of the
forces?
1) 12, 5 2) 14, 4 3) 5, 13 4) 10, 8
  
4. Resultant of two vectors A and B is of magnitude P. If B is reversed, then resultant is of magnitude
Q. What is the value of P2 + Q2.
1) 2  A2  B2  2) 2  A2  B2  3) A 2  B2 4) A 2  B2

5.     
A particle moves from a point 2iˆ  5jˆ to 4ˆj  3kˆ when a force of 4iˆ  3jˆ N is applied. How 
much work has been done by the force?
1) 8 J 2) 11 J 3) 5 J 4) 2 J
6. A man holds an umbrella at 30 with the vertical to keep himself dry. He, then, runs at a speed of 10ms–
o
1
and finds the rain drops to be hitting vertically. Speed of the rain drops w.r.t. the running man and w.r.t.
earth are

1) 20ms–1, 10ms–1 2) 10ms–1, 20 3ms 1

3) 10 3ms 1 , 20ms–1 4) 20ms–1, 10 3ms 1

7. A stone projected at angle 53o attains maximum height 25 m during its motion in air. Then its distance
from the point of projection where it will fall is

400
1) 75 m 2) m 3) 50 m 4) 60 m
3
8. A particle is projected with certain velocity at two different angles of projections with respect to horizontal
plane so as to have same range ‘R’ on a horizontal plane. If ‘t1’ and ‘t2’ are the time taken for the two
paths, then which one of the following relations is correct?

2R R R 4R
1) t1t 2  2) t1t 2  3) t1t 2  4) t1t 2 
g g 2g g

63
Brilliant STUDY CENTRE

9. A ball of mass M is thrown vertically upwards. Another ball of mass 2M is thrown at an angle  to
vertical. Both of them stay in air for the same period of time. The heights attained by the two are in the
ratio:
1) 1 : 2 2) 2 : 1 3) 1 : 1 4) 1: cos 
10. An aeroplane is flying horizontally with a velocity of 600 km/h and at a height of 1960 m. When it is
vertically above a point A on the ground, a bomb is released from it. The bomb strikes the ground at
point B. The distance AB is
1) 1200 m 2) 0.33 m 3) 3.33 km 4) 33 km
11. A body is projected, with velocity v1, at an angle of 30o with the horizontal. Another body is projected
vertically upwards with velocity v2 from a point on horizontal line, vertically below the highest point. If
v2
the two bodies collide at highest point, then v should be
1

1) 2 2) 0.5 3) 3 2 4) 2 3
12. A projectile is fired from the surface of the earth with a velocity of 5 ms–1 and angle  with the horizontal.
Another projectile fired from another planet with a velocity of 3 ms–1 at the same angle follows a
trajectory which is identical with the trajectory of the projectile fired from the earth. The value of the
acceleration due to gravity on the planet (in ms–2) is: (Given g = 9.8 ms–2)
1) 3.5 2) 5.9 3) 16.3 4) 110.8
13. A particle is thrown with velocity u making an angle  with the vertical. It just crosses the top of two
poles each of height h after 1 s and 3 s respectively. The maximum height of projectile is

1) 9 . 8 m 2) 19 . 6 m
3) 39 . 2 m 4) 4 . 9 m
14. A projectile can have same range from two angles of projection with same initial speed. If h1 and h2 be
the maximum heights, then

1) R  h1h 2 2) R  2h1h 2 3) R  2 h1h 2 4) R  4 h1h 2

15. A football is kicked 20 m/s at a projection angle of 45o. A receiver on the goal line 25 metres away in the
direction of kick runs the same instant to meet the ball. If he is to catch the ball before it hits the ground,
his speed must be ?
1) 5.483 m/s 2) 6.283 m/s 3) 5.112 m/s 4) 6.112 m/s

64
[Study Package - Physics - Class XI]

1 3
16. A particle is projected with a velocity of 30m/s, at an angle 0  tan   . After 1 s, the particle is
4
moving at an angle  to the horizontal, where tan  will be equal to (g = 10m/s2)

1 1
1) 1 2) 2 3) 4)
2 3
17. Two stones are projected with the same velocity but making different angles with the horizontal. Their
ranges are equal. If angle of projection of one is 30o and its maximum height is y, then the maximum
height of the other will be

y y
1) 3y 2) 2y 3) 4)
2 3

18. Two particles are projected from the ground simultaneously with speeds 30 ms–1 and 10 3 ms 1 at
angles 30o and 60o with the horizontal in the same direction. The maximum distance between them till
both of them strike the ground is approximately:

1) 15 3m 2) 20 3m 3) 30 3m 4) 40 3m
19. A projectile is fired at an angle of 45o with the horizontal. Elevation angle of the projectile at its highest
point as seen from the point of projection is

1 1
1) 60o 2) tan  
2

1
 3
3) tan   4) 45o
 2 
20. A point moves along a circle with speed v = at. The total acceleration of the point at a time when it has
traced 1/8th of the circumference is:

v a
1) 2) 2a 4  2 3) a 4) 4  2
8a 2
21. A point P moves in counter-clockwise direction on a circular path as shown in the figure. The movement
of P is such that it sweeps out a length s = t3 + 5, where s is in metres and t is in seconds. The radius
of the path is 20 m. The acceleration of P when t = 2 is nearly

1) 14 ms 2 2) 13 ms 2 3) 12 ms 2 4) 7.2 ms 2

65
Brilliant STUDY CENTRE

KEY WITH HINTS


LEVEL - I
     
1. 3 S  F1  F2 and D  F1  F2
 
As two vectors are perpendicular to each other, hence S.D  0 .
     2  2
or  F  F  . F  F   0 ;
1 2 1 2 or F   F 
1 2 0
     
or | F1 |2  | F2 |2  0 ; or | F1 |2  | F2 |2 ; or | F1 |  | F2 |
2. 2 Let the magnitudes of the forces be F1 and F2.
F1 3 2 2 o 2
Then F  5 and F1  F2  2F1F2 cos 60   42  or F12  F22  F1F2  1764
2

Solving these, we get F1 = 18 N, F2 = 30 N


2
3. 4 10 A   2A 
2
  
2A  2  2A  2A cos 

1
Square both sides cos      45o
2
   
4. 4 AB  AB
  2   2
Squaring both side A  B  A  B
     
A2  B2  2ABcos   A2  B2  2ABcos 
cos   0


2
5. 2 Position vector r   a cos t  ˆi   a sin t  ˆj

 dr d
 Velocity, v    a cos t  ˆi   a sin t  ˆj
dt dt

  a sin t  iˆ   a cos t  ˆj 

v.r   a 2 cos t sin t  a 2 sin t cos t  0


Here, v  r , i.e., velocity vector is perpendicular to position vector..

6. 2 x component: - v x  u x  a x t  2  t

at vx = 0, 2 – t = 0;  t  2 sec

y component : v y  u y  a y t  1  3  2  7 m / s

66
[Study Package - Physics - Class XI]

7. 2
N

W E

S
Assume +ve x-axis along east and +ve y-axis along north.
 
Vi  5iˆ ; Vf  5jˆ
 
 Vf  Vi 5jˆ  5iˆ  1  ˆ  1  ˆ
a     i   j
t 10  2  2

1
 Average acceleration has magnitude m / s 2 along N – W
2
8. 2
v

Initial Final

v
   
v f  vjˆ ; v i   vjˆ ;  Change in velocity = v f  vi  2vjˆ ; Change in speed = 0

1 1
9. 2 Velocity of rain  v r   4ms vertically downwards. Velocity of wind  v w   3ms from north to
south direction. A rain drop is acted upon by two velocities vr and vw as shown in figure. From
the traingle law, the resultant velocity of the rain drop is v = OW. In order to protect himself from
rain, he must hold his umbrella at an angle  with the vertical (towards north) given by

RW v w 3
tan    
OR v r 4

67
Brilliant STUDY CENTRE

10. 2 Here, velocity of water flowing in river, vr = 3 ms–1


Velocity of swimmer in still water, vs = 4 ms–1

From figure, the resultant velocity of the swimmer is,

2 2
v  vs2  v 2r   4    3   25  5 ms 1

  dr
11. 1 r   t 2  4t  6  ˆi   t 2  ˆj ; v   2t  4  iˆ   2t  ˆj
dt

 dv  
a
dt
 
 2iˆ  2ˆj m / s 2 ; v and a becomes perpendicular


 v.a  0 ;   2t  4  ˆi   2t  ˆj .  2iˆ  2ˆj  0

 2  2t  4   2  2t   0 ;  t  1
12. 3 From the figure the X-component remain unchanged, while the Y-component is reverse. Then,

the velocity at ponit B is 2iˆ  3jˆ ms 1 
13. 2 From the equation of motion, we knows v = u + at

 
Given, u  2iˆ  3jˆ and a  0.3iˆ  0.2ˆj  
v   2iˆ  3jˆ    0.3iˆ  0.2ˆj  10 ; v  5iˆ  5ˆj  v  u  at  ;
v  5 2 units

2u sin 
14. 4  4sec ;  u sin   20 ;
g

u 2 sin 2  400
 H   20 m
2g 2 10
15. 2 Projectile possess minimum kinetic energy at the highest point of trajectory. i.e., at the horizontal
distance R/2.

gx 2
16. 2 Given y  3 x 
2

gx 2
Comparing with the equation of trajectory, y  tan   x    tan   3
2u 2 cos2 

68
[Study Package - Physics - Class XI]

17. 3

2h 2  78.4
Time of flight    4s ; Horizontal range R = 150 × 4 = 600 m
g 9.8

dx dy
18. 1 x = 36t;  36   x ; y  48t  4.9t 2 ;  48  9.8t   y ;
dt dt
y 48 4 4
At t  0,  y  48; tan     ;  sin  
x 36 3 5

1
19. 2 K.E at the point of projection  mu 2  E ;
2
1 1 3
K.E at the topmost point  m(u cos ) 2  mu 2 cos 2 300  E
2 2 4
20. 4 The horizontal velocity of the projectile remains constant throughout the journey. Since the body
is projected horizontally, the initial velocity will be same as the horizontal velocity of any point.
dx
Since x = 2t, 2  Horizontal velocity = 2m/s;  Initial velocity = 2m/s
dt
21. 3 Here r = 1 km = 1000 m; u = 900 km h–1 = 250 ms–1
The centripetal acceleration of the air craft is given by

v 2 2502 a 62.5
a   62.5 ms2 ; g  9.8 ms 2    6.4
r 1000 g 9.8
22 1 1 2
22. 2 Here, r = 1 m;    s ; Acceleration, a  4 2  2 r  4 2 1 / 2   1  2 ms 2
44 2
It is directed along the radius towards the centre.
23. 4 In uniform circular motion the angular velocity remains a constant i.e., the direction is always
along the axis perpendicular to the plane of motion, and the magnitude remains constant. So
the angle will be zero.

v2
24. 3 Centripetal acceleration of a particle moving on a circular path is given by a c 
R
v2
o o
a c  a cos 30  15cos 30 m / s ;  2
 15cos 30o
R

3 3
 v 2  R  15  2.5  15   v  5.7 m / s
2 2
69
Brilliant STUDY CENTRE

LEVEL - II
 
  A.B
 
1. 2 A  4iˆ  4ˆj  4kˆ ; B  3iˆ  ˆj  4kˆ ; cos   | A | | B|
 
  
A.B  4iˆ  4ˆj  4kˆ . 3iˆ  ˆj  4kˆ  12  4  16  0
 
so, cos   0 ; i.e., angle between A and B is 90o

OB OC 1
2. 4 According to the question, OC  ;  
2 OB 2

 cos  
1 
   60o ;  angle between A 
and B is 150o.
2
3. 3 Let the force be A and B and A  B
A + B = 18; B = 18 – A
 
A  B  12; A 2  B2  2ABcos   12 ----- (1)

o Bsin 
Also tan 90  ; A  Bcos   0
A  Bcos 
A
cos    ---- (2)
B
Put eq. (2) in eq. (1)

B2  A 2  144
18  A 2  A 2  144
A = 5; B = 18 – A = 13
   
4. 1 Let  is the angle between A and B . Resultant of A and B -is P  A 2  B2  2ABcos  ---(1)
  
When B is reversed, the angle between A and B is 180o    .
 
Resultant of A and B is

Q  A 2  B2  2ABcos 180     A 2  B2  2ABcos  ------- (2)

 P 2  Q 2  2  A 2  B2 

70
[Study Package - Physics - Class XI]

5. 3  Displacement of the particle s  r2  r1  4ˆj  3kˆ   2iˆ  5jˆ   2iˆ  ˆj  3kˆ

Force on the particle, F  4iˆ  3jN


ˆ

 Work done W  F.s   4iˆ  3jˆ  .  2iˆ  ˆj  3kˆ   8  3  5 J

 o vm
6. 3 Velocity of man v m  10m / s ; sin 30 
v re

vm = velocity of man; vre = velocity of rain w.r.t earth; vrm = velocity of rain w.r.t man;
vm o v rm
v re   20m / s ; As, cos 30  v rm  v re cos 30  10 3ms1
sin 30 v re ;
7. 1

u 25m

u 2 sin 2 53o 25 5 2u 2 sin 53o cos 53o


H max   25m ; u m/sec ;  R   75m
2g 2 g

For a projectile the range is same for angle of projection  and  90   


o
8. 1

u 2 sin 2
Horizontal Range R 
g
Let t1 and t2 be the time of flights for the two paths.
2u sin 
t1 
g

2u sin  90 o    2u cos 
t2  
g g

 2u sin   2u cos   2u 2 2sin .cos  2  u 2 sin 2  2R


 t1.t 2       
 g  g  g2 g g  g

71
Brilliant STUDY CENTRE

2v1 2v 2 sin 
9. 3 Time of flight T1 = T2; T1  T2  ;  v1  v 2 sin 
g g

v12
Max. height produced H1 
2g

2
 v sin  H1 v12 / 2g v12 H1
H2  2 ;   ; But v1  v 2 sin  ;  1
2g H 2 v 22 sin 2  / 2g v 22 sin 2  H2
10. 3
500
v m/sec
m
path followed
1960 m by bomb
A
R

2h 2 1960
Time of flight    20 sec
g 9.8

500 10000
Range = vx × Time of flight =   20  m;  AB = 3.33 km
3 3
11. 2 If two bodies reach the highest point at the same time they will collide
2
 v sin 30  v 22
t1  1  For body projected at  ; t 2   For body projected vertically upwards
2g 2g

 For collision, t1  t 2 ;
 v sin 30 
 1 
v 22
; 
v2
 sin 30  0.5
2g 2g v1

gx 2
12. 1 The equation of trajectory is y  x tan   2
2u cos2 
where  is the angle of projection and u is the velocity with which the projectile is projected.
For equal trajectories for same angles of projection,

g
= constant
u2

9.8 g
As per question, 
5 2 32
where g' is acceleration due to gravity on the planet.

9.8  9
g   3.5 ms 2
25

72
[Study Package - Physics - Class XI]

1 1
13. 2 h  u cos  t1  gt12 (1); h  u cos  t 2  gt 22 (2)
2 2
Equating (1) and (2) and substituting the value of t 1 and t2 we get, u cos   19.6ms 1
u 2cos 2
Maximum height =  19.6m
2g

u 2 sin 2
14. 4 R at an angle  and  90    ;
g
u 2 sin 2 
Now h1 
2g
u 2 sin 2  90    u 2 cos2 
h2   ;
2g 2g

u2 u2
h1h 2    sin  cos   sin  cos  
2g 2g
2
u 2 u 2  2sin  cos  
 
2g 2g 4
2
 u 2 sin 2  1 R 2
h1 h 2    .  ;  R  4 h1 h 2
 g  16 16
u 2 sin 2θ
15. 1 Here u = 20 m/s, θ  45 , d = 25 m;o Horizontal range : R 
g
2
 20  sin 2  45o 400 400
R   sin 90o   40.82 m
9.8 9.8 9.8
2u sin θ 2  20sin 45 2  20 1
Time of flight, T    or T = 2.886 s
g 9.8 9.8  2
Since the goal man is already 25 m away in the direction of the ball. So to catch the ball, he is to
cover a distance = 40.82 – 25 = 15.82 m, in time 2.886 sec.
 the velocity of goal man to catch the ball is
15.82
v  5.483 m/s
2.886

3 vy 30sin 0
16. 4 tan o  ; 
4 vx 30cos 0

gt 3 10 3 5 10 3 5 1
We know that tan   tan o        
u cos o 4 30  4 4 30  4 4 12 3
5

73
Brilliant STUDY CENTRE

17. 1 As horizontal range of the two stones is same. So the sum of angles of projection of two
stones must be 900,
2
1 2
2 2 u  
According to question , y  u sin 30   2
2g 2g
2
 3 2
u  
1 u 2 sin 2 60  2  y1
And y   ;   3 or y1  3y
2g 2g y
2
u 2 sin 2  30  sin 60
18. 3 R1    45 3
g 10

2 sin120 3
R 2  10 3   300   15 3
10 2  10

 Smax  R1  R 2  30 3

19. 2

u 2 sin 2 
Height of porjectile H 
2g

u 2 sin 2 45o u 2
H 
2g 4g

u 2 sin 2 u 2 sin 0o u 2


Range of projectile R   
g g g

R u2
 
2 2g

H u 2 / 4g
 tan    2
R / 2 u / 2g

1 1
tan   ;   tan 1  
2 2

74
[Study Package - Physics - Class XI]

dv V2 a 2t 2
20. 4 at   a ---- (1); ac   ------- (2)
dt R R

1 2R 1 2
S  ut  at 2 ;  at
2 8 2

R a 2  R  a
t  ------ (3); Put eqn. (3) in (2); a c   
2a R  2a  2

2 2 a
Total acceleration a  a t  a c  4  2
2

ds d 3
21. 1 s = t3 + 5;  v    t  5  3t 2
dt dt
dv d
Tangential acceleration, a t    3t 2   6t
dt dt
At t = 2 s, v = 3(2)2 = 12 m/s, at = 6(2) = 12 m/s2
2
v 2 12  144
Centripetal acceleration, a c     7.2 m/s 2
R 20 20
2 2 2 2
Net acceleration, a   a c   a t    7.2  12  14 m/s2 \

75
Brilliant STUDY CENTRE

CHAPTER - 04
LAWS OF MOTION

INTRODUCTION
Sir Isaac Newton (1642-1727) presented these principles to the world in his book Philosophiae Naturalis
Principia Mathematica (The Mathematical Principles of Natural Philosophy), which was published in 5 July
1687 and is often referred to as the Principia, procedures for finding the motion of an object.
LINEAR MOMENTUM
Linear momentum of a body is the quantity of motion contained in the body. It is measured as the
 
product of the mass of the body and its velocity i.e., Momentum = mass  velocity, p  m  v
Momentum is a vector quantity, the direction is same as that of velocity.

 
The dimensional formula of momentum is M1L1T 1 , unit : kg-m/sec [S.I.], g-cm/sec [C.G.S.]
If two objects of different masses have same momentum, the lighter body possesses greater velocity.
v1 m 2 1
p  m1 v1  m 2 v 2 = constant  v  m --------(1); i.e. v 
2 1 m
Momentum of a body is measured by the force required to stop the body in unit time.
NEWTON’S FIRST LAW
According to Newton’s first law of motion, a body continues to be in a state of rest or of uniform motion
along a straight line, unless it is acted upon by an external force to change the state. This means force
applied on a body alone, can change its state of rest or state of uniform motion along a straight line. Hence
we define force as an external effort in the form of a push or pull which moves or tries to move a body at
rest ; stops or tries to stop a body in motion ; changes or tries to change the direction of motion of a body.
Newton’s first law defines inertia and is rightly called the law of inertia. Inertia are of three types : Inertia
of rest, Inertia of motion, Inertia of direction
1. Inertia of rest : It is the inability of a body to change by itself, its state of rest. This means a body at rest
remains at rest and cannot start moving by its own.
Example : A person who is standing freely in a bus is thrown backward, when the bus starts suddenly.
2. Inertia of motion : It is the inability of a body to change itself its state of uniform motion i.e., a body in
uniform motion can neither accelerate nor retard by its own.
Example : (i) When a bus or train stops suddenly, a passenger sitting inside tends to fall forward. This
is because the lower part of his body comes to rest with the bus or train but the upper
part tends to continue its motion due to inertia of motion.
(ii) A person jumping out of a moving train may fall forward.

76
[Study Package - Physics - Class XI]

3. Inertia of direction : It is the inability of a body to change by itself direction of motion.


Example :(i) When a stone tied to one end of a string is whirled and the string breaks suddenly, the
stone flies off along the tangent to the circle.
(ii) When a car goes round a curve suddenly, the person sitting inside is thrown outwards.
NEWTON’S SECOND LAW
According to this law, the rate of change of linear momentum of a body is directly proportional to the
external force applied on the body, and this change takes place always in the direction of the force applied.
EXPLANATION OF NEWTON’S SECOND LAW
According to Newton’s second law of motion, the rate of change of linear momentum of a body is directly
proportional to the external force applied on the body, and this change takes place always in the direction of the
applied force.
  
m = mass, v  velocity;  The linear momentum of the body p  mv -------(2)
 
Let F is external force applied on the body, and dp - change in linear momentum in time dt.
t.
     
dp dp dp
 F or F i.e., F  k -------(3); where k is a constant of proportionality..
dt dt dt

 
d  dv   dv
F  k  mv   km ; i.e., F  kma ------(4); where a  is acceleration.
dt dt dt
The unit of force is so chosen that the value of k =1.
 
 F  ma -------(5)
If the acceleration produced is in three dimensions having components ax, ay, az along X-axis, Y-axis,
Z-axis respectively, then
  
a  îa x  ĵa y  k̂a z -----(6);  
 F  m î a x  ĵa y  k̂a z -----(7); F  îFx  ˆjFy  k̂Fz -----(8)

Fx  ma x , Fy  ma y and Fz  ma z ------(9)
As acceleration is a vector quantity and mass is a scalar, therefore force F = ma, is a vector and

direction is same as that of a .
Writing eqn. (4) in scalar form F = ma ------ (10).
Thus force is product of mass and acceleration of the body.
Hence second law of motion gives a measure of force.
Dimension : Force = mass  acceleration; [F] = [M1][L1T-2] = [M1L1T–2]
Units : Absolute units : (i) Newton (S.I.) (ii) Dyne (C.G.S)
Newton : One newton is that force which produces an acceleration of 1ms–2 on a body of mass
1 Kilogram. 1 Newton = 1 kg  1 ms-2=1kgm/s2
Dyne : One dyne is that force which produces an acceleration of 1cm/s2 in a body of mass 1 gram.
1 dyne = 1 g  1 cm s-2 = 1 g cm s–2.
Relation between Newton and dyne, 1 N = 1 kg  1 ms–2 = 103 g  102 cm s–2 = 105 dynes

77
Brilliant STUDY CENTRE

Gravitational units
The gravitational unit of force in S.I. is 1 kilogram weight (kg wt) or 1 kilogram force (kg f). It is that
much force which produces an acceleration of 9.81 ms–2 on a body of mass 1kg., kg wt = 9.81 N.
The gravitational unit of force in C.G.S. system is 1 gram weight (g wt) or one gram force (1 gf). It is that
much force which produces an acceleration of 981 cm s–2 on a body of mass 1 gram. 1 gm-wt = 981 dyne.
Relation between gravitational units of force : 1 kg-f =107gm-f
Note: (1) F = ma formula is valid only if force is changing the state of rest or motion and the mass of the
body is constant and finite.
 d 
 dv  dm
(2) If m is not constant F   
mv  m v ------------(11)
dt dt dt
(3) If force and acceleration have three component along x, y and z axis, then
 
F  Fx ˆi  Fy ˆj  Fz kˆ and a  a x ˆi  a y ˆj  a z kˆ

From above it is clear that Fx  ma x , Fy  ma y and Fz  ma z

(4) No force is required to move a body uniformly along a straight line.


  
F  ma F  0 (As a = 0 )
(5) When force is written without direction then positive force means repulsive while negative force
means attractive.
Example : Positive force - Force between two similar charges. Negative force - Force between
two opposite charges
(6) Out of so many natural forces, for distance 10-15 metre, nuclear force is strongest while gravitational
force is weakest. FNuclear  FElectromagnetic  FGravitational

FE
(7) Ratio of electric force and gravitational force between two electrons  10 43  F  F
FG E G

(8) Constant force : If the direction and magnitude of a force is constant, then it is said to be a
constant force.
(9) Variable or dependent force :
(i) Time dependent force : In case of impulse or motion of a charged particle in an alternating
electric field force is time dependent.

Gm1m 2
(ii) Position dependent force : Gravitational force between two bodies FG 
r2

1 q1q 2
or Force between two charged particles FE 
40 r 2

(iii) Velocity dependent force : Viscous force  6rv  .

Force on charged particle in a magnetic field  qvBsin  

78
[Study Package - Physics - Class XI]

(10) Central force : If a position dependent force is always directed towards or away from a fixed point
it is said to be central otherwise non-central. Example : Motion of earth around the sun, motion
of electron in an atom.
(11) Conservative and non conservative forces : If under the action of a force the work done in a
round trip is zero or the work is path independent, the force is said to be conservative otherwise
non conservative.
Example : Conservative force : Gravitational force, electric force, elastic force.
Non conservative force : Frictional force, viscous force.
Equilibrium of Concurrent Forces
(1) The necessary condition for the equilibrium of a body under the action of concurrent forces is that the
vector sum of all the forces acting on the body must be zero.

(2) Mathematically for equilibrium F net 0


(3) Three concurrent forces will be in equilibrium, if they can be represented completely by three sides of
a triangle taken in order.

(4) Lami’s Theorem : For equilibrium of concurrent forces

  
F1 F2 F3
  --------(12)
sin  sin  sin 

NEWTON’S THIRD LAW


To every action, there is always an equal (in magnitude) and opposite (in direction) reaction.
(1) When a body exerts a force on any other body, the second body also exerts an equal and opposite
force on the first.
(2) Action and reaction never act on the same body.
 
(3) If FAB = force exerted on body A by body B (Action) and FBA = force exerted on body B by body A
 
(Reaction). Then according to Newton’s third law of motion FAB  FBA

(4) Example : A book lying on a table exerts a force on the table which is equal to the weight of the book.
This is the force of action. The table supports the book, by exerting an equal force on the book. This is
the force of reaction. As the system is at rest, net force on it is zero. Therefore force of action and
reaction must be equal and opposite.

79
Brilliant STUDY CENTRE

PRINCIPLE OF CONSERVATION OF LINEAR MOMENTUM


According to this principle, in an isolated system, the vector sum of the linear momenta of all the
bodies of the system is conserved and is not affected due to their mutual action and reaction.
Consider an isolated system consisting of n bodies of masses m1, m2 .......... mn moving with velocities,
   
v1 , v 2 ,.........v n respectively. The vector sum of linear momenta i.e., the total linear momentum P of all the
   
bodies in the system is given by p  m1 v1  m 2 v 2  ..........  m n v n

Let M be the total mass of the system and Vcm be the velocity of the centre of mass of the system.
 
p  MVcm ------------(13)
 
dp d  dVcm 
Differentiating (13) with respect to time, we get, 
dt dt

MVcm  M
dt
  Ma cm ------------(14)


where a cm is the acceleration of centre of mass of the system.
 
According to Newton’s second law of motion Ma cm is equal to the external force F .

dp 
 From (11),  F -----------(15)
dt

In the case of an isolated system, no external force is acting on the system. i.e., F  0.

dp 
From (15), 0 Hence p  a constant ,
dt
  
m1v1  m2 v 2  .......mn v n  constant-----------(16)
which proves the principle of conservation of momentum
Practical applications of the law of conservation of linear momentum
(i) When a man jumps out of a boat to the shore, the boat slightly moves away from the shore.
(ii) An astronaut in open space, who wants to return to the spaceship, throws some objects in a direction
opposite to the direction of motion of the spaceship. By doing so, he gains a momentum equal and
opposite to that of the thrown object and so he moves towards the spaceship.
(iii) Recoiling of a gun : For bullet and gun system, the force exerted by trigger will be internal so the
momentum of the system remains unaffected.
 
Let mG = mass of the gun, m B = mass of the bullet, v G = velocity of the gun, v B = velocity of
the bullet
 
Initial momentum of the system = 0; Final momentum of the system = mG v G  m B v B
By the law of conservation of linear momentum

   mB 
mG vG  m B v B  0 ; so recoil ; v G   m v B --------(17)
G

80
[Study Package - Physics - Class XI]


a) Here negative sign indicates that the velocity of recoil v G is opposite to the velocity of the bullet

 1
b) vG 
m G i. e. higher the mass of gun, lesser the velocity of recoil of gun.

(iv) Rocket propulsion : The initial momentum of the rocket on its launching pad is zero. When it is fired
from the launching pad, the exhaust gases rush downward at a high speed and to conserve momen-
tum, the rocket moves upwards.
Let m0 = initial mass of the rocket; m = mass of the rocket at any instant ‘t’ (instantaneous mass)
mr = residual mass of the empty container of the rocket;

u = velocity of exhaust gases,

v = velocity of rocket at any instant ‘t’ (instantaneous velocity)

dm
= rate of change of mass of the rocket = rate of fuel consumption = rate of ejection of the fuel
dt

 dm
a) Thrust on the rocket : F   u  mg
dt

Here negative sign indicates that the direction of thrust is opposite to the direction of escaping gases.

 dm
F  u (if effect of gravity is neglected)
dt

u dm
b) Acceleration of the rocket: a = g
m dt

u dm
and if effect of gravity is neglected a = ----------(18)
m dt

 m0 
c) Instantaneous velocity of the rocket : v = u log e    gt
 m 

81
Brilliant STUDY CENTRE

 m0   m0 
and if effect of gravity is neglected v  u log e    2.303u log10   ---------(19)
 m   m 
The speed attained by the rocket when the complete fuel gets burnt is called burnt out speed of the
rocket. It is the maximum speed acquired by the rocket.
APPARENT WEIGHT OF A MAN IN A LIFT/ELEVATOR
Suppose a person of mass m is standing on a weighing machine placed in an elevator/lift. The actual
weight of the person = mg. This acts on the weighing machine which offers a reaction R given by the
reading of the weighing machine. This reaction exerted by the surface of contact on the person is the
apparent weight of the person.
(i) When the elevator is at rest

Acceleration of the person = 0; Net force on the person f = 0

R  mg  0 ; R = mg-----------------(20)
i.e. apparent weight is equal to the actual weight of the person.
(ii) When the elevator is moving uniformly in the upward or downward direction
In uniform motion, acceleration of the person = 0. Proceeding as in case (i) R = mg i.e., apparent
weight = actual weight of the person.
(iii) When the elevator is accelerating upwards
Suppose the uniform upward acceleration of the person in the lift = a

 Net upward force on the person f = ma, f  R1  mg

R1  mg  f  mg  ma  m  g  a  ------------(21) Thus R 1  mg

Hence apparent weight of the person becomes more than the actual weight when the elevator is
accelerating upwards.
(iv) When the elevator is accelerating downwards
Suppose the uniform downward acceleration of the person in the lift = a

 Net downward force on the person f = ma; f  mg  R 2

R 2  mg  f  mg  ma  m  g  a  ------------(22) Thus R 2  mg
Hence apparent weight of the person becomes less than the actual weight when the elevator is
accelerating downwards.

82
[Study Package - Physics - Class XI]

In free fall of a body under gravity, a = g  From (22), R 2  mg  g   0


i.e. apparent weight of the body becomes zero or the body becomes weightless.
Frame of Reference
(1) A frame in which an observer is situated and makes his observations is known as his ‘Frame of
reference’ and is associated with a coordinate system and a clock.
(2) Frames of reference are of two types : (i) Inertial frame of reference (ii) Non-inertial frame of refer-
ence.
(i) Inertial frame of reference :
(a) A frame of reference which is at rest or which is moving with a uniform velocity along a straight line is
called an inertial frame of reference.
(b) In an inertial frame of reference Newton’s laws of motion holds good.
Example : The lift at rest, lift moving (up or down) with constant velocity, car moving with constant
velocity on a straight road.
(ii) Non inertial frame of reference :
(a) Accelerated frames of reference are non-inertial frames of reference.
(b) Newton’s laws of motion are not applicable in non-inertial frame of reference.
Example : Car moving in uniform circular motion, lift which is moving upward or downward with some
acceleration, plane which is taking off.
Impulse.
When a large force acts on a body for a very small time interval, it is called impulsive force.

 t2 
Impulse of a force is a measure of total effect of the force. I   Fdt
t1

Impulse is a vector quantity and its direction is same as that of force.


The dimensional formula of impulse is M1L1T 1 
Units : Newton-second or kg.m.s-1 (S.I.) and Dyne-second or g.cm.s-1 (C.G.S.)
Force-time graph :
Impulse is equal to the area under F-t curve.
If we plot a graph between force and time, the area under the curve and time axis gives the value of
impulse.

1 1
I = Area between the curve and time axis =  Base  Height  Ft
2 2

 dp t2  p2     
From Newton’s second law F  or  Fdt   dp  I  p2  p1  p
t1 p1
dt

83
Brilliant STUDY CENTRE

 
i.e. The impulse of a force is equal to the change in momentum.  F t   p 
This statement is known as Impulse momentum theorem.
Examples : Hitting, kicking, catching, jumping, diving, collision etc.

In all these cases an impulse acts. I  Fdt  Fav .t  p  constant ----------(23)

(i) In hitting or kicking a ball we decrease the time of contact so that large force acts on the ball
producing greater acceleration.
(ii) In catching a ball a player by drawing his hands backwards increases the time of contact and
so, lesser force acts on his hands and his hands are saved from getting hurt.
Motion of bodies connected by strings
Acceleration =a

R3 R1
R2

T1 T2 T3
F

m2g m1 g
m3g

F
a 
 m1  m2  m3  ----------(24)
m3 F
Tension T1  m 3a  ----------(25)
 m1  m2  m3 
 m 2  m3  F
Tension T2   m 2  m 3  a  ----------(26)
 m1  m 2  m3 
Masses suspended over a pulley in a vertical plane:
Consider two masses m1 and m2 connected at the two ends of a massless inextensible string passing over
a frictionless pulley. Let the pulley is massless.

T1
a T2
m2 m1
m2g a

m1g

84
[Study Package - Physics - Class XI]

Equations of motion are : m1g  T1  m1a T2  m 2 g  m 2 a ---------(27)

 m1  m2  g 2m1m 2 g
If the mass of the pulley is zero, T1 = T2.; a  --------(28); T1  T2 
m1  m 2 m1  m 2 --------(29)

1 24m m g
Reaction at pulley R  T1  T2 ; R  m  m --------(30)
 1 2
If the two masses are equal, put m1  m 2  m ; Then a = 0; T1  T2  mg

Reaction at pulley R  T1  T2  2mg


FRICTION
Friction is an opposing force that comes into play when one body actually moves (slides or rolls) or
even tries to move over the surface of another body. This resistance is represented by a single force called
friction. The force of friction is parallel to the surface and opposite to the direction of intended motion.
TYPES OF FRICTION
(1) Static friction : Let us consider a block of weight mg lying on a horizontal surface as shown in Fig.R is
the normal reaction between the block and the horizontal surface. It balances mg, i.e., R  mg ----(1)
Suppose a small force P is applied on the block to the right as shown. The force of friction F opposes
the motion. F = P -------(2)

This means as we increase P, friction F also increases, remaining equal to P always.


The opposing force that comes into play when one body tends to move over the surface of another, but
the actual motion has yet not started is called Static Friction.
Static friction is a self-adjusting force because it changes itself in accordance with the applied force.
(2) Limiting friction : As we increase the applied force, a stage comes when the body is just at the verge
of moving over the other. The static friction at this stage is obviously maximum. This maximum value of
static friction is called Limiting Friction.
Hence Limiting Friction is the maximum opposing force that comes into play, when one body is just at
the verge of moving over the surface of another body..
The magnitude of limiting friction between any two bodies in contact is directly proportional to the nor-
mal reaction between them. F  R or F   R -------(3)
Direction of the force of limiting friction is always opposite to the direction in which one body is at the
verge of moving over the other
Coefficient of static friction :  s is called coefficient of static friction and defined as the ratio of force
F
of limiting friction to the normal reaction μ s  -------(4)
R

85
Brilliant STUDY CENTRE

Dimensions of coefficient of static friction : [MoLoTo]. It has no unit. Value of coefficient of static friction
lies in between 0 and 1. Its value depends on material and nature of surfaces in contact.The value of  s
does not depend upon apparent area of contact.
(3) Kinetic or dynamic friction: If the applied force is increased further and sets the body in motion, the
friction opposing the motion is called kinetic friction. Kinetic friction depends upon the normal reaction.
Fk  R or Fk  μ k R where μ k is called the coefficient of kinetic friction.
The value of μ k depends upon the nature of the surfaces in contact. Kinetic friction is always lesser
than limiting friction Fk < Fl  k   s
Types of kinetic friction
a) Sliding friction: The opposing force that comes into play when one body is actually sliding over the
surface of the other body is called sliding friction. e.g. A flat block is moving over a horizontal table.
b) Rolling friction: When objects such as a wheel (disc or ring), sphere or a cylinder rolls over a
surface, the force of friction comes into play is called rolling friction. Rolling friction is directly propor-
tional to the normal reaction (R) and inversely proportional to the radius (r) of the rolling cylinder or
R
wheel. Frolling = μ r -------(5)
r

r is called coefficient of rolling friction. It would have the dimensions of length and measured in metre.
Rolling friction is often quite small as compared to the sliding friction.
Advantages and Disadvantages of Friction
(1) Advantages of friction
(i) Walking is possible due to friction.
(ii) Two bodies stick together due to friction.
(iii) Brake works on the basis of friction.
(iv) Writing is not possible without friction.
(v) The transfer of motion from one part of a machine to other part through belts is possible by friction.
(2) Disadvantages of friction
(i) Friction always opposes the relative motion between any two bodies in contact. Therefore extra
energy has to be spent in overcoming friction. This reduces the efficiency of machine.
(ii) Friction causes wear and tear of the parts of the machinery in contact. Thus their lifetime reduces.
(iii) Frictional force result in the production of heat, which causes damage to the machinery.
Angle of Friction.
Angle of friction may be defined as the angle which the resultant of limiting friction and normal reaction
makes with the normal reaction.
By definition angle  is called the angle of friction

F F
tan θ  -------(6); tan θ  μ [As we know μ]
R R

Hence coefficient of limiting friction is equal to tangent of the angle of friction.

86
[Study Package - Physics - Class XI]

Angle of Repose
Angle of repose is defined as the angle of the inclined plane with the horizontal such that a body placed
on it just begins to slide. By definition  is called the angle of repose
In limiting condition, F  mg sin  and R  mg cos 

F
So  tan α
R

F F
  μ  tan θ  tan α -------(7) [As we know  μ  tan θ ]
R R
Thus the coefficient of limiting friction is equal to the tangent of angle of repose. As well as    i.e.
angle of repose = angle of friction.
Maximum Length of Hung Chain
A uniform chain of length l is placed on the table in such a manner that its l  part is hanging over the
edge of the table without sliding. Since the chain have uniform linear density therefore the ratio of mass
or ratio of length for any part of the chain will be equal

m 2 mass hanging from the table


We know μ  
m1 mass lying on the table

 For this expression we can rewrite the above expression in the following manner
length hanging from the table
μ [ As chain have uniform linear density]
length lying on the table

l l
 -------(8) by solving l 
l  l 1    -------(9)
Stopping of block due to friction
1. On horizontal surface
(i) Distance travelled before coming to rest: A block of mass m is moving initially with velocity u on
a rough surface and due to friction it comes to rest after covering a distance S.

Retarding force F  ma   R

87
Brilliant STUDY CENTRE

 ma   mg
ag

From v 2  u 2  2aS  0  u 2  2  gS [As v = 0, a   g ]

u2 P2
S -------(10) or S= -------(11) [momentum P = mu]
2μg 2μm 2g
(ii) Time taken to come to rest
From equation v  u  at  0  u   gt [As v = 0, a   g ]

u
t -------(12)
μg
(iii) Force of friction acting on the body
We know, F  ma

Fm
v  u F m
u
So, ; [As v = 0]
t t

 u
F   mg -------(13)  As t  μg 
 
Dynamics of Circular Motion
In circular motion or motion along any curved path, Newton’s law is applied in two perpendicular directions
one along the tangent and the other perpendicular to it i. e. towards centre. The component of net force
along the centre is called centripetal force. The component of net force along the tangent is called tangential
force.

dv mv2
Tangential force  Ft   ma t  m
2
 mr ; Centripetal force  FC   m r 
dt r
Motion in a Vertical Circle
Consider the motion of a small body (say stone) tied to a string and whirled in a vertical circle. If at any
time the body is at angular position q, as shown in the figure, the forces acting on it are the tension T in the
string along the radius towards the centre and the weight of the body mg acting vertically downwards.

mv2 mv2
Applying Newton’s law towards centre, T  mg cos   or T=  mg cos 
r r

88
[Study Package - Physics - Class XI]

The body will move on the circular path only if Tmin > 0(as if Tmin  0) the string will slack and the body
will fall down instead of moving on the circle.) So for completing the circle, i.e. looping the loop, the minimum
mv2H
tension in the string T = 0    180o  . r
 mg  0 ie, v H  gr ---------- (14)

Now applying conservation of mechanical energy between highest point H and lowest point L.

1 1 1 1
mg  2r  mv 2H  mv 2L ; 2mgr  mrg  mv 2L ; 5mgr  mv 2L ; v L  5rg
2 2 2 2

i.e, for looping the loop, the minimum velocity at lowest point must be v L  5gr ---------- (15)

In case of motion in a vertical plane, the tension is maximum at lowest position and in the above case
of looping the loop Tmax = 6 mg --------- (16)
Motor Car moving over a convex bridge
The motion of a car over a convex bridge AB is the motion along the tangent of a circle AB. The centripetal
force is provided by the difference of weight mg of the car and the normal reaction R of the bridge.

mv 2 mv 2
mg  R  or R  mg  -------- (17)
r r
Clearly, R  mg ,i,e,. the weight of the moving car is less than the stationary car..

If R  mg , the car will move along the track and if R  mg , the car will be detached from the bridge
Circular Turning on Roads
When vehicles go through turnings, they travel along a nearly circular arc. There must be some force
which will produce the required centripetal acceleration. If the vehicles travel in a horizontal circular path,
this resultant force is also horizontal. The necessary centripetal force is being provided to the vehicles by
following three ways;
 By friction only  By banking of roads only  By both friction and banking of roads.
By Friction Only
Suppose a car of mass m is moving at a speed v in a horizontal circular arc of radius r. In this case, the
necessary centripetal force to the car will be provided by the force of friction f acting towards the centre.

mv2
Thus, f 
r

Further, limiting value of f is N or f L  N  mg  (N = mg)

89
Brilliant STUDY CENTRE

mv 2 mv2
Therefore, for a safe turn without sliding,  f L ----- (18) i.e.,  mg or v  rg ----- (19)
r r
Here, two situations may arise. If m and r are known to us, the speed of the vehicle should not exceed
v2
rg and if v and r known to us, the coefficient of friction should be greater than .
rg
Bending of a cyclist
A cyclist provides himself the necessary centripetal force by leaning inward on a horizontal track,
while going round a curve. Consider a cyclist of weight mg taking a turn of radius r with velocity v. In
order to provide the necessary centripetal force, the cyclist leans through angle  inwards as shown
in figure.
The cyclist is under the action of the following forces:
The weight mg acting vertically downward at the centre of gravity of cycle and the cyclist.
The reaction R of the ground on cyclist. It will act along a line-making angle  with the vertical.
The vertical component R cos  of the normal reaction R will balance the weight of the cyclist, while
the horizontal component R sin  will provide the necessary centripetal force to the cyclist.

mv 2
R sin   ------(i)
r
and R cos   mg ------(ii)

R sin  mv 2 / r v2
Dividing equation (i) by (ii), we have  or tan   -------- (iii)
R cos  mg rg

2
1  v 
Therefore, the cyclist should bend through an angle   tan  
 rg 
It follows that the angle through which cyclist should bend will be greater, if
(1) The radius of the curve is small i.e., the curve is sharper.
(2) The velocity of the cyclist is large.
Note: For the same reasons, an ice skater or an aeroplane has to bend inwards, while taking a turn.

90
[Study Package - Physics - Class XI]

By Banking of Roads Only


Friction is not always reliable at circular turns if high speeds and sharp turns are involved. To avoid
dependence on friction, the roads are banked at the turn so that the outer part of the road is some what lifted
compared to the inner part.

Applying Newton’s second law along the radius and in the vertical direction

mv 2 v2
Nsin  = and N cos   mg , then tan   or v  rg tan  -------- (20)
r rg
Centrifugal Force
When a body is rotating in a circular path and when the centripetal force vanishes, the body would
leave the circular path. To an observer A who is not sharing the motion along the circular path, the body
appears to fly off tangentially at the point of release. To another observer B, who is sharing the motion along
the circular path (i.e., the observer B is also rotating with the body which is released), it appears to B, as if
it has been thrown off along the radius away from the centre by some force. This inertial force is called
centrifugal force.

mv 2
Its magnitude is equal to that of the centripetal force =
r
Centrifugal force is a fictitious force which has to be applied as a concept only in a rotating frame of
reference to apply Newton’s law in that frame.

FBD of the ball w.r.t non inertial frame rotating with the ball.
Suppose we are working from a frame of reference that is rotating at a constant angular velocity w with
respect to an inertial frame. If we analyse the dynamics of a particle of mass m kept at a distance r from the
axis of rotation, we have to assume that a force mrw2 react radially outward on the particle. Only then we
can apply Newton’s laws of motion in the rotating frame. This radially outward pseudo force is called the
centrifugal force.

91
Brilliant STUDY CENTRE

QUESTIONS
LEVEL - I
1. A player stops a football weighing 0.5 kg, which comes flying towards him with a velocity of 10 m/s. If
the impact lasts for 1/50th sec. and the ball bounces back with a velocity of 15 m/s, then the average
force invovled is
1) 250 N 2) 1250 N 3) 500 N 4) 625 N
2. Two skaters A and B of weights 40 kg wt and 60 kg wt respectively stand facing each other 5 m apart.
They then pull a light rope stretched between them. Where they meet is
1) 2.5 m from A 2) 2 m from A 3) 3 m from A 4) 1.5 m from A
3. A player caught a cricket ball of mass 150 gram, which was moving with a velocity of 20 m/s. If the
catching process is completed in 0.1 seconds, the blow exerted by the ball on the hands of the player
is
1) 100 N 2) 30 N 3) 20 N 4) 15 N
4. The average force necessary to stop a bullet of mass 20 g moving with a speed of 250 m/s as it
penetrates into wood for a distance of 12 cm is
1) 2.2 × 103 N 2) 3.2 × 103 N
3) 4.2 × 103 N 4) 5.2 × 103 N
5. A dynamometer D is attached to two bodies of masses M = 6 kg and m = 4 kg. Forces F = 20 N and
f = 10 N are applied to the masses as shown in figure. The dynamometer reads:

D
M M
f = 20N f = 10N
1) 10 N 2) 20 N 3) 6 N 4) 14 N
6. The weight of a man in a lift moving upwards is 608 N, while the weight of the same man in the lift
moving downwards with same acceleration is 368 N. The normal weight of the person is
1) 488 N 2) 588 N 3) 480 N 4) 240 N

7. The coefficient of static friction, s , between the block A of mass 2 kg and the table as shown in figure
is 0.1. What would be the maximum mass value of block B so that the two blocks do not move? The
string and the pulley are assumed to be smooth and massless (g = 10 m/s2)

2kg A

1) 0.2 kg 2) 0.4 kg 3) 2.0 kg 4) 4.0 kg


8. A light string passing over a smooth light pulley connects two blocks of masses m1 and m2 (vertically).
g
If the acceleration of the system is , then the ratio of masses is
8
1) 8 : 1 2) 9 : 7 3) 4 : 3 4) 5 : 3

92
[Study Package - Physics - Class XI]

9. Three forces acting on a body are shown in figure. To have the resultant force only along the y-
direction, the magnitude of the minimum additional force needed along X-axis is
y

4N
o 1N
30
60o x

2N

3
1) 0.5 N 2) 1.5 N 3) N 4) 3N
4
10. A block of mass m1, of 3 kg, is lying on a frictionless plane inclined at 30o with the horizontal. It is
connected to a mass m2, of 4 kg, with the help of a string passing over a pulley as shown in figure.
The acceleration of each block will be

a
T
T m2
m1 a
o
m2g
30

1) 2.55 ms–2 2) 3.55 ms–2 3) 4.55 ms–2 4) 5.55 ms–2

11.  
A block of mass 4 kg is kept at rest in a big box moving with velocity 2iˆ m/s and having acceleration

 2iˆ  3jˆ  m / s .Find pseudo force acting on the block w.r.t. box.
2

 
1) 8iˆ N  
2) 8iˆ  12jˆ N 
3) 8iˆ  12ˆj N   
4) 8iˆ  12jˆ N


12. Velocity of particle of mass 2 kg varies with time t according to the equation v  2tiˆ  6ˆj m/s. Here  
t is in seconds. The impulse imparted to the particle in time interval from t = 0 to t = 2 s is:

 
1) 2iˆ kg m / s  
2) 4iˆ kg m / s  
3) 8iˆ kg m / s  
4) 16iˆ kg m / s

13. Find frictional force and acceleration of block if F is equal to 60 N

1) 20 N and 1 m/s2 2) 20 N and 2 m/s2


3) 40 N and 2 m/s2 4) 40 N and 1 m/s2

93
Brilliant STUDY CENTRE

14. A force of 52 N acts on a particle placed at point (3, 4) towards a point (–2, –8). If mass of particle is
2kg, then its acceleration vector is:

1) 20iˆ  48ˆj 2) 10iˆ  24ˆj 3) 5iˆ  12ˆj 4) 48iˆ  20ˆj

15. A bridge is in the form of a semi-circle of radius 40 m. The greatest speed with which a motor cycle can
cross the bridge without leaving the ground at the highest point is (g = 10 ms–2) (frictional force is
negligibly small)

1) 40 ms 1 2) 20 ms 1 3) 30 ms 1 4) 15 ms 1
16. In the system shown, the mass 30 kg is pulled by a force 210 N. At the instant when the 15 kg mass
has acceleration 6 m/s2, the acceleration of 30 kg mass will be: (Assume the spring to be massless)

1) 2 m/s2 2) 3 m/s2
3) 3.4 m/s2 4) 4 m/s2

3
17. A person wishes to slide down a rope whose breaking load is of the weight of the person. Minimum
5
acceleration by which the person should slide down without breaking the rope is:
1) 0.8 g 2) 1.2 g 3) 0.6 g 4) 0.4 g
18. Consider a system of masses M1 and M2 (M1 > M2) connected by a massless string. In figure (A), the
system is pulled by a force F from the side of mass M1 and in (B), the system is pulled by the same
force F from the side of mass M2, then:

1) acceleration in the two cases is same and tension in the string connecting M1 and M2 is also same.
2) acceleration of system in (A) is more than in (B) while tension in the string is same in both cases
3) acceleration of system in both cases is same but tension in the string in (A) is more than in (B)
4) acceleration of system in both cases is same but tension in the string in (A) is less than in (B).
19. A train having 60 compartments is pulled by engine with a force 6 × 104 N. If mass of each compartment
is 4000 kg, the tension in the coupling between 40th and 41st compartment is:
1) 4 × 104 N 2) 3 × 104 N
3) 2.5 × 104 N 4) 2 × 104 N
20. A man measures time period of a pendulum (T) in stationary lift. If the lift moves upwards with an
g
acceleration , then new time period will be
4
2 5 2T
1) 2) 5T / 2 3) 4)
5T 2T 5

94
[Study Package - Physics - Class XI]

21. Two blocks A and B of masses 2 m and m, respectively, are connected by a massless and inextensible
string. The whole system is suspended by a massless spring as shown in the figure. The magnitudes
of acceleration of A and B, immediately after the string is cut, are respectively:

g g g g
1) g, 2) ,g 3) g, g 4) ,
2 2 2 2
22. For a rocket propulsion velocity of exhaust gases relative to rocket is 2 km/s. If mass of rocket
system is 1000 kg, then the rate of fuel consumption for a rocket to rise up with acceleration 4.9 m/s2
will be :
1) 12.25 kg/s 2) 17.5 kg/s 3) 7.35 kg/s 4) 5.2 kg/s

23. A body, under the action of a force F  6iˆ  8ˆj  10kˆ , acquires an acceleration of 1 m/s2. The mass of
the this body must be :

1) 10 2 kg 2) 2 10 kg 3) 10 kg 4) 20 kg

24. A balloon with mass ‘m’ is descending down with an acceleration ‘a’ (where a < g). How much mass
should be removed from it so that it starts moving up with an acceleration ‘a’?

2ma 2ma ma ma
1) 2) 3) 4)
ga ga ga ga

LEVEL - II
1. The force ‘F’ acting on a particle of mass ‘m’ is indicated by the force-time graph shown below. The
change in momentum of the particle over the time interval from zero to 8 s is :

1) 24 N-s 2) 20 N-s 3) 12 N-s 3) 6 N-s


2. A block takes twice as much to slide down a 45o rough inclined plane as it takes to slide down a
similar smooth plane. The coefficient of friction is:

3 3 1 1
1) 2) 3) 4)
4 2 4 3

95
Brilliant STUDY CENTRE

3. In figure, coefficient of friction between the block and the floor is ‘  ’. Force F that will move the block
on the floor with a uniform speed is:

mg mg
1)   sin   cos   mg 2)   sin   cos   mg 3) 4)
cos    sin  sin    cos 
4. Two blocks, 4 kg and 2 kg are sliding down an inclined plane as shown in figure. The acceleration of
2 kg block is:

1) 1.66 m/s2 2) 2.66 m/s2 3) 3.66 m/s2 4) 4.66 m/s2


5. A conveyor belt is moving at a constant speed of 2 m/s. A box is gently dropped on it. The coefficient
of friction between them is   0.5 . The distance that the box will move relative to the belt before
coming to rest on it, taking g = 10 ms–2 is:
1) 0.4 m 2) 1.2 m 3) 0.6 m 4) zero
6. A plank with a box on it at one end is gradually raised about the other end. As the angle of inclination
with the horizontal reaches 30o, the box starts to slip and slides 4.0 m down the plank in 4.0 s. The
coefficient of static and kinetic friction between the box and the plank will be, respectively:

1) 0.4 and 0.3 2) 0.6 and 0.6 3) 0.6 and 0.5 4) 0.5 and 0.6
7.

Three forces start acting simultaneously on a particle moving with velcoity, v . These forces are
represented in magnitude and direction by the three sides of a triangle ABC. The particle will now move
with velocity.

1) less than v
 2) greater than v

3) |v| in the direction of the largest force BC



4) v , remaining unchanged

96
[Study Package - Physics - Class XI]

8. A particle of mass 50 g moves on a straight line. The variation of speed with time is shown in the
figure. The force acting on the particle at time t = 2 s, 4 s and 6 s will be

1) 0.25 N along the motion, 0 N and 0.25 N opposite to the motion respectively
2) 0.25 N opposite to the motion, 0 N and 0.25 N along the motion respectively
3) 0 N, 0.25 N opposite to the motion and 0.25 N along the motion respectively
4) 0.25 N along the motion 0 N and 0.25 N along the motion respectively
9. A spring of force constant k is cut into two pieces such that one piece is double the length of the other.
Then the longer piece will have a force constant of
1) (2/3) k 2) (3/2) k 3) 3k 4) 6 k
    
10. Five forces F1 , F2 , F3 , F4 and F5 are acting on a particle of mass 2 kg so that it is moving with an

acceleration of 4ms 2 in the east direction. If F1 is removed, then the acceleration becomes 7 ms–2

due north. Acceleration of the block of only F1 is acting, will be

1) 16 ms 2 2) 65 ms 2 3) 260 ms 2 4) 33 ms 2

11. Starting from rest a body slides down a 45o inclined plane in twice the time it takes to slide down the
same distance in the absence of friction. The co-efficient of friction between the body and the inclined
plane is
1) 0.75 2) 0.33 3) 0.25 4) 0.80
12. A 60 kg body is pushed horizontally with just enough force to start it moving across a floor and the
same force continues to act afterwards. The coefficient of static friction and sliding friction are 0.5
and 0.4 respectively. The acceleration of the body is
1) 6 ms–2 2) 4.9 ms–2 3) 3.92 ms–2 4) 1 ms–2
13. A man sits on a chair supported by a rope passing over a frictionless fixed pulley. The man who
weighs 1000 N exerts a force of 450 N on the chair downwards while pulling the rope on the other
side. If the chair weighs 250 N, then the acceleration of the chair is

9
1) 0.45 m s–2 2) zero 3) 2 m s–2 4) m s 2
25
14. A body of mass m is placed on a rough surface with coefficient friction  inclined at an angle  . If the
mass is in equilibrium, then

1  1  1 m 1 
1)   tan 1  2)   tan   3)   tan 4)   tan
  m

97
Brilliant STUDY CENTRE

15. Three equal weights of mass 2 kg each are hanging on a string passing over a fixed pulley as shown
in fig. What is the tension in the string connecting weights B and C?

1) Zero 2) 13 N 3) 3.3 N 4) 19.6 N


16. A body of mass 10 kg is acted upon by two perpendicular forces 6 N and 8 N. The resultant acceleration
of the body is:

1 4
(i) 1 ms–2 at an angle of tan   w.r.t. 6 N force
3

1 4
(ii) 0.2 ms–2 at an angle of tan   w.r.t. 6 N force
3

1 3
(iii) 1 ms–2 at an angle of tan   w.r.t. 8 N force
4

1 3
(iv) 0.2 ms–2 at an angle of tan   w.r.t. 8 N force
4
1) (i, iii) 2) (i, iv) 3) (ii, iii) 4) (ii, iv)
o
17. A body of mass 3 kg hits a wall at an an angle of 60 and returns at the same angle. The impact time
was 0.2 s. Calculate the force exerted on the wall:

1) 150 3 N 2) 50 3N

3) 100 N 4) 75 3N

98
[Study Package - Physics - Class XI]

18. The upper half of an inclined plane with inclination  is perfectly smooth while the lower half is rough.
A body starting from rest at the top will again come to rest at the bottom, if coefficient of friction for the
lower half is give by
1) 2 tan  2) tan  3) 2sin  4) 2 cos 
19. If the coefficient of friction between an insect and bowl surface is  and the radius of the bowl is r, the
maximum height to which the insect can crawl in the bowl is

r  1 
r 1
1) 2 2)  2 3) r 1  2 4) r  1   2  1
1   1  
20. A system consists of three masses m1, m2 and m3 connected by a string passing over a pulley P. The
mass m1 hangs freely and m2 and m3 are on a rough horizontal table (the coefficient of friction =  ).
The pulley is frictionless and of negligible mass. The downward acceleration of mass m1 is: (Assume
m1 = m2 = m3 = m)

g 1  2  2g g 1  2  g 1  2 
1) 2) 3) 4)
9 3 3 2

99
Brilliant STUDY CENTRE

KEY WITH HINTS


LEVEL - I

1
1. 4 Here m = 0.5 kg; u = –10 m/s; t = s ; v = +15 ms–1
50

Force  m  v  u  / t  0.5 10  15  50  625N

v A 60 3
2. 3 4v A  60 v B or  
v B 40 2

3
Since the distance between the two is 5 m, therefore, distance travelled by A  times that of
2
B, i.e., A will be meeting B travelling a distance = 3m.

m  u  v  0.150  20  0 
3. 2 F   30 N
t 0.1

4. 4 Here, m  20 g  20  10 3 kg ; v = 250 m/s; s = 12 cm = 0.12 m.

3 2
1 2 mv 2 20  10  250 
From F  s  mv ; F    5.2  103 N
2 2s 2  0.12

net force 20  10
5. 4 If a is acceleration of the system, then a    1 ms 2
total mass 64

It is along the direction of F .

If R is reading of the dynamometer, then R + Ma = F; R  F  Ma  20  6  1  14 N


6. 1 If a is acceleration of the lift, then as per question

976
R1  608  m  g  a  ; R 2  368  m  g  a  ; R1  R 2  608  368  2 mg ; mg   488 N
2

7. 1 Here, m1  2kg , m 2  ? ; The two blocks would not move, when

T  m 2 g  fs  s R  s m1g ;  m 2  s m1  0.1 2  0.2 kg

 m1  m 2  g g  m1  m 2  g m1 9
8. 2 For the given arrangement, a =   ;  
 m1  m 2  ; 8  m1  m 2  m2 7
9. 1 To have the resultant force only along the y-direction, resultance force along X-axis must be
zero. This is possible, when a force F is applied along X-axis, such that

1 1
F  1cos 60o  2 sin 30o ; F  2 sin 30o  1cos 60o  2  2  2  0.5N

100
[Study Package - Physics - Class XI]

10. 2 At equilibrium, m2g – T = m2a; T – m1gsin  = m1a

25
m2g – m1g sin30° = (m1 + m2) a;  a   3.55ms 2
7
 
11. 2   
Pseudo force Fpseudo   ma frame  4 2iˆ  3jˆ  8iˆ  12jˆ 
 
12. 3
  
Impulse = Change in momentum  m  v 2  v1   2  4iˆ  6ˆj  6jˆ   8iˆ kg m / s
  
13. 3 fSL  S N  S Mg   0.5 10 10   50N

Therefore, motion will start if F  50N ; If F = 60 N, motion will start so kinetic friction acts

f  f K 60  40
f K   K mg   0.4 10 10   40N ; Acceleration a    2 m / s2 ’
m 10

 F 52  5iˆ  12ˆj 
14. 2 a     10iˆ  24ˆj
m 2  13 

15. 2 v  gr  10  40  20 ms 1

16. 4 Let spring force be kx then 210 – kx = 30 a30 and kx = 15a15 = 15(6) = 90

So, 210  90  30a 30  a 30  4m / s 2

 3 
17. 4 mg  Tmax  ma min  a min   mg  mg  / m  0.4g
 5 

F
18. 4 In both cases, acceleration a 
M1  M 2

M2F M1F
In (A), TA  M 2 a  ; In (B), TB  M1a 
M1  M 2 M1  M 2

6 104
19. 4 Acceleration of train  m / s2
60  4000

 6 104  4
th st
Tension in the coupling between 40 and 41 component   20  4000   60  4000   2 10 N
 

101
Brilliant STUDY CENTRE

l 4l T 2 2T
20. 4 T  2 l / g, T  2  2 ;  T 
gg/4 5g T 5 5
21. 2 Due to non-impulsive nature of spring and impulsive nature of string,

mg mg g
aB   g  and a A   
m 2m 2
dm dm dm
22. 3 Here, thrust  v   2000  ; For rocket, v  m g  a 
dt dt dt
dm dm
 2000  1000  9.8  4.9    7.35kg / s
dt dt
 2 2
| F| 62   8   10  36  64  100 200
23. 1 Mass m       10 2 kg
|a | 1 1 1
24. 1 Let upthrust of air be Fa, then for downward motion , mg – Fa = ma
2ma
for upward motion Fa   m  m    m  m  a ; Therefore, m 
ga
LEVEL - II
1
1. 3 Change in momentum, p  Fdt  Area of F-t graph 
  2  6  3  2  4  3  12N-s
2
2
2. 1 By using 2nd equation of motion s  ut  1/ 2  t

1
For smooth inclined plane : s  0   g sin   t12
2
1
For rough inclined plane : s  0   g sin   g cos   t 22 ;
2
3
But t 2  2t1 so  
4
3. 4 Normal reaction N  mg  cos  ;For constant speed Fnet  0  Fsin   N  0

mg
 Fsin     mg  Fcos    0 ;  F 
sin    cos 
4. 2 Both blocks move together so,

 4  2  g sin 30o  0.3  4g cos 30o   0.2  2g cos 30o 


a  2.66 m / s 2
42
5. 1 By using v 2  u 2  2as ; where v  0, u  2m / s, a  g  5m / s 2 ;
2
We have 0 2   2   2  5  s  s  0.4m

102
[Study Package - Physics - Class XI]

o 1
6. 3 Coefficient of static friction, s  tan 30   0.6 ; a  g sin 30o   g cos 30 o
k
3

1 1  g kg 3 
S  ut  at 2 ;  4      16   k  0.5
2 2 2 2 
7. 4 As shown in the figure, the three forces are represented by the sides of a triangle taken in the
same order.
 
Therefore the resultant force is zero. Fnet  ma -
Therefore acceleration is also zero. i.e., velocity remains unchanged.

15  50 
8. 1 For 0 < t < 3 s, acceleration   5 ms 2 ;  force, F  ma   kg   5ms 2   0.25N
3  1000 
For 3 s < t < 5 s, acceleration = 0.  F  0 ; For 5 s < t < 8s, retardation = 5 ms–2
F  0.25N opposite to motion.
L 2L
9. 2 Original length of the spring = L; Length of two pieces are l1  and l2 
3 3
1 k L
k  1  ;  k  3k ; Similarly,, k 2  L  k 2  3 k .
L k l1 1
k l2 2
10. 2 Let east to be x-direction and north be the y-direction.
        
   
F1  F2  F3  F4  F5  m 4iˆ  8iˆ -----(i); and F2  F3  F4  F55  m 7ˆj  14ˆj ----(ii)


(i) - (ii) gives: F1  8iˆ  14jˆ ;  a 
 82  142 N 65 ms 2
2kg

1 2x 1 t 2
11. 1 x  ut  at 2 ; If u  0; t  ;  time to slide  ; rough  ;
2 a a t smooth 1
a smooth 2 g sin 
    4 ; Put   45o ; 4  4  1    0.75
a rough 1 g  sin    cos  
12. 4 Applied force (F) for motion to start is F  s mg  0.5mg
Once it starts moving, friction becomes kinetic. f k   k mg  0.4mg ;  m a  F  f k

a  0.5g  0.4g  0.1g  1ms 2


1000
13. 3 For man, T + 450 - 1000 = a; T = 550 + 100a -----(1)
10
 250 
For a chair T  450  250   a ; T = 700 + 25a ----- (2); From (1) and (2), a =
 10 
2ms–2

103
Brilliant STUDY CENTRE

14. 1 At the equilibrium point O,


In case of limiting condition,
F  mg sin  ...... (i)
Normal force, R  mg cos  ......... (ii)
Now, comparing Eqs. (i), (ii), we get
F mg sin 
 ( force of friction F = R )
R mg cos 
  tan ,   tan 1 
2g  2g  2g g g 4g
15. 2 Acceleration, a   ; for ‘C’, 2g  Tc  2   ;  Tc 
2 2 2 3 3 3

10N
16. 1 Acceleration   1 m / s 2 ; tan 1  8  4 ; tan 2  6  3
10kg 6 3 8 4

17. 1
p 2mv cos 30o 2  3  10  3 / 2
F    150 3N
t t 0.2

18. 1 Let speed of body at point B be v then

v 2  2  g sin    and v 2  2  g sin   g cos   


 2 sin    cos  ;    2 tan 

19. 2

N  mg cos  ; N  mg sin   tan   


Net force in the direction of motion m1g    m 2  m3  g g
20. 3 Acceleration    1  2 
Total mass of system m1  m 2  m3 3

104
[Study Package - Physics - Class XI]

CHAPTER - 05
WORK, ENERGY AND POWER

Work Done by Constant Force


In everyday life, the term ‘work’ is considered to be synonym of ‘labour’, ‘toil’, ‘effort’ etc. In physics,
there is a specific way of defining work.
Work is said to be done by a force when the force produces a displacement in the body on which it
acts in any direction except perpendicular to the direction of the force
For work to be done, following two conditions must be fulfilled.
(i) A force must be applied.
(ii) The applied force must produce a displacement in any direction except perpendicular to the
direction of the force.
 
Suppose a force F is applied on a body in such a way that the body suffers a displacement S in the direction
of the force. Then the work done is given by

W = FS F S

However, the displacement need not always take place in the direction of the force. Suppose a constant
   
force F , applied on a body, produces a displacement S in the body in such a way that S is inclined to F at
an angle  . Now the work done will be given by the dot product of force and displacement.
 
W  F . S , since work is the dot product of two vectors therefore it is a scalar quantity..

W = FS cos  , or W = (F cos  )S ------ (1)


\ W = component of force in the direction of displacement × magnitude of displacement.
So work is the product of the component of force in the direction of displacement and the magnitude
of the displacement.
Also, W = F(S cos  ) ------ (2)
or work is product of the component of displacement in the direction of the force and the magnitude of
the force.


os
F Sc
 F

S S
F cos 

105
Brilliant STUDY CENTRE

Special Cases:
Case (i) When  = 90o, then W = FS cos 90o = 0
So, work done by a force is zero if the body is displaced in a direction perpendicular to the direction of the
force.
Work done by a variable force
Often the force applied to an object varies with position. Important examples include electric and
gravitational force, which vary with the distance between interacting objects. The force of a spring is another
example; as the spring stretches, the force increases.

   
Here we cannot apply W  F . S . Since F is a variable. So we take a small part dS of its path. This

dS is very small so that force may not vary during this displacement. So the work done during this displacement
is
 
dW  F . dS = F dS cos 
The total work done in going from A to B as shown may be calculated by summing up i.e. integrating
the work done during all its small fractions.
B
  B
i.e. WA  B   F . dS    F cos   dS , In terms of rectangular components,
A A

 
F  Fx ˆi  Fy ˆj  Fz kˆ and dS  dx ˆi  dy ˆj  dz kˆ
xB yB zB

therefore, WA  B   F dx   F dy   F dz
x y z
xA yA zA

Work done as Area under the force displacement graph


Suppose a particle moving along a straight line and a variable force acting on it.
xf

W  F .dx = Area under F - x graph from x = xin to xf.


x in

106
[Study Package - Physics - Class XI]

In general, the work done from initial point xin to final point xf is given by the area under the force-
displacement curve as shown in the figure.
Area (work) above the x-axis is taken as positive, and below x-axis as negative
Units of work:
1. Unit of work:
I. In CGS system, the unit of work is erg.
One erg of work is said to be done when a force of one dyne displaces a body through one
centimetre in its own direction.  1 erg = 1 dyne × 1 cm = 1 g cm s–2 × 1 cm = 1 g cm2s–2
II. In SI i.e., the unit of work is joule.
One joule of work is said to be done when a force of one Newton displaces a body through one
metre in its own direction. 1 Joule = 1 Newton × 1 metre = 1 kg × 1 m/s2 × 1 m = 1 kg m2 s–2.
Relation between joule and erg
1 joule = 1 newton × 1 metre; 1 joule = 105 dyne × 102 cm = 107 dyne cm
1 joule = 107 erg; 1 erg = 10 –7 joule
Dimensions of work
[Work] = [Force] [Distance] = [MLT–2] [L] = [ML2T–2]
On the basis of dimensional formula, the unit of work is kg m2 s–2.
Conservative and Non-conservative Forces
Gravitational, electrostatic, and restoring force of a spring are some of the natural forces produces
work done that depends only on the locations of the initial and final points and not on the path followed. On
the other hand, there are forces such as friction, whose work depends on path followed. Accordingly, forces
are divided into two categories - one whose work is path independent called conservative and other whose
work is path dependant called non-conservative forces. The forces of the former category are known as
conservative forces and of the latter one as non-conservative forces.
A force is conservative if the work done by it on a body moving it from one position to another position
depends only on the initial and final positions and not on the path followed by it.
or The net work done by the force on a body that moves through any closed path is zero.
A force is said to be non conservative if the work done by it on a body between two positions depends
on the path followed by the body between the two positions.
or The workdone by the force on a body that moves through a closed path in non-zero.
Conservative Force and Potential Energy
For a conservative force F that depends upon position r, there is a potential energy function U which
also depends on r. When a conservative force does positive work, the potential of the system decreases,
i.e.,

dU
Work done = decrease in potential energy or Fdr = – dU or F   -----(6)
dr
Hence the negative derivative of the potential energy function with respect to position gives the
conservative force acting on the system.
b

The change in potential energy when the body is displaced from r = a to r = b is U b  U a   F.dr
a

Note: F is negative if r is opposite to F and positive if r is in the same direction as F.

107
Brilliant STUDY CENTRE

Energy
Definition: Energy is defined as internal capacity of doing work.
Energy appears in many forms such as mechanical, electrical, chemical, thermal (heat), optical (light),
acoustical (sound), molecular, atomic, nuclear etc., and can change from one form to the other.
Unit: erg is the CGS unit of energy and joules is the SI unit.
1 erg = 1 g × (1 cm/s)2; 1 J = 1 kg × (1 m/s)2
Kinetic Energy
The energy possessed by a body by virtue of its motion is called kinetic energy.
Examples: (i) Flowing water possesses kinetic energy which is used to run the water mills.
(ii) Moving vehicle possesses kinetic energy.
(iii) Moving air (ie, wind) possesses kinetic energy which is used to run wind mills.
1) Expression for kinetic energy :
Let m = mass of the body, u = initial velocity of the body ( = 0), F = force acting on the body,
a = Acceleration of the body, s = Distance travelled by the body, v = Final velocity of the body

v2
2 2
From v = u + 2as  v 2  0  2as s 
2a
Since the displacement of the body is in the direction of the applied force, then work done by the force
is

v2 1
W  F  s  ma  ;  W mv 2
2a 2

1
This work done appears as the kinetic energy of the body KE  W  mv 2 ------ (7)
2
2) Work-energy theorem: From equation (7) dW = mv dv
Work done on the body in order to increase its velocity from u to v is given by
v v
 v2  1
W  m  v dv  m   ;  W m  v 2  u 2  ------ (8)
u  2 u 2 

Work done = change in kinetic energy W  E ------ (9)


This is work energy theorem, it states that work done by a force acting on a body is equal to the
change produced in the kinetic energy of the body.
Examples:
(i) In case of vertical motion of a body under gravity when the body is projected up, force of gravity is
opposite to motion and so kinetic energy of the body decreases and when it falls down, force of gravity
is in the direction of motion so kinetic energy increases.
(ii) When a body moves on a rough horizontal surface, as force of friction acts opposite to motion, kinetic
energy will decrease and the decrease in kinetic energy is equal to the work done against friction.
3) Relation between kinetic energy and linear momentum: As we know

1 1 p 1
E mv 2    v 2  As p  mv  E  pv
2 2 v 2
108
[Study Package - Physics - Class XI]

1 1 p2 2E
 KE, E  mv 2  pv  ------ (10) and Momentum p   2mE ------ (11)
1)
2 2 2m v
From above relation it is clear that a body can not have kinetic energy without having momentum and
vice-versa.
Potential Energy
The difference in potential energy between two points A & B is equal to the work done by external force

 
Fext against the conservative forces in moving a particle slowly (that is without developing kinetic energy)
between those two points.
B
 
U B  U A  U  Wext.  A  B    Fext . dr
slowly A

     
B
 
 U    FC . dr  U   W
Fext  FC  ma  Fext.  FC  0  Fext.   FC ; C
A

Change in potential energy between two points is equal to the negative of work done by conservative
forces.
Gravitational potential energy for uniform gravitational force:
Near the earth surface, the variation in the gravitational force between a body of mass m and the
ground can be neglected. For such a system, change in gravitational potential energy in any vertically upward
displacement h of mass m is given by U  mgh and in vertical downward displacement h is given by
U   mgh
Elastic Potential Energy
(1) Restoring force and spring constant: When a spring is stretched or compressed from its normal
position (x = 0) by a small distance x, then a restoring force is produced in the spring to bring it to the
normal position. According to Hooke’s law this restoring force is proportional to the displacement x
and its direction is always opposite to the displacement.

  
ie, F   x ; or F   k x ------ (14); where k is called spring constant.
if x = 1, F = k (Numerically); or k = F
Hence spring constant is numerically equal to force required to produce unit displacement in the
spring.
Actually k is a measure of the stiffness/softness of the spring.

109
Brilliant STUDY CENTRE

2
F
 k 
 F  MLT 
   MT 2 
Dimension : As k 
x  
x L
Units : S.I. unit is newton/metre, C.G.S. unit is dyne/cm
(2) Expression for elastic potential energy: When a spring is stretched or compressed from its
  
normal position (x = 0), work has to be done by external force against restoring force. Fext  Frestoring  kx .
Let the spring is further stretched through the distance dx, then work done
 
dW  Fext . dx  Fext . dx cos 0o  kx dx  As cos 0o  1
Therefore total work done to stretch the spring through a distance x from its mean position is given by
x x x
 x2  1
W   dW   kx dx  k    kx 2 ------ (15)
0 0  2 0 2
This work done is stored as the potential energy of the stretched spring.

1 2 1  F F2
 Elastic potential energy U 
2
kx ; U  Fx
2  As k  x 
; Also U 
 2k

1 2 1 F2
 Elastic potential energy U  kx  Fx  ------ (16)
2 2 2k
Note: If the spring is stretched from initial position x1 to final position x2 then work done
1
= Increment in elastic potential energy = k  x 22  x12 
2
(3) Energy graph for a spring: If the mass attached with the spring performs simple harmonic motion
about its mean position then its potential energy at any position (x) can be given by

1 2
U kx ------ (17); So for the extreme position
2

1 2
U
2
ka  As x  a for extreme

This is the maximum potential energy or the total energy of the mass.

1 2
 Total energy E  ka ------ (18) (v = 0 at extreme so KE = 0)
2
110
[Study Package - Physics - Class XI]

1 2 1 2 1
Now kinetic energy at any position K  E  U  ka  kx =  k  a 2  x 2  ------ (19)
2 2 2

1 2
 U max  ka  At extreme x  a  and U min  0 [At mean x = 0]
2

1 2
K max  ka  At x  0 and K min  0 [At extreme x   a ]
2
It means kinetic energy changes parabolically with respect to position but total energy remains always
constant irrespective to position of the mass.
VARIOUS FORMS OF ENERGY : THE LAW OF CONSERVATION OF ENERGY
Mechanical energy can be classified into two distinct categories : one based on motion, namely kinetic
energy; the other on configuration (position), namely potential energy. Energy comes in many a forms
which transform into one another
The Equivalence of Mass and Energy
Till the end of the nineteenth century, physicists believed that in every physical and chemical process,
the mass of an isolated system is conserved. Albert Einstein (1879-1955), showed that mass and energy
are equivalent and are related by the relation E = m c2
where c, the speed of light in vacuum is approximately 3 ×108 m s–1.
E = 1× (3 ×108)2 J = 9 ×1016 J. This is equivalent to the annual electrical output of a large (3000 MW)
power generating station.
The principle of conservation of energy
The total mechanical energy is conserved, if the forces doing work are conservative. If some of the
forces involved are non conservative, part of the mechanical energy may get transformed into other forms
such as heat, light and sound. The total energy of an isolated system does not change. Energy may be
transformed from one form to another but the total energy of an isolated system remain constant. Energy
can neither be created, nor be distroyed.
Conservation of Mechanical Energy

The work-energy theorem, shows that the change KE in a body’s kinetic energy is equal to the net
work done on it: KE  Wnet

Consider separately the work Wc done by conservative force and the work Wnc done by nonconservative
forces. Then KE  Wc  Wnc

111
Brilliant STUDY CENTRE

We’ve defined the change in potential energy U as the negative of the work done by conservative
forces. So we can write.

KE  U  Wnc ; or KE  U  Wnc


We define the sum of the kinetic and potential energy as the mechanical energy. The Equation shown
that the change in mechanical energy is equal to the work done by non-conservative forces.

i.e. E  Wnc ;  E  0 if W nc = 0

Thus if work done by non-conservative forces is zero the mechanical energy of the system is
unchanged. This is called law of conservation of mechanical energy. It may also be written as

U  KE  0 ; or U  KE ( Increase in PE = Decrease in KE)


or U + KE = constant; or U in  KE in  U f  KE f

Power
Power of a body is defined as the rate at which the body can do the work.

W W
Average power  Pav   
t t
 
dW F. ds  
Instantaneous power  Pin st.     As dW  F . ds 
dt dt  


    ds 
Pinst  F .v ------ (23)  As v  
 dt 

ie, power is equal to the scalar product of force with velocity.


Important points

(1) Dimension:  P    F v    MLT 2   LT 1    P    ML2T 3 


(2) Units : Watt or Joule/ sec [S.I.], Erg/sec [C.G.S.]
Practical units: Kilowatt (kW), Mega watt (MW) and Horse Power (hp)
Relations between different units: 1 watt = 1 Joule / sec = 107 erg/sec
1 hp = 746 watt; 1 MW = 106 watt; 1 kW = 103 watt

1
(3) If work done by two bodies is same then power 
time
ie, the body which perform the given work in lesser time possess more power and vice-versa.
(4) As power = work/time, any unit of power multiplied by a unit of time gives unit of work (or energy),
ie, kilowatt-hour or watt-day are units of work or energy.

J
1kWh  103   60  60 sec   3.6  106 joule
sec

112
[Study Package - Physics - Class XI]

dW
(5) The slope of work-time curve gives the instantaneous power. As P   tan 
dt

dW
(6) Area under power time curve gives the work done as P 
dt

 W   P dt  W  Area under P-t curve

COLLISION
Collision

Collision in a system takes place when there is an interactive force acts for a short time between two or
more bodies.

(i) In a collision, a relatively large force acts on each colliding particle for a relatively short time.

(ii) In collision, it is not necessary that the colliding particles come in contact physically.

(iii) In a collision, if the motion of colliding particles before and after the collision is along the same line, the
collision is said to be head on or one dimensional. Particles move in other directions than initial direction
after collisions is called oblique impact.

(iv) The law of conservation of momentum holds good for any type of collision (elastic, inelastic or perfectly
inelastic). The total momentum of the system after the collision must be equal to the total momentum
of the system before collision. Although the momentum of individual particles within the system may be
changed, but the total momentum remains constant.

(v) A perfectly elastic collision: If in a collision, along with momentum KE is also conserved, the collision is
said to be perfectly elastic.

(vi) An inelastic collision: If in a collision, some kinetic energy is lost, the collision is said to be inelastic. All
real collisions belong to this category. Here, KE appears in other forms.

(vii) A perfectly inelastic collision: If in a collision, two bodies stick together or move with same velocity after
the collision, the collision is said to be perfectly inelastic.

(viii) The conservation of momentum and the conservation of total energy holds for all the three types of
collisions, but KE conservation hold only for perfectly elastic collision.

(ix) The collisions of elementary particles, like electrons, protons with the nucleus are examples of nearly
perfectly elastic collisions. In real collisions, like collision of a ball with a wall, etc., some KE is lost in the
form of heat and sound. However, these are usually neglected and collisions are treated as elastic
collisions.

113
Brilliant STUDY CENTRE

Elastic collision
During collision if KE is conserved the collision is elastic, i.e., Initial KE will be equal to the final KE. For
an inelastic collision the KE is not conserved, i.e., the final KE will not be equal to the initial KE. The loss in
KE appears in other forms of energy.
Inelastic collision

If two bodies after collision stick together the collision is perfectly in-elastic.

The ratio of relative velocity after collision to the relative velocity before collision is called coefft. of
restitution e.
 
|v v |  
e   2  1 ------(26); e  0, v 2  v1 , perfectly inelastic collision.
| u 2  u1 |

   
e  1 | v 2  v1 |  | u 2  u1 | , perfectly elastic collision.; For all other collision, 0 < e < 1.

Collision in one dimension

Elastic collision

Consider two bodies moving along a line with velocities u1 & u2 collide elastically, after collision they
travel with velocities v1 & v2.

m1 m1
u1 u2 v1 v2

m2 m2

By law of conservation of momentum

m1u1  m 2 u 2  m1v1  m 2 v 2 ; m1  u1  v1   m 2  v 2  u 2  ---------(29)

since collision is elastic, KE is conserved.

1 1 1 1
m1u12  m 2 u 22  m1v12  m 2 v 22 ; m1  u12  v12   m 2  v 22  u 22  ---------(30)
2 2 2 2

Dividing eqn. (30) by (29); u1  v1  v 2  u 2 ;  u1  u 2   v2  v1 ---------(31)


Thus in one dimesnional elastic collision, relative velocity of approach = relative velocity of recession.

Multiplying eqn. (31) with m2 and subtracting it from eqn. (29)

 m  m2  2m 2
 m1  m2  u1  2m 2u 2   m1  m2  u1 ; v1   1  u1  u2 ---------(32)
 m1  m 2  m1  m 2

Similarly multiplying eqn. (31) with m1 and adding to (29)

114
[Study Package - Physics - Class XI]

2m1 m  m1
2m1u1   m 2  m1  u 2   m1  m 2  v 2 ; v2  u1  2 u2
m1  m 2 m1  m 2 ---------(33)

If the masses are equal, m1 = m2 = m; then v1 = u2 and v2 = u1

ie, For an elastic collision of two bodies of equal masses, in one dimension the velocities are interchanged,
initially if u2 = 0, then v1 will become zero ie, if the second body was at rest initially then the Ist body stops
motion and the second body moves with the velocity of the first body.

Loss of kinetic energy

(i) Elastic collisions: The total KE in an elastic collision is conserved. However, individual particles may
gain or lose KE. Suppose a particle of mass m1, moving with velocity u1 collides with a particle of mass
m2 at rest.

1
The KE of particle 1 before collision is: K i  m1u12
2

1   m1  m 2  
The KE of particle 1 after collision is, K f  m1v12 ; But, v1    u1 ;
2   m1  m 2  

2 2
 m1  m 2  K f  m1  m 2  Ki  K f 4m1m 2
So, K f    Ki    ---------(34) or 
 m1  m 2  ------(34)
2
Ki
 m1  m 2  K i  m1  m 2 

The loss is maximum (=100%) when m1 = m2.

Inelastic collision - Loss of KE due to collision.

In case of inelastic collision, after collision the bodies move with same velocity or they stick together.

m1 m2
u1 u2
A B A B

Before collision After collision

Two particles of masses m1 & m2, moving with velocities u1 & u2 (u1 > u2) along the same line collide
head on. And after collision they have same common velocity v, then by conservation of linear momentum.

m1u1  m 2 u 2
m1u1  m 2 u 2   m1  m 2  v ; v
m1  m 2

1 1 1
KE i  m1u12  m 2 u 22 ; KE f   m1  m 2  v 2 ; loss in KE due to collision.
2 2 2
115
Brilliant STUDY CENTRE

1 1 1
KEi  KEf  m1u12  m 2 u 22   m1  m 2  v 2
2 2 2

1  m1m 2  u1  u 2  2u1u 2  
2 2 2
1  m u  m2u 2  
  m1u12  m2 u 22  1 1    
2  m1  m 2 
 2
  m1  m 2  

1 m1m 2 2 1 m1m 2
k   u1  u 2  --------(35); If m2 is at rest, u2 = 0; k  u12
2  m1  m 2  2  m1  m 2 

k m2  1 2 k
  k  m1u1  ; If m2 is a massive target (m2 >> m1);  1; 100%
k m1  m 2  2  k

ie, if a light moving body strikes a heavy body at rest and sticks to it, all its KE will be lost.
Impulse momentum principle
When a system changes its configuration, its particles change their location or momenta. Sum of
linear momenta of all the particles equals to the linear momentum of centre of mass due to translation.
Principle of impulse and momenta states that net impulse of all the external forces is equal to change in
momentum of the centre of mass.
  
  Fi dt  p c2  p c1

116
[Study Package - Physics - Class XI]

QUESTIONS
LEVEL - I
1. The kinetic energy of a particle moving along a circle of radius R is K. The force acting on the particle is

K 2K K K
1) 2) 3) 4)
R R 2R R2
2. Two bodies of masses 1 kg and 2 kg moving with same velocities are stopped by the same force.
Then the ratio of their stopping distances is

1) 1 : 2 2) 2 : 1 3) 2 :1 4) 1: 2

3. A force F = 20 + 10 y acts on a particle in y-direction where F is in newton and y in meter. Work done
by this force to move the particle from y = 0 to y = 1 m is
1) 20 J 2) 30 J 3) 5 J 4) 25 J
4. A gardener pushes a lawn roller through a distance 20 m. If he applies a force of 20 kgwt in a direction
inclined at 60o to the ground, the work done by him is
1) 1960 J 2) 196 J 3) 1.96 J 4) 196 kJ

5.
2
 
The potential energy of a conservative system is given by V  x   x  3x joule, where x is measured
in metre. Then its equilibrium position is at
1) 1.5 m 2) 2 m 3) 3 m 4) 1 m
6. A bread gives 2 kcal of energy to a boy. How much height he can climb by using this energy, if his
efficiency is 28% and mass is 60 kg?
1) 15 m 2) 5 m 3) 2.5 m 4) 10 m
7. A particle is acted upon by a constant power. Then, which of the following physical quantity remains
constant?
1) Speed 2) Rate of change of acceleration
3) Kinetic energy 4) Rate of change of kinetic energy
8. A body projected vertically upward from the earth reaches a height equal to earth’s radius before
returning to the earth. The power exerted by the gravitational force is greatest.
1) at the highest position of the body
2) at the instant just before the body hits the earth
3) it remains constant all through
4) at the instant just after the body is projected
9. Which of the following is true for any collision?
1) Both linear momentum and kinetic energy are conserved
2) Neither linear momentum nor kinetic energy may be conserved
3) Linear momentum is always conserved, however, kinetic energy may or may not be conserved
4) Kinetic energy is always conserved, but linear momentum may or may not be conserved

117
Brilliant STUDY CENTRE

10. A particle is placed at the origin and a force F = kx is acting on it (where k is a positive constant). If u(0)
= 0, the graph of u(x) versus x will be (where u is the potential energy function)

11. A body of mass 2 kg is projected at 20 m/s at an angle 60o above the horizontal. Power delivered to
the block due to the gravitational force at its highest point is

1) 200 W 2) 100 3 W 3) 50 W 4) zero

12. A light body A and a heavy body B have equal linear momentum. Then the kinetic energy of the body
A is
1) greater than that of B 2) equal to that of B 3) zero 4) smaller than that of B
13. A toy gun has a spring of force constant k. After charging the spring by compressing it through a
distance of x, the toy releases a shot of mass m vertically upwards. Then the shot will travel a vertical
height of

2mg kx 2 kx kx 2
1) 2) 3) 4)
kx 2 mg mg 2mg
14. A worker pushes a wheelbarrow with a horizontal force of 50 N on level ground over a distance of 5.0
m. If a friction force of 43 N acts on the wheelbarrow in a direction opposite that of the worker, what
work is done on the wheelbarrow by the worker?
1) 250 J 2) 215 J 3) 35 J 4) 10 J

15.  
A particle is moved from (0, 0) to (a, a) under a force F  3iˆ  4ˆj . By two paths, path 1 is OP and
path 2 is OQP. Let W 1 and W 2 be the work done by this force in these two paths. Then

1) W 1 = W 2 2) W 1 = 2W 2
3) W 2 = 2W 1 4) W 2 = 4W 1
16. A body of mass (4 m) is lying in x-y plane at rest. It suddenly explodes into three pieces. Two pieces
each of mass (m) move perpendicular to each other with equal speeds (v). The total kinetic energy
generated due to explosion is

3
1) mv2 2) mv 2 3) 2mv 2 4) 4mv 2
2

118
[Study Package - Physics - Class XI]

17. On a frictionless surface, a block of mass. M moving at speed v collides elastically with another block
of same mass M which is initially at rest. After collision the first block moves at an angle  to its initial
v
direction and has a speed . The second block’s speed after the collision is:
3

3 2 2 3 3
1) v 2) v 3) v 4) v
2 3 4 2
18. Two identical balls A and B having velocities of 0.5 m/s and –0.3 m/s respectively collide elastically in
one dimension. The velocities of B and A after the collision respectively will be
1) –0.3 m/s and 0.5 m/s 2) 0.3 m/s and 0.5 m/s 3) –0.5 m/s and 0.3 m/s 4) 0.5 m/s and –0.3 m/s
19. Which of the following graphs is correct between kinetic energy (E), potential energy (U) and height
(h) from the ground of the particle.

20. A ball is dropped from a height of 10 m. It is embedded 1 m in sand and stops,


1) only momentum remains conserved 2) only K.E. remains conserved
3) both momentum and K.E. are conserved 4) neither momentum nor K.E. is conserved
21. A billiard ball moving with a speed of 5 m/s collides with an identical ball, originally at rest. If first ball
stops dead after collision, then second ball will move forward with a speed of
1) 10 m/s 2) 5 m/s 3) 2.5 m/s 4) 1 m/s
22. If two balls, each of mass 0.06 kg, moving in opposite directions with speed 4 m/s, collide and rebound
with same speed, then impulse imparted to each ball due to other is
1) 0.48 kg m/s 2) 0.24 kg m/s
3) 0.81 kg m/s 4) zero
23. A body of mass m1 is moving with a velocity v. It collides with another stationary ball of mass m2. They
get embedded. At the point of collision, velocity of the system will
1) increase 2) decrease but not to zero
3) zero 4) remain the same
24. A child is sitting on a swing. Its minimum and maximum heights from the ground are 0.75 m and 2 m
respectively, its maximum speed will be
1) 10 m/s 2) 5 m/s 3) 8 m/s 4) 15 m/s
25. A man racing with his son has half the kinetic energy of the son, who has half the mass of the father.
The man speeds up by 1 m/s and has the same kinetic energy as the son. What was the original
speed of the man?
1) 4.8 m/s 2) 3.6 m/s 3) 2.4 m/s 4) 1.2 m/s

119
Brilliant STUDY CENTRE

a b
26. If potential energy is given by U   . Then find out maximum force. (Given a = 2, b = 4)
r2 r
16 32 32 16
1)  N 2)  N 3)  N 4)  N
27 27 27 27
27. Consider a drop of rain water having mass 1 g falling from a height of 1 km. It hits the ground with a
speed of 50 ms–1. Take ‘g’ constant with a value 10 ms–2. The work done by the (i) gravitational force
and the (ii) resistive force of air is
1) (i) 1.25 J (ii) – 8.25 J 2) (i) 100 J (ii) 8.75 J
3) (i) 10 J (ii) –8.75 J 4) (i) –10 J (ii) –8.25 J
LEVEL - II
1. A stone is projected vertically upto reach maximum height h. The ratio of its kinetic energy to its
4
potential energy at a height h , will be
5
1) 5 : 4 2) 4 : 5 3) 1 : 4 4) 4 : 1
2. The heart of a man pumps 5 litres of blood through the arteries per minute at a pressure of 150 mm
of mercury. If the density of mercury be 13.6 × 103 kg/m3 and g = 10 m/s2 then the power of heart in
watt is
1) 3.0 2) 1.50 3) 1.70 4) 2.35
3. A body of mass 103 kg moving with speed 2 ms–1 collides with a body of mass 15 × 103 kg inelastically
and sticks to it. Then loss in KE of the system will be
1) 1.875 kJ 2) 15 kJ
3) 10 kJ 4) 5 kJ
4. A moving block having mass m, collides with another stationary block having mass 4m. The lighter
block comes to rest after collision. When the initial velocity of the lighter block is v, then the value of
coefficient of restitution (e) will be
1) 0.5 2) 0.25 3) 0.8 4) 0.4
5. A machine which is 75 percent efficient, uses 12 joules of energy in lifting up a 1 kg mass through a
certain distance. The mass is then allowed to fall through that distance. The velocity at the end of its
fall is (in ms–1)

1) 24 2) 32 3) 18 4) 9
6. A ball of mass 0.2 kg is thrown vertically upwards by applying a force by hand. If the hand moves 0.2
m which applying the force and the ball goes to 2 m height further, find the magnitude of the force.
Consider g  10 m / s 2 .

1) 22 N 2) 4 N 3) 16 N 4) 20 N
7. The work done in dragging a stone of mass 100 kg up an inclined plane 1 in 100 through a distance of
10 m is (take g = 9.8 m/s2)
1) zero 2) 980 J 3) 9800 J 4) 98 J

120
[Study Package - Physics - Class XI]

8. A body of mass 5 kg is moving with a momentum of 10 kg-m/s. A force of 0.2 N acts on it in the
direction of motion of the body for 10 seconds. The increase in its kinetic energy is
1) 2.8 joules 2) 3.2 joules
3) 3.8 joules 4) 4.4 joules
9. A mass of 0.5 kg moving with a speed of 1.5 m/s on a horizontal smooth surface collides with a nearly
weightless spring of force constant k = 50 N/m. The maximum compression of the spring would be :

1) 0.15 m 2) 0.12 m 3) 1.5 m 4) 0.5 m


10. An explosion blows a rock into three parts. Two parts go off at right angles to each other. These two
are 1 kg first part moving with a velocity of 12 ms–1 and 2 kg second part moving with a velocity of 8
ms–1. If the third part flies off with a velocity of 4 ms–1, its mass would be
1) 5 kg 2) 7 kg 3) 17 kg 4) 3 kg
11. If 4 J of work is required to stretch a spring through 10 cm beyond its unstretched length. The extra
work required to stretch it through additional 10 cm shall be
1) 4 J 2) 8 J 3) 12 J 4) 16 J
12. The bob of a pendulum is released from a horizontal position A as shown in figure. The length of the
pendulum is 2 m. If 10% of the initial energy of the bob is dissipated as heat due to the friction of air,
what would be the speed of the bob when it reaches the lowermost point B? Take g = 10 ms–2.

1) 3 ms–1 2) 4 ms–1
3) 5 ms–1 4) 6 ms–1
13. A car of mass 1000 kg moving with a speed 18 km h–1 on a smooth road collides with a horizontally
mounted spring of spring constant 6.25 × 103 N m–1. The maximum compression of the spring is
1) 1 m 2) 2 m 3) 3 m 4) 4 m
14. A 30 m deep well is having water upto 15 m. An engine evacuates it in one hour. The power of the
engine, if the diameter of the well is 4 m is
1) 11.55 kW 2) 1155 kW
3) 23.10 kW 4) 2310 kW

121
Brilliant STUDY CENTRE

15. A body moves from rest with a constant acceleration. Which one of the following graphs represents
the variation of its kinetic energy K with the distance travelled (x)?

K K

1) 2)
x x

K K

3) 4)

x x

16. A uniform chain of length 2 m is kept on a table such that a length of 60 cm hangs freely from the edge
of the table. The total mass of the chain is 4 kg. The work done in pulling the entire chain on the table
is (Take g = 10ms–2)

1) 12.9 J 2) 6.3 J 3) 3.6 J 4) 2.0 J

17. An engine pumps water through a hose pipe. Water passes through the pipe and leaves it with a
velocity of 2 m/s. The mass per unit length of water in the pipe is 100 kg/m. What is the power of the
engine?

1) 800 W 2) 400 W 3) 200 W 4) 100 W



18. A particle moves in XY plane under the influence of a force F such that its instantaneous momentum

is p  ˆi2 cos t  ˆj2sin t . What is the angle between the force and instantaneous momentum?

1) 0o 2) 180o 3) 90o 4) 45o

19. A particle of mass ‘m’ moving along X-axis under a force F  k x . Work done for the motion of
particle from x = 4 m to x = 9 m is nearly

1) 19.2 k 2) 8.6 k

3) 32.3 k 4) 12.7 k

20. A stone of mass m is thrown from earth’s surface at an angle  to the horizontal with an initial
velocity v0. Ignoring the air drag, the power developed by gravity force t second after the beginning of
motion is

1) mg  g / 2  v 0 sin   2) mgv0 sin .t

3) mg  g  v sin .t  4) zero

122
[Study Package - Physics - Class XI]

21. A body starts from rest and acquires a velocity v in time t. The work done on the body in times T is
proportional to

v v2 v2 2 v 2T 2
1) T 2) T 3) T 4)
t t2 t2 t
22. A bullet when fired at a target has its velocity decreased to 50% after penetrating 30 cm into it.
Additional thickness it will penetrate in cm before coming to rest is
1) 10 cm 2) 30 cm 3) 40 cm 4) 60 cm
23. Consider two incline rough surfaces A and B with angles of inclination 45o and 60o, respectively.
Height of the top edge of each surface from ground is same. A particle placed on top of the incline ‘A’
reaches the bottom with a speed vA. When the same particle is placed on top of the incline B, it
reaches the bottom with speed vB. Assuming the same coefficient of friction in the two cases?
1) vA = vB 2) vA > vB

3) v A  v B 4) It cannot be decided

24. A 2 kg block slides on a horizontal floor with a speed 4 m/s. It strikes an uncompressed spring and
compresses it till the block is motionless. The kinetic energy is 15 N and spring constant is
10, 000 N / m . The spring compresses by

1) 8.5 cm 2) 5.5 cm 3) 2.5 cm 4) 11.0 cm


25. If kinetic energy of a body is increased by 300% then percentage change in momentum will be
1) 100% 2) 150% 3) 265% 4) 73.2%
26. A body of mass 1 kg is thrown upwards with a velocity of 20 m/s. It momentarily comes to rest after
attaining a height of 18 m. How much energy is lost due to air frcition? (g = 10 m/s2)
1) 10 J 2) 20 J 3) 30 J 4) 40 J

123
Brilliant STUDY CENTRE

KEY WITH HINTS


LEVEL I
1 2
1. 2 Kinetic energy of a particle, K  mv ...(i)
2
where m is the mass and v is the speed of the particle. Force acting on the particle is

mv 2 2K
F or F  (Using (i))
R R
2. 1 If two bodies of masses m1 and m2 moving with the same velocities are stopped by the same
force,

1
ds m 1 kg 1
then the ratio of their stopping distances is d  m  2 kg  2
1

s2 2

3. 4 Given: F = 20 + 10 y; Work done, W  F.dy 


1 1
 10  10
   20  10y  dy   20y  y 2   20   25 J
0  2 0 2

4. 1 The work done by the gardener is W  FScos  , F = 20 × 9.8 N; S = 20 m;   60o

 W  196  20  cos 60o  1960 J


dV
5. 1 
2

Here, V  x   x  3x J ; For a conservative field, force, F  
dx
d 2
F 
dx
 x  3x     2x  3   2x  3
3
At equilibrium position, F = 0;   2x  3  0 or x  m  1.5 m
2
6. 4 E = 5000 cal = 5000 × 4.2 = 21 × 103 J
28
By conservation of energy mgh   21 103 ; h = 10 m.
100
7. 4 since power is rate of change of energy. Therefore, if power is constant then rate of change of
kinetic energy will also be constant.


8. 2 Power, P  F.v  Fv cos 
Just before hitting the earth   0o . Hence, the power exerted by the gravitational force is greatest
at the instant just before the body hits the earth.
9. 3 Linear momentum is conserved in all type of collisions but kinetic energy is not conserved in all
type of collisions. Kinetic energy is conserved in elastic collisions but not conserved in inelastic
collisions.
124
[Study Package - Physics - Class XI]

du
10. 1 From F   ; du  Fx
dx
ux x x 2
u  du    F dx    kx dx ; u   kx ; As u(0) = 0, and u  x 2
0 0 0 2

11. 4 Velocity of highest point ; v x  v cos   20  cos 60 o  10 m / s

Now the angle between horizontal velocity and gravitational force   90o

Power = fv cos   mgv cos 90o  0

1
12. 1 Since KE  ; Lighter body has greater kinetic energy than heavier.
m
13. 4 According to the law of conservation of energy, elastic potential energy stored in the spring =

1 2 kx 2
gravitational potential energy acquired by shot. kx  mgh ; h 
2 2mg
14. 1 The work one on the wheelbarrow by the worker is

W   Fcos   x   50N  5.0m   250 J

15. 1 Force is a constant and work done is path independent under a constant force. Hence W1  W2 .
16. 2 Initial linear momentum = Pi = 0

Final momentum Pf  0  mviˆ  mvjˆ  p3 ;  P3  mv 2

P32 1 1 2m 2 v 2 3mv 2
Total KE   mv 2  mv 2   mv 2 
2  2m 2 2 4m 2

17. 2 In elastic collision energy of system remains the same, so,  KE before collision   KE after collision
2
1 2 1v 1 2
Let speed of second body after collision is v ; mv  0     m  v  ;
2 2 3 2

2 2
 v  v
3
18. 4 Since both bodies are identical and collision is elastic. Therefore velocities will be interchanged
after collision. vA = –0.3 m/s and vB = 0.5 m/s
19. 1 Potential energy increases and kinetic energy decreases when the height of the particle increases.
20. 1 As the collisin is inelastic, therefore, only momentum is conserved.
21. 2 As the balls are identical, their collision is perfectly elastic. The velocities are just exchanged.
22. 1 As each ball rebounds with the same speed, therefore, impulse imparted = change in linear
momentum of the ball = m (v – u) = 0.06 (4 + 4) = 0.48 kg m/s

125
Brilliant STUDY CENTRE

23. 2 If v' is velocity of the system, then from principle of conservation of linear momentum,
m1v
m1v  0   m1  m 2  v ; v 
m1  m 2 ; Clearly, v  v but v  0

1
24. 2 mv 2  mg  h 2  h1  ; v  2g  h 2  h1   2 10  2  0.75   25  5 m / s
2
Ks M
25. 3 Given, K m  and M s  m
2 2
1 2
Let the original speed of man be vm. Then M m  v m  1  K s  2 K m
2
1  Ks  1
M m v m2  K m   ; or  v m  1  2v m2 or v m  1  2 v m ;or v m   2.4 m/s
2
or 2 
2  2  2 1
dU  2a b  2a b
26. 1 F    3  2   3  2
dr  r r  r r

dF 6a 2b 6a 3a 6 3 22 4 16
 4  3 0  2b  r    ;  Fmax  3
 2

dr r r r b 4 2 3 / 2 3 / 2  27

27. 3 (i) The work done by the gravitational force  mgh  103  10  1000  10 J
(ii) The total work done by gravitational force and the resistive force of air is equal to change in
kinetic energy of rain drop. Wg  mgh  103 10 103  10

1 1
 Wg  Wr  mv 2  0 ; 10  Wr   103  502 or Wr  8.75 J
2 2
LEVEL - II
4 4 4
1. 3 At a height h .; Potential energy  mg  h  mgh ; Total energy = mgh
5 5 5
4 1
 Kinetic energy at that height  mgh  mgh  mgh
5 5

4 K.E. 1/ 5mgh 1
 At a height h,  
5 P.E. 4 / 5mgh 4
2. 3 Volume of blood pumped by man’s heart, V  5 litres  5  10 3 m3
Time in which this volume of blood pumps, t = 1 min = 60 s
Pressure at which the blood pumps, P = 0.15 m of Hg = 15 × 136 × 103 × 10 = 20.4 × 103 N/m2

PV  20.4  10  5 10   1.70 W


3 3

 Power of heart  
t 60 s

1 MM 1 10 15 10 2 3 3
Loss in KE  2 M  M  u1  u 2   
1 2 2
3. 1   2   1.875 kJ
 1 2 2 16 103

126
[Study Package - Physics - Class XI]

4. 2 Final velocity of the block of mass 4 m = v'


Initial velocity of block of mass 4 m = 0; Final velocity of block of mass m = 0
v
According to law of conservation of linear momentum, mv + 4m × 0 = 4mv' + 0; v 
4
Relative velocity of separation v / 4
Coefficient of restitution, e    0.25
Relative velocity of approach v
5. 3 From work energy theore, work done = change in P.E.
So, W = mgh; 0.75 × 12 = 1 × 10 × h;  h  0.9 m
1
From conservation of energy; KE = PE; mv2  mgh
2
v  2gh  2 10  0.9  18

6. 1 W  k ; Wh  Wg  k ; Since k  0 ; Wh  Wg  0

i.e., F.s1  mgs 2  0 ; F   0.2   0.2 10  2.2  0  sin ce s 2  s1  h  ;  F  22N

1
7. 4 Slope of inclined plane, sin   ; Component of weight down the inclined plane
100
1
F  mg  sin   100  9.8   9.8 N ; s = distance moved = 10 m; W  Fs  9.8  10  98 J
100
8. 4 Change in momentum = Force × time; P2  P1  F  t  0.2  10  2

 P2  2  P1  2  10  12 kg-m / s
1 1  2 2 44
Increase in K.E. 
2m
 P22  P12  
25 
12   10   
 10
 4.4J

9. 1 By the law of conservation of energy,

 1 2 1 2  1 1
i.e., K  U  0 ;  0  mv    kx  0   0 ; mv 2  kx 2
 2  2  2 2

 0.5 1.5  1.5 


x   ;   0.15 m
 50 
10. 1 Momentum of first part = 1 × 12 = 12 kg ms–1. Momentum of the second part = 2 × 8 = 16 kg
ms –1
2 2
 Resultant momentum  12   16   20 kg ms 1
Let the mass of the third part be M, then 4 × M = 20; M = 5 kg

127
Brilliant STUDY CENTRE

1
11. 3 Work done in stretching a spring W  kx 2
2
where k is the spring constant and x is the extension in the spring.

1 1
 W1  kx12 ...(1) and W2  kx 22 ...(ii)
2 2
2 2
W2  x 2   20 cm 
Divide (ii) by (i), we get     4
W1  x1   10 cm 

W2 = 4 W1
Extra work done = W 2 – W 1 = 4W 1 – W 1 = 3W 1 = 3 × 4 J = 12 J
( W1  4 J (Given))

90 1
12. 4 PE at A = mgh. Since 10% of this energy is lost, KE at point B  mgh   0.9 mgh  mv 2 .
100 2

or v 2  1.8 gh  1.8  10  2  36 ,  v = 6 ms–1.

13. 2 Here, m = 1000 kg, v = 18 km h–1 =  5 ms 1 ; k = 6.25 × 103 N m–1


At maximum compression xm, the kinetic energy of the car is converted entirely into the potential

1 1 m
energy of the spring.  mv 2  kx 2m or x m  v
2 2 k

1000 kg
Substituting the given values, we get x m   5ms 1  0.4  5m  2m
6.25 103 Nm 1
2
14. 1 Mass of water, m = volume × density  r 2l      2   15  1000 kg

h
30  15
 22.5 m ; W mgh   2   15  1000  9.8  22.5
P     11.55 kW
2 t t 3600

1
15. 3 K  mv 2 , where v  0  at
2

1 1
K m(at) 2  ma 2 t 2
2 2
As a = [LT–2] = constant

L  T 2

1
K  ma 2 . L or K  L or x
2

128
[Study Package - Physics - Class XI]

4  0.60
16. 3 Mass of length 2 m of the chain = 4 kg, Mass of 60 cm chain   1.2 kg
2
Weight of the hanging part of the chain = 1.2 × 10 = 12 N
Since the centre of gravity of the hanging part lies at its mid-point, i.e., 30 cm or 0.30 m
 W  12  0.30  3.6 J

v 2 Alv 2
17. 1 P  Fv  mav  V   Av3
t t
m V
Mass per unit length     ;  P  100  2 3  800 W
l l

 ˆ  dp
18. 3 Here, p  i2 cos t  ˆj2sin t ; F  2iˆ sin t  2ˆjcos t
dt


F.p  2iˆ sin t  2jcos
ˆ  
t . 2iˆ cos t  2ˆjsin t  4sin t cos t  4sin t cos t  0 ;   90o

9
2  3/ 2 9 38
19. 4 Work done   Fdx   k x dx  k x 4  k  12.7 k
4 3 3

1 1
mv22  mv12 2 2
m   u cos     u sin   gt   u 
2
20. 1 W 2 2  gt 
Pav       mg   u 0 sin  
t t 2 t  2 
2
1 2 1 2 1  v  mv 2T 2
21. 3 Work done = Change in KE  mv T  m  aT   m  T  
2 2 2 t  2t 2
22. 1 Workdone = Change in KE implies
2
1 u 1 3 1
F  30   m    mu 2   mu 2 ; and F  d   0  mu 2
2 2 2 8 2
Therefore d = 40 cm so additional thickness = 10 cm

23. 3

1 h
W  KE  mv 2A  mgh   mg cos 45o   mgh  mgh cot 45o  mgh  mgh
2 sin 45o
1 mgh
and mv 2B  mgh  mg cot 60o  mgh  ; Therefore v B  v A
2 3
129
Brilliant STUDY CENTRE

1 1
24. 2 W  KE  Wspring  Wfriction  mv 22  mv12
2 2

1 1 2
  10000  x 2  15  x  0   2  4 
2 2

 1000x 3  30x  16
 x  5.5 cm

25. 1 As p  2mK
where p = momentum and K = kinetic energy

p 2  p1 2mK 2  2mK1  K1  3K1   K1


100   100   100  100%
p1 2mK1 K1
26. 2 By using Work-Energy Theorem
W  KE
1
mgh  Wfriction  0  mu 2
2
1 1 2
 Loss in energy  mu 2  mgh  1  20   110 18   20 J
2 2

130
[Study Package - Physics - Class XI]

CHAPTER - 06
SYSTEMS OF PARTICLES AND ROTATIONAL MOTION

A rigid body is defined as a body in which the distance between the particles never changes, whatever
be the force applied on it or it may not deform under the influence of forces.
A rigid body can have translation motion and rotational motion and rotation and translation together. In
translation motion at any instant of time all particles of the body have the same velocity. In rotation motion the
particles of the body rotates about an axis called axis of rotation. In rotation of a rigid body, the particles of
the body moves in a circle, about an axis.
A. CENTRE OF MASS
Centre of Mass of a body or system is a point where the whole mass of the body or system is supposed
to be concentrated, and forces are directly applied to this point for translational motion.

For a system of particles of masses m1, m2, ..... mn whose position vectors are given by r1 , r2 , r3 ...........rn
respectively.

Y
m1
m2 n

xcm m r i i
r1 r2 m3 m r  m 2 r2  .......m n rn R cm  i 1
mn R cm  11 n
r3 m1  m 2  m3  ......m n ; m i
rn i 1
O X

1 n n
R   mi ri , when m i M
M i 1 i

1 n 1 n 1 n
In three dimensional form X  i i
M i 1
m x , Y   mi y i ;
M i1
Z  mi z i
M i 1
For a system of continuous distribution of mass, consider an elemental mass dm at a distance r from
the origin.
y
dm

O x
131
Brilliant STUDY CENTRE

1 1 1 1
R rdm ; Then X  x dm ; Y  ydm ; Z z dm
M M M M

n origin
m1 x m2
If the centre of mass is at the origin O, then m r  0 ,
i 1
i i
r1 O r2
r

Here the centre of mass is the point about which the sum of the moments of the mass of the system is
zero.

Consider a system of two particles of mass m1 & m2 separated by a distance r. The centre of mass
lies between the particles at distances r1 & r2 from m1 & m2 respectively. The distance of the centre of mass
is inversely proportional to the mass of the particles.

m1r1  m 2 r2 r1  r2
r when m1 = m2; r 
m1  m 2 2

r
If m1 = m2, r1 = r2 = ; ie, centre of mass lies at a point midway between the masses.
2

1 m2 m1
For a two particle system r  ; m1r1  m2 r2 ; r1  r ; r2  r
m m1  m 2 m1  m 2

Centre of mass of a uniform rod

For rod lying along the x-axis, and length x= L and mass M

dx
x
x=0 x=L

L
1 M L
The element is at (x, 0, 0) then x      dx 
M 0 L  2

For homogeneous bodies the center of mass coincides with the geometric centre.

The sum of the moments of masses of a system about the centre of mass is always zero.

The centre of mass of a body is the point at which its whole mass is assumed to be concentrated.
Centre of gravity of the body is the point at which whole of its weight is assumed to be concentrated.

 
 mi ri  mi g i ri
R CM  R CG 
 mi  mi g i

For a body in uniform gravitational field g is a constant, then, COM & COG coincides.

132
[Study Package - Physics - Class XI]

Uniform circular disc Solid hemisphere Hollow hemisphere


y y y

cm
cm
y x
R x x

3R R
4R x  0, y  x  0, y 
y 8 2
3

Triangule
Solid cone Hollow cone
y y y

h h l
cm cm h
cm

x x 0 x

For equilateral triangle


h h
x  0, y  x  0, y 
4 3 y
2 3

Motion of the centre of mass


Motion of the centre of mass can describe the motion of the body and the forces acting on the body are
directly applied to the centre of mass. For particles of mass m1, m2, ... mn having position vectors r1, r2,..., rn.

dr1 dr dr
m1  m 2 2  m3 3  ............
d dt dt dt
Velocity of centre of mass VCM   R CM  
dt m1  m 2  m3  ........

m1 v1  m 2 v2  m3 v3  ......

m1  m 2  m3  .....

 MVCM  m1v1  m 2 v 2  m3 v3  .......... ; since p = MV,,


     
MVCM  p is the linear momentum.  p  p1  p 2  p3  .........
 
If VCM  0, p  0 ie, in the frame of reference of the centre of mass, the momentum of a system is
zero. Due to this the centre of mass frame is called zero momentum frame.

dVCM 1    1 
Acceleration of the centre of mass a CM    m1a1  m 2 a 2  .......m n a n  =  mn a n
dt M M

133
Brilliant STUDY CENTRE

 
  MdVCM dp 
Consider the equation MVCM  p ; differentiating   Fext
dt dt
 
Fext is the external force applied. Ma CM  Fext
This equation is called the equation of motion of the centre of mass of a system of particles.
It shows that the centre of mass of a system of particles moves as though all the mass of the system
is concentrated at it and all the external forces were applied to it.
  
1) If Fext  0, a CM  0 , then VCM  a constant

ie, If no external forces are acting on a system, the velocity of the centre of mass remains a constant.
It shows that the velocity of the centre of mass is unaffected by the internal forces. So if the centre of mass
of a system is at rest it will remain at rest unless acted by an external force.
    
2) If Fext  0, a CM  0, VCM  a constant ; So that p  MVCM  a constant

We can see that equation of motion of centre of mass leads to conservation of linear momentum.
When no external forces are acting on a system of particles, the linear momentum remains conserved
which leads to Newtons third law of motion.
Centre of mass at rest
1) Explosion of a Bomb at rest. The explosion is due to internal forces, The centre of mass of the
Bomb remain at the same position, so the fragments will fly in all directions.
2) A Boat floating in a lake has net zero momentum even if the people on it changes the position.
3) Objects initially at rest, if moving under mutual forces (electrostatic, gravitational) have net zero
momentum.
4) A compressed spring by two masses on either side of the spring when released, the masses
acquire velocities in opposite direction, so that the net momentum is zero.
Centre of mass moving with uniform velocity
If Fext = 0, Vcm is a constant, so also the net momentum of the system remains constant. ie, the centre
of mass of the system continues to move with initial velocity.
Examples: 1) A bomb moving in a straight line with constant velocity when explodes the fragments move in
all directions. So that the velocity of centre of mass remains constant, and the momentum
remains conserved.
2) A man jumping on a cart exert internal forces, therefore the net momentum of the system and
hence the motion of the centre of mass remains conserved.

Linear displacement s
Angular displacement      , unit-radian.
Radius r


The angular displacement per unit time is defined as angular velocity,   Unit radian/sec
t

   2
Linear velocity v   r ,    2 [where T = time period,  = frequency]
T

134
[Study Package - Physics - Class XI]

The magnitude of angular velocity is called the angular speed which is also represented by  .
The rate of change of angular velocity is defined as angular acceleration.

If a particle has angular velocities 1 and 2 at time t1 and t2, then


 
 2  1
Angular acceleration   2
t 2  t1 Unit - rad/sec
 
 d d 2
Instantaneous angular acceleration   
dt dt 2
2  1
Average angular acceleration  av 
t 2  t1
  
Linear aceleration a    r
Angular acceleration is an axial vector whose direction is along the change in direction of angular
velocity ie, normal to the rotational plane, outward or inward along the axis of rotation.
Moment of a force about a point or Torque

Y F
F sin 
 F cos 
F
P
r 

r O X
O A
d N

Fig.(1) Fig.(2)
The moment of the force about a point is the product of the force and the perpendicular distance from
the point of the line of action of the force. Moment of force = F  OA

When the force makes an angle  with the rod. Moment of the force about O  F  ON
 
 r F sin   r  F.

The moment of a force about a point is also called torque     r  F 



Torque is an axial vector, ie, its direction is always perpendicular to the plane containing vector r and F
in accordance with right hand screw rule.

  r F sin  ; or   F  r sin    Fd [As d  r sin  ]

ie, Torque = Force × Perpendicular distance of line of action of force from the axis of rotation.
Torque is also called as moment of force and d is called moment of lever arm. Unit: newton-metre (SI)

If there are a number of forces acting on the body then,   1  2  3  ..... .

135
Brilliant STUDY CENTRE

Work done by a torque

If a force F acts on a body at a perpendicular distance r  r  from the axis of rotation, the work done by

the force in rotating the body through an angle  is given by W  F r     

dW d
Power =    , where  is the angular velocity..
dt dt
Angular momentum
The turning momentum of a particle about the axis of rotation is called the angular momentum of the
particle.

  
L  r  P  r P sin  nˆ
Angular momentum is an axial vector ie, always directed perpendicular to the plane of rotation and
along the axis of rotation. S.I. Unit : kgm 2 s 1 or J sec
    
In case of circular motion, L  r  P  m  r  v   mvr sin 

 L  mvr  mr 2  (as r  v )
 
In vector form L  I ( I- M.I.)
  
  dL d    d   
From L  I  I  I    As dt   and   I 
dt dt  
ie, the rate of change of angular momentum is equal to the net torque acting on the particle. [Rotational
analogue of Newton’s second law]
The angular momentum of a system of particles is equal to the vector sum of angular momentum of
    
each particle ie, L  L1  L2  L3  ...........  Ln .

Law of Conservation of Angular Momentum



 dL
Newton’s second law for rotational motion   .
dt

dL
If the net external torque on a particle (or system) is zero then 0
dt
   
ie, L  L1  L 2  L3  ....... = constant

136
[Study Package - Physics - Class XI]

Angular momentum of a system remains constant if resultant torque acting on it zero.

  I
1
As L  I so if   0 then I = constant

Angular Impulse

dp
Impulse I  Fdt 
  dt dt ; dp = change in momentum.

dL
In rotational motion angular impulse J is, J   dt   dt  dL = change in angular momentum.
dt
Parallel forces
If the lines of action of a set of forces are parallel, they are called parallel forces. If the parallel forces
are in the same plane, they are called coplanar parallel forces. If all the forces are in the same direction,
they are called like parallel forces. If they are in opposite directions, they are called unlike parallel forces.
condition for equilibrium of a set of coplanar parallel forces

P Q

C A D B E O

R S W

Two conditions are to be satisfied.


1. Translational equilibrium 2. Rotational equilibrium
1. Translational equilibrium
For translational equilibrium, the algebraic sum of the forces acting on the body =0
i.e.; P+Q-R-S-W = 0, P + Q = R + S + W
The total force acting along a direction= the total force acting in the opposite direction. If this condition
is satisfied, the body is either at rest or moving with uniform linear velocity.
2. Rotational equilibrium
For rotational equilibrium, the algebraic sum of the moments of forces about any point is equal to zero .
About O,
P  OA + Q  OB = R  OC + S  OD + W  OE.
Total anticlock wise moments = Total clock wise moments. About D,
Q  DB + R  DC = P  DA + W  DE.
If this condition is satisfied, the body is either not rotating or rotating with constant angular velocity.
A body is said to be in rotational equilibrium if resultant torque acting on it is zero ie,   0 .

In case of beam balance or see-saw the system will be in rotational equilibrium if,
 
1  2  0 or F1l1  F2 l2  0  F1l1  F2l2

137
Brilliant STUDY CENTRE

   
However if, 1  2 , L.H.S. will move downwards and if 1  2 . R.H.S. will move downward and the
system will not be in rotational equilibrium.

Couple

F
A B
F O

Two equal unlike parallel forces acting at different points in a body constitute a couple.
Consider a body in which two equal forces (F) acting at points A and B in opposite directions. Let O be
a point in between A and B.
Moment of the couple about O= OB  F+OA  F = F(OA+OB) = F  AB

Moment of the couple is called torque  , Therefore   F  AB

Thus torque is the product of one of the forces and the perpendicular distance.
Generally both couple and torque carry equal meaning. The basic difference between torque and couple is
that in case of couple both the forces are externally applied while in case of torque one force is externally
applied and the other is reactionary.

1 2
Work done by torque in twisting the wire W  C
2
Where   C ; C is known as twisting coeffcient or couple per unit twist.
Centre of gravity
The centre of gravity is the point where the total weight of the body acts. The centre of mass is the
location where the entire mass is concentrated. For every orientation of the body this is where the gravitational
force of the body acts. In uniform gravitational field the centre of mass is identical to centre of gravity, but
they may not coincide always. In a symmetrical object the centre of gravity coincide with the geometric
centre.
The centre of gravity of a body is defined as the point where the gravitational torque on the body is zero.

i.e., I g  i  ri  m i g  0

138
[Study Package - Physics - Class XI]

g  0,  mi ri  0 , ri is the position vector of the ith particle w.r. to centre gravity..
The position of centre of gravity.
The position of centre of gravity of an object affects its stability. The lower the centre of gravity, the more
stable the object. It is higher, the object is more likely to topple.
Equations of Rotational motion
If angular acceleration is O, then  = const. and   t
If angular acceleration =  = constant, then

1  2   1
 t;   2 ; 2  1  t
2 t

1 
  1t  t 2 ; 22  12  2 ; mth  1  2  n  1
2 2
If  is not constant, then

d d d 2
 ;   ;  d    d
dt dt dt 2
Concept of moment of inertia
According to Newton’s Ist law in rotational motion, there is an inability of a body to change by itself its
state of rest or of uniform rotational motion. This inability is called rotational inertia or moment of inertia.
Moment of Inertia
Moment of inertia plays the same role in rotational motion as mass plays in linear motion. It is the
property of a body due to which it opposes any change in its state of rest or of uniform motion.
(1) Moment of inertia of a particle I = mr2; where r is the perpendicular distance of the particle from the
rotational axis.

(2) Moment of inertia of a body made up of a number of particles, I  m1r12  m 2 r22  m 3 r32  ......
(3) Moment of inertia of a continuous distribution of mass, treating the element of mass dm at position r as
particle. dI  dm r 2 ie, I   r 2 dm

Unit : kg m2
Radius of Gyration
Radius of gyration of a body about a given axis is the perpendicular distance of a point from the axis,
where if whole mass of the body were concentrated, the body shall have the same moment of inertia as it
has with the actual distribution of mass.

139
Brilliant STUDY CENTRE

When square of radius of gyration is multiplied with the mass of the body, it gives the moment of inertia
of the body about the given axis.

I
I  Mk 2 or k  -----(13); Here k is called radius of gyration.
M

From the formula of discrete distribution,

I  mr12  mr22  mr32  ...........  mrn2

2 2 2
 2
Radius of gyration, k, then I  Mk  m r1  r2  ...........  rn 
nmk 2  m  r12  r22  r32  .............  rn2  (As M = nm)

r12  r22  r32  .........  rn2


 k
n
Hence radius of gyration of a body about a given axis is equal to root mean square distance of the
constituent particles of the body from the given axis.
Through in this concept a real body (particularly irregular) is replaced by a point mass for dealing its rotational
motion.
Example: In case of a disc rotating about an axis through its centre of mass and perpendicular to its
plane,

k
I

1/ 2  MR 2 
R
M M 2

 
So instead of disc we can assume a point mass M at a distance R / 2 from the axis of rotation for
dealing the rotational motion of the disc.
Theorem of Parallel Axes
Moment of inertia of a body about a given axis I is equal to the sum of moment of inertia of the body
about an axis parallel to given axis and passing through centre of mass of the body, Ig and Ma2 where M is the
mass of the body and a is the perpendicular distance between the two axes.

I  Ig  Ma 2

140
[Study Package - Physics - Class XI]

1 3
I  Ig  Ma 2  MR 2  MR 2  M R 2
2 2
Theorem of perpendicular axes

According to this theorem the sum of the moment of inertia of a plane lamina about two mutually
perpendicular axes lying in its plane is equal to its moment of inertia about an axis perpendicular to the plane
of lamina and passing through the point of intersection of first two axes.
Iz = Ix + Iy --------- (18)

Kinetic Energy and Power for Rotating Body

Work: If the body is initially at rest and angular displacement is d due to torque then work done on the body..

W    d

Analogue to work in translatory motion W  F dx 


Kinetic Energy: The energy, which a body has by virtue of its rotational motion is called rotational kinetic
energy. A body rotating about a fixed axis possesses kinetic energy because its constituent particles are in
motion, even though the body remains in place.

Analogue to Translatory
Rotational Kinetic Energy
Kinetic Energy
1 2 1
KR  I KT  mv2
2 2
1 1
K R  L K T  Pv
2 2
L2 P2
KR  KT 
2I 2m

141
Brilliant STUDY CENTRE

Power: Rate of change of kinetic energy is defined as power.

d d 1 d
P  K R    I2   I  I  1  
dt dt  2  dt
   
In vector form Power, P   .  [Analogue to power in translatory motion P  F. v ]

Slipping: When the body slides on a surface without rotation then its motion is called slipping motion.

1
Body posess only translatory kinetic energy K T  mv 2 .
2
Here, friction between the body and surface F = 0.

Rolling: If in case of rotational motion of a body about a fixed axis, the axis of rotation also moves, the
motion is called combined translatory and rotatory.

Body possess both translational and rotational kinetic energy.


Net kinetic energy = (Translatory + Rotatory) kinetic energy.

1 1 1 1 K2
K N  KT  KR  mv 2  I2  mv 2  mv 2 2
2 2 2 2 R

1  K2 
 KN  mv 2  1  2 
2  R 

Here, condition of force between the body and surface F  0


Spinning: When the body rotates in such a manner that its axis of rotation does not move then its motion is
called spinning motion. Body possess only rotatory kinetic energy.

142
[Study Package - Physics - Class XI]

2 2
1 2 1 2K  K 1 5
K R  I  mv  2  ;  1 for ring  for disc and  for solid sphere
2 2 R  R 2 2
Rolling without slipping
Rolling without slipping is when an object rolls across and there is no relative motion of object and
surface, at the point of contact.
It may be treated as a pure rotation about an axis passing through point of contact with same angular
velocity  .

1 1 1 1
By the law of conservation of energy: K N  mv 2  I2  mR 2 2  I2
2 2 2 2

1 2 2 1
  I  mR 2   I p 2 [As I P  I  mR 2 ]
2 2
where I is moment of inertia of rolling body about its centre ‘O’ and IP is moment of inertia of rolling body
about point of contact ‘P’.
Linear velocity of different points in rolling: In case of rolling, all points of a rigid body have same
angular speed but different linear speed.
Example: Let A, B, C and D are four points then their velocities are shown in the following figure.

For pure rolling vA = 0; v  R  0 ; v  R

v B  v  R  2v
* If v > Rw, the body undergoes forward slipping.
* If v < Rw the body undergoes backward supply.
Acceleration of a point is rolling motion.

Translation Rotation Rolling

2R

vcm = const. + =
2R 2R
2
R

143
Brilliant STUDY CENTRE

Velocity of a part is Rolling wheel.

P
vtranslation = vcm
 vrotation = R
R

2 2 2
v p  v t  v r  v cm  v cm  2vcm cos   2 vcm 1  cos 


v p  2vcm cos  
2
Rolling on an inclined plane

k rolling  k t   k R  cm

1 1 1 1 v2
k rolling  mv 2cm  Icm 2  mv 2cm  mk 2 cm2
2 2 2 2 R

1 2  k2  k2 kt 1
k rolling  mvcm 1  let   1  
2  R 2  ;
R2
; k rolling  ; k rolling  k t

A body of mass m and radius R rolls down an inclined plane of height ‘h’ at an angle of inclination  ,
loses potential energy and acquires both linear and angular speeds.
Hence, gains kinetic energy of translation and that of rotation.

1 2 k2 
By conservation of mechanical energy: mgh  mv  1  2 
2  R 

2gh 2gh
(1) Velocity at the lowest piont: v  2

k 
1 2
R

g sin  g sin 
(2) Acceleration in motion: a  
k2 
1 2
R

1 2h  k 2  1 2h
(3) Time of descent: t  1 2  
sin  g  R  sin  g

1 1 k2
Therefore v  ; a ; t  1 
k2 k2 R2
1 1 2
R2 R

144
[Study Package - Physics - Class XI]

QUESTIONS
LEVEL - I

1. Two bodies of mass 1 kg and 3 kg have position vectors ˆi  2ˆj  kˆ and 3iˆ  ˆj  2kˆ , respectively. The
centre of mass of this system has a position vector.

1) 2iˆ  ˆj  kˆ 2) 2iˆ  ˆj  2kˆ 3) ˆi  ˆj  kˆ 4) 2iˆ  2kˆ

2. Three identical spheres, each of mass 1 kg are kept as shown in the fig. touching each other, with
their centres on a straight line. If their centres are marked P, Q, R respectively, the distance of centre
of mass of the system from P is :

PQ  PR  QR PQ  PR
1) 2)
3 3

PQ  QR PQ  QR  PR
3) 4)
3 3
3. A ‘T’ shaped object with dimensions shown in figure is lying on a smooth floor. A force ‘F’ is applied at
the point P parallel to AB, such that the object has only the translational motion without rotation. Find
the location of P with respect to C.

4 3 2
1) l 2) l 3) l 4) l
3 2 3
4. Two particles which are initially at rest, move towards each other under the action of their internal
attraction. If their speeds are v and 2v at any instant, then the speed of centre of mass of the system
will be
1) 2 v 2) zero 3) 1.5 v 4) v
145
Brilliant STUDY CENTRE

5. A thin circular ring of mass M and radius r is rotating about its axis with a constant angular velocity  .
Four objects each of mass m, are kept gently to the opposite ends of two perpendicular diameters of
the ring. The angular velocity of the ring will be

M M  M  4m    M  4m  
1) 2) 3) 4)
4m M  4m M M  4m
6. A rope is wound around a hollow cylinder of mass M = 3 kg and radius R = 40 cm. If the rope is pulled
with a force F = 30 N, find the angular acceleration of the cylinder.
1) 10 m/s2 2) 25 rad/s2 3) 10 rad/s2 4) 25 m/s2
7. A circular platform is mounted on a frictionless vertical axle. Its radius R = 2 m and its moment of inertia
about the axle is 200 kg m2. It is initially at rest. A 50 kg man stands on the edge of the platform and
begins to walk along the edge at the speed of 1 ms–1 relative to the ground. Time taken by the man to
complete one revolution is

3 
1)  s 2) s 3) 2 s 4) s
2 2
8. A uniform rod AB of length  and mass m is free to rotate about point A. The rod is released from rest

m 2
in the horizontal position. Given that the moment of inetia of the rod about A is , the initial angular
3
acceleration of the rod will be

mg 3 3g 2g
1) 2) g 3) 4)
2 2 2 3
9. A ring starts from rest and acquires an angular speed 10 rads–1 in 2s. The mass of the ring is 500 gm
and its radius is 20 cm. The torque on the ring is
1) 0.02 Nm 2) 0.20 Nm 3) 0.10 Nm 4) 0.01 Nm

10. A round disc of moment of inertia I2 about its axis perpendicular to its plane and passing through its
entre is placed over another disc of moment of inertia I1 rotating with an angular velocity  about the
same axis. The final angular velocity of the combination of discs is

I2 I1  I1  I2  
1) 2)  3) 4)
I1  I2 I1  I2 I1
11. Two bodies have their moments of inertia I and 2I respectively about their axis of rotation. If their
kinetic energies of rotation are equal, their angular momenta will be in the ratio
1) 2 : 1 2) 1 : 2 3) 2 :1 4) 1: 2

146
[Study Package - Physics - Class XI]

12. A person sitting firmly over a rotating stool has his arms stretched. If he folds his arms, his angular
momentum about the axis of rotation
1) Increases 2) Decreases 3) Remains unchanged 4) Doubles
13. Consider a body shown in figure, consisting of two identical balls, each of mass M connected by a
light rigid rod. If an impulse J = MV is imparted to the body at one of its ends, what would be its angular
velocity?

L
M M
J=MV

V 2V V V
1) 2) 3) 4)
L L 3L 4L
14. (a) Centre of gravity (C.G.) of a body is the point at which the weight of the body acts.
(b) Centre of mass coincides with the centre of gravity if the earth is assumed to have infinitely large
radius
(c) To evaluate the gravitational field intensity due to any body at an external point, the entire mass of
the body can be considered to be concentrated at its C.G.
(d) The radius of gyration of any body rotating about an axis is the length of the perpendicular dropped
from the C.G. of the body to the axis
Which one of the following pairs of statements is correct?
1) (d) and (a) 2) (a) and (b) 3) (b) and (c) 4) (c) and (d)

15. Four solid spheres each of diameter 5 cm and mass 0.5 kg are placed with their centres at the
corners of a square of side 4 cm. The moment of inertia of hte systme about the diagonal of the
square is

1) 3  104 kg m 2 2) 6  104 kg m 2 3) 9  104 kg m 2 4) 4  10 4 kg m 2

16. A thin uniform rod XY of length 2 is hinged at one end X to the floor and stands vertical. When
allowed to fall, the angular speed with which the rod strikes the floor is

g 2g 3g g
1) 2) 3) 4)
 3 2 3
17. The ratio fo the radii of gyration of a circular disc about a tangential axis in the plane of the disc and of
a circular ring of the same radius about a tangential axis in the plane of the ring is

1) 2 : 3 2) 2 : 1 3) 5: 6 4) 1: 2

18. A small object of a uniform density rolls up a curved surface with an initial velocity v. It reaches upto a
3v 2
maximum height of with respect to the initial position. The object is [NEET 2013]
4g
1) hollow sphere 2) disc 3) ring 4) solid sphere

147
Brilliant STUDY CENTRE

19. A solid sphere is rolling on a frictionless surface as shown in figure, with translational velocity v m/s.
If it is to climb the inclined surface, then v should be

v h

10 10
1)  gh 2)  2gh 3) 2gh 4) gh
7 7
20. A circular disc is rolling down an inclined plane without slipping. If the angle of inclination is 300, the
acceleration of the disc down the incline plane is

g g 2
1) g 2) 3) 4) g
2 3 3
21. A solid cylinder and a hollow cylinder, both of the same mass and same external diameter are released
from the same height at the same time on an inclined plane. Both roll down without slipping. Which
one will reach the bottom first?
1) Both together only when angle of inclination of plane is 45o
2) Both together
3) Hollow cylinder
4) Solid cylinder
22. A solid cylinder of mass M and radius R rolls without slipping down an inclined plane of length L and
height h. What is the speed of its centre of mass when the cylinder reaches its bottom?

3 4
1) 2gh 2) gh 3) gh 4) 4gh
4 3
23. A solid cylinder of mass 3 kg is rolling on a horizontal surface with velocity 4 ms –1. It collides with a
horizontal spring of force constant 200 Nm–1. The maximum compression produced in the spring will
be
1) 0.5 m 2) 0.6 m 3) 0.7 m 4) 0.2 m
LEVEL - II
1. Three bricks each of length L and mass M are arranged as shown from the wall. The distance of the
centre of mass of the system from the wall is [AIIMS 2018]

L/4
L/2
L

L L 3  11 
1) 2) 3)   L 4)  L
4 2 2  12 

148
[Study Package - Physics - Class XI]

2. From a thin uniform square lamina of side ‘a’ a square of side a/2 is removed from its corner as
shown in figure. Find the centre of mass of the remaining portion (shown shaded) of the lamina.

a a a a
1) 2) 3) 4)
2 4 6 12
3. A car of mass 1000 kg is moving with a velocity of 10 ms–1 towards another car of mass 1500 kg
moving with a velocity of 15 ms–1 in the same direction. Find the velocity of the centre of mass of the
two cars
1) 15 m/s 2) 13 m/s 3) 17 m/s 4) 11 m/s
o
4. A cracker is thrown into air with a velocity of 10 m/s at an angle of 45 with the vertical. When it is at
a height of (1/2)m from the ground, it explodes into a number of pieces which follow different parabolic
paths. What is the velocity of centre of mass, when it is at a height of 1 m from the ground ? (g = 10
ms –2 )

1) 4 5 ms 1 2) 2 5 ms 1 3) 5 4 ms 1 4) 10ms 1
5. A wheel has angular acceleration of 3.0 rad/sec2 and an initial angular speed of 2.00 rad/sec. In a time
of 2 sec it has rotated through an angle (in radian) of
1) 10 2) 12 3) 4 4) 6
  
6. If F is force acting on a particle having position vector r and  -be the torque of this force about the
origin, then
     
1) r.   0 and F.   0 2) r.   0 and F.   0
     
3) r.   0 and F.   0 4) r.   0 and F.   0
7. The instantaneous angular position of a point on a rotating wheel is given by the equation
(t)  2t 3  6t 2 . The torque on the wheel becomes zero at
1) t =1 s 2) t = 0.5 s 3) t = 0.25 s 4) t = 2s

8.  
A force F  2iˆ  3jˆ newton acts on a particle whose position vector with respect to origin O is

 
r  4iˆ  5jˆ metre. Find the magnitude and direction of the torque

1) 22 Nm iˆ 2) 22 Nm ˆj 3) 22 Nm kˆ 4) 2 Nm kˆ

149
Brilliant STUDY CENTRE

9. An automobile moves on a road with a speed of 54 km h–1. The radius of its wheel is 0.45 m and the
moment of inertia of the wheel about its axis of rotation is 3 kg m2. If the vehicle is brought to rest in
15 s, the magnitude of average torque transmitted by its brakes to the wheel is

1) 10.86 kg m 2s 2 2) 2.86 kg m 2s 2 3) 6.66 kg m 2 s 2 4) 8.58 kg m 2s 2

10. A mass is whirled in a circular path with constant angular velocity and its angular momentum is L. If
the string is now halved keeping the angular velocity the same, the angular momentum is

L L
1) 2) 3) L 4) 2L
4 2
11. A particle of mass m moves along line PC with velocity  as shown in Fig. What is the angular
momentum of the particle about P?

P L

r
l
O

1) m  l 2) m  L 3) Zero 4) m  r
12. A particle of mass m moves in the XY plane with a velocity v along the straight line AB. If the angular
momentum of the particle with respect to origin O is L A when it is at A and LB when it is at B, then

1) LA > LB
2) LA = LB
3) the relationship between LA and LB depends upon the slope of the line AB
4) LA < LB
13. If I1 is the moment of inertia of a thin rod about an axis perpendicular to its length and passing through
the centre of mass and I2 is is the moment of inertia of the ring formed by bending this rod in the form
I1
of a ring, then
I 2 is

1) 1 : 1 2) 2 : 3 3)  : 4 3) 3 : 5

150
[Study Package - Physics - Class XI]

14. Two rings of radius R and nR made up of same material have the ratio of moment of inertia about an
axis passing through centre as 1 : 8. The value of n is

1
1) 2 2) 2 2 3) 3 4)
2
15. Four identical thin rods each of mass M and length  , form a square frame. Moment of inertia of this
frame about an axis through the centre of the square and perpendicular to its plane is

2 13 1 4
1) M 2 2) M 2 3) M 2 4) M 2
3 3 3 3

R
16. From a circular disc of radius R and mass 9 M, a smal disc of mass M and radius is removed
3
concentrically. The moment of inertia of the remaining disc about an axis perpendicular to the plane
of the disc and passing through its centre is

40 4
1) MR 2 2) MR2 3) 4 MR2 4) MR 2
9 9
17. A solid cylinder and a disc of same radii are allowed to roll down a smooth inclined plane from the top
of a plane. The ratio of their times taken to reach the bottom of the inclined plane is
1) 1 : 1 2) 2 : 3 3) 2 : 7 4) 1 : 2
18. Point masses m1 and m2 are placed at the opposite ends of a rigid rod of length L, and negligible
mass. The rod is to be set rotating about an axis perpendicular to it. The position of point P on this rod
through which the axis should pass so that the work required to set the rod rotating with angular
velocity 0 i s minimum, is given by

m2 m2L
1) x  L 2) x 
m1 m1  m 2

m1L m1
3) x  4) x  L
m1  m2 m2

19. Three objects, A : (a solid sphere), B : (a thin circular disk) and C : (a circular ring), each have the
same mass M and radius R. They all spin with the same angular speed  about their own symmetry
axes. The amounts of work (W) required to bring them to rest, would satisfy the relation
1) W C > W B > W A 2) W A > W B > W C 3) W B > W A > W C 4) W A > W C > W B

151
Brilliant STUDY CENTRE

20. A ball rolls without slipping. The radius of gyration of the ball about an axis passing through its centre
of mass is K. If radius of the ball be R, then the fraction of total energy associated with its rotational
energy will be

K2  R 2 K2 K2 R2
1) 2) 3) 4)
R2 R2 K2  R 2 K2  R 2
21. A wheel of mass 5 kg and radius 0.40 m is rolling on a road without sliding with angular velocity 10 rad
s–1. The moment of inertia of the wheel about the axis of rotation is 0.65 kg m2. The percentage of
kinetic energy of rotation in the total kinetic energy of the wheel is
1) 22.4 % 2) 11.2% 3) 88.8% 4) 44.8%
22. A solid sphere rolls down from the top of an inclined plane. Its velocity on reaching the bottom of the
plane is v. When the same sphere slides down from the top of the plane, its velocity on reaching the
bottom is v'. The ratio v'/v is

3 7 3
1) 2) 1 3) 4)
5 5 5

152
[Study Package - Physics - Class XI]

KEY WITH HINTS


LEVEL - I

1. 1 r1 for M1  ˆi  2jˆ  kˆ for M1  1 kg ; r2 for M 2  3iˆ  2ˆj  kˆ for M 2  3 kg

mi ri
rC.M. 
m

 rC.M. 
1iˆ  2ˆj  1kˆ  1   3iˆ  2ˆj  kˆ   3 ;  rC.M. 
8iˆ  4ˆj  4kˆ
 2iˆ  ˆj  kˆ
4 4
2. 2 As is clear from symmetry of Fig., centre of mass of the system is at Q. Its distance from P is

m  0  m  PQ  m  PR PQ  PR
PQ (x)  ; x
3m 3
3. 2 The object will have translational motion without rotation, when force F is applied at the centre
of mass of the system. If m is mass per unit length, then mass of AB, m1 = ml at O and mass
of OC, m2  m  2l  at D where CD = l.

Let DP = x ; As P is the centre of mass, therefore,

l 4l
ml  l  x   2mlx; 3x  l; x  l / 3;  CP  CD  DP  l  
3 3
4. 2 As no external force is acting on the system, the centre of mass must be at rest i..e, vCM = 0

5. 2 According to law of conservation of angular momentum I11  I2 2

As I1  MK 2 , 1   and I 2   M  4m  K 2

2 2 M
 MK     M  4m  K 2  2 
M  4m
6. 2 For a hollow cylinder I = MR2 ; Torque on cylinder is   FR  I  FR.

FR FR F 30
Therefore a   2
   25 rad s 2
I MR MR 3  0.4

153
Brilliant STUDY CENTRE

7. 3 Li = 0, also by conservation Lf = 0
 Angular momentum = Angular momentum of platform and man in opposite direction.
mvR 50  1 2 1
i.e., mvR  I or     rad s 1
I 200 2

v 1 1
Angular velocity of man relative to platform is r       1 rad s 1
R 2 2

2 2
Time taken by the man to complete one revolution is T    2 s
r 1

 mg
8. 3 Torque about A,   mg   ; Also   I
2 2

 mg / 2 3 g A
 Angular acceleration,    
I m 2 / 3 2 

 10
9. 3    5 rad / s 2 ;   I  MR 2   0.500   0.20  2  5  0.10 Nm
t 2

10. 3 From conservation of angular momentum, I11   I1  I 2  2

I11 I1
 2  
 I1  I2   I1  I 2 
1 2
11. 4 Kinetic energy of rotation K  I ; Angular momentum L  I
2

L2 L1 2I1K1 I I 1
 K ; Given : K1  K 2 ; I1  I, I2  2I ;    1  
2I L2 2I 2 K 2 I2 2I 2
12. 3 As there is no torque applied to the system, so angular momentum about the axis of rotation is
conserved.
13. 1 Let  be the angular velocity of the rod about the centre of mass C. As angular impulse =
change in angular momentum about the centre of mass of the system, therefore,

M M
C
L/2 L/2 J=MV

L L  ML2  V
J.  I c  ; MV    2  ; 
2 2  4  L
14. 1

154
[Study Package - Physics - Class XI]

5
15. 3 M = 0.5 kg; R   102 m ;
2
a = 4 × 10–2 m

Moment of inertia of the system about the diagonal of the square is

2 2
2 2
2 2  a   2 2
2 2  a   8 2 2
I  MR   MR  M     MR   MR  M     MR  Ma
5 5  2  5 5  2  5

2
8  5 
2 4
   0.5     0.5  4  10  1  8  104 kgm 2  9  10 4 kg m 2
5  2  

16. 3 The moment of inertia of the rod about a horizontal axis passing through point X is

1 2 4
Ix  m  2   m 2  m 2 ------ (i)
12 3

Now, since centre of gravity falls through a height  , the change in gravitational potential energy
1
of the rod is mg . According to the conservation of energy mg  I x 2
2

2mg 2mg 6g 3 g 3g
2     or  
Ix 4 2 4 2  2
 m 
3 

5
17. 3 The radius of gyration of disc about a tangential axis in the plane of the disc K1  R
2

3
The radius of gyration of circular ring of same radius about tangential axis K 2  R
2

K1 5 2 5
 R 
K2 2 3R 6

155
Brilliant STUDY CENTRE

3v 2
18. 2 The kinetic energy of the rolling object is converted into potential energy at height h 
4g

; So, by the law of conservation of mechanical energy, we have

2
1 1 1 1 v  3v 2   v
   
2
Mv 2  I2  Mgh ; Mv  I    Mg  ;
2 2 2 2 R  4g   R

1 v2 3 1 1 v2 1 1
I 2  Mv 2  Mv 2 ; I 2  Mv 2 or I  MR 2 .
2 R 4 2 2 R 4 2
Hence the object is disc.

1 1
19. 1 For solid sphere, total energy E  E t  E r  mv2  I2
2 2

1 12  7
E mv 2   mr 2  2  mv 2 ;
2 2 5  10

7 10
To climb up, E  mv 2  mgh ; v  gh
10 7

g sin 
20. 3 The acceleration down the plane is given by a 
 I 
1  2 
 MR 

1
For disc, I  MR 2 . Using this and   300 , we get a = g/3
2

 K2 
2  1  2 
21. 4 Time taken to reach the bottom of inclined plane t   R .
g sin 

2 R2
Here,  is length of incline plane. For solid cylinder K 
2

For hollow cylinder = K 2  R 2

Hence, solid cylinder will reach the bottom first.

156
[Study Package - Physics - Class XI]

1 1 1 v
22. 3 KE of cm when reached bottom  mv2  I2 ; I  MK 2 and  
2 2 2 R

1 1 v2 1  K2 
 K.E.  mv 2  MK 2 . 2  mv 2 1  2 
2 2 r 2  R 

L1 2IK1 1 I I 1
But for solid cylinder, L   1  ;
2 2IK
2 2
I2 2I 2

3
 K.E.  mv 2 ----- (i)
4
P.E. of the solid cylinder at height h is P.E. = mgh ------ (ii)

3 2 4
From (i) and (ii), mgh  mv  v  gh
4 3
23. 2 At maximum compression, the solid cylinder will stop. According to law of conservation of
mechanical energy, loss in kinetic energy of cylinder = gain in potential energy of spring
2
1 2 1  MR 2   v  1 2 1
Mv       kx ( v  R and for solid cylinder, I  MR 2 )
2 2 2  R  2 2

1 1 1
Mv 2  Mv 2  kx 2 ;
2 4 2
3 1 3 Mv 2
Mv 2  kx 2 or x 2 
4 2 2 k
M = 3 kg, v = 4 ms–1, k = 200 N m–1
3 3  4  4 36
x2   or x  0.6 m
2  200 100
LEVEL - II
L L L L L L 5
1. 4 From figure, x1  , x 2    L and x 3     L
2 2 2 2 4 2 4

L 5L 11
M  ML M ML
m x  m 2 x 2  m3 x 3 2 4  4 11
 X CM  1 1   L
m1  m 2  m3 MMM 3M 12

157
Brilliant STUDY CENTRE

M
2. 4 Let M be the mass of the complete lamina. Mass of the removed portion is m  . The centre
4
of mass of the complete lamina is taken to be at origin O (x = 0, y = 0). The coordinates of the
a a
centre of mass O' of the removed portion are x   and y   .
4 4
The x and y coordinates of the centre of mass of the remaining portion are

M a M a
M0   M  0   
Mx  mx 4 4  a My  my 4 4  a
x CM   y CM  
Mm M 12 and Mm M 12
M M
4 4

m11  m2 2 1000  10  1500  15


3. 2 CM   13 ms 1
m1  m2 1000
4. 1 Motion of the centre of mass is exactly similar to that of translatory motion of a body that is
thrown into air.

10 10 10
u x  u cos   m / s; u y  u sin   m / s ; ux  m/s
2 2 2

2 2 100
v 2y  u 2y  2  g  h ; v y  v y  2gh   2  10  1  30
2

2 2 100
 Net velocity of CM  v x  v y   30  80  4 5 m / s
2

5. 1   3 rad / sec 2 ; i  2 rad / sec

1 2
Time t = 2 sec;   i t  t
2

1
   2  2   3  4  4  6  10 radian
2

6. 2 Torque is always perpendicualr to F as well as r .
  
 r .   0 as well as F.   0

d d2 
7. 1 Given:   t   2t  6t ; 
3 2  6t 2  12t ;  12t  12  
dt dt 2

When angular acceleration (  ) is zero, then the torque on the wheel becomes zero.    I 

 12t  12  0 or t  1 s

158
[Study Package - Physics - Class XI]

8. 3    
  r  F  4iˆ  5jˆ  2iˆ  3jˆ  4iˆ  2iˆ  12iˆ  ˆj  10ˆj ˆi  15ˆj  ˆj  0  12 kˆ  10kˆ  0  22 kˆ Nm

The magnitude of torque is 22 Nm and its direction is along the positive z-axis

9. 3 v  54 kmh 1  15 ms 1 ; R = 0.45 m; I = 3 kg m2; t = 15 s

v 15 1500 100
i    rad s 1  rad s 1 ;
R 0.45 45 3

f  0 (as the vehicle comes to rest)

 The angular retardation of the wheel is

100
0
f  i 3   100 rad s 2
 
t 15s 45

 100  20
The magnitude of required torque is   I |  |   3k    kgm 2s 2  6.66 kg m 2s 2
 45  3

10. 1 L  mr 2  . for given m and , L  r 2 . If r is halved, the angular momentum L becomes one-
fourth.
11. 3 As P lies on the line along which the particle moves, angular momentum of the particle about P
is L  m r  m   0  0
   
12. 2 From the definition of angular momentum,  
L  r  p  rmv sin   k

Therefore, the magnitude of L is L  mvr sin   mvd


where d  r sin  is the distance of closest approach of the particle to the origin. As d is the
same for both the cases, hence LA = LB.
13. 2 Let M be the mass and L be the length of the rod.
2
1 L  L  I1  2
 I1  ML2 ;  L  2r or r  ;
2
 I2  Mr  M   ; 
12 2  2  I2 3

159
Brilliant STUDY CENTRE

14. 1 The moment of inertia of circular ring whose axis of rotation is passing through its centre is,
2
I = mR2;  I1  m1R 2 and I2  m 2  nR 

m2 m1
Since, both have same density; 
2  nR   A 2R  A

I1 m1R 2 m1R 2 1
 m 2  nm1 ;   2
 2
 3
I 2 m 2  nR  m1n  nR  n

I1 1 1 1
   Given    or n  2
I2 8 8 n3
15. 4 Moment of inertia for the rod AB rotating about an axis through the mid-point of AB perpendicular
M 2
to the plane of the paper is
12

A  B


D C

2   2 2  M 2
 Through the axis through the centre parallel to this axis, I  I0  Md  M   
 12 4  3

4
For all the four rods, I  M 2
3
16. 1 Mass of the disc = 9 M; Mass of removed portion of disc = M
The moment of inertia of the complete disc about an axis passing through its centre O and
9
perpendicular to its plane is I1  MR 2
2

O R/3

2
1 R 1 2
Now, the moment of inertia of the disc with removed portion I2  M    MR
2  3  18
40mR 2
Therefore, MI of the remaining portion of disc  I : I1  I2 
9

160
[Study Package - Physics - Class XI]

17. 1 Time taken by the rolling body to reach the bottom of the inclined plane is

1 2h  k 2 
t 1  
sin  g  R 2  ; where h is the height and  be angle of inclination of the inclined plane.

k2 1 1 3h
For solid cylinder, 2   t cylinder 
R 2 sin  g

k2 1 1 3h t cylinder
For disc,  ;  t disc   1
R2 2 sin  g t disc
18. 2 Moment of inertia of the system about the axis of rotation (through point P) is
2
I  m1 x 2  m 2  L  x 

By work energy theorem, work done to set the rod rotating with angular velocity 0  Increase
in rotational kinetic energy.
1 2 1 2 dW
W 0  m1 x 2  m 2  L  x   02 ; For W to be minimum, 0
2 2  dx
1
i.e.,  2m1x  2m 2  L  x  1  02  0
2
m2L
or m1 x  m 2  L  x   0 or  m1  m 2  x  m 2 L or x   0  0 
m1  m 2
1 2
19. 1 Work done required to bring an object to rest; W  KE   I ;
2

2
For same , W  I ; For a solid sphere, I A  MR 2 ; For a thin circular disk, I B  1 MR 2
5 2
For a circular ring, I C  MR 2  IC  IB  IA  WC  WB  WA

1 2 1 1
20. 3 Total energy  I  mv 2  mv 2 1  K 2 / R 2 
2 2 2

1 2 1 mK 2 v 2
Rotational energy  I 
2 2 R2

K2 / R 2 K2
Required fraction  
1  K2 / R 2 R 2  K2

161
Brilliant STUDY CENTRE

21. 4 M = 5 kg, R = 0.40 m,   10 rad / s , I  0.65 kg m 2

1 1 2 1 2
K.E. of translation  Mv 2  M  R  ; Et   5   0.4  10   40 J
2 2 2
1 2 1 2
K.E. of rotation, E r  I   0.65 10   32.5 J
2 2
Total energy, E  E t  E r  40  32.5  72.5 J

E r 32.5
  100%  44.8%
E 72.5

1 2 1 2 1 2 1 2 2 v2
22. 3 For rolling: Mgh  Mv  I  Mv    MR   2
2 2 2 2 5  R

1 1 7  2 v
Mv 2  Mv 2  Mv 2  I  MR and   
2

2 5 10  5 R

1 1 7 v 7
For sliding : Mgh  Mv2 . Therefore, Mv2  Mv 2 ; or 
2 2 10 v 5

162
[Study Package - Physics - Class XI]

CHAPTER - 07
GRAVITATION

NEWTON’S LAW OF GRAVITATION


Newton’s law of gravitation states that every body in the universe attracts every other body with a force,
which is directly proportional to the product of their masses and inversely proportional to the square of the
distance between their centres. The direction of the force is along the line joining the particles.

m1m 2 m1m 2
For masses m1 and m2 at a distance r apart, F  2 or F  G
r r2
G is the universal gravitational constant, when m1 = m2 = 1, r = 1, F = G

VECTOR FORM OF NEWTON’S LAW OF GRAVITATION


Consider two bodies A and B of masses m1 and m2, placed at a distance r apart,

 Gm1m 2 Gm m 
Gravitational force on A by B,
F12  2
rˆ21   13 2 r21 . r̂21 = unit vector from B to A
r r21

 Gm1m 2 Gm m 
F
Gravitational force on B by A, 21
 2
rˆ12   1 3 2 r12
r r12
 
F12  F21 . By Newtons third law of motion.
Properties of Gravitational Force
1) It is always attractive in nature.
2) It is independent of the medium between the particles.
3) It holds good over a wide range of distances. It is found true for interplanetary to inter atomic distances.
4) It is a central force, ie, acts along the line joining the centres of two interacting bodies.

5) It is the weakest force in nature : As Fnuclear  Felectromagnetic  Fgraviational .

6) It is always conservative in nature.

163
Brilliant STUDY CENTRE

Acceleration Due to Gravity


The force of attraction exerted by the earth on a body is called gravitational pull or gravity.
When a force acts on a body, it produces acceleration. Therefore, a body under the effect of gravitational
pull must accelerate.
The acceleration produced in the motion of a body under the effect of gravity is called acceleration due
to gravity, it is denoted by g.
Consider a body of mass m, lying on the surface of earth then gravitational force on the body is given by

GMm
F ; Where M = mass of the earth and R = radius of the earth.
R2
If g is the acceleration due to gravity
Force on the body due to gravity = mass × acceleration, or F = mg

GMm
mg 
R2

GM 4  4 3 
 g  g GR  M  R  
R2 3  3 
It is clear that g value depends upon the mass, radius and density of planet and it is independent of
mass, shape and density of the body placed on the surface of the planet.
Acceleration due to gravity is a vector quantity and its direction is always towards the centre of the
planet.
Average value of g = 9.8 m/s2 at the surface of earth.

gR 2
Mass of earth, M  ; M = 6.018  1024 kg
G

3g R 2 G 3g
Density of earth,   3
  5.5  103 kg m 3
4R 4RG
VARIATION OF ACCELERATION DUE TO GRAVITY
The value of acceleration due to gravity changes with height (i.e. altitude), depth, shape of the earth and
rotation of earth about its own axis.
Variation of g with height
Consider earth to be a sphere of mass M, radius R with centre at O. Let g be the value of acceleration due
to gravity at a point A on the surface of earth.,

GM
 g
R2

164
[Study Package - Physics - Class XI]

GM
g  2
R  h
g GM R2 R2 g   h  2
    ;  1
g  R  h  2 GM  R  h  2 g  R 

h g 2h  2h 
If  1 , then  1 or g  g 1  
R g R  R
Acceleration due to gravity decreases with height.
1
g decreases with height as g  and at r   , g' = 0
r2

 2h  2hg
If h << R, then g  g 1   , then g  g  g 
 R R
Variation in g with Depth
Consider earth to be a homogeneous sphere of radius R and mass M with centre at O. Let g be the value
GM
of acceleration due to gravity at a point A on the surface of earth, (Fig. below). Then g
R2

A
g
d B
g'
R
(R–d)
4 O
g GR
3

The body at B will experience gravity pull due to shaded portion of earth whose radius is (R – d) and
mass is M'

GM 4
g  2 and M  
3
R  d 
then
 R  d  3

4
 g  G  R  d  
3

g R  d  d
 ; g   g 1  
g R  R
We note that the value of acceleration due to gravity decreases with depth. At the centre of the earth, d  R

 R
g  g  1    0
 R

165
Brilliant STUDY CENTRE

The acceleration due to gravity is zero at the centre of earth. Therefore the weight of the body of mass
m at the centre of earth = mg0 = 0, but the mass of the body will not be zero.
The value of g decreases on going below the surface of the earth.

At the centre of the earth d = R  g  0

g g  g d
% Decrease in the value of g with depth. 100   100   100
g R R

gd
Absolute decrease g  g  g 
R
Graph between g and r

r=R r

Variation of g due to rotation of earth.

Earth is rotating about its own axis. So earth is a non inertial frame, so pseudo force is acting there.
Due to this g vary due to the rotation of earth.

g  g  2 R cos 2 

at equator   0 , g  g  2 R

at poles   90, g  g

If g  2 R at equator, then g' = 0;    g / R the apparent weight of the body at equator will be
zero.
Variation of g Due to Shape of Earth.
Earth is elliptical in shape. It is flattened at the poles and bulged out at the equator. The equatorial
radius is about 21 km longer than polar radius,

GM
from g 
R2

166
[Study Package - Physics - Class XI]

GM GM
At equator ge  gp 
R 2e ; At poles R 2p

2
ge R p
 R  R pole  g pole  g equator and g p  g e  0.018 ms2
g p R e2 ; Since equator

Inertial and Gravitational Masses


i) Inertial mass: It is the mass of the material body, which measures its inertia.
If an external force F acts on a body of mass mi, then according to Newton’s second law of motion

F
F  mi a or mi 
a
Hence inertial mass of a body may be measured as the ratio of the magnitude of the external force
applied on it to the magnitude of acceleration produced in its motion.
It is the measure of ability of the body to oppose the production of acceleration in its motion by an
external force.
It is proportional to the quantity of matter contained in the body, independent of size, shape etc.
2) Gravitational Mass: It is the mass of the material body, which determines the gravitational pull acting
upon it.
If M is the mass of the earth and R is the radius, then gravitational pull on a body of mass mg is given by

F F
mg  
 GM / R 2  g
Here mg is the gravitational mass of the body.
3) Comparison between inertial and gravitational mass
i) Both are measured in the same units.
ii) Both are scalars.
iii) Both do not depends on the shape and state of the body.
iv) Inertial mass is measured by applying Newton’s second law of motion where as gravitational mass
is measured by applying Newton’s law of gravitation.
v) Spring balance measure gravitational mass and inertial balance measure inertial mass.

167
Brilliant STUDY CENTRE

4. Comparison between mass and weight of the body

Mass (m) Weight (W)


It is a quantity of matter contained in a body It is the attractive force exerted by earth
on any body
Its value does not change with g Its value changes with g
Its value can never be zero for any at infinity and at the centre of earth
material particle its value is zero
Its unit is kilogram and its dimension is [M] Its unit is Newton or kg-wt and
–2
dimension are [MLT ]
It is determined by a physcial balance It is determined by a spring balance
It is a scalar quantity It is a vector quantity

Gravitational Field
The space surrounding a material body in which gravitational force of attraction can be experienced is
called its gravitational field.
INTENSITY OF GRAVITATIONAL FIELD
The intensity of gravitational field at a point in a gravitational field is defined as the force experienced
by a body of unit mass placed at that point provided the presence of unit mass does not disturb the original
gravitational field. It is always directed towards the centre of gravity of the body whose gravitational field is
considered. Intensity of gravitational field at a point is a vector quantity and is denoted by E.

  F
So if a test mass m at a point in a gravitational field experiences a force F then E
m
It is a vector quantity and is always directed towards the centre of gravity of the body whose gravitational
field is considered. Unit. N/kg

F GMm / r 2 GM 1
then intensity of gravitational field E    E 2 , E 2
m m r r
E = 0 at r  
Intensity at a given point (P) due to the combined effect of different point masses can be calculated by vector
sum of different intensities.

   
For a system of point masses, E net  E1  E 2  E3  .............

168
[Study Package - Physics - Class XI]

Gravitational field intensity due to point mass


r
B
Force acting on a mass m placed at B A
M I

GMm
F
r2
F GM GM
  2 , directed towards M. I  2
m r r
Gravitational field intensity due to uniform solid sphere.

r
O A
R

For a mass m placed at A, directed towards centre of sphere.


GMm F GM GM
F 2
;  2 ; I 2
r m r r
at the surface of the sphere, r = R
GM
I , directed towards the centre of the sphere.
R2

m'

r r
m
R m
R

3
M 4 r 
Inside the sphere,   ; m   r 3 ; m  M  
4 3 3 R
R
3
GMm F Gm
Force F  ;  2
r2 m r

GMr 3 GMr
I  3 directed towards the centre of the sphere.
r 2R 3 R

|I|

r=R r

169
Brilliant STUDY CENTRE

Gravitational field intensity due to spherical shell.


A point outside the shell.

r A
R M

GM
I directed toward the centre of sphere.
r2
GM
at surface r = R; I 
R2

|I|
I=0
r A
R
At a point inside the sphere

r=R r

Gravitational field intensity due to ring.

M
dm
R2  r2
F
R R
A  A
r m r m

Consider a small element dm of the ring.


G.dm M
Force due dm is given by dF 
R 2  r2
This can be resolved to vertical and horizontal components.
Vertical components cancel each other and horiozntal components added together.
Total force due to M is given by
r
F   dF cos ; cos  
R  r2
2

G.dm M r FGM r
F .   dm
2
R r 2 2
R r 2
 R 2  r 2 3/2
F Gr
I  M
m  R 2  r 2 3/2 ; directed towards the centre of ring.

170
[Study Package - Physics - Class XI]

Gravitational Potential
At a point in a gravitational field, potential V is defined as negative of work done per unit mass in shifting
a test mass from infinity to the given point ie,
 
F . dr  
V      E . dr
m

dV GM
 E , V
dr r
ie, negative gradient of potential gives intensity of field or potential is a scalar function of position whose
space derivative gives intensity. Negative sign indicates that the direction of intensity is in the direction
where the potential decreases.
Gravitational potential due to a uniform solid sphere
Outside the sphere

r GM
R V
r

At the surface

R GM
V
R

Inside the sphere

r
R GM  2 2 
V 3R  r
2R 3

GM  2
At the centre of the sphere Vcentre  3R  0 
2R 3
3GM 3
Vcentre    Vsurface
2R 2

r=R
r
GM

R
3GM

2R

171
Brilliant STUDY CENTRE

Gravitational potential due to a spherical shell


At a point outside

r
R GM
V
r

R GM
at surface V
R

O r GM
Inside the shell V
R

Potential inside the shell is same as that on the surface.


Gravitational potential due to a ring

dm

R2  r2
a R
r A
r

Consider a small element of mass dm in that ring; potential at that point due to small element.
G dm
dv  
R 2  r2
G GM
v   dv    dm  
R 2  r2 R 2  r2
GM
v
R2  r2

Gravitational potential difference : It is defined as the work done to move a unit mass from one point to
the other in the gravitational field.

WA B 1 1
V  VB  VA   GM   
m  rB rA 

172
[Study Package - Physics - Class XI]

Gravitational Potential Energy


Gravitational potential energy of a body at a point in a gravitational field of another body is defined as
the amount of work done in bringing the given body from infinity to that point without acceleration.
The gravitational potential energy of a body at a point is defined as the amount of work done in bringing
the body from infinity to that point against the gravitational force.

r
GMm GMm GMm
W dx ; W  u
 x2 r r

Gravitational potential energy of a system of three particles


m3

r3 r2

m1 m2
r1

mm m m mm 
U  G  1 2  2 3  1 3  ; when m = m , and r = r = r
 r1 r2 r3  1 2 1 2 3

Gm 2
U 3
r
Gravitational potential energy of four particles at the corners of a square.
m a m

a a Gm 2 Gm 2
U 4  2
a 2a
m a m

 Gm1m 2 Gm 2 m 3 
Gravitational potential energy U   u i      .........
 r12 r23 

GMm  GM 
Relation between gravitational potential energy and potential U    m 
r  r 
 U = mV
Work Done Against Gravity
If the body of mass m is moved from the surface of earth to a point at a distance h above the surface of
the earth, then change in potential energy or work done against gravity will be

1 1  mgh
W  U  GMm    , when r = R and r = R + h; W
 r1 r2  1 2 h
1
R

173
Brilliant STUDY CENTRE

ESCAPE VELOCITY
The escape velocity of a body is defined as the minimum velocity with which the body has to be
projected vertically upwards from the surface of a planet so that it just crosses the gravitational field of that
planet and never returns on its own.

Q
dx
P
xA
R
O

If ve is the escape velocity of the body projected from the surface of earth, then
1
Kinetic energy of the body  mve2
2

1 GMm 2GM 2GM


 mv 2e  or v 2e  or ve 
2 R R R

GM 2gR 2
As g Ve   2gR
R2 R
Kepler’s Laws of Planetary Motion
Johannes Kepler, a German mathematician and astronomer (1571-1630) after a life time study worked out three
empirical laws which govern the motion of the planets and are known as Kepler’s laws of planetary motion. These
are,
(1) The law of Orbits : Every planet moves around the sun in an elliptical orbit with sun at one of the foci.

(2) The law of Area : The line joining the sun to the planet sweeps out equal areas in equal interval of time.
i.e. areal velocity is constant. According to this law planet will move slowly when it is farthest from sun
and more rapidly when it is nearest to sun. It is similar to the law of conservation of angular momentum

dA 1 r  vdt  1 dA L L
Areal velocity    rv   [ As L = mvr; rv  ]
dt 2 dt 2 dt 2m m
(3) The law of periods: The square of period of revolution (T) of any planet around sun is directly proportional
to the cube of the semi-major axis of the orbit.
3
r r 
T 2  a 3 or T 2   1 2 
 2 
174
[Study Package - Physics - Class XI]

r1  r2
From the figure AB = AF + FB; 2a  r1  r2  a  where a = semi - major axis.
2
r1 = Shortest distance of planet from sun (perigee); r2 = Largest distance of planet from sun
(apogee)
SATELLITE
A satellite is a body which is revolving continuously in an orbit around a comparatively much larger
body. For example, earth is a satellite of sun and moon in turn is a satellite of earth.
Orbital Velocity of Satellite
Orbital velocity of a satellite is the velocity required to put the satellite into its orbit around the earth.
For revolution of satellite around the earth, the gravitational pull provides the requied centripetal force.

mv 2 GMm GM
 ;  v
r r2 r

gR 2 g
v R [As GM = gR2 and r = R + h]
R h R h
Orbital velocity is independent of the mass of the orbiting body and is always along the tangent of the
orbit.


For a given planet, greater the radius of orbit, lesser will be the orbital velocity of the satelite v  1/ r 
GM ve
Close to the surface of planet v0  ;  v0  i.e., v escape  2 v orbital
R 2
Time Period of Satellite
It is the time taken by satellite to go once around the earth.

Circumference of the orbit 2r r


 T orbital velocity
  2r
v GM

175
Brilliant STUDY CENTRE

3 3/2
r3 R  h  R  h
 T  2 ;  T  2  2 1  R  [As r = R + h]
GM gR 2 g  

Time period of nearby satellite

R3 R
T  2 2
 2
gR g

For earth R = 6400 km and g = 9.8 ms–2.


T = 84.6 minutes = 1.4 hr
Height of Satellite

3
r3 R  h
As we know, time period of satellite T  2  2
GM gR 2

1/3
 T 2 gR 2 
From the above rearranging the equation h   2 
R
 4 
Geostationary Satellite
The satellite which appears stationary relative to earth is called geostationary or geosynchronous satellite.
It is used as a communication satellite. Such a satellite appears stationary due to its zero relative velocity
with respect to that place on earth. The orbit of a geostationary satellite is known as the parking orbit
Height of geostationary satellite:

As T  2
r3
 2
R  h  24hr
GM GM
Susbstituting the value of G and M we get R + h = r  42000 km  7 R
 height of geostationary satellite from the surface of earth h  6R  36000 km.

GM
Orbital velocity of geostationary satellite, v  , v = 3.08 km/sec.
r
Polar satellite
These satellites are revolving in a plane containing axis of rotation of earth. They go round poles of earth
in north south direction. Polar satellites are launched at low altitudes around 500-800 km.
Time period of revolution is around 100 minutes.
Angular Momentum of Satellite

GM
Angular momentum of satellite L = mvr;  Lm r  L  m 2GMr
r

176
[Study Package - Physics - Class XI]

Energy of a satellite
2
1 2 GMm 1  GM  GMm
Total energy of a satellite ; E = KE + PE  mv   m   
2 r 2  r  r

GMm GMm GMm


  
2r r 2r
Weightlessness in a Satellite.
A satellite, which does not produce its own gravity moves around the earth in a circular orbit under the
GM
action of gravity. The acceleration of satellite is towards the centre of the earth.
r2 r

If a body of mass m placed on a surface inside a satellite moving around the earth. Then force on the body
are
GMm
(i) The gravitational pull of earth  (ii) The reaction by the surface = R
r2

GmM GmM  GM 
By Newtons law  R  ma ; 2
 R  m 2   R=0
r2 r  r 
Thus the surface does not exert any force on the body and hence its apparent weight is zero.
A body needs no support to stay at rest in the satellite and hence all position are equally comfortable.
Such a state is called weightlessness.

177
Brilliant STUDY CENTRE

QUESTIONS
LEVEL - I
1. Two spheres of masses m and M are situated in air and the graviational force between them is F. The
space around the masses is now filled with a liquid of specific gravity 3. The gravitational force will
now be

F F
1) 3 F 2) F 3) 4)
3 9
2. A spherical planet has a mass Mp and diameter Dp. A particle of mass m falling freely near the surface
of this planet will experience an acceleration due to gravity, equal to

4GM p GM p m GM p 4GM p m
1) 2 2) 2 3) 2 4)
D p D p D p D 2p
3. A small body of superdense material whose mass is twice the mass of the earth but whose size is
very small compared to the size of the earth, starts from rest at a height H << R above the earth’s
surface, and reaches the earth’s surface in time t. Then t is equal to:

1) H/g 2) 2H / g 3) 2H / 3g 4) 4H / 3g
4. If the distance between the sun and the earth is increased by three times then attraction between two
will:
1) remain constant 2) decrase by 63%
3) increase by 63% 4) decrease by 89%
5. Suppose that the angular velocity of rotation of earth is increased. Then, as a consequence
1) there will be no change in weight anywhere on the earth
2) weight of the object, everywhere on the earth, will increase
3) except at poles, weight of the object on the earth will decrease
4) weight of the object, everywhere on the earth, will decrease

g
6. The height at which the acceleration due to gravity becomes (where g = the acceleration due to
9
gravity on the surface of the earth) in terms of R, the radius of the earth is

R R
1) 2R 2) 3) 4) 2R
2 2
7. If the Earth has no rotational motion, the weight of a person on the equator is W. Determine the speed
with which the earth would have to be rotated about its axis so that the person at the equator will weigh
3
W . Radius of the Earth is 6400 km and g = 10 m/s2.
4
1) 0.63 103 rad / s 2) 0.83  103 rad / s 3) 0.28  103 rad / s 4) 1.1 103 rad / s
8. A man can jump to a height of 1.5m on a planet A. What is the height he may be able to jump on another
planet whose density and radius are, respectively, one-quarter and one-third that of planet A
1) 1.5 m 2) 15 m 3) 18 m 4) 28 m

178
[Study Package - Physics - Class XI]

th
 1 
9. The acceleration due to gravity at a height   of the radius of the earth above the earth surafce is
 20 
9 ms 2 . Its values at a point at an equal distance below the surface of the earth in ms–2 is about
1) 8.5 2) 9.5 3) 10.5 4) 11.5
10. Imagine a new planet having the same density as that of earth but it is 3 times bigger than the earth in
size. If the acceleration due to gravity on the surface of earth is g and that on the surface of the new
planet is g', then

g
1) g  2) g  27g 3) g' = 9g 4) g' = 3g
9
11. A box weighs 196 N on a spring balance at the north pole. Its weight recorded on the same balance if
it is shifted to the equator is close to (Take g = 10 ms–2 at the north pole and the radius of the earth =
6400 km)
1) 194.32 N 2) 194.66 N 3) 195.66 N 4) 195.32 N
12. Infinite number of bodies, each of mass 2 kg are situated on x-axis at distances 1 m, 2 m, 4 m, 8 m,
....., respectively, from the origin. The resulting gravitational potential due to this system at the origin
will be

4 8
1)  G 2) –4G 3) –G 4)  G
3 3
13. A particle of mass M is situated at the centre of a spherical shell of same mass and radius a. The
a
gravitational potential at a point situated at distance from the centre, will be
2

3GM 2GM GM 4GM


1)  2)  3)  4) 
a a a a
14. The mass of the earth is 6.00 × 1024 kg and that of the moon is 7.40 × 1022 kg. The constant of
gravitation G  6.67  1011 Nm 2 / kg 2 . The potential energy of the system is –7.79 × 1028 joule. The
mean distance between earth and moon is:
1) 3.80 × 108 metres 2) 3.7 × 106 metres 3) 7.60 × 104 metres 4) 1.90 × 102 metres
15. The change in potential energy when a body of mass m is raised to a height nR from earth’s surface
is : (R=radisu of the earth)

n n n2
1) mgR n  1 2) mgR 3) mgR n  1 4) mgR
     n 2  1
16. A satellite of mass m is orbiting the earth (of radius R) at a height h from its surface. The total energy
of the satellite in terms of g0, the value of acceleration due to gravity at the earth’s surface, is

mg 0 R 2 mg 0 R 2 2mg 0 R 2 2mg 0 R 2
1) 2 R  h 
2) 2 R  h 3) 4) 
    Rh Rh

179
Brilliant STUDY CENTRE

17. A particle of mass 10 g is kept on the surface of a uniform sphere of mass 100 kg and radius 10 cm.
Find the work to be done against the gravitational force between them, to take the particle far away
from the sphere: (you may take G = 6.67 × 10–11 Nm2/kg2)
1) 13.34 × 10–10 J 2) 3.33 × 10–10 J 3) 6.67 × 10–9 J 4) 6.67 × 10–10 J
18. A satellite is revolving round the earth with orbital speed v0. If it stops suddenly, the speed with which
it will strike the surface of earth would be (ve = escape velocity of a body on earth’s surface)

v e2
1)
v0
2) v 0 3) v 2e  2v 02 4) v 2e  v 20

19. A body is projected vertically upward from the surface of the earth with a velocity equal to half the
escape velocity. If R is radius of the earth, the maximum height attained by the body is

R R 2
1) 2) 3) R 4) R
6 3 3
20. The ratio of escape velocity at earth (ve) to the escape velocity at a planet (vp) whose radius and mean
density are twice as that of earth is:

1) 1 : 2 2) 1: 2 2 3) 1 : 4 4) 1: 2

21. Average distance of the earth from the sun is L1. If one year of the earth = D days, one year of another
planet whose average distance from the sun is L2 will be
1/ 2 3/ 2 2/3
 L2   L2   L2   L2 
1) D   days 2) D   days 3) D   days 4) D  L  days
 L1   L1   L1   1

22. A body is orbiting very close to the surface of earth with kinetic energy K. The energy required to
escape from the surface of earth is:

1) 2K 2) K 3) K / 2 4) 2 K

23. Two particles of equal mass go round a circle of radius R under the action of their mutual gravitational
attraction. The speed of each particle is

1 1 Gm 1 Gm 4Gm
1) v  2) v  3) v  4) v 
2R Gm 2R 2 R R
24. Two satellites of same mass m are launched in the same orbit of radius r around the earth of mass M
to rotate opposite to each other. If they collide inelastically and stick together as wreckage, the total
energy of the system just after the collision is:

GMm 2GMm GMm GMm


1) 2) 3) 4)
r r 2r r
25. The radius of a planet is R1 and a satellite revolves around it in a circular orbit of radius R2. The time
period of revolution is T. What is the acceleration due to gravity on the surface of the planet?

42 R 32 42 R 22 42 R 22 42 R 22


1) 2 2 2) 2 3 3) 4) 2 2
T R1 T R1 T2 T R1

180
[Study Package - Physics - Class XI]

26. A synchronous satellite goes around the earth once in every 24 h. What is the radius of orbit of the
synchronous satellite in terms of the earth’s radius? (Given mass of the earth, M e  5.98  10 24 kg ,

radius of the earth, R e  6.37  10 6 m , Universal constant of gravitation, G  6.67  1011 Nm 2 kg 2 )


1) 2.4 Re 2) 3.6 Re 3) 4.8 Re 4) 6.6 Re
27. A geostationary satellite is orbiting the earth at a height of 5R above that surface of the earth, R being
the radius of the earth. The time period of another satellite in hours at a height of 2R from the surface
of the earth is

6
1) 5 2) 10 3) 6 2 4)
2
LEVEL - II
1. The ratio of the weights of a body on the Earth’s surface to that on the surface of a planet is 9 : 4. The
1
mass of the planet is th of that of the Earth. If ‘R’ is the radius of the Earth. What is the radius of the
9
planet? (Take the planets to have the same mass density)

R R R R
1) 2) 3) 4)
3 9 4 2
2. A fully dressed astronaut can jump on the earth a maximum vertical height 0.6 m. If the mean density
of the moon is (2/3) time that of the earth and radius of moon is (1/4) times that of the earth, the
maximum vertical distance through which he can jump on the moon is:
1) 3.0 m 2) 3.6 m 3) 4.2 m 4) 2.4 m
3. An object of mass m is projected vertically upwards from the surface of earth of radius R with velocity
nve, where ve is the escape velocity and n < 1. Neglect the air resistance, to what height from the
surface of earth will the object rise?

Rn 2 R Rn 2 Rn 2
1) 2) 1  n 2 3) 4) 1  n 2
1  n 
2
  1 n2  
4. The density of newly discovered planet is thrice that of the earth. The acceleration due to gravity at the
surface of that planet is equal to that at the height R from the surface of earth. If radius of the earth is
R, the radius of the planet would be

R R R
1) 2) 3) 4) 3R
3 6 12
5. An accurate pendulum clock is mounted on the ground floor of a high building. How much time will it
lose or gain in one day if it is transferred to top story of a building which is 200 m higher than the
ground floor? Radius of earth is 6.4 × 106 m. It will
1) loss 2.7 s 2) loss 5.4 s 3) gain 2.7 s 4) gain 5.4 s
6. The depth d at which the value of acceleration due to gravity becomes 1/n times the value at the
surface, is [R = radius of the earth]

R  n  1 R  n 
1) 2) R   3) 4) R  
n  n  n2  n  1

181
Brilliant STUDY CENTRE

7. Taking the radius of the earth to be 6400 km, by what percentage will the acceleration due to gravity at
a height of 100 km from the surface of the earth differ from that on the surface of the earth?
1) about 1.5 % 2) about 5% 3) about 8% 4) about 3%
8. The acceleration due to gravity on the planet A is 9 times the acceleration due to gravity on planet B. A
man jumps to a height of 2 m on the surface of A. What is the height of jump by the same person on the
planet B?
1) 2/9 m 2) 18 m 3) 6 m 4) 2/3 m
9. The density of a newly discovered plaent is twice that of earth. The acceleration due to gravity at the
surface of the planet is equal to that at the surface of the earth. If the radius of the earth R, the radius
of the planet would be

1 1
1) 2R 2) 4R 3) R 4) R
4 2
10. An object of mass 2 kg is moved from infinity to a point P. Initially that object was at rest but on
reaching P its speed is 2 ms–1. The work done in moving with object is –4J. Then gravitational potential
at P is:
1) 4 J/kg 2) 8 J/kg 3) –4 J/kg 4) –8 J/kg
11. Two masses m1 and m2 are initially at rest and are separated by a very large distance. If the masses
approach each other subsequently, due to gravitational attraction between them, their relative velocity
of approach at a separation distance of d is:

1/2
2Gd  m1  m2  G  2G 
3)  m1  m 2 
1/ 2
1) m  m 2) 4)  m1  m 2  .2Gd
 1 2 2d  d 

12. If g is the acceleration due to gravity on the earth’s surface, the gain in the potential energy of an object
of mass m raised from the surface of the earth to a height equal to the radius R of the earth is:

1 1
1)   mgR 2) 2mgR 3) mgR 4)   mgR
 2  4
13. The work done in shifting a particle of mass m from the centre of the earth to the surface of the earth
is (Where M is the mass of the earth and R is the radius of the earth)

GMm GMm GMm GMm


1)  2)  3)  4) 
R R 2R 2R

 
14. The gravitational field in a region of space is given by I  4iˆ  ˆj Nkg . The work done by this field is
1

zero when the particle is moved along the line:


1) y + 2x = 4 2) y + 4x = 3 3) 4y + x = 6 4) y – 4x = 3
15. If an antificial satellite is moving in a circular orbit around the earth with a speed equal to one-third the
magnitude of escape velocity from the earth, the height of the satellite above the surface of earth is:
(Given R is the radius of the earth)
1) 1.5 R 2) 2.5 R 3) 3.5 R 4) 4.5 R

182
[Study Package - Physics - Class XI]

16. Two satellites of earth, S1 and S2 are moving in the same orbit. The mass of S1 is four times the mass
of S2. Which one of the following statements is true?
1) The potential energies of earth and statellite in the two cases are equal
2) S1 and S2 are moving with the same speed
3) The kinetic energies of the two satellites are equal
4) The time period of S1 is four times that of S2.
17. A body is projected up from the surface of the earth with a velocity equal to 3/4th of its escape velocity.
If R be the radius of earth, the height it reaches is:

3R 9R 8R 9R
1) 2) 3) 4)
10 7 5 5
18. Kepler’s third law states that square of period of revolution (T) of a planet around the sun, is proportional
to third power of average distance r between sun and planet i.e., T 2  Kr 3 here K is constant.
If the masses of sun and planet are M and m respectively then as per Newton’s law of gravitation force
of attraction between them is

GMm
F , here G is gravitational constant.
r2
The relation between G and K is described as

1
1) K = G 2) K  3) GK  4 2 4) GMK  4 2
G
19. A particle of mass m is thrown upwards from the surface of the earth, with a velocity u. The mass and
the radius of the earth are respectively, M and R. G is the gravitational constant and g is the acceleration
due to gravity on the surface of the earth. The minimum value of u so that the particle does not return
back to earth is:

2GM 2GM 2gM


1) 2) 3) 4) 2gR 2
R2 R R2
20. A body is projected vertically upwards from the surface of the earth with a velocity kve where k < 1 and
ve is the escape velocity. If R is the radius of the earth, the maximum height from the surface of the
earth a body will rise is:

1 k2 k2R 1 k2 k 2R
1) 2 2) 3) 2 4) 1  k 2
kR 1 k2 k R  
21. Two plaents are at mean distances d1 and d2 from the sun and their frequencies of revolution are n1
and n2 respectively. Then

1) n12 d12  n 22 d 22 2) n 22 d 32  n12 d13 3) n1 d12  n 2 d 22 4) n12 d1  n 22 d 2


22. The earth is assumed to be a sphere of radius R. A platform is arranged at a height R from the surface
of the earth. The escape velocity of a body from this platform is fv, where v is the escape velocity from
the surface of the earth. The value of f is:

1 1 1
1) 2 2) 3) 4)
2 3 2

183
Brilliant STUDY CENTRE

23. Two planets of masses M1 and M2 have satellites of masses m1 and m2 respectively, revolving around
them at the same radius r. The period of the first satellite (of mass m1) is twice as that of the second.
Which one of the following relations is correct?
1) 4M1 = M2 2) 2M1 = M2 3) M1 = 2M2 4) m1M1 = m2M2
24. The mass of a spaceship is 1000 kg. It is to be launched from the earth’s surface out into free space.
The value of ‘g’ and ‘R’ (radius of earth) are 10m/s2 and 6400 km respectively. The required energy of
for this work will be
1) 6.4 × 1011 Joules 2) 6.4 × 108 Joules 3) 6.4 × 109 Joules 4) 6.4 × 1010 Joules
25. Two bodies each ofm ass M, are kept fixed with a separation 2 L. A particle of mass m is projected
from the midpoint of the line joining their centres, perpendicular to the line. The gravitational constant
is G. The correct statement(s) is (are)
1) the minimum initial velocity of the mass m to escape escape the gravitational field of the two
bodies is 4 GM / L

2) the minimum initial velocity of the mass m to escape the gravitational field of the two bodies is
2 GM / L
3) the minimum initial velocity of the mass m to escape the gravitational field of the two bodies is
2GM / L
4) the energy of mass m remains constant.
26. Out of the following, the only correct statement about satellites is:
1) A satellite can not move in a stable orbit in a plane passing through the earth’s centre
2) Geostationary satellites are launched in the equatorial plane
3) We can use just one geostationary satellite for global communication around the globe
4) The speed of a satellite increases with an increase in the radius of its orbit.

184
[Study Package - Physics - Class XI]

KEY WITH HINTS


LEVEL - I
1. 2 Gravitational force does not depend upon the medium in which objects are placed.

GM p m
2. 1 Gravitational force acting on particle of mass m is
F  2
 Dp / 2 
F GM p 4GM p
Acceleration due to gravity experience by the particle is
g  2

m  Dp / 2 D 2p

3. 3 As the masses of the body and earth are comparable, both will move towards their centre of
2m  0  m  H H
mass. Let the distance of the centre of mass from the body be r, then r  
2m  m 3
It means the body will move towards earth through a distance H/3, under gravity. As H << R, so
the acceleration of the body is equal to the acceleration due to gravity.

2  H / 3 2H
Hence, time taken by body is t  
g 3g

Gm1m 2 Gm1m 2 F1
4. 4
F1  2
; F2  2
 ; % Decrease in force of attraction
r  3r  9

F1  F2  1 800
 100  1   100   89%
F1  9 9

5. 3 Effect of rotation of earth on acceleration due to gravity is given by g  g  2 R cos 2 

Where  is latitude. There will be no change in gravity at poles as   90o , at all other points as
 increases, g will decrease.
gs g g
6. 1 gh  2
   h  2R
 h 9  h 2
1  R  1  R 
   

3W
7. 1 Here, the weight of person on the equator = W. If earth rotate about its axis, then weight 
4
The acceleration due to gravity at the equator, g e  g  2 R cos 2 

3 g
g  g  2 R cos 2 0o or 2 R 
4 4

g 10
  3
 0.63 10 3 rad / s
4R 4  6400 10

185
Brilliant STUDY CENTRE

 1
u2 1  R
H H ; HA gB 4 3 1  4 
8. 3     g  3 GR 
2g g HB gA R 12  

 HB  12HA  12  1.5  18 m

gR 2
9. 2 Acceleration due to gravity at a height h above the surface of the earth is g h 
 R  h 2
where R is the radius of the earth.

2
gR 2  20 
 9 2
 g   ---- (i)
 R  21 
R  
 20 

 d   R / 20    19 
g d  g 1    g 1   ; gd  g  
 R  R   20 

g d 19 21 21
    or g d  9.5 ms 2
9 20 20 20

4
G R 2  4
10. 4 GM 3 4 ; g  G  3R    3g
g 2   GR  3
R R2 3
11. 4 Weight at north pole, mg = 196 N at north pole, g = 10 m/s 2; Re = 6400 km

Now, acceleration due to gravity at equator, g equator =  g  R e 2

mg equator  mg  mR e 2  195.32N

12. 2 The resulting gravitational potential at the origin O due to each of mass 2 kg located at positions
as shown in figure is

G2 G2 G2 G2  1 1 1   1 


V     ........  2G 1     ....  2G   4G
1 2 4 8  2 4 8  1
1  
 2

 1 1 1   1  2
 2G 1     .........  2G    2G    4G
 2 4 8  1 1
1  
 2

186
[Study Package - Physics - Class XI]

13. 1 Mass of the particle = M. Mass of the spherical shell = M, Radius of shell = a.

a
Point P is at a distance from the centre of the shell as shown in figure.
2

GM
Gravitational potential at point P due to particle at O is V1  
 a / 2

GM
Gravitational potential at point P due to spherical shell is V2  
a
2GM GM 3GM
Total gravitational potential at the point P, V  V1  V2    
a a a
GMm  GMm
14. 1 U or r 
r U

6.67  1011  6  1024  7.4  10 22


r  3.8  108 m
7.79  1028

GMm GMm GMm  1 


15. 3 Change in P.E., U  U 2  U 1         1
 R  nR  R R  1  n  


R g m 
2
n  n 
 mgR  
R  n  1  n 1 
16. 2 PE + KE of satellite at height h from the earth surface
GMm 1
E  mv 2 ------ (i)
Rh 2

mv 2 GMm GM
Also,  2
or v 2  ...(ii)
(R  h) (R  h) Rh

GMm 1 GMm 1 GMm


From eqns. (i) and (ii), E   
(R  h) 2 (R  h) 2 (R  h)

1 GM mR 2 mg 0 R 2
   
2 R 2 (R  h) 2 (R  h)

187
Brilliant STUDY CENTRE

17. 4 Work done against gravitational force = Change in PE

 GMm  GMm  6.67  10  100  10 10   6.67 10


11 3
10
 0   J
 R  R  0.1
18. 3 Let v be the velocity with which the satellite strikes the surface of the earth. According to law of
conservation of total mechanical energy, we get

GMm 1 GMm 2GM 2GM GM 2GM


 mv 2  ;  v0 
2
 ; v   , ve 
Rh 2 R R Rh R h R

 v  ve2  2v 02

19. 2 Let the body is projected from the surface of the earth with the velocity v and reaches maximum
height h.
According to law of conservation of mechanical energy we get

1 2 GMm GMm 1 2 GM GM
mv   0 or v  
2 R R  h  2 R R  h

v e 1 2GM 1  1 GM  GM GM
v  ;    
2 2 R 22 R  R R  h 

GM GM GM 3 1 R
  ;   ; 3h = R, h 
4R R R h 4R R  h 3

2GM 2G  4 3  ve
20. 2 Escape velocity    R    R  ;  Ratio,  1: 2 2
R R 3  vp

21. 2 According to Kepler’s third law, T 2  r 3 or T  r 3/2

3/ 2 3/ 2
D  L 2  L 
   or D  D  2  days
D  L1   L1 

1 1 GM
22. 2 KE of body orbiting close to earth’s surface,  mv02  m  K (given)
2 2 R

1 1 2GM mGM
KE of escaping body  mv 2e  m   2K
2 2 R R
 Energy required to escape the body = 2 K – K = K

Gm2 mv 2 Gm 1 Gm
23. 3 2
 v 
 2R  R 4R 2 R

188
[Study Package - Physics - Class XI]

24. 2 Energy of each satellite in the orbit will be PE and KE


Let V be the velocity of wreckage just after the collision. According to the law of conservation of
momentum; mv0  mv 0  mV or V  0 ; Here KE is zero.

GM 2GMm
The energy (PE only) of wreckage just after collision Ef    2m   
r r

2 4 2 4 2 3
25. 1 Time period of satellite around the plaent is T   R 32 ; GM p  R2
GM p T2
GM p 4 2 R 32
g is the acceleration due to gravity on the surface of planet.  g 
R 12 T 2 R 12

r3 4 2 r 32
26. 4 The time period of satellite T  2 ; Squaring both sides, we get T 
GM e GM e

GM e 4r 3
2
As g  T 
R e2 gR 2e

2 4 2 r 3
Substituting the given values in above equation,  86400   ; r  6.6 R e
98 R e2
27. 3 According to Kepler’s third law T 2  r 3/2
3/ 2 3/ 2
T r   R  2R  1 T2 1 24 24
 2  2     ;  3/ 2 or T2  3/2   6 2 hours
T1  r1   R  5R  23/ 2 24 2 2 2 2
LEVEL - II
1. 4 Mass of object remains same. Weight of object  acceleration due to gravity..
W earth  9 g  earth 
 
W planet  4 g  pl anet 

2 2
9 GM  earth  R  planet  M earth  R  planet  R 2planet  R  earth  R
      9 2  R (planet )  
4 GM  planet  R 2earth  M  planet  R 2earth  R  earth  2 2

GM  G 4 4 4 R 2  g
2. 2 On moon g  2
 2  R 3  GR   G       
R R 3 3 3  4  3  6
4
On earth g  GR
3
u2 1
Now, h max  or h max  or h max g  a constant
2g g
hg hg
 hg  hg or h     6h  6  0.6  3.6 m
g g / 6

189
Brilliant STUDY CENTRE

3. 3 According to the law of conservation of mechanical energy, we have

1 2  GMm  GMm 1  2GM  1 1  GMmh


m  nve       or mn 2    GMm   
2  R  R  h 2  R   R  R  h   R R  h 

Rh 1 R 1  n2 Rn 2
 2 or  2 ; or h 
h n h n 1  n2
GM G 4 3 4
4. 3 On the surface of earth, g  2
 2  R   GR
R R 3 3

R2 g 4 1 1

Acceleration due to gravity at height R is g  g 2
  GR   GR
R  R  4 3 4 3

G 4 3
On the surface of planet. Acceleration due to gravity is g p  R 2 3 R p  3   4GR p
p

1 R
As g p  g so 4G R p  GR or R p 
3 12
2
 1 T g R  h Rh h Th
5. 1 T  2 or T  ;      1  ; T  T 
g g T g R 2
R R R
As T' > T, so the clock will lose the time.

 T  T  T  T h / R  2  200 /  6.4  10 6 

T 2  200   24  60  60 
Loss of time in 1 day  × time of one day   2.7 s
T 2  6.4 106

 d g  d d 1  n  1
6. 2 g  g  1   ;   g 1     1  ;  d  R  
 R n  R R h  n 

gR 2
7. 4 Acceleration due to gravity at a height h from the surface of the earth is g h 
 R  h 2

 6400  2  6400 
2

gh  g 2
 g  -------- (i)
 6400  100   6500 
2
g  gh  
% change in acceleration due to gravity  100%   1   6400    100%  3%
g   6500  
8. 2 The velocity of the mass while reaching the surface of both the planets will be same.
gA
i.e., 2g A h A  2g B h B or h B  h A  2  9  18 m
gB

190
[Study Package - Physics - Class XI]

GM e G  4 / 3 R e3
9. 4 On earth, g e   e ; g e  R e e
R 2e R e2

Acceleration due to gravity of planet g p  R p p

1
R e  e  R p  p  R e  e  R p 2 e  R p  Re
2
10. 3 Let EP be the potential energy of the body while reaching P. Then, Workdone = PE + KE

1 1
4  E P  mv 2  E P   2  22  E P  4 ; or EP = –8 J
2 2
EP 8
Gravitational potential at P, VP     4 J / kg
m 2

11. 3

By the law of conservation of momentum m1v1  m 2 v 2 -------(1)

Gm1m 2 1 1
and by the conservation of mechanical energy 0  0    m1v12  m 2 v 22 ------(2)
d 2 2

2G  m1  m 2 
From above equations, v1  v 2 
d
GMm GM
12. 1 Potential energy at the surface of the earth     2 .mR   gmR
R R
GMm GMm gmR
P.E. at a distance R from the surface of the earth   R  R   2R 
  2

gmR 1
 Gain in the potential energy      gmR   mgR
2 2
3GMm
13. 3 Gravitational potential energy at the centre of the earth is U i  
2R
GMm
Gravitational potential energy at the surface of the earth is U f  
R
GMm  3Gm 
Work done, W  U f  U i    
R  2R 
GMm 3GMm GMm
W  
R 2R 2R

191
Brilliant STUDY CENTRE


14. 2 I  4iˆ  ˆj
Let a particle be moved in this gravitational field through a distance dx along x-axis and dy along


y-axis. Then displacement ds  dx ˆi  dy ˆj 
   
 
Work done by the field is dW  F.ds  m0 I .ds  m 0 4iˆ  ˆj . dx iˆ  dy ˆj  m 0  4 dx  dy  
Since work done is zero, so W  0 or  dW  0
or   4dx  dy   0 or  d  4x  y   0 ; or 4x + y = a constant.
As 4x + y = 3 satisfies the above condition.

2GM GM
15. 3 Escape velocity, v e  ; Orbital velocity, v 0 
R R h

ve GM 1 2GM GM 1 2GM
Given, v 0  or  or   ; Solving we get, h = 3.5 R
3 R h 3 R Rh 9 R
16. 2 The satellite of mass m is moving in a circular orbit of radius r.

GMm
 Kinetic energy of a satellite, K  ------- (i)
2r

GMm
Potential energy of a satellite, U   ------ (ii)
r

GM
Orbital speed of satellite, v  ------ (iii)
r
1/ 2
  4 2  3 
Time-period of satellite, T   r  ------- (iv)
 GM  

Given ms1  4ms 2 ; Since M, r is same for both the satellites S1 and S2.

U s1 ms1
 From equation (ii), we get U  m ;   4 or, U s1  4U s2
U s2 m s2
Option (1) is wrong.
From (iii), since v is independent of the mass of a satellite, the orbital speed is same for both
satellites S1 and S2. Hence option (2) is correct.

Ks ms
From (i), we get K  m ;   1  4 or K s  4K s
1
Hence option (3) is wrong.
K s 2 ms2 1 2

From (iv), since T is independent on the mass of a satellite, time period is same for both the
satellites S1 and S2. Hence option (d) is wrong.
192
[Study Package - Physics - Class XI]

17. 2 By the conservation of mechanical energy


2
GMm 1  3 2GM  GMm
  m     0  h  9 R
R 2 4 R  Rh 7

18. 4 Gravitational force of attraction between sun and planet provides centripetal force for the orbit of
planet.

GMm mv 2 2 GM 2r 2 42 r 2


  ; v  ----- (i); Time period of the planet is given by T  ,T  2
r2 r r v v

4 2 r 2 4 2 r 3
T2   ----- (ii)
 GM  GM
 
 r 

According to question, T 2  Kr 3 ---- (iii)

4 2
Comparing equations (ii) and (iii), we get K   GMK  42
GM

1  GMm  2GM
19. 2 By the conservation of mechanical energy mu 2      00  u 
2  R  R
20. 2 Let h be the maximum vertical height attained by the projected body. According to the law of
conservation of mechanical energy, we have

1 2  GMm   GMm  1 2  2GM  GMm GMm GMmh


m  kve          0 or 2 mk  R   R  R  h  R  R  h 
2  R   Rh   

h k 2R
or k 2

R  h
or k 2
R  k 2
h  h or h 1  k 2
  k 2
R ; or h 
1  k2
2
T22 d 32 1/ n 2  d 32
 3 or n 22 d 32  n12 d13
21. 2 As, T 2  d 3 or 2
1 1 1/ n1  d1
GMm 1 2
22. 2 By using the conservation of mechanical energy  R  R  2 m  fv   0  0
 

2GM 1
Where, v  ; f
R 2

23. 1
2r 2r 2r 3/2 T M2 ; 2T2 M 2 ;  M  4M
T    1    2 1
v GM GM T2 M1 T2 M1
r

193
Brilliant STUDY CENTRE

24. 4 m  1000 kg; g  10m / s2 ; R  6400km

Gm
Energy =  gmR  10  1000  6400  103  6.4  1010 Joules
R
25. 2 Let v be the minimum initial velocity of the mass m to escape. According to law of conservation
of total energy, we have

2GMm 1 4GM GM
  mv 2  0 ; or v 2  or v  2
L 2 L L

GM 2r
26. 2 Orbital velocity, v 0  and T 
r v0

If r increases, v0 decreases. For geostationary satellite, if r increases, v0 also increases for a


fixed value of T. i.e., for geostationary satellite the speed of a satellite increases with an increase
in the radius of its orbit. In fact, a geostationary satellite is launched in the equatorial plane.

194
[Study Package - Physics - Class XI]

CHAPTER - 08
MECHANICAL PROPERTIES OF SOLIDS

SOLIDS
A solid is that state of matter in which its constituent atoms or molecules are held strongly at the positions of
minimum potential energy and it has a definite shape and volume.
Elasticity : The property of matter by virtue of which a body tends to regain its original shape and size after the
removal of deforming force is called elasticity.
Elastic limit:
The maximum deforming force upto which a body retains its property of elasticity is called elastic limit of the
material of body.
Stress
The internal restoring force acting per unit area of cross section of the deformed body is called stress.
At equilibrium, restoring force is equal in magnitude to external force, stress can therefore be defined as external
force per unit area on a body that tends to cause it to deform.
If external force F is applied on the area A of a body then,

Force F
Sress  
Area A
Strain
The ratio of change in configuration to the original configuration is called strain.
It has no dimensions and units.
a) Linear strain : When the deforming force produces a change in length of the body alone, the strain produced
Change in length  l 
is called linear strain or tensile strain. Linear strain 
Original length  l 

Linear strain in the direction of deforming force is called longitudinal strain and in a direction perpendicular to
force is called lateral strain.
b) Volumetric strain : When the deforming force produces a change in volume of the body alone, the strain
Change in volume  V 
produced is called volumetric strain. Volumetric strain  Original volume V
 
195
Brilliant STUDY CENTRE

c) Shearing strain : When the deforming force produces a change in the shape of the body without changing its
volume, strain produced is called shearing strain.
It is defined as the angle in radians through which a plane perpendicular to the fixed surface of the cubical body
gets turned under the effect of tangential force.
x

L

Note: When a beam is bent both compression strain as well as an extension strain is produced.

Hooke’s law and Modulus of Elasticity


According to this law, within the elastic limit, stress is directly proportional to the strain.
stress
ie, stress  strain or  costant=E , E is modulus of elasticity..
strain
E-depends on nature of the material, temperature of the body and the manner in which body is deformed and is
independent of dimensions.

Stress-strain Curve
a) Stress-strain curve vary from material to material.
b) In region OP stress is proportional to strain, obeying Hookes law. Point P is called limit of proportionality and
slope of line OP gives the Young’s modulus Y of the material of the wire. Also Y  tan  .
c) In the region PE, the stress is not proportional to strain. The wire still regains its original length after the removal
of stretching force. E is known as elastic limit or yield-point. The region OPE represents the elastic beahviour of
the material of wire.
d) Beyond the elastic limit E, ie, between EA, the strain increases much more rapidly and if the stretching force is
removed the wire does not come back to its natural length and permanent increase in length takes place.

196
[Study Package - Physics - Class XI]

e) Beyond A, for a very small increase in stress a very large increase in strain is produced (region AB) and after B,
the strain increases even if the wire is unloaded and ruptures at C. In the region BC the wire flows. The
maximum stress (corresponding to B) after which the wire begins to flow and breaks is called breaking or
tensile strength. The force applied is called the breaking force.
Important points.
 Breaking force  A
 Breaking force = P × A; P is the breaking stress.

breaking stress
 The safety factor = working stress

 Breaking of wire under its own weight.


Breaking force = Breaking stress × Area of cross section.
Weight of wire = Mg = ALdg = PA

P
 Ldg  P  L ; This is the length of wire if it breaks by its own weight.
dg

Brittle material
A brittle material when subtended to stress, it breaks without significant plastic deformation. e.g., ceramics,
glass etc.

The plastic region between E and C is small for brittle material and it will break soon after the elastic limit is
crossed.
Ductile material
Ductility is a measure of materials ability to undergo significant plastic deformation before rupture. Malleability
is materials ability to deform under compressive stress, can be made a sheet by hammering. Gold has good ductility
and malleability while lead has low ductility but high malleability.

The material of the wire have a good plastic range and such materials can be easily changed into different shapes
and can be drawn into thin wires.

197
Brilliant STUDY CENTRE

Elastomers
They are polymers capable of recovering their original shape after being stretched to great extents called elastomer.

Stress strain curve is curved within the elastic limit and strain produced is much larger than the stress applied and
have no plastic range. The breaking point lies very close to elastic limit. Example rubber.
Elastic Moduli
There are three moduli of elasticity namely Young’s modulus (Y), Bulk modulus (K) and modulus of rigidity (  )
corresponding to the three types of the strains.
Young’s Modulus (Y)
It is defined as the ratio of normal stress to longitudinal strain within limit of elasticity.
Normal stress F / A FL
Y  
longitudinal strain l / L Al

MgL
If force is applied on a wire of radius r by hanging a weight of mass M, then Y 
r 2l
Important points
 2L  L
(i) If the length of a wire is doubled, longitudinal strain   1
L L

 y  stress  strain  1
FL L
(ii) Increment in the length of wire l  when F and Y are constants.
r 2 Y r2
(iii) When two wires of same length  , same area of cross-section A, Young’s modulii Y1 and Y2 are connected in
2Y1Y2
series. Then the equivalent Young’s modulus is Y  Y  Y
1 2

Y1  Y2
(iv) When two wires are connected in parallel, then equivalent Young’s modulus is Y 
2
Elongation in a wire by its own weight: The weight of the wire Mg act at the centre of gravity of the wire so that
length of wire which is stretched will be L/2.
FL Mg  L / 2  MgL L2 dg
 Elongation l 
AY

AY

2AY

2Y
 M  AL  d
Thermal stress : If a rod is fixed between two rigid supports, due to change in temperature, its length will change
and so it will exert a normal stress (compressive if temperature increases and tensile if temperature decreases) on the
supports. This stress is called thermal stress.

198
[Study Package - Physics - Class XI]

l l
Coefficient of linear expansion   ,  thermal strain  
L L
So thermal stress =  Y
And tensile or compressive force produced in the body  YA
Note: In case of volume expansion Thermal stress = K .
{ K = Bulk modulus,  = cubical expansion coefficient}.
Force constant of wire: Force required to produce unit elongation in a wire is called force constant (k) of material
F F/ A F YA
of wire.  k ----------(i) Also young’s modulus Y    ----------(ii)
l l/L l L

YA
from (i) and (ii) k 
L
Actual length of the wire : If the actual length of the wire is L, then under the tension T1, its length becomes L1 and
under the tension T2, its length becomes L2.
T1 T
L1  L  l1  L1  L  ----(i) and L 2  L  l2  L 2  L  2 ----(ii)
k k

L1T2  L 2 T1
From (i) and (ii) we get L  T2  T1
Work Done in Stretching a Wire
In stretching a wire work is done against internal restoring forces. This work is stored in the wire as elastic
potential energy or strain energy.
FL YA
For a wire of length L streched by x, by a force F, Y  F .x
Ax L

YA
So the work done for an additional small increase dx in length, dw  Fdx  x.dx
L
Hence the total work done in increasing the length by l,
1 1
YA 1 YA 2
W   dW   . x dx  l ; This work done is stored in the wire.
0 0 L 2 L

1 YAl 2 1
 Energy stored in wire U   Fl
2 L 2
Dividing both sides by volume of the wire we get energy stored in per unit volume of wire.

199
Brilliant STUDY CENTRE

1 F l 1 1 2 1 2
U V      stress  strain   Y   strain    stress 
2 A L 2 2 2Y
[As AL = volume of wire]

Bulk Modulus
The ratio of normal stress to the volumetric strain within the elastic limits is called as Bulk modulus (k).

Normalstress F/A  PV
K K 
Volumetric strain ; V / V V
(– sign shows as P increases, V decreases)

where P = increase in pressure ; V = original volume; V = change in volume.

1 V
The reciprocal of bulk modulus is called compressibility. C = 
K PV
Modulus of Rigidity
Within limits of elasticity, the ratio of tangential stress to the shearing strain is called modulus of rigidity of the
Shearing stress
material of the body    ie,   Shearing strain

Consider a cube of material fixed at its lower face and acted upon by a tangential force F at its upper surface
F
having area A. The shearing stress, will be 
A
 is angle of shear..

200
[Study Package - Physics - Class XI]

QQ x
Shearing strain =   
PQ L

shear stress F / A F
So   shear strain    A , shearing is observed only for solids.

Poisson’s Ratio

Lateral strain
   dr / r {–ve sign indicate r decreases as it is streched)
Longitudinal strain dL / L

Poisson’s ratio is a dimensionless and a unitless quantity.

(i) Practical value of Poisson’s ratio 0    0.5

dV dL dL dL 1
(ii) As there is no change in the volume of the wire,   2  1  2   0 , 1  2  0,   
V L L L 2

Relation between Y, K,  and 

Elastic constants are Y, K,  and  . Poisson’s ratio is the ratio of two strains. Elastic constants are related by:
Y  3K 1  2  and Y  2 1    .

Practical Applications of Elasticity

a) The metallic parts of machinery are subjected to a stress within elastic limit, to avoid permanent deformation.

b) The thickness of the metallic rope used in order to lift a load is decided by considering the elastic limit of the
material and the factor of safety.

c) The bridges are declared unsafe after long use since it loses the elastic strength.

I form girders

In designing a beam for its use to support a load (in construction of roofs and bridges), it is advantages to
increase its depth rather than the breadth of the beam.

Consider a bar of length l, breadth b and depth d supported horizontally at its two ends, Let Y be the Young’s
modulus of the material of the bar. When a load W is attached at its middle point, the depression  produced at the
middle point of the bar is given by

Wl 3

4Ybd3
201
Brilliant STUDY CENTRE

b
(a) (b) (c)
In order to have smaller depression  , for a given load, (i) l should be small (ii Y should be large (iii) b should
large and (iv) d should be large. The most effective method to reduce depression in the beam of given length and
1
material is to make depth of the beam large as compared to its breadth because   d3 . But on increasing the
depth too much, the beam shown in Fig.(a) may bend, as shown in Fig. (b). This bending is called buckling. To
check this buckling, a compromise between breadth and depth of a beam is made by using I shaped girder
shown in Fig. (c), with a large load bearing surface.
WL3
For a beam with circular cross-section depression is given by  
12r 4 Y

202
3rd rearrangement 24-2

[Study Package - Physics - Class XI]

QUESTIONS
LEVEL - I
1. A wire is suspended from the ceiling and stretched under the action of a weight F suspended from its
other end. The force exerted by the ceiling on it is equal and opposite to the weight. then
F
1) Tensile stress at any cross section A of the wire is
A
2) Tensile stress at any cross section is zero
2F
3) Tensile stress at any cross section A of wire is
A
4) Tension at any cross section a of the wire is 2F
2. The strain-stress curves of three wires of different materials are shown in the figure. P, Q and R are
the elastic limits of the wires. The figure shows that

1) Elasticity of wire P is maximum 2) Elasticity of wire Q is maximum


3) Tensile strength of R is maximum 4) Elasticity of wire P is minimum
3. A rod is fixed between two points at 20oC. The coefficient of linear expansion of material of rod is 1.1 ×
10–5/oC and Young’s modulus is 1.2 × 1011 N/m2. Find the stress developed in the rod if temperature of
rod becomes 10oC
1) 1.32 × 107 N/m2 2) 1.10 × 1015 N/m2 3) 1.32 × 108 N/m2 4) 1.10 × 106 N/m2
4. A substance breaks down under a stress of 105 Pa. If the density of the substance is 2 × 103 kg/m3,
find the minimum length of the wire made of this substance which will break under its own weight
(g = 10 m/s2)
1) 10 m 2) 2.5 m 3) 4 m 4) 5 m
5. A cable that can support a load of 800 N is cut into two equal parts. The maximum load that can be
supported by either part is
1) 100 N 2) 400 N 3) 800 N 4) 1600 N
6. Identify the incorrect statement
1) Young’s modulus and shear modulus are relevant only for solids
2) Bulk modulus is relevant for solids, liquids and gases
3) Alloys have larger values of Young’s modulus than metals
4) Metals have large values of Young’s modulus than elastomers

203
Brilliant STUDY CENTRE

7. Two wires A and B are of same materials. Their length are in the ratio 3 : 4 and diameters are in the
ratio 5 : 1 when stretched by force FA and FB respectively they get equal in increase in their lengths.
The ratio of FB/FA is
1) 0.01 2) 0.03 3) 0.04 4) 0.05
8. A copper wire and a steel wire of the same diameter and length are connected end to end and a force
is applied, which stretches their combined length by 1 cm. The two wires will have
1) Different stresses and strains 2) The same stress and strain
3) The same strain but different stresses 4) The same stress but different strains
9. A wire is stretched by 0.01 m when it is stretched by a certain force. Another wire of same material
but double the length and double the diameter is stretched by the same force. The elongation in
meters is
1) 0.005 2) 0.01 3) 0.02 4) 0.04
10. What percent of length of a wire will increase by applying a stress of 1 kg weight/mm2 on it. Y = 1 × 1011
Nm–2 and 1 kg wt = 9.8 N
1) 0.0078% 2) 0.0088% 3) 0.0098% 4) 0.0067%
11. Rigidity modulus of steel is  and its Young’s modulus is Y. A piece of steel of cross-sectional area a
is stretched into a wire of length L and area a/10. Then
1) Y increases and  decreases 2) Y and  remain the same
3) Y decreases and  increases 4) Both Y and  increases
12. A thick rope of density  and length L is hung from a rigid support. The increase in length of the rope
due to its own weight is (Y is the Young’s modulus)

1 1 L2g Lg
1) L2 g 2) L2 g 3) 4)
4Y 2Y Y Y
13. A metallic rod of length l and cross-sectional area A is made of a material of Young modulus Y. If the
rod is elongated by an amount y, then the work done is proportional to
1) Y 2) 1/Y 3) y2 4) 1/y2
14. Identical springs of steel and copper (Ysteel > Ycopper) are equally stretched
1) Less work is done on copper spring 2) Less work is done on steel spring
3) Equal work is done on both the springs 4) Data is incomplete
15. If S is stress and Y is Young’s modulus of material of a wire, the energy stored in the wire per unit
volume is :

S2
1) 2 Y/S 2) S/2Y 3) 2S2Y 4)
2Y
16. If a spring extends by x on loading, then the energy stored by the spring is (if T is tension in the spring
and k is spring constant)

T2 T2 2x 2T 2
1) 2) 3) 4)
2x 2k T2 k

204
[Study Package - Physics - Class XI]

17. A wire of length L, modulus Y and cross-section a is stretched through l (when l < < L). If the elastic
limit is not exceeded, the potential energy of the wire increases by

Yal 2 Yal 2 YaL2 YaL2


1) 2) 3) 4)
2L 4L 2l l
18. A wire suspended vertically from one of its ends is stretched by attaching a weight of 200 N to the
lower end. The weight stretches the wire by 1 mm. Then the elastic energy stored in the wire is
1) 0.2 J 2) 10 J 3) 20 J 4) 0.1 J
19. When the pressure on a metal cube is increased by 107 Pa, its volume decreases by 0.015%. The
bulk modulus of the metal (in Nm–2) is
1) 1.5 × 1010 2) 3.33 × 1010 3) 6.67 × 1010 4) 7.5 × 1010
20. The bulk moduli of ethanol, mercury and water are given as 0.9, 25 and 2.2 respectively in units of
V
109 Nm 2 . For a given value of pressure, the fractional compression in volume is V . Which of the

V
following statements about for these three liquids is correct?
V
1) Ethanol > Water > Mercury 2) Water > Ethanol > Mercury
3) Mercury > Ethanol > Water 4) Ethanol > Mercury > Water
21. A solid sphere of radius 20 cm is subjected to a uniform pressure of 106 N m–2. If the bulk modulus of
the solid is 1.7 × 1011 Nm–2, the decrease in the volume of the solid is approximately equal to
1) 0.2 cm3 2) 0.3 cm3 3) 0.4 cm3 4) 0.5 cm3
22. A metal rod of length 1.0 m and cross-sectional area 1.0 mm2 is fixed horizontally between two rigid
supports. The tension in the rod is zero when the temperature is 40oC. Find the tension in the rod
when the temperature falls to 20oC. Young’s modulus of metal = 2.0 ×1011 Nm–2 and coefficient of
linear expansion  1.0  105 K 1

1) 20 N 2) 40 N 3) 4 N 4) 400 N
23. The increase in length of a wire on stretching is 0.025%. If its Poisson’s ratio is 0.4, then the percentage
decrease in diameter is
1) 0.01 2) 0.02 3) 0.03 4) 0.04

1
24. A material has Poisson’s ratio . If a uniform rod of it suffers a longitudinal strain of 3 × 10–2, then the
2
percentage change in volume is
1) 1% 2) 0% 3) 5% 4) 7%
25. The relation between Y,  and B is

1 1 1 9 1 3 1 1 1 9 3 1
1)   2)   3)   4)  
Y 3 9B Y  B  B Y Y  B

205
Brilliant STUDY CENTRE

26. If the Young’s modulus of the material is 6 times its modulus of rigidity, then its volume elasticity will
be

2Y Y Y
1) 2)  3) 4)
3 9 9
27. A beam of metal supported at the two ends is loaded at the centre. The depression at the centre is
proportional to :

1  1 
1) Y2 2) Y 3)   4)  2 
Y Y 

LEVEL - II
1. A cube of metal is subjected to a hydrostatic pressure of 4 GPa. The percentage change in the length
of the side of the cube is close to
(Given bulk modulus of metal, B = 8 × 1010 Pa)
1) 1.67 2) 5 3) 20 4) 0.6

2. A steel wire of cross-sectional area 3  106 m 2 can withstand a maximum strain of 10–3. Young’ss
modulus of steel is 2 × 1011 Nm–2. The maximum mass the wire can hold is (Take g = 10 ms–2)
1) 40 kg 2) 60 kg 3) 80 kg 4) 100 kg
3. The length of the wire is increased by 2% by applying a load of 2.5 kg w.t What is the linear strain
produced in the wire?
1) 0.1 2) 0.01 3) 0.2 4) 0.02
4. The elastic limit of brass is 379 MPa. What should be the minimum diameter of a brass rod if it is to
support a 400 N load without exceeding its elastic limit?
1) 0.90 mm 2) 1.36 mm 3) 1.16 mm 4) 1.00 mm
5. A steel wire with cross-section 3 cm2 has elastic limit 2.4 × 108 N m–2. The maximum upward
acceleration that can be given to a 1200 kg elevator supported by this cable wire if the stress is not to
exceed one-third of the elastic limit is (Take g = 10 ms–2)
1) 12 ms–2 2) 10 ms–2 3) 8 ms–2 4) 7 ms–2
6. The Young’s modulus of three materials are in the ratio 2 : 2 : 1. Three wires made of these materials
have their cross-sectional areas in the ratio 1 : 2 : 3. For a given stretching force the elongation’s in the
three wires are in the ratio
1) 1 : 2 : 3 2) 3 : 2 : 1 3) 5 : 4 : 3 4) 6 : 3 : 4
7. The ratio of diameters of two wires of same material is n : 1. The length of wires are 4 m each. On
applying the same load, the increase in length of thin wire will be :
1) n2 times 2) n times 3) 2n times 4) None of the above
8. If the work done in stretching a wire by 1 mm is 2 J, the work necessary for stretching another wire of
same material but with double radius of cross-section and half the length by 1 mm is

1
1) 16 J 2) 8 J 3) 4 J 4) J
4
206
[Study Package - Physics - Class XI]

9. The radii and Young’s moduli of two uniform wires A and B are in the ratio 2 : 1 and 1 : 2 respectively.
Both wires are subjected to the same longitudinal force. If the increase in length of the wire A is one
percent, ,the percentage increase in length of the wire B is :
1) 1.0 2) 1.5 3) 2.0 4) 3.0
10. Two wires of same length and radius are joined end to end and loaded. The Young’s moduli of the
materials of the two wires are Y1 and Y2. The combination behaves as a single wire then its Young’s
modulus is

2Y1Y2 Y1Y2 YY
1 2 2Y Y
1 2
1) Y  2) Y  3) Y  2 Y  Y 4) Y  3 Y  Y
Y1  Y2 Y1  Y2  1 2  1 2
11. Young’s moduli of two wires A and B are in the ratio 7 : 4. Wire A is 2 m long and has radius R. Wire
B is 1.5 m long and has radius 2 mm. If the two wires stretch by the same length for a given load,
then the value of R is close to
1) 1.9 mm 2) 1.3 mm 3) 1.5 mm 4) 1.7 mm
12. Two springs of equal lengths and equal cross-sectional areas are made of materials whose Young’s
modulii are in the ratio of 2 : 3. They are suspended and loaded with the same mass. When stretched
and released, they will oscillate with time periods in the ratio of

1) 3: 2 2) 3 : 2 3) 3 3 : 2 2 4) 2: 3
13. A uniform heavy rod of weight 10 kg ms–2, cross-sectional area 100 cm2 and length 20 cm is hanging
from a fixed support. Young modulus of the material of the rod is 2 × 1011 Nm–2. Neglecting the lateral
contraction, find the elongation of rod due to its own weight.
1) 5  1010 m 2) 4  108 m 3) 5  108 m 4) 2  109 m
14. The product of Young’s modulus of the material of the wire with its cross-sectional area is equal to its
length. Find the parameters representing x and y-axes of the curve as shown:

o
45

1) load and increase in length 2) stress and strain


3) Young’s modulus and stress 4) Young’s modulus and strain
15. Young’s modulus for a steel wire is 2 × 1011 Pa and its elastic limit is 2.5 × 108 Pa. By how much can
a steel wire 3 m long and 2 mm in diameter be stretched before the elastic limit is exceeded?
[AIIMS 2015]
1) 3.75 mm 2) 7.50 mm 3) 4.75 mm 4) 4.00 mm

16.  10 2

A 5 m long aluminium wire Y  7 10 N / m of diameter 3 mm supports a 40 kg mass. In order to

 10 2

have the same elongation in a copper wire Y  12  10 N / m of the same length under the same
weight, the diameter should now be, in mm
1) 1.75 2) 1.5 3) 2.3 4) 5.0

207
Brilliant STUDY CENTRE

17. A wire fixed at the upper end stretches by length  by applying a force F. The work done in stretching
is

F F
1) 2) F 3) 2F 4)
2 2
18. A rubber cord catapult has cross sectional area 25 mm2. Initial length of rubber cord is 10 cm. It is
stretched by 5 cm and then released to project a stone of mass 5 g. Taking the Young’s modulus for
rubber to be 5 × 108 Nm–2, the velocity of the stone is
1) 20 ms–1 2) 100 ms–1 3) 250 ms–1 4) 150 ms–1
19. A brass rod of cross-sectional area 2 cm2 and length 2 m is compressed length wise by a weight of 10
kg. If Young’s modulus of elasticity of brass is 1011 N/m2 and g = 9.8 m/s2 then increase in energy of
the rod will be
1) 480.2 J 2) 490.1 J 3) 1500 J 4) None

20. The force (F) - extension    , graphs shows that the strain energy stored in the material under test,
for an extension of 4 mm, is greater than which of the following values?

1) 80 mJ 2) 60 mJ 3) 40 mJ 4) None of these
21. Two wires of the same material and same length but diameters in the ratio 1 : 2 are stretched by the
same force. The potential energy per unit volume of the two wires will be in the ratio.
1) 1 : 2 2) 4 : 1 3) 2 : 1 4) 16 : 1
22. The normal density of gold  and its bulk modulus is B. The increase in density of gold when a pressure
P is applied uniformly on all sides is :

P 2P B B
1) 2) 3) 4)
B B P P
23. The compressibility of water is 4 × 10–5 per unit atmospheric pressure. The decrease in volume of
100 cubic centimetre of water under a pressure of 100 atmosphere will be
1) 0.4 cc 2) 4 × 10–5 cc 3) 0.025 cc 4) 0.004 cc
24. The base of a rubber cube of side 3.0 cm is clamped. A horizontal force F of 2.7 N is applied on the top
face. Calculate the horizontal displacement of the top face of the eraser. Shear modulus of rubber =
1.5 × 105 N/m2.
1) 0.2 mm 2) 0.4 mm 3) 0.6 mm 4) 0.8 mm
25. A cube of aluminium of side 6 cm is subjected to a tangential force such that top face shears through
0.012cm relative to the bottom face. The tangential force is k × 1010 dyne. What is the value of k?
[Shear modulus of the material is   2  1011 dyne cm 2 ]
1) 144 2) 14.4 3) 1.44 4) 0.144

208
[Study Package - Physics - Class XI]

KEY WITH HINTS


LEVEL - I
1. 1 When a wire is suspended from the ceiling and stretched under the action of a weight (F)
suspended from its other end, the force exerted by the ceiling on it is equal and opposite to the
weight. However, the tension at any cross section A of the wire is just F and not 2 F. Hence,
tensile stress which is equal to the tension per unit area is equal to F/A
2. 4 As stress is shown on x-axis and strain on y-axis
1 1
So Y  cot    ; So elasticity of wire P is minimum and of wire R is maximum
tan  slope
F
3. 1 Thermal stress  Y    1.2 1011  1.110 5   20  10   1.32  10 7 N / m 2
A
4. 4 Stress at B is maximum, so the wire will break first at B if stress here becomes 105 Pa

B
5
mg ALg 10 m
 105 ;   105  L   5m
A A g
A
5. 3 Breaking stress does not depend upon the length of the cable
6. 3 Metals have larger values of Young’s modulus than alloys and elastomers

YAL r2
7. 2 F F
L L
2
F r2 L 1 3 3 1
 B  B 2  A        0.03
FA (rA ) L B  5  4 4 25

Force Stress
8. 4 Stress  . Since, F and area are same,  stress has to be the same ; Strain 
area Y
Since the Ysteel > Ycopper , strain of steel < strain of copper.
4 FL 4F (2L)  0.01
9. 1 Y  or  2  1   0.005 m
 D  1  (2 D)   2
2 2
2 2

  Stress 9.8  106


10. 3 Strain =    9.8  105 ;  % increase in length
 Y 1  1011


 100  9.8  10 5 100  0.0098%

11. 2 Modulus of rigidity and Young’s modulus of elasticity depend only on material not dimension.
12. 2 As the weight of wire acts at centre of gravity.  Only half the length of wire gets extended.

F  L / 2  Mg  L / 2  MgL ALgL L2 g


Now, Y  .   l =  l   l =
A l Al 2AY 2AY 2Y

209
Brilliant STUDY CENTRE

2
1 1 y
13. 3 W   Stress  Strain  Volume   Y     A  W  y 2
2 2 
1 1
14. 2 Work done, W   F  L ; For a given F, W  L ; But L 
2 Y
1
W ; Ycopper  Ysteel ; W steel < W copper ; So, less work is done on steel spring
Y

1 Stress  stress S2
15. 4 Energy stored per unit volume   stress  strain  
2 2Y 2Y

1 2 1 (kx)2 1 F2 T 2
16. 2 U  kx   
2 2 k 2 k 2k
l
 
17. 1 Work done,  F . dl

0

 l
F L Yal Yal Ya Ya 2
but Y or F  W   L dl  L 0 ldl  2L
A l L 0

Yal 2
 P.E. increases by
2L

18. 4 Elastic energy per unit volume  1  stress  strain


2
1
 Elastic energy  × stress × strain × volume
2
1 F L 1 1
     AL   FL   200  103  0.1J
2 A L 2 2
V 0.015
19. 3 P  107 Pa,  ;
V 100

P 107
B  4
 6.67 1010 Nm 2
V / V 1.5 10
1
20. 1 Compressibility 
Bulk modulus
As bulk modulus is least for ethanol (0.9) and maximum for mercury (25) among ethanol, mercury
and water.
V
Hence compression in volume
V
Ethanol > Water > Mercury

210
[Study Package - Physics - Class XI]

p
21. 1 The bulk modulus is given by B  V / V
 
Vp 4 3
 Decrease in volume V  where V  r
B 3
3 6
4  0.2   10
 V   11
 0.2  106 m3  0.2 cm3
3 1.7  10
T
22. 2 Thermal stress = YT ;    YT ;
A

 T  AYT  1.0  10 5   1.0  10 6    2.0  1011    40  20   40N

D / D D   
23. 1 Poisson ratio,   ;  % decrease in diameter    100      100 = 0.01%
 /  D   
24. 2 Volume of a rod  r 2 h

V 2r L dr / r
  ; 
V r L dL / L
dV dL  1 dV
 (1  2)   1  2   3 102 ;  100  0
V L  2 V
9 1 3
25. 4 The relation between Y,  and B is  
Y B 
26. 4 As we know that Y  2G 1   

Y
6G  2G 1    ;    2 ; Y  3B 1  2   B 
9

Mg 3 1
27. 3  
4bd 3 Y Y
LEVEL - II
1. 1 Given : P  4GPa  4 109 Pa
B = 8 × 1010 Pa

P.V V P 4 109 1
According to bulk modulus formula, B  ,   10
 ...(i)
V V B 8 10 20

V 3a 2 3a 2 a 1 V
V  a3,  . a  3 . a , 100  100
V V a a 3 V
1 1
  100 1.67% (using (i))
3 20
211
Brilliant STUDY CENTRE

2. 2 Maximum stress = Young’s modulus × maximum strain = 2  1011  103  2 108 Nm 2


 Maximum force (F) = maximum stress × area = 2 ×108 × 3 × 10–6 = 600 N
F 600
Maximum mass    60 kg
g 10
2 102
3. 4 Let L be the original length of the wire. Increase in length, L  L  L L  1.02L
100 100
L 0.02L
 Change in length, L  L  L  1.02L  L  0.02L ; Strain    0.02
L L

4. 3 For minimum diameter, P  F or d 2 


F 4

400  4
or d  1.16 mm
2
d  P 3.14  379 106

4
1
5. 2 Maximum tension in the wire is T   stress × area of cross-section
3
1
 (2.4 108 )  (3 104 )  2.4 10 4 N
3
If a is the maximum upward acceleration of elevator, then T = m (g + a)
or 2.4 × 104 = 1200 (10 + a)
On solving, we get a = 10 ms–2
FL 1
6. 4 l and for a given stretching force l 
AY AY
Let three wires have young’s modulus 2Y, 2Y and Y and their cross sectional areas are A, 2A
and 3A respectively.
1 1 1 1 1 1 1 1 1
l1 : l2 : l3  : :  : :  : :  6:3: 4
A1Y1 A 2 Y2 A 3 Y3 A  2Y 2A  2Y 3A  Y 2 4 3
2 2
1   d 2   1 
7. 1   2          2  n 2  1
d  2  d1   n 

Yr 2 L
8. 1 Stretching force, F  ; Both the wires are of same material, so Y will be equal,
L
extension in both the wires is same, so L will be equal
2
r2 F  2r  L
F ;    2  8 ; or F  8F ------(i)
L F L / 2 r

1
Work done in stretching a wire, W   F  L
2
W F
For same extension W  F ;    8 [Using (i)]; W  8W  8  2J  16J
W F

212
[Study Package - Physics - Class XI]

FL L F
9. 3 Young’s modulus, Y  or  2
r 2 L L r Y
L 1
For the same force,  2
L r Y
L B
2 2
LB  rA   YA   2   1  L B  L 
or    2
L A  rB   YB   1   2  or 2 A 
LB  LA 
LA

L B  L 
 100  2  A 100   2(1%)  2%
LB  LA 
10. 1 Let the length of both the wires is  and radius is r..
Let the load is F, extension in the respective wries is 1 and  2 .
So, by the formula of Young’s modulus,
F
Y
A. 

  ...... (i)
Y
Since, the wires are in series combination, so equivalent length,  eq      2

Now, using (i)


 eq  
 eq   1   2   
Yeq Y1 Y2

2   2Y1Y2
   Yeq 
Yeq Y1 Y2 Y1  Y2

FL FL
11. 4 Young’s modulus of wire, Y  
AL R 2 L
2
Y1 L1  R 2 
For the same load and stretch,   
Y2 L 2  R 1 
1/ 2
L1Y2  4 2 
or R1  R 2  2  103    1.75 mm
L 2 Y1  7 1.5 

213
Brilliant STUDY CENTRE

F L F YA
12. 1 Young’s modulus Y  . ; Force constant k  
A l l L
where l is the extension in the spring of original length L and cross-sectional area A when a force
F = Mg is applied. Now, the time period of vertical oscillations is given by

M ML  T1  Y2  3
T  2  2 ;
k YA T2 Y1 2

13. 1 Weight, W = 10 kg m/s2

Area, A = 100 cm2; length, L = 20 cm

Young’s modulus, Y  2  1011 N / m 2

Weight acts at the mid point of the rod.

W.  L / 2  WL WL
Y   or L 
A. L 2AL 2AY

where L is the elongation of the rod due to its own weight.

10  0.2
 L  4 11
 5  1010 m
2 100  10  2  10

stress F F F
Y  ; YA  ; given YA   ;   
    F
14. 1 ;
strain A  

 y
Also  1 ; i.e.,  tan 45o  1 . So x and y axes represent F and  .
F x

15. 1 The cross-sectional area of a wire of radius r = 1 mm = 10–3 m is A  r 2  3.14  106 m 2

The maximum force that can be applied without exceeding the elastic limit is therefore

F
F      A    2.5 108 Nm 2  3.14 106 m 2   785N
 A max

When this force is applied, the wire will stretch by

L 
L F

 3m  785N   3.75  103 m  3.75 mm
Y A  2 10 N / m 2  3.14 106 m 2 
11

214
[Study Package - Physics - Class XI]

FL 1
16. 3 l 2
 r2  (F, L and l are constant)
r Y Y
1/ 2 1/ 2
r2  Y1   7 1010 
   10 
r1  Y2   12  10 
1/ 2
 7
 r2  1.5     1.145 mm ;  dia = 2.29 mm
 12 

FL
17. 4 Young’s modulus Y  ........ (i)
A

YA
F
L


YA  d  YA YA 2
or dW  F d  or  dW   d  
L L 0 2L

YA 2
or work done  ........ (ii)
2L

F
From (i) and (ii); Work done 
2

1
18. 3 W mv 2 Kinetic energy of the stone.
2

1 YA 2
U
2 L

1 1 YA 2
mv 2  YA2 / L ; v  250 m / s
2 2 mL

1 (Stress)2 1 F2  A  L
19. 1 Work done    Volume  
2 Y 2 A2  Y

1 F2 L 1 (10  9.8) 2  2 9604


W     
2 AY 2 2 104 1011 2 107

W = 4802 × 10–7 Joule

W  480.2 J

215
Brilliant STUDY CENTRE

20. 3 Area under the curve represents strain energy stored and it is greater than 40 mJ.

1 1
Area under curve (straight line)  .F.l  .  20  .  4  103  J  40mJ
2 2

2
1 Stress  1 S2
21. 4 Potential energy per unit volume of the wire is U  
2 Young 's modulus 2 Y 2

Force S1  F1  A 2 
As stress, S      
Area S2  F2  A1  ;

S1 A 2
But, F1 = F2 (Given)  
S2 A1 -----(i)

As the two wires are of the same material, therefore their Young’s moduli are the same. i.e., Y1
= Y2.

2 2 2 4 2
U1  S1   A 2   D 2    D 2 
          
U 2  S2   A1   D1    D1 
 

4
 2  16  D1 1 
      given   (Using (i))’
1 1  D2 2 

M
22. 1  ...(1) (Mass is a constant quantity)
V

  V
 ...(2)
 V

P  P P
B  or   
   B B

1 V / V
23. 1 C   V  CP  V  4  105 100  100  0.4 cc
K P

216
[Study Package - Physics - Class XI]

24. 3

x 3 cm

L = 3 cm

Area of top face (A) = 3 cm × 3 cm = 9 cm2 = 9 ×10–4 m2

F 2.7
Shearing stress =  4
 3  103 N / m 2
A 9  10

shearing stress 3  103 x


Angle of shear    5
 2.0  102 ; tan  
shear modulus 1.5  10 L

x  L tan   L   is small  = 3cm × 2.0 × 10 = 6 × 10 cm = 0.6 mm


–2 –2

25. 3 Lateral displacement, x = 0.012 cm  Shearing strain,

x 0.012 F
   0.002 Shearing stress,  
L 6 a

 2  1011  0.002  4  108 dynecm 2

Shearing force, F = shearing stress × area

2 2
 4  108   L   4  108   6   1.44  1010 dyne  K  1.44

217
Brilliant STUDY CENTRE

CHAPTER - 09
MECHANICAL PROPERTIES OF FLUIDS

HYDROSTATICS AND HYDRODYNAMICS


Any state of matter that can flow is a fluid. Study of fluids at rest is fluid statics and fluids in motion is
called fluid dynamics or hydrodynamics.
Pressure

F
It is defined as the magnitude of the normal force  F  acting on unit surface area  A  , P 
A

The pressure is a scalar. Unit of pressure N/m2, or Pascal ; 1 Pascal = 1 N/m2


Atmospheric pressure
The pressure exerted by the atmosphere is called atmospheric pressure. At sea level 1 atm. = 1.013 ×
105 Pa.
Gauge Pressure
The excess of pressure of a system above Atmospheric pressure is called Gauge pressure. e.g., Tyre
Pressure.
Absolute pressure
Absolute pressure = Atmospheric pressure + Gauge Pressure
Variation of pressure with depth
Consider a column of a liquid of height h, Let P1 is the pressure on the top of the column and P2 is the
pressure on the bottom of the column of area A. P2 > P1.

218
[Study Package - Physics - Class XI]

Force exerted by the liquid column at the bottom = Force due to atmosphere + Force due to liquid at
the bottom.

P2 A  P1A  hAg ; where  is the density of the liquid.  P2  P1  hg

dP
So along the liquid column the pressure gradient is  g .
dh
Pascals law: The pressure applied to an enclosed fluid is transmitted equally to every part of the fluid and
to the walls of the container.
At the same level in a liquid the pressure is the same.
Hydraulic lift

It is used to lift heavy objects. A liquid is enclosed in a vessel, by two pistons of area A1 & A2,
where A2 > A1.

F1
When a force F1 is applied on the liquid at the piston of smaller area, then P 
A1 .

The pressure is transmitted equally to the other part of the liquid having larger area. The force applied
F1
at the other end F2 = P × A2  F2   A2 then F2 > F1.
A1

So that a large force is applied on the second piston.


Note: Hydraulic brakes in automobiles work on this principle.
Relative density
The ratio of the density of a material to the density of a reference material (usually water at 4oC) is
density of substance
called the relative density. RD 
density of a reference liquid

Weight of body

Weight of equal volume of water

Weight of body Weight of body in air W1


  
Loss of weight in water Weight in air  Weight in water W1  W2

219
Brilliant STUDY CENTRE

Density of a mixture of substance

Let m1, m2, m3 etc. are the masses and 1 ,  2 , 3 etc. are the densities of the substances respectively,,
when mixed together. Then the total mass of mixture = m1 + m2 + m3

Total volume of mixture  m1  m 2  m3


1 2 3

m1  m 2  m 3
 Density of the mixture is   m m m
1
 2 3
1  2 3

m1  m 2
for two substance    
 m12  m21  1 2
Buoyant force (upthrust)
If a body is partially or wholly immersed in a fluid, it experiences an upward force, due to the fluid
surrounding it. This force exerted on the body by the fluid is called Buoyant force and the phenomenon is
called Buoyancy.
Archimedes’ Principle
It states that when a body is partially or totally immersed in a liquid, it loses a weight equal to the weight
of the liquid displaced by it.

Let a body of density  immersed in a liquid of density  .

Force experienced on the top of the body F1  P1  A  A  h1g  P0 

upward force experienced on the bottom of the body  F2  P2 A  A  h 2 g  P0 

 net upward force = F = F2 – F1  Ag  h 2  h1  ;  F  AgL  Vg {  AL = V

Features of Archimedes Principle


(1) Apparent weight of the body of density (  ) when immersed in a liquid of density (  ).
Apparent Weight = Actual Weight – Upthrust force

   
 W  Fup  Vg  Vg  Vg 1    WAPP  W  1  
   
220
[Study Package - Physics - Class XI]

(2) If a body of volume V is immersed in a liquid of density  then its weight reduces.
W 1 = Weight of the body in air.; W 2 = Weight of the body in water.

W1  W2
Then apparent (loss of weight) weight W1  W2  Vg V 
g
Floatation

When a body of density  B and volume V is immersed in a liquid of density  , the forces acting are

1) The weight of the body W = mg = VB g acting vertically downwards through the centre of gravity of
the body.

2) The upthrust (Th) = Vg acting vertically upwards through the centre of gravity of the displaced liquid
(centre of buoyancy)
Then according to the following situations the body will sink or float.

a) If  B   , then weight of the body is greater then the upthrust, the body will sink.

b) If  B   , then W = Th, so the body will be completely submerged, and there will be neutral
equilibrium any where in the liquid.

c) If  B   , then W < Th, the body moves up and will be in equilibrium and will float in the liquid such
that

W  VA g (where VA is the volume of the body inside the liquid.)

 V B g  VA g ; V B  VA 
 The law of floatation states that, a body will float in a liquid if weight of the liquid displaced by the
immersed part of the body is equal to the weight of the body.
Hydrodynamics
1. Steady flow
It is defined as the type of flow of a fluid in which the fluid characteristics does not change with time. In
unsteady flow, velocity, pressure and density at a point in the liquid flow varies with time.
Stream line flow
In a steady flow, all the particles passing through a given point follow the same path and hence a
unique line of flow, then the flow is called streamline. The velocity of the particles in stream line remain
constant or vary in a regular manner.
Laminar and Turbulent flow
The flow of a viscous fluid in which particles of the fluid move in parallel layers, each of which has a
constant velocity but is in motion relative to its neighbouring layers. Laminar flow over a horizontal surface
may be thought of as consisting of thin layers or laminae, all parallel to each other. The layer near the
surface is stationary and all the other layers slide over each other.
Turbulant flow of a liquid in which the fluid undergoes irregular fluctuations or mixing takes place. The
speed of the fluid at a point is continuously undergoing changes in magnitude and direction.

221
Brilliant STUDY CENTRE

Critical velocity
During the steady flow of a liquid, if the velocity of the liquid is below a particular value, the motion of the
liquid is streamline. If it crosses this velocity the flow of the liquid becomes Turbulent. Then the fluid particles
don’t follow the path of their preceeding particles. The maximum velocity upto which fluid motion is streamline
is called critical velocity (vc).
Equation of continuity
For a steady flow of an incompressible and non viscous fluid through a tube of non-uniform cross
section, the product of the area of cross section and the velocity of flow is same at every point in the tube.
ie, A × v = a constant.

A2
v2
A1 C
v1
B

A1 and A2 are the area of cross section of a tube of non uniform cross section (A2 > A1) at B & C
respectively. Let v1 is the velocity of liquid at B, and v2 is the velocity of liquid flowing at C. The volume of
liquid flowing through the tube at B, per sec = A1v1,
Since the liquid is in compressible, and the liquid cannot accumulate in the tube, mass of the liquid
entering the tube must be equal to mass of the liquid coming out per sec.

 A1 v1  A 2 v 2  Av  a constant
This is the equation of continuity and represents the conservation of mass in case of moving fluids.
Energy of a flowing fluid
1. Pressure energy
If P is the pressure on the area A of a fluid and the liquid moves through a distance due to this pressure,
then the pressure energy of liquid = work done = force × displacement = P × A × L

PAL
 pressure energy per unit volume = P
AL
Kinetic energy
1m 2 1 2
For a volume V of liquid of mass m and velocity v, then KE per unit volume    v  v
2 V 2
Potential energy

m
For a liquid of volume V and mass m at a height, PE per unit volume    gh  gh
V
Bernoullis Theorem
According to Bernoullis theorem for the steady flow of an incompressible and non-viscous fluid through
a tube of non-uniform cross section, the sum of pressure energy per unit volume, the potential energy per
unit volume and kinetic energy per unit volume is same at every point in the tube. i.e., P  gh  ½v 2 = a
constant.
 Lifting force on an aeroplane wings works on this principle.
222
[Study Package - Physics - Class XI]

Application of Bernoullis theorem


1) Venturimeter

It is used to measure the flow speed of a liquid through a pipe of non-uniform cross section. At the
points A and B, applying Bernoulis equation

P1  ½v12  P2  ½ v 22 since h1 = h2

A1 v1
from equation of continuity A1 v1  A 2 v 2 ; v2 
A2

 A2 
 P1  P2  ½v12  12  1 = gh is the pressure difference.
 A2 

2gh dV 2gh
then v1  ; The volume flow rate  A1v1  A1 2
 A1 
2 dt  A1 
  1   1
 A2   A2 
2) Toricellis’ Theorem (speed of efflux)

Liquid is flowing out through the orifice B. The pressure at A & B are the same, P0, atmospheric pressure.
The speed at A is very small (vA = 0) compared to speed at B. By Bernoullis’ theorem.

P0  gH  ½vA2  P0  g  H  h   ½v B2 

½v 2B  gh, v B  2gh


It is the velocity acquired by a liquid when it is freely falling through (h) from the surface to the orifice and
is called velocity of efflux.

2H  h
Note:  For a uniformly accelerated motion in the y-direction, H  h  0  ½gt 2  t 
g

223
Brilliant STUDY CENTRE

2H  h
  Horizontal Range R  v  t  2gh   2 h H  h
g

d
 Range is maximum when R2 is maximum. i.e.,
dh
 R2   0

d H
ie, 4
dh
 hH  h 2   0 ; i.e., H – 2 h = 0, h 
2

H H
 Rmax = 2 2  H  2   H So the maximum range a liquid can have is the height of the liquid
 

A 2H
 If the hole is at the bottom of the tank, time ‘t’ to make the tank empty t  A g
, where A0 is the
0

area of orifice and A is the area of the container.

VISCOSITY
Viscosity is the property of the fluid by virtue of which an internal frictional force comes into play when
the liquid is in motion and opposes the relative motion between adjacent layers. Force between the layers
opposing the relative motion is called viscous force.

According to Newton the tangential viscous force F acting on any layer of a fluid is directly proportional
dv dv dv
to its area A and to the velocity gradient at the layer. F A or F  A
dx dx dx
Velocity gradient at a layer of a fluid is the rate of change of speed with distance in a direction
dv
perpendicular to the flow given by ; dv - change in velocity for a distance dx between two layers.
dx
 is the coefficient of viscosity..

dv
When  1 ,and A = 1, F   ,  coefficient of viscosity is the viscous force acting between two
dx
layers of a liquid having unit area and unit velocity gradient.

Dimension is ML–1T–1, and unit is poise (cgs) dyne sec-cm–2 and in SI units Poisenille (Nsm–2). 1 pl = 10
Poise

Note: (1) Viscosity of liquids is much greater than that of gases, L  G (about 100 times or more).

(2) When temperature increases viscosity of liquids decreases, and viscosity of gases increases.

(3) When density increases, viscosity of liquids increases while in gases viscosity decreases.

(4) With increase in pressure, the viscosity of liquids except water increases. While in gases viscosity
has no variation. For water viscosity decrease with pressure.

224
[Study Package - Physics - Class XI]

Reynold’s number

Reynold’s showed that when fluids flow through the tubes, the critical velocity depends on density  

Viscosity    of the fluid and on the diameter (D) of the tube.

      v cD
vc    or v c  N R    NR 
 D   D  

NR is the Reynolds number, if the value of NR is less than 2000, the flow is streamline.
If NR > 3000 the flow is turbulent.
If the value of NR is between 2000 to 3000, the flow can be streamline or turbulent.
Poiseuille’s Formula
For the steady flow of a liquid through a capillary tube of length L and radius r, under a pressure
Pr 4
difference of P across it, the volume of the liquid flowing per second is given by Q 
8 L

This is Poiseuille’s Formula.


Stokes Law
When a body moves through a fluid, the fluid opposes the motion of the body due to viscosity. The
magnitude of the viscous force depends on shape and size of the body and also on the velocity of the body
and viscosity of the liquid.
Stokes stated that if a sphere of radius r moving with a velocity v, through a liquid of viscosity  , then
the viscous force opposing the motion of the sphere is F  6rv

4 3
When the liquid is moving down the weight of the body acts downwards W  mg  r g and the
3
4 3
upthrust of the liquid U  r g acting upwards, where  and  are the densities of sphere and liquid
3
respectively.

4 3 4 4 3
Net force acting downwards  r g  r 3g  r g     
3 3 3
At a certain state the viscous force balances the net downward force and after that the body falls with
4 3
constant velocity called Terminal velocity (vT), then 6rv T  r      g
3

2 r2
vT       g );  vT  r 2
9

ie, when radius increases the terminal velocity increases.

225
Brilliant STUDY CENTRE

SURFACE TENSION
Intermolecular Force
The force of attraction or repulsion acting between the molecules are known as intermolecular force.
The nature of intermolecular force is electromagnetic.
The intermolecular forces of attraction may be classified into two types.

Cohesive force Adhesive force


The force of attraction between molecules of The force of attraction between the molecules of
same substance is called the force of the different substances is called the force of
cohesion. This force is lesser in liquids and adhesion.
least in gases gases.

Surface Tension
The property of a liquid by which its free surface tends to have minimum surface area and behaves as
if it were under tension acting like a stretched membrane is called surface tension. Due to this liquid drops
takes spherical shape to reduce surface area.
Surface tension of a liquid is the force acting per unit length on an imaginary line, drawn on the free
surface of the liquid. The direction of the force is perpendicular to the line and tangential to the surface of
the liquid.

F
Let F is the force acting on a line of length L, then T  . Unit: N/m
L
Factors Affecting Surface Tension
(1) Temperature: The surface tension of liquid decreases with rise of temperature. The surface tension
of liquid is zero at its boiling point and it vanishes at critical temperature.
(2) Impurities: The presence of impurities, considerably affect the force of surface tension, depending
upon the degree of contamination.
Applications of Surface Tension

(1) The oil and grease spots on clothes cannot be removed by pure water because pure water-does not wet
oily clothes. On the other hand, when detergents (like soap) are added in water, the surface tension of
water decreases. As a result of this, wetting power of water increases. Thus, oil, grease and dirt particles
get mixed with water easily. Hence clothes are washed easily.

(2) The antiseptics have very low value of surface tension. Due to low surface tension, the antiseptics
spreads properly over wound.

226
[Study Package - Physics - Class XI]

Surface Energy

The molecules on the liquid surface experience net downward force. So to bring a molecule from the
interior of the liquid to the free surface, some work is required to be done against the intermolecular force of
attraction, which will be stored as potential energy of the molecule on the surface. The potential energy of
surface molecules per unit area of the surface is called surface energy.

Unit : Joule / m2 (S.I.) erg / cm2 (C.G.S.); Dimension : [MT–2]

If a rectangular wire frame ABCD, equipped with a sliding wire LM dipped in soap solution, a film is
formed in the frame. Due to the surface tension, the film will have a tendency to shrink and thereby, the
sliding wire LM will be pulled in the inward direction. However, the sliding wire can be held in this position
under a force F, which is equal and opposite to the force acting on the sliding wire LM all along its length due
to surface tension in the soap film.

If T is the force due to surface tension per unit length, then F = T × 2l.

Here, l is length of the sliding wire LM. The length of the sliding wire has been taken as 2l for the reason
that the film has got two free surfaces.
Suppose that the sliding wire LM is moved through a small distance x, so as to take the position L'M'. In
this process, area of the film increases by 2l × x (on the two sides) and to do so, the work done is given by

W  F  x   T  2l   x  T   2lx   T  A

 W  T  A [ A = Total increase in area of the film from both the sides.’

If temperature of the film remains constant in this process, this work done is stored in the film as its
surface energy.

W
From the above expression T  or T  W [If A  1]
A
ie, surface tension may be defined as the amount of work done in increasing the area of the liquid
surface by unity against the force of surface tension at constant temperature.
Work done in Blowing a Liquid Drop or Soap Bubble
If the initial radius of liquid drop is r1 and final radius of liquid drop is r2 then

W = T × Increment in surface area  T  4  r22  r12  [drop has only one free surface]

W  T  8  r22  r12  [Bubble has two free surface]

227
Brilliant STUDY CENTRE

Splitting of Bigger Drop


When a drop of radius R splits into n smaller drop, (each of radius r) then surface area of liquid increases.
Hence the work is to be done against surface tension.

4 3 4
Since the volume of liquid remains constant therefore R  n r 3
3 3

 R 3  nr 3

or R  n1/3r

Work done = T  A  T  T  n4r 2  4 R 2   4T  nr 2  R 2 

Excess Pressure
Due to the property of surface tension a drop or bubble tries to contract and so compresses the matter
enclosed. This in turn increases the internal pressure which prevents further contraction and equilibrium is
achieved. So in equilibrium the pressure inside a bubble or drop is greater than outside and the difference of
pressure between two sides of the liquid surface is called excess pressure.

2T
Note:  Excess of pressure inside a liquid drop P 
R

4T
 Excess of pressure inside a soap bubble P 
R

2 2
 Radius of the new bubble formed when two bubbles coalesce, r  r1  r2

r1r2
 Radius of interface when two soap bubbles of different radii are in contact, r  r  r
2 1

Angle of Contact
Angle of contact between a liquid and a solid surface is defined as the angle enclosed between the
tangents to the liquid surface and the solid surface inside the liquid, both the tangents being drawn at the
point of contact of the liquid with the solid.

  90o   90o   90o

concave meniscus. plane meniscus. convex meniscus.


Liquid wets solid surface. Liquid does not wet solid surface. Liquid does not wet solid surface.
On increasing the temperature, angle of contact decreases.

228
[Study Package - Physics - Class XI]

Capillarity
If a tube of very narrow bore (called capillary) is dipped in a liquid, it is found that the liquid in the capillary
either ascends or descends relative to the surrounding liquid. This phenomenon is called capillarity.
Examples of capillarity :
(i) Ink rises in the fine pores of blotting paper leaving the paper dry.
(ii) A towel soaks water.
(iii) Oil rises in the long narrow spaces between the threads of a wick.
Ascent Formula
When one end of a capillary tube of radius r is immersed into a liquid of density d which wets the sides
of the capillary tube (water and capillary tube of glass), the shape of the liquid meniscus in the tube becomes
concave upwards.
R = radius of curvature of liquid meniscus; T = surface tension of liquid; P = atmospheric pressure

2T
Pressure at point A = P, Pressure at point B  P  .
R

Pressure at points C and D just above and below the plane surface of liquid in the vessel is also P
(atmospheric pressure). The points B and D are in the same horizontal plane in the liquid but the pressures
at these points are different.
In order to maintain the equilibrium the liquid level rises in the capillary tube into height h (Tube 2).
Pressure due to liquid column = pressure difference due to surface tension.

2T 2T 2T cos   r 
 hg  ;  h   As R  cos  
R Rg rg  

 If   90o , h is positive i.e., liquid rises in capillary tube.

 If   90o , h is negative i.e, liquid falls in a capillary tube.


Note: When a capillary tube dipped vertically in a liquid is tilted, length of liquid (  ) in capillary tube increases
but vertical height of liquid (h) in the tube above the surface of liquid in trough remains the same.
h
Then,   or h   cos 
cos 

229
Brilliant STUDY CENTRE

QUESTIONS
LEVEL - I
1. In a hydraulic press the small cylinder has a diameter of ‘d1’ cm, while the large piston has a diameter
of ‘d2’ cm. If a force ‘F1’ applied to a small piston, the force on the large piston ‘F2’ is given by

d12 1 d 22 d12 1 d12


1) F2  2) F2  F1 3) F2  4) F2  F1
d 22 F1 d12 d 22 F1 d 22
2. An ice cube floats in water. What percentage of volume is outside water. Density of water = 1 g/cc,
density of ice = 0.9 g/cc.

1) 10% 2) 90% 3) 1% 4) 15%

3. 
A height H of water   1000 kg m
3
 column which can produce the same pressure as that of a

mercury column   1300 kg m
3

of height 1 m at the bottom of a vessel is

1) 0.136 m 2) 1.36 m 3) 13.6 m 4) 136 m


4. A body measures 5 N in air and 2 N when put in water. The buoyant force is
1) 7 N 2) 9 N 3) 3 N 4) 6 N
5. For a body immersed in a liquid, when the weight of the body is less than the upthrust then the body
will
1) float partially immersed 2) sink
3) float fully immersed 4) be of zero weight
6. A cubical block of wood 10 cm along each side floats at the interface between an oil and water with its
lowest surface 2 cm below the interface. If the heights of oil and water columns are 10 cm each and
oil  0.8 g / cc , find the mass of the block.
1) 408 gm 2)804 gm 3) 480 gm 4) 840 gm
7. A large cylindrical tank has a hole of area A at its bottom. Water is poured in the tank by a tube of equal
cross-sectional area A ejecting water at the speed v.
1) The water level in the tank will keep in rising
2) No water can be stored in the tank
3) The water level will rise to a height v2/2g and then stop
4) The water level will oscillate
8. An old age arteries carrying blood in the human body become narrow resulting in an increase in the
blood pressure. This follows from
1) Pascal’s law 2) Stoke’s law
3) Bernoulli’s principle 4) Archimede’s principle

230
[Study Package - Physics - Class XI]

9. Air is streaming past the wings of an aeroplane with a speed of 90 ms–1 below and 120 ms–1 above the
surface. If the wing is 15 m long and has an average width of 2 m, then :[Density of air = 1.2 kgm–3]
1) pressure difference between the surface of the wing is 4090 Pa
2) up-lift on the wing is 81900 N
3) up-lift on the wing is 113.4 kN
4) pressure difference between the surface of the wings is 2680 Pa
10. Water is flowing streamline motion through a horizontal tube. The pressure at a point in the tube is p
where the velocity of flow is v. At another point, where the pressure is p/2, the velocity of flow is
(density of water=  )

p p 2p 2p
1) v2  2) v2  3) v2  4) v2 
   
11. Water is flowing through a channel that is 12 m wide with a speed of 0.75 ms–1. The water then flows
into four identical channels that have a width of 4.0 m each. The depth of the water does not change
as it flows into the four channels. The speed of the water in one of the smaller channels is

1) 0.56 ms 1 2) 2.3 ms 1 ` 3) 0.25 ms 1 4) 0.75 ms 1


12. Two spheres of the same material, but of radii R and 3R are allowed to fall vertically downwards
through a liquid of density  . The ratio of their terminal velocities is
1) 1 : 3 2) 1 : 6 3) 1 : 9 4) 1 : 1
13. Adding detergents to water helps in removing dirty greasy stains. The is because
(a) It increases the oil-water surface tension
(b) It decreases the oil-water surface tension
(c) It increases the viscosity of the solution
(d) Dirt is held suspended surrounded by detergent molecules
1) b and d 2) a only 3) c and d 4) d only

14. A rectangular film of liquid is extended from (4 cm × 2 cm) to (5 cm × 4 cm). If the work done is 3  104 J ,
the value of the surface tension of the liquid is
1) 0.250 N m–1 2) 0.125 N m–1
3) 0.2 N m–1 4) 8.0 N m–1
15. Two small drops of mercury, each of radius R, coalesce to form a single large drop. The ratio of the
total surface energies before and after the change is
1) 1 : 21/3 2) 21/3 : 1 3) 2 : 1 4) 1 : 2

231
Brilliant STUDY CENTRE

16. A tall cylinder is filled with viscous oil. A round pebble is dropped from the top with zero initial velocity.
From the plot shown in figure, indicate the one that represents the velocity (v) of the pebble as a
function of time (t).

17. Two water droplets coalesce to form a large drop. In this process
1) energy is liberated
2) energy is absorbed
3) energy is neither liberated nor absorbed
4) a small amount of mass is converted into energy in accordance with Einstein’s mass energy
equivalence relation E = mc2.
18. A solid sphere with a terminal velocity of 20 m/s in air. If it is allowed to fall in vacuum
1) The terminal velocity will be 20 m/s
2) The terminal velocity will be less than 20 m/s
3) The terminal velocity will be more than 20 m/s
4) There will be no terminal velocity
19. The surface tension of soap solution at a temperature 20oC is 2.5 × 10–2 N m–1. The excess pressure
inside a bubble of soap solution of radius 6 mm is
1) 12.5 Pa 2) 14.2 Pa
3) 15.5 Pa 4) 16.7 Pa
20. Choose the correct statement.
1) Terminal velocities of rain drops are proportional to square of their radii
2) Water proof agents decrease the angle of contact between water and fibres
3) Detergents increase the surface tension of water
4) Hydraulic machines work on the principle of Torricelli’s flow
21. The ratio of radii of two soap bubbles is 2 : 1. The ratio of excess pressures inside them is
1) 1 : 2 2) 1 : 4 3) 2 : 1 4) 4 : 1
22. The wettability of a surface of a liquid depends primarily on
1) density
2) angle of contact between the surface and the liquid
3) viscosity
4) surface tension

232
[Study Package - Physics - Class XI]

23. If two soap bubbles of different radii are connected by a tube


1) air flows from the bigger bubble to the smaller bubble till the sizes become equal
2) air flows from the bigger bubble to the smaller bubble till the sizes are interchanged
3) air flows from the smaller bubble to the bigger
4) there is no flow of air
24. Two soap bubbles of radii r1 and r2 equal to 4 cm and 5 cm are touching each other over a common
surface S1S2 (shown in figure). Its radius will be

1) 4 cm 2) 20 cm 3) 5 cm 4) 4.5 cm
25. A glass rod of diameter d = 2 mm is inserted symmetrically into a glass capillary tube of radius r = 2
mm. Then the whole arrangement is vertically dipped into liquid having surface tension 0.072 N/m.
3
The height to which liquid will rise on capillary is (Take g = 10 ms–2, liq  1000 kg / m . Assume
contact angle to be zero of capillary tube to be long enough)
1) 1.44 cm 2) 6 cm 3) 4.86 cm 4) None of these
26. Water rises to a height h in capillary tube. If the length of capillary tube above the surface of water is
made less than h, then
1) water rises upto a little below the top and stays there
2) water does not rise at all
3) water rises upto the tip of capillary tube and then starts overflowing like a fountain
4) water rises upto the top of capillary tube and stays there without overflowing
27. If the surface tension of water is 0.06 Nm–1, then the capillary rise in a tube of a diameter 1 mm is
1) 1.22 cm 2) 2.44 cm 3) 3.12 cm 4) 3.86 cm
28. A barometer tube reads 76 cm of mercury. If the tube is gradually inclined at an angle of 60o with
vertical, keeping the open end immersed in the mercury reservoir, the length of the mercury column
will be

1) 152 cm 2) 76 cm 3) 38 cm 4) 38 3

LEVEL - II
1. The cross-section of two pistons in a hydraulic lift are 100 cm2 and 800 cm2 respectively. What is the
minimum force required to lift 5000 kg-wt on large piston surface? (in kN)
1) 392 2) 400 3) 412 4) 500
2. The force acting on a window of area 50 cm × 50 cm of a submarine at a depth of 2000 m in an
ocean, interior of which is maintained at sea level atmospheric pressure is (Density of sea water =
103 kg m–3, g = 10 ms–2)
1) 106 N 2) 5 × 105 N 3) 25 × 106 N 4) 5 × 106 N

233
Brilliant STUDY CENTRE

3. A cylindrical vessel is filled with a liquid of density  to a height h such that the force exerted by the
liquid on the bottom is equal to the force exerted on the walls of the vessel. Then h should be
1) equal to radius 2) more than the radius
3) less than the radius 4) two times the radius
4. A piece of metal weighs 155 g in air, 150 g in water and 130 g in liquid. The relative density of the
metal and that of liquid are respectively
1) 5, 5 2) 30, 5 3) 31, 5 4) 10, 15
5. A body floats in water with 40% of its volume outside water. When the same body floats in an oil, 60%
of its volume remains outside oil. The relative density of oil is
1) 0.9 2) 1.0 3) 1.2 4) 1.5
6. A hollow sphere of inner and outer diameter of 12 cm and 16 cm respectively floats half submerged in
water. The specific gravity of the material of the sphere is
1) 8.65 2) 6.85 3) 0.865 4) 0.685
7. An iceberg floats on sea water with a portion of it being submerged. If the relative density of ice is 0.9
and that of sea water is 1.125, the fraction of the whole volume of the iceberg that appears on the
surface of sea water is

2 3 1 4
1) 2) 3) 4)
5 5 5 5
8. A cubical block of wood of specific gravity 0.5 and a chunk of concrete of specific gravity 2.5 are
fastened together. The ratio of mass of wood to the mass of concrete which makes the combination to
float with its entire volume submerged in water is:

1 1 3 2
1) 2) 3) 4)
5 3 5 5

1
9. Two blocks A and B are made of different kinds of wood. Block A floats in water with th of its volume
4
2
above the surface of water. Block B floats in water with rds of its volume below the surface of water..
3
The ratio of the densities of A and B is
1) 3 : 2 2) 5 : 3 3) 9 : 8 4) 4 : 3
–2 –2
10. Two water pipes P and Q having diameter 2 × 10 m and 4 × 10 m respectively are joined in series
with the main supply line of water. The velocity of water flowing in pipe P is :
1) Four times that of Q 2) Two times that of Q

1 1
3) time that of Q 4) times that of Q
2 4
11. The reading of a manometer fitted to a closed tap is 3.5 × 105 Pa. If the valve is opened, the reading
of the manometer falls to 3 × 105 Pa. The velocity of water is
1) 1 ms–1 2) 10 ms–1 3) 100 ms–1 4) 0.1 ms–1

234
[Study Package - Physics - Class XI]

12. At what speed, the velocity head of the water is twice than that of pressure head of 75 cm of Hg?
1) 10 m/s 2) 20 m/s 3) 30 m/s 4) 40 m/s
13. Fig. shows a venturimeter, through which water is flowing. The speed of water at X is 2 cm/s. The
speed of water at Y (taking g = 10 ms–2) is

1) 0.23 ms–1 2) 0.32 ms–1 3) 1.01 ms–1 4) 10.24 ms–1


14. Water is filled in a cylindrical container to a height of 3 m. The ratio of the cross-sectional area of the
orifice and the beaker is 0.1. The square of the speed of the liquid coming out from the orifice is
(g=10 m/s2)

1) 50 m 2 / s 2 2) 50.5 m 2 / s 2 3) 51m 2 / s 2 4) 52 m 2 / s 2
15. A cylindrical vessel filled with water upto the height H becomes empty in time t0 due to a small hole at
the bottom of the vessel. If water is filled in the same vessel to a height 4H it will flow out in time:
1) 8 t0 2) 4 t0 3) 2 t0 4) t0
16. Water from a pipe is coming at a rate of 100 litres per minute. If the radius of the pipe is 5 cm, the
Reynolds number for the flow is of the order of (density of water = 1000 kg m–3, coefficient of viscosity
of water = 1 m Pa s)
1) 102 2) 104 3) 103 4) 106
17. A square plate of 0.1 m side moves parallel to a second plate with a velocity of 0.1 m/s, both plates
being immersed in water. If the viscous force is 0.002 N and the coefficient of viscosity is 0.01 poise,
distance between the plates in m is
1) 0.1 2) 0.05 3) 0.005 4) 0.0005
18. The rate of steady volume of water through a capillary tube of length l and radius r under a pressure
difference P is V. This tube is connected with another tube of same length but half the radius in series.
Then the rate of steady volume through them is (the pressure difference across the combination is P)

V V 16V 17V
1) 2) 3) 4)
16 17 17 16
19. A small sphere of radius ‘r’ falls from rest in a viscous liquid. As a result, heat is produced due to
viscous force. The rate of production of heat when the sphere attains its terminal velocity, is proportional
to
1) r3 2) r2 3) r5 4) r4

235
Brilliant STUDY CENTRE

20. Two solid spheres of the same metal but of mass M and 8 M fall simultaneously in a viscous liquid. If
their terminal velocities are v and nv, then the value of n will be
1) 16 2) 8 3) 4 4) 2
21. Consider a soap film on a rectangular frame of wire of area 4 × 4 cm2. If the area of the soap film is
increased to 4 × 5 cm2, the work done in the process will be (The surface tension of the soap film is
3 × 10–2 N/m)
1) 12 × 10–6 J 2) 24 × 10–6 J 3) 60 × 10–6 J 4) 96 × 10–6 J
22. The soap bubble formed at the end of the tube is blown very slowly with time. Which group is best
suited for the excess pressure inside the bubble with time.

23. Work done in increasing the size of a soap bubble from a radius of 3 cm to 5 cm is nearly (Surface
tension of soap solution = 0.03 Nm–1)
1) 4 mJ 2) 0.2 mJ 3) 2 mJ 4) 0.4 mJ
24. A frame made of metallic wire enclosing a surface are a is covered with a soap film. If the area of the
frame of metallic wire is reduced by 50%, the energy of the soap film will be changed by
1) 100% 2) 75% 3) 50% 4) 25%
25. The air pressure inside a soap bubble of radius R exceeds the outside air pressure by 10 Pa. By how
much will the pressure inside a bubble of radius 2R exceed the outside air pressure?
1) 20 Pa 2) 40 Pa 3) 2.5 Pa 4) 5 Pa
26. The excess pressure inside a spherical drop of water is four times that of another drop. Then their
respective mass ratio is :
1) 1 : 16 2) 1 : 64 3) 1 : 4 4) 1 : 8
27. If ‘M’ is the mass of the water that rises in a capillary tube of radius ‘r’, then mass of water which will
rise in a capillary tube of radius ‘2r’ is

M
1) 2) 4 M 3) M 4) 2 M
2
28. A capillary tube of radius r is immersed vertically in a liquid such that the liquid rises in it to height h
(less than the length of the tube). Mass of liquid in the capillary tube is m. If radius of the capillary
tube is increased by 50%, then mass of liquid that will rise in the tube, is

2 4 3 9
1) m 2) m 3) m 4) m
3 9 2 4
29. The lower end of a glass capillary tube is dipped in water. Water rises to a height of 8 cm. The tube is
then broken at a height of 4 cm. The height of water column and angle of contact will be
1) 2 cm, 60o 2) 4 cm, 120o 3) 4 cm, 60o 4) 2 cm, 30o

236
[Study Package - Physics - Class XI]

KEY WITH HINTS


LEVEL - I
1. 2

F2 F
In hydraulic press, pressure on larger piston = pressure on smaller piston,  1
A 2 A1

A 2 F1 d 22 4 d 22 d 22 F1
 F2     F1  F1 2  F2  2
A1 4 d12 d1 d1

2. 1

In equilibrium FB = W;  Vin g  ice Vg

Vin ice 0.9 Vin


  Volume outside water  1   1  .9  .1 ie, 10%
V  1 V

3. 3 The pressures are the same at the bottom of the vessel. Therefore,  w gH  m gh m

 m h m 13600  1
 H   13.6 m
w 1000
4. 3 Buoyant force = Weight of liquid displaced = True weight – Observed weight = 5 N – 2 N = 3 N
5. 1 When the weight of the body is less than the upthrust, then the body will float partially immersed.
6. 4 (a) For the floating cube,

Weight of the cube = Buoyant force  Voil g  Vw g

 mg  Boil  Bwater  8 102 oil g  2 102  w  g

 m  800  0.8  200 1  640  200  840 gm

v2
7. 3 Av  A 2gh max ; h max 
2g

237
Brilliant STUDY CENTRE

8. 3 Bernoulli’s principle,
In old age arteries get thicker and narrow. Due to narrowness and other obstruction the velocity
of the flow of blood gets decreased. This results in increased pressure inside the blood vessel.

 2 1.2
9. 3 p 
2
 v 2  v12  
2
1202  902  = 0.6 (14400 – 8100) = 3780 Pa
Force = p  area  3780  l  b  3780 15  2 = 113400 N = 113.4 kN

1 1 p 1
10. 1 According to Bernoulli’s theorem, p  v 2 = constant;  p  v 2   v2
2 2 2 2

2 p p
or v   v 2 or v  v 2 
 
11. 1

Av  A1v1  A 2 v 2  A3 v3  A 4 v 4 Av  4  A1v1 

3
12  0.75  4 (4  v) ; v  0.75  0.75  0.75 ; v = 0.56 ms–1
4
12. 3 Terminal velocity of a body varies directly as the square of the radius of the body. i.e, v  r 2
2
v
 1 
 R  or v : v  1: 9
1 2
v 2  3R  2
13. 1
14. 2 Workdone = Surface tension of film × Change in area of the film. T × A

A1  8cm 2 , A 2  20 cm 2 , A  2  A 2  A1   24  10 4 m 2 , W = 3 × 10–4 J

W 3 104 1
 T  4
  0.125 Nm 1
A 24 10 8

R2
15. 2
1/3 1/3 2 2/3 2
As R   n R  2 R  R   2 R  2
 2 2/3
R

Initial surface energy 2  4R T 


2
 R2  2/3 1/3
  2  2   2  2  2
Final surface energy  4R  T 
2
R 
16. 3 When a round pebble is dropped from the top of a tall cylinder, filled with viscous oil the pebble
acquires terminal velocity after some time.
17. 1 When a big drop breaks up into smaller drops, the total surface area of the smaller drops is
more that the surface area of the big drop. The increase in the surface area can be brought
about by supplying energy. Thus a big drop has to absorb energy to break up into smaller drops.
On the other hand, when smaller drops coalesce to form a big drop, there is a decrease in the
surface area. Hence energy is liberated in this process.

238
[Study Package - Physics - Class XI]

18. 4 When a body falls through a fluid it drags the layer of the fluid in contact with it. In vacuum,
there is no drag and hence no terminal velocity.

4S 4  2.5 102
19. 4 Excess pressure inside the soap bubble, P    16.7 Pa
r 6 103

2r 2      g
20. 1 (a) The terminal velocity of a rain drop is v t  ; Thus v t  r 2
9

(b) Water proof agents increase the angle of contact between the water and fibres.
(c) Detergents decrease the surface tension of water.
(d) Hydraulic machines work on the Pascal’s law

4T 4T 4T
21. 1 Excess pressure inside the soap bubble is P  ;  P1  -----(i) and P2 
R R1 R 2 ------(ii)

P1 R 2  1   R1 2 
     Given  
Divide (i) by (ii), we get
P2 R1  2   R2 1 
22. 2

4T
23. 3 Pressure inside the bubble  P0 
r
Smaller the radius, greater will be the pressure. Air flows from higher pressure to lower pressure.
Hence air flows from the smaller bubble to the bigger.

r2 r1 5 4
24. 2 Radius of curvature of common surface of double bubble r    20 cm
r2  r1 5  4

25. 1 Here, surface tension force support the weight of liquid rising in the tube.

2T
h , where r  r2  r1
rg

.072  2
h  .0144 m  1.44 cm
1 103 1000 10

26. 4 Water will not overflow but will change in its radius of curavture.

2Scos  2  0.06  cos 0o


27. 2 h  3 3
 2.44 102 m  2.44 cm
rg 0.5 10 10  9.8

h h 76
28. 1 cos 60o  l   152 cm
l 1/ 2 1 / 2

239
Brilliant STUDY CENTRE

LEVEL - II

1. 1 As we know F1A1  F2 A 2

A2 800
F2  F2   5000  9.8 
A1 100

F1  5000  9.8  8 ; F1  392000N

2. 4 h = 2000 m,   103 kgm 3 , g  10 ms 2 . The pressure outside the submarine is P  Pa  gh

where P is the atmospheric pressure. Pressure inside the submarine is Pa.


Hence, net pressure acting on the window is gauge pressure.

Gauge pressure, Pg  P  Pa  gh  103 kgm 3  10ms 2  2000m  2  107 Pa

Area of the window is A  50cm  50cm  2500  10 4 m 2

7 4 2 6
Force acting on the window is F  Pg A  2 10 Pa  2500  10 m  5  10 N

3. 1 Force exerted by the liquid on the bottom. F1 = pressure × area = gh  r 2

The force exerted on the walls of the vessel.

h
F2  g    2r  h
 2

hg
[Mean pressure on the walls of the beaker is ]
2

But F1  F2  gh r 2  gh 2 r hr

weight in air
4. 3 Relative density of solid 
weight in air  weight in water

155
 Relative density of solid 
155  150

155
R.D.   31
5

weight in air  weight in liquid 155  130 25


Relative density of liquid    5
weight in air  weight in water 155  150 5

240
[Study Package - Physics - Class XI]

5. 4 According to Archimedes’ principle. Weight of a body = Weight of the liquid displaced.


Let V be volume of the body.

In water Vbody g  0.6V water g -------(i); In oil V body g  0.4Voil g ---------(ii)

0.6 water  3
Divide (i) by (ii), we get 1  or oil   1.5 = Relative density of the oil.
0.4oil water 2

6. 3

For equilibrium, Fg = Fbuoy  mg  Fbuoy ; So, Vg  Vw  w g

4 3  2 3
 3   R  r      3 R   w
3
This implies
   

 1 1
Solving this,  3
  0.865
w  r    6 3 
2 1  3  2 1    
 R    8  

7. 3 Weight of iceberg = weight of the liquid displaced by the submerged part of the ice berg mg = m'g

V ice
Vice g  V w g  
V  water , where V' is the volume of the submerged portion of the ice berg
and V is the total volume of the ice berg.

V 0.9 V
 , where is the fraction of the volume submerged. Fraction of the whole volume
V 1.125 V
V V 0.9
that appears above the sea water is  1  1 ;
V V 1.125
V 0.225 225 1
  
V 1.125 112.5 5

241
Brilliant STUDY CENTRE

8. 3 Let volume of block = V1; Volume of concrete = V2

 Displaced volume of water   V1  V2 

Weight of the combination = Bouyant force;  0.5  V1  g  2.5  V2  g   V1  V2   1 g

V1 3 M 0.5  V1 3
 ; 1  
V2 1 M 2 2.5  V1 5

9. 3 Let V1 be the volume of block A and 1 is density. Then, from the principle of flotation, we have

 1 3
1V1g   1   V1   g or 1 
 4 4
where  is the density of water. Similarly for block B we have

2V2 2 1 3 3 9
 2 V2 g  g or  2     
3 3 2 4 2 8
10. 1 Rate of flow through both pipes will be same, i.e., Q1 = Q2
2
V1 V2 r12 1 r22 2 d2  d2  4 10 2 
 ;  ;  1 vP  2  v Q ;  v P   2 
vQ ;  vP  4 v Q
t t t t 4 4  2  10 

1 1 1 2 1
11. 2 p1  12  p2  22 ; 2   p1  p 2   12   p1  p 2   1  0 
2 2 2 2

2  p1  p 2  2   3.5  105  3  105 


or 2   3 = 10 ms–1
 10

P V2 V 2 2P 4P
12. 2 Pressure head  ; velocity of head  ;   V2 
g 2g 2g g 

4  75 102 13.6 103  9.8


V2  3
 39984  102  399 ; V = 20 m/s
10

 2  2
13. 2 Px  Py 
2
 v y  v x2  ; hg 
2
 v y  v2x 

1 2 1
hg 
2
 v y  v x2   0.51 1000   v 2y  2 2  ; v 2y  1024 ;
2
v y  32 cm / s

2gh 2 10   3  0.525  2


14. 1 Velocity of efflux, v 2   ; v 2  50  m / sec 
2 2
a 1   0.1
1  
A

242
[Study Package - Physics - Class XI]

A 2h t 4H
15. 3 t  t h   2
A0 g t0 H

dV dx
16. 2 The rate of flow  r 2  r 2 v
dt dt

1 dV 100 103 1 2
v  2
  4
 m/s
r dt 60  25  10 3

v cD 2 103  5  2 10 2 2 105


So, the Reynolds number,  . 3
  2.12  104
 3 10 3

dv
17. 4 F  A (drag force or viscous force)
dx

Adv 0.001 0.01 0.1


dx    0.0005 m [As 0.01 Poise = 0.001 decapoise (SI unit)
F 0.002
2
A   0.1  0.01m 2 ]

 Pr 4
18. 2 V ; when tubes are connected in series, then the rate of flow of water through each
8l
tube (V1) is same. Let (P – P1) be the difference in pressure across the first tube and P1 be the
difference in pressure across the second tube. Then

4
  P  P1  r 4 P1  r / 2  P1 16P
V1   (i); or P  P1  or P1 
8l 8l 16 17

 16P /17   r 4 /16  1  Pr 4 V


From (i), V1    
8l 17 8l 17

19. 3 The viscous drag force, F  6rv ; where v = terminal velocity..

 The rate of production of heat = power = force × terminal velocity..

 Power  6rv.v  6rv 2 ..... (i)

2r 2      g
 Terminal velocity v  ;  v  r2
9

 4r 4      2 2 
Now, power = 6r  812 g ; or Power  r5.
 

243
Brilliant STUDY CENTRE

20. 3  The solid spheres are of same metals therefore, have same density, i.e.,   

M 8M R
    2  R  2r
4 3 4 3 2
r R
3 3
For solid sphere of mass M, terminal velocity

2R 2     
v g ..... (i)
9
For solid sphere of mass 8 M terminal velocity
2
2  r      g
v  ..... (ii)
9

 v  nv  n  4 (from (i) and (ii)

21. 2  Initial surface area of the film  2  4  4cm 2  32cm 2

Final surface area of the film  2  4  5cm 2  40cm 2

4
4 2 4
Increase in surface area  40  10  32  10 m  8  10 m
2

Work done W = Surface tension × Increase in surface area
  3  10 2    8  10 4   24  10 6 J

22. 3 As the soap bubble come to the end of tube slowly its radius increases slowly.

1
P  . Hence pressure decrease with time. Slope is parabolic downward.
r
2 2
23. 4 S  0.03Nm 1 , r1  3  10 m, r2  5  10 m

Initial surface area of the bubble  2  4r12  72  10 4 m 2

Final surface area of the bubble  2  4r22  200 10 4 m 2

Increase in surface area  200104  72 10 4  128 10 4


 Work done = S × increase in surface area
 0.03  128  10 4  3.84 104  4 104 J  0.4mJ
24. 3 Surface energy = surface tension × surface area or U = S × 2A.

A
New surface energy, U1  S  2    S A
2

U  U1 2SA  SA
% decrease in surface energy   100%   100%  50%
U 2SA

244
[Study Package - Physics - Class XI]

4S
25. 4 Excess pressure inside the soap bubble P  Pi  P0 
R
where S is the surface tension of soap solution and R is the radius of the bubble.

P2 R 1 R 1 1 1
    P  P   10Pa  5Pa
P1 R 2 2R 2 ; or 2 2 1 2

2T
26. 2 P 
R

1
Pressure  [Where, P0 = Express pressure, T = Surface Tension, R = Radius]
Radius

P1 R 2 4P2 R2 R1 1
  ; P1  4P2     M V
P2 R1 P2 R1 ; R2 4

3 3
M1  R1   1  1
3
And V  R  M  R ; 3
MR ;3 So,     
M2  R2   4  64

2
27. 4 h cos  , h  1
rg r

h 2 h
If r  2r, h  , now, M    V   r 2 h ;  M    r 2 h     2r   2M
2 2

2Tcos  1 h r 2
28. 3 h h  2  1 
rg r h1 r2 3

 3 
 r1  r, r2  r  50% of r  r1 
 2 

2
3  2  3 3
New mass m 2  r2 h 2     r1   h1     r1 h1    m
2 2

2  3  2 2

29. 3 Water rises in capillary tube due to force of adhesion between solid and liquid molecules. Tube
is broken at height of 4 cm, so force of adhesion is present till 4 cm.
So height of water column is 4 cm.

h h  h 2S 
  1  
cos  cos   cos  rg 

8 4 1
 o
  cos      60o
cos 0 cos  2

245
Brilliant STUDY CENTRE

CHAPTER - 10
THERMAL PROPERTIES OF MATTER

Heat
Heat is a form of energy that is transferred between systems with different temperatures (flowing from
high to low temperature system.). Due to heat there is the sensation of hotness or coldness in a body.
Temperature
Degree of hotness or coldness of a body is due to its temperature. It is a measure of the average KE of
the syste.
Thermometry
Thermometry is the branch of Physics concerned with the measurement of temperature and the design
and use of thermometers.
Scales of temperature
The SI unit of temperature is kelvin which is defined as (1/273.16)th part of thermodynamic temperature
of triple point of water (ice, water and water vapour co-exist in dynamic equilibrium at this temperature.)
Celsius scale or centigrade scale is also common.
Conversion between different scales.

TC  0 T  32 TK  273.15
 F  --------(1)
100  0 212  32 373.15  273.15
The change in temperature in different scales will be

TC TF TK 9 9


  ; F  C  32 , C  K  273 , F  32   K  273 
100 180 100 5 2
Triple point of water 273.16 K (0.01oC) at a pressure of 6.11 × 102 P is taken as the reference point.
THERMOMETERS
A thermometer is used to measure the temperature of a body. The variation of some physical property
of a substance is used to measure temperature.
Liquid Thermometer
Thermal expansion of a liquid, usually mercury or alcohol is used to measure temperature. Mercury
has the advantage that it has uniform and large expansion and high thermal conductivity and low specific
heat capacity. The range of such thermometer is –50oC to 350oC.

246
[Study Package - Physics - Class XI]

Gas Thermometer

An ideal gas at constant volume or at constant pressure can be used for measuring temperature.

For constant volume gas thermometer P  T

P  P0
TC   100o C
P100  P0

F or constant pressure V  T , Based on Charles law..

Resistance Thermometer

Variation of resistance of metals with temperature is used to measure temperature. Since platinum has
high melting point and expansion coeffecient large, is used in resistance thermometry.

R  R0
TK   100o C
R 100  R 0

Thermoelectric Thermometer

Thermocouples which apply Seebeck effect is used to measure temperature. Thermo-emf generated
at a temperature is given by e  aTC  bTC2 , a & b are constants.

Pyrometers

Pyrometers are used to measure very high temperature such as of a furnace or glowing filament.
These are primarily based on ‘Stefan’s law’ of thermal radiations, viz., R  eATK4

THERMAL EXPANSION
When matter is heated the interatomic distance and the matter as a whole expands. Thermal expansion
for solids are minimum and gases are maximum.

Solids have linear expansion, superficial expansion and volume expansion while liquids and gases
have only volume expansion.

L
For eg. coefficient of linear expansion   ; L  L 1  T 
L  T

A
Coefficient of superficial expansion   ; A  A 1  T 
A  T

V
Coefficient of volume expansion   ; V  V 1  T 
V  T

Relation between ,  and 

2   ; 3   ;  :  :   1: 2 : 3

247
Brilliant STUDY CENTRE

Application of thermal expansion in solids


Bimetallic strip
It is used in a thermostat. Two strips of equal length but different expansions are joined together. When
heated the strip will bend with metal of greater  on the outer side.

This type of arrangement is used in electrical heating circuits.


A Scale gives correct reading at a temperature  , at a different temperature    , the scale expands and
the reading will be less than actual length.
True value  SR 1   ; SR - scale reading
However, if    , due to contraction of scale, scale reading will be more than true value. So true value
will be lesser than scale reading and is given by True Value, TV  SR 1   
When object only is expanding
Measured value = True value 1  0 t  ,    1  0 t 

When object and scale are expanding    1    0   s   t ; 0 -  of object; s -


scale
A pendulum clock gives correct time at a temperature  . At a temperature    , due to linear expansion
the length of the pendulum increases and hence the time period will increase. Due to this time interval
measured will be less.
Time loss or gain in pendulam clocks
Period of pendulam clock

 0 1  t 
T0  2  / g ; when temperature increases by t ; T  2
g

 1  1 
By binomial theorem T  T0 1   t  ; T  T0  T0   t 
 2  2 
T 1 1
  t where t is the change in time period.; T   t  T0
T0 2 2
when t is positive, clock goes slow; when Temperatures decreases the clock goes fast.
Thermal Stress
The ends of a rod is rigidly fixed to prevent expansion or contraction, when undergoes a change of
temperature, a stress is developed on the rod. Due to this stress, the rod will exert a large force on the
supports.
L
Thermal strain =   ; Thermal stress = Y and force F  YA
L
248
[Study Package - Physics - Class XI]

Thermal Expansion in Liquids


Liquids have only volume expansion. When a liquid is heated in a container, the container also expands
with liquid.
The actual increase in volume of liquid
= The apparent increase in volume of liquid + The increase in the volume of the vessel.

Apparant Expansion in Volume V


Coefficient of apparent expansion of liquid      
Initial Volume  rise of temperature V  

Re al increase in volume V
Coefficient of Real expansion of liquid   r   
Initial volume  rise in temperature V  

Vvessel
Coefficient of expansion of vessel   V 
V  

 r   v   s ;  r     3 vessel

m
Density of a liquid  
V

When temperature increases by  , mass remains same. But volume increases V  V 1   

m m 
   
V V 1    1    temperature rises, density decreases.

Thrust due to a liquid of volume V is given by, T  Vg (  is density)

When temperature rises, New thrust T  Vg

T Vg V 1   v 
   ; If  v   r , T  T
T Vg V 1   r 
With rise in temperature, thrust will decrease.
This implies that when a body is weighed in a liquid at different temperatures, when temperature
increases the weight of the body decreases.

 1   V  
W  Vg  V 1   V    g  Vg  
1   r    1   r  

1 1
 W0 1   V  1   r     W0 1   V  1   r     W0 1   r    V    V  r  2 
   

Since  V and  r are small,  V  r is very small, so that it is neglected.  W  W0 1    V   r    -


----(7)

249
Brilliant STUDY CENTRE

Anomolous expansion in liquids


Normally matter expands on heating and contracts on cooling. For water it expands on heating if its
temperature is greater than 4oC. In the range of 0oC to 4oC water contracts on heating and expands on
cooling (coefficient of volume expansion in this range is negative). This behaviour of water in the range 0oC
to 4oC is called anomolous expansion. At 4oC density of water is maximum and specific volume is minimum.

CALORIMETERY
Heat
Heat is a form of energy. SI unit of heat is Joule. Also heat can be measured in calories. 1 cal = 4.18 J
Calorie: It is defined as the amount of heat required to raise the temperature of 1g of water by 1oC.
International Calorie : It is the heat required to raise the temperature of 1 g of water from 14.5oC to 15.5oC
at a pressure of 1 atm.
work done
When heat is given to a body work is done. The ratio of  4.18 J per cal
heat produced
is called mechanical equivalent of heat.
Specific heat capacity: It is the amount of heat required to raise the temperature of a unit mass of the
substance through unit degree.

1 dQ
The specific heat of a substance s 
m dT
The heat required to raise the temperature of mass m by  is Q  ms
Molar heat capacity (C)
The molar heat capacity of a substance is the amount of heat required to raise the temperature of one
mole of a substance through 1oC.
Molar heat capacity C = Molecular Weight (M) × specific heat capacity (s)
Let molecular mass of the substance is (M) and mass of the substance is (m), the number of moles in
m
the substance  
M
1 dQ M  dQ 
C   
 dT m  dT 
SI unit : - J/mole - K
A gas has two specific heat capacities depending on whether the gas is expanding at constant volume
or at constant pressure. (specific heat at constant volume CV and specific heat at constant pressure Cp).

250
[Study Package - Physics - Class XI]

Thermal Capacity
The quantity of heat required to raise the temperature of the whole body through 1oK is known as its
thermal capacity.
Thermal capacity = mass × specific heat capacity. Unit - Cal/K
Water equivalent of a body
Mass of water having the same thermal capacity as the body is called its water equivalent.
Water equivalent = mass of body × sp. heat of the material (w = ms)
Latent heat
The heat energy to be supplied to a substance to change its state without change of temperature is
called latent heat.
Latent heat of vapourisation
The heat energy required to convert unit mass of a liquid to the vapour state without change of temperature
is latent heat of vapourisation. For water it is 2.26 × 106 Jkg–1 or 536 cal/g.
Latent heat of Fusion
It is the heat energy required to convert unit mass of a solid into the liquid state without change of
temperature.
L.H. of fusion of ice is 3.36 × 105 Jkg–1 or 80 cal/g
Regelation
It is the process of melting of ice by applying pressure and its resolidification when pressure is removed.
Variation of melting point and boiling point with pressure applied.
Increase of pressure lowers melting point of water. Increase of pressure increases the boiling point of
water.
Phase diagram
A phase diagram is a graph drawn between Temperature and pressure of a substance. A phase of a
substance is a region of a material that is chemically uniform, physically distinct and mechanically separable.
ice, water and steam are different phases of water.
Phase diagram for water

On each part of the curve the substance is in a particular phase and all the three phases co-exist at a
point called triple point where the solid, liquid and gaseous states co-exists. The ice line, steam line or
sublimation line separates the two states of matter.

251
Brilliant STUDY CENTRE

Principle of calorimetry
When two bodies, one being a liquid and the other being a solid or both being liquid, at different
temperatures are mixed, heat will be transferred from body at a higher temperature to the body at a lower
temperature till both acquire the same temperature, so that Heat lost = Heat gained.

The heat given to a body Q  mcT , where c is the specific heat capacity and T is change in
temperature.

TRANSFER OF HEAT
Heat is transferred from a body at a higher temperature to a body at lower temperature.
The transfer of heat from one body to another takes place by conduction, convection or radiation.
Conduction
Conduction is the process of transfer of heat from particle to particle without the actual movement of
the particles. In solids and mercury heat is transferred by conduction.
Consider a slab of face Area A, and lateral thickness L, whose face have temperatures TH & TC, (TH >
TC )

L
TC

A
TH
Q2
Q1
x
dx

For a small element dx of a slab. If Q is the heat crossing through the area A, for a time t, then
Q dT dT
  KA , where K is the thermal conductivity. is the temperature gradient.
t dx dx

dT
Q = K when A = 1,  1 and t = 1
dx

Q
K  Js 1m 1K 1 or Wm 1K 1
 dT 
A 
 dx 

Q  KA
 TH  TC  t
L
Thermal Resistance
It is the ability to oppose heat energy flows through the conductor.

Temperature difference T
Heat current = 
Thermal resistance R

252
[Study Package - Physics - Class XI]

kAT
We have H 
L
L
Thermal Resistance is R
KA
where L is the length of conductor, A is the Area of the conductor and K is the thermal conductivity.

T kelvin
Unit. R  
H Watt
a) Two slabs joined in series

L1 L2

A A L1 L
; Req = R1 + R2   2
K1 A K 2 A
1 K1 K2 2

L1  L 2 L1K 2  L 2 K1 K1K 2  L1  L 2  2k1k 2



K1K 2 A ;
K eq  ; If  1   2 ;  k eq
K eq A  L1K 2  L2 K1  k1  k 2

 1  2 
In series the total heat conducted H1  H 2  H   
 R eq. 

b) When they are in parallel

K2 A2
1 2
K1 A1

1 1 1 K eq  A1  A 2  K1 A1 K 2 A 2 K1A1  K 2 A 2
    K eq 
R eq R1 R 2 ; L L L
;
A1  A 2

k1  k 2
If A1  A 2 , k eq 
2

1 2  
In parallel the heat conducted H  H1  H 2  R
eq.

Convection
When heat is transferred from one place to another by actual movement of heated substance (fluids) is
called convection. Convection requires a medium.

253
Brilliant STUDY CENTRE

Growth of ice in ponds

1u@0oC y

0oC dy

Ice starts forming in a pond at a temperature below 0oC. To find the time later for growth of ice with a
thickness y, consider a small thickness dy.
Let dQ is the heat released when a thickness of dy is formed.
Heat energy transferred to the environment.

kA  0   
dQ  dt
y

For melting d  mL  Ady  L

kA
dt  AdyL , dt  L ydy
y k
y
L
The total time taken t   dt   y dy  1 L y 2 ; -density of ice ; L - Heat of fusion;  - at
0
k 2 k
temperature

Time taken to grow ice upto a thickness, t  y 2 .


Due to convection (a) Land and Sea breeze occur, b) Trade winds are formed,
c) Monsoons are originating, d) Ventillation becomes possible
Radiation
Thermal radiation is the transfer of heat by means of electromagnetic radiation, generated by the thermal
motion of particles in matter. The energy is usually transferred in the infrared region. For radiative transfer it
does not require a material medium.
a) Radiant energy Travels in straight lines. b) It is reflected and refracted as in the case of light.
c) Intensity of radiation obeys inverse square law. d) Every body above absolute zero emits radiations.
Thermal radiation falling on a surface exerts pressure on the surface, called radiation pressure. When
radiation passes through a medium it absorbs radiation and due to this the temperature of the medium
increases. Bolometer is used to measure radiation intensity.
A perfect Black body is one which completely absorbs radiations falling on it.
A good absorber is a good emitter of radiation. A black body when heated to required temperature will
emit radiation absorbed by it
If Q is the total incident energy, then part of its is absorbed, partily reflected and the remaining transmitted.
 Q  A  R  T ; A - Energy absorbed; R - Reflected; T - Transmitted.
254
[Study Package - Physics - Class XI]

A R T A
1   ; a  - Absorptance (absorptive power)
Q Q Q Q

R
r = Reflective
Q

T
t - transmittance a  r  t 1
Q
Absorptive power
A
a ; for a perfect black body a = 1; for others a < 1.
Q
Emissive power
Radiant energy emitted per unit area per unit time
Q Energy Power
   ; Unit W / m 2
t Area  time Area
Spectral emissive power
Emissive power per unit wavelength change at wavelength 

d
E   E  d ;  E
 d
Relative Emissivity or Emissivity (er)
It is the ratio of energy emitted by a body to the energy emitted by a black body in the same conditions.
Q energy emitted by a body
er  
Q BB energy emitted by black body ; 0 < er < 1

Prevost’s theory of energy exchange


Above absolute zero of temperature there is a continuous exchange of heat energy between a body
and its surroundings. Heating or cooling of a body takes place depending on its temperature difference with
the surroundings. When temperatures are equal, there is no cooling or heating, the temperature remains
the same.
Kirchhoff’s Law
The ratio of the emissive power to the absorptive power of all bodies at any given temperature and
wavelength is a constant and it is equal to the emissive power of a black body at the same temperature.
e
 a constant = E (E is the emissive power of a black body at the same temperature).
a
Good absorbers are good emitters.
Stefans’ Law
The emissive power of a black body is directly proportional to fourth power of its absolute temperature.
E  T4 where T = temperature of ideal black body in Kelvin.
–8 2 4 2
E  T 4 ,  = Stefan’s constant = 5.67 × 10 w/m k ; Unit of E = watts/m .

255
Brilliant STUDY CENTRE

The total energy radiated by a surface of area A in a time t by a black body is Q BB  AT 4 t
For any other body the total energy radiated by an area A in a time t

QGB  eAT 4 t (e - emissive power)


When a black body at a temperature T Kelvin surrounded by another body at a temperature.
T0 Kelvin (where To < T),
4
4

Heat lost by black body per unit area E   T  T0 . This is Stefan - Boltzmann law..

If the body is not a perfect black body, then E  e  T 4  T04  (e - is the emissivity)

4 4
For a body of area A, the heat energy radiated by the body in time t, is  eA T  T0 t  
Applications of Stefan’s law
Cooling by Radiation
If a body at temperature T is in an environment of temperature T0 (< T), the body is losing energy by
emitting radiations at a rate., P1  eAT 4

and is receiving energy by absorbing radiations at a rate P2  eAT04

4 4
So, net rate of loss of energy by the body through radiation, P  eA T  T0  
Let m is the mass of the body and c its specific heat, the rate of loss of heat at temeprature T,
dQ dT
 mc
dt dt
and if the body is cooling by radiation, from above equation

dT eA 4
or   T  T04 
dt mc
i.e., when a body cools by radiation the rate of cooling depends on:
1) Nature of radiating surface, e
2) Area of radiating surface, A

1
3) Mass of radiating body, 
m

1
4) Specific heat of radiating body, 
c
5) Temperature of radiating body, (T)
6) Temperature of surrounidngs, (T0)
Newton’s Law of cooling
When temperature difference between the surrounding and body is not very large, the rate of cooling
is proportional to the temperature difference betwen body and surroundings.

256
[Study Package - Physics - Class XI]

Q  1  2 
R    0 ; t - time of cooling; 1 - initial and 2 at temperature of body
t  2 

d  1  2 
mc    0 R0 - surrounding temperature.
dt  2 

       
mc  1 2    1 2   0
 t   2 

d d
 k  2  1  ;  mc  k  2  1 
dt dt

Total Time of cooling dT


 k  T  T0 
From Newton law of cooling dt
T2
dT T2  T0 1 T2  T0
 TT  k  dt ; n   kt ; t n
T1 0 T1  T0 k T1  T0

d
 kdt
2  1

n  2  1  x
x
x
x

n  2  1    kt  c ; y = mx + c

Newton’s law of cooling can be used to compare the specific heats of two liquids as : if equal
masses of two liquids having same surface area and same initial temperature 1 goes to same final
temperature 2 with same temperature of surroundings, ie, 0 in time intervals t1 and t2 respectively, then
according to Eqn.

t1 K 2 c t
 or 1  1 [ K  1 ]
t 2 K1 c2 t 2 c

257
Brilliant STUDY CENTRE

Wein’s Displacement Law


Wiens displacement law states that the black body radiation curve for different temperatures peaks at
a wavelength inversely proportional to the temperature. The shift of the peak is a direct consequence of
planck radiation law which describes the spectral brightness of black body radiation as a function of wavelength
at any given temperature.
 m T  b  a constant

 m is the wavelength at which maximum emission takes place at a Temperature T Kelvin.


1
The value of b = 2.89 × 10–3 mK or  m 
T

Solar Constant
The solar radiant energy received per unit area per unit time by a black surface held at right angles to
the suns rays and placed at the mean distance of the earth is called solar constant. Solar constant is taken
as 1340 watts/m2.
Temperature of sun
Let R be the radius of the sun and d be the radius of the Earth’s orbit around the sun, E is the energy
emitted by the sun per second per unit area.

ER 2
S ; S is the energy falling per unit area of earth per unit time,
d2

T 4 R 2
By Stefans law S 
d2

258
[Study Package - Physics - Class XI]

QUESTIONS
LEVEL - I
1. Oxygen boils at –183oC. This temperature is approximately.
1) 215oF 2) –297oF 3) 329oF 4) 361oF
2. The triple point of carbon dioxide is 216.55 K the corresponding temperature on the Celsius and
Fahrenheit scale respectively are
1) 56.45oC, –69.61oF 2) –56.45oC, 69.61oF
3) 56.45oC, 69.61o F 4) –56.45oC, –69.61oF

th
2
3. When a thermometer is taken from the melting ice to a warm liquid, the mercury level rises to of
5
the distance between the lower and the upper fixed points. The temperature of liquid in K is
1) 217.15 2) 313.15 3) 220 4) 330
4. A constant volume gas thermometer shows pressure reading of 50 cm and 90 cm of mercury at 0oC
and 100oC respectively. When the pressure reading is 60 cm of mercury, the temperature is
1) 25oC 2) 40oC 3) 15oC 4) 12.5oC

5. Two rods, one of aluminium and the other made of steel, having initial lengths 1 and  2 are connected
together to form a single rod of length 1   2 . The coefficient of linear expansion for aluminium and
steel are  a and  s respectively. If the length of each rod increases by the same amount when their

1
temperature are raised by toC, then find the ratio
1   2

s a a s
1) 2) 3) 4)
a s  a  s  a  s

6. A 1 L glass flask contains some mercury. It is found that at different temperatures the volume of air
inside the flask remains the same. What is the volume of mercury in this flask if coefficient of linear
expansion of glass is 9 × 10–6/oC while of volume expansion of mercury is 1.8  104 / o C ?

1) 50 cc 2) 100 cc 3) 150 cc 4) 200 c


7. If on heating liquid through 80oC, the mass expelled is (1/100)th of mass still remaining, the coefficient
of apparent expansion of liquid is

1) 1.25  104 / o C 2) 12.5  104 / o C 3) 1.25  105 / o C 4) None of these

8. A steel tape measures the length of a copper rod as 90.0 cm when both are at 10oC, the calibration
temperature for the tape. What would be the tape reading for the length of the rod when both are 30oC.
Given,  for steel 1.2  105 o C 1 and  for copper is 1.7  105 o C 1

1) 90.01 cm 2) 89.90 cm 3) 90.22 cm 4) 89.80 cm

259
Brilliant STUDY CENTRE

9. The pressure that must be applied to the ends of a steel wire of length 10 cm to keep its length
constant when its temperature is raised by 100oC is
(For steel Young’s modulus is 2 × 1011 Nm–2 and coefficient of thermal expansion is 1.1 × 10–5 K–1)

1) 2.2  108 Pa 2) 2.2  109 Pa 3) 2.2 107 Pa 4) 2.2 106 Pa


10. A metal rod of length L and cross-sectional area A is heated through ToC. What is the force required to
prevent the expansion of the rod lengthwise?

YAT YAT 1  T  1  T 
1) 1  T 2) 1  T 3) 4)
    YAT YAT
11. The moment of inertia of a body is I and its coefficient of linear expansion is  . If the temperature of
the body rises by a small amount T , then change in its moment of inertia about the same axis is

IT
1)  I T 2) 2 IT 3) 4IT 4)
2

12. Coefficient of linear expansion of brass and steel rods are 1 and  2 . Lengths of brass and steel rods

are 1 and  2 respectively. If   1   2  is maintained same at all temperatures, which one of the following
relations holds good?

1) 1 2   2 1 2) 1 22   2  12 3) 12  2   22  1 4) 11   2  2

13. When m g of water at 10oC is mixed with m g of ice at 0oC, which of the following statements are
false?
(i) The temperature of the system will be given by the equation,

m  80  m 1  T  0   m 1 10  T 


(ii) Whole of ice wll melt and temperature will be more than 0oC but lesser than 10oC
(iii) Whole of ice will melt and temperature will be 0oC
(iv) Whole of ice will not melt and temperature will be 0oC
1) i, ii 2) i, iii 3) ii, iv 4) i, ii, iii
14. A 10 kW drilling machine is used to drill a bore in a small aluminium block of mass 8 kg. Find the rise
in temperature of the block in 2.5 minutes, assuming 50% power is used up in heating the machine
itself or lost to the surroundings. (Specific heat of aluminium = 0.91 J g–1 oC–1)
1) 100 oC 2) 103 oC 3) 150 oC 40 155 oC
15. 4 gm of steam at 100oC is added to 20 gm of water at 46oC in a container of negligible mass.
Assuming no heat is lost to surrounding, the mass of water in container at thermal equilibrium is
[latent heat of vapourisation = 540 cal/gm. Specific heat of water = 1 cal/gm-oC.]
1) 18 gm 2) 20 gm 3) 22 gm 4) 24 gm
16. Three bodies of the same material and having masses m, m and 3m are at temperature 40oC, 50oC
and 60oC respectively. If the bodies are brought in terminal contact, the final temperature will be
1) 45oC 2) 54oC 3) 52oC 4) 48oC

260
[Study Package - Physics - Class XI]

17. Two identical plates of metal are welded end to end as shown in figure. 20 cal of heat flows through it
in 4 minute. If the plates are welded as shown in figure, the same amount of heat will flow through the
plates:

1) 1 minute 2) 2 minute 3) 4 minute 4) 16 minute


18. Consider boiling water converting into steam. Under this condition, the specific heat of water is
1) Less than zero 2) Zero
3) Slightly greater than zero 4) Infinite
19. The latent heat of vaporisation of a substance is always [AIIMS 2010]
1) Greater than its latent heat of fusion 2) Greater than its latent heat of sublimation
3) Equal to its latent heat of sublimation 4) Less than its latent heat of fusion
20. The sprinkling of water reduces slightly the temperature of a closed room because
1) temperature of water is less than that of the room
2) specific heat of water is high
3) water has large latent heat of vaporisation
4) water is a bad conductor of heat
21. Two rods A and B of different materials are welded together as shown in figure. Their thermal
conductivities are K1 and K2. The thermal conductivity of the composite rod will be

[NEET 2017]

3(K1  K 2 ) K1  K 2
1) 2) K1 + K2 3) 2 (K1 + K2) 4)
2 2
22. A body cool from 90oC to 70oC in 10 minutes if temperature of surrounding is 20oC find the time taken
by body to cool from 60oC to 30oC. Assuming Newton’s law of cooling is valid
1) 10 min 2) 24 min 3) 36 min 4) 8 min
23. Certain quantity of water cools from 70oC to 60oC in the first 5 minutes and to 54oC in the next 5
minutes. The temperature of the surroundings is
1) 45oC 2) 20oC 3) 42oC 4) 10oC
24. Two solid spheres, of radii R1 and R2 are made of the same material and have similar surfaces. The
spheres are raised to the same temperature and then allowed to cool under identical conditions.
Assuming spheres to be perfect conductors of heat, their initial rates of loss of heat are

R 12 R1 R2 R 22
1) 2 2) 3) 4) 2
R2 R2 R1 R1

261
Brilliant STUDY CENTRE

25. If the temperature of the Sun gets doubled, the rate of energy received on the Earth will increase by a
factor of
1) 2 2) 4 3) 8 4) 16

26. Which one of the following is  m  T graph for perfectly black body?  m is the frequency of radiation
with maximum intensity. T is the absolute temperature

1) A 2) B 3) C 4) D
27. A black body is at a temperature of 5760 K. The energy of radiation emitted by the body at wavelength
250 nm is U1, at wavelength 500 nm is U2 and that at 1000 nm is U3. Wien’s constant, b = 2.88 × 106
nm K. Which of the following is correct? [NEET 2016]
1) U1 > U2 2) U2 > U1 3) U1 = 0 4) U3 = 0
28. We consider the radiation emitted by the human body. Which of the following statements is true?
1) The radiation emitted is in the infrared region
2) The radiation is emitted only during the day
3) The radiation is emitted during the summers and absorbed during the winters
4) The radiation emitted lies in the ultraviolet region and hence is not visible
29. Temperature of a room is –15oC and outside temperature is 10oC. If room temperature is made 40oC,
then find out the outside temperature, if rate of heat flow is same in both cases
1) 10oC 2) 5oC 3) 15oC 4) 20oC
30. The rate of cooling at 600 K, if surrounding temperature is 300 K is H. The rate of cooling at 900 K is

16 2
1) H 2) 2 H 3) 3 H 4) H
3 3
LEVEL - II
1. The reading of Centigrade thermometer coincides with that of Fahrenheit thermometer in a liquid.
The temperature of the liquid is
1) –40oC 2) 0oC 3) 100oC 4) 300oC
2. Mercury boils at 367oC. However, mercury thermometers are made such that they can measure
temperature upto 500oC. This is done by
1) Maintaining vacuum above mercury column in the stem of the thermometer
2) Filling nitrogen gas at high pressure above the mercury column
3) Filling nitrogen gas at low pressure above the mercury level
4) Filling oxygen gas at high pressure above the mercury column

262
[Study Package - Physics - Class XI]

3. In the phase diagram shown, the point Q corresponds to the triple point of water. The regions I, II and
III respectively correspond to phases

1) liquid, solid, vapour 2) solid, liquid, vapour 3) liquid, vapour, solid 4) solid, vapour, liquid
4. Two absolute scale A and B have triple points of water defined to be at 200 A and 350 B. The relation
between TA and TB is

4 4 2 2
1) TA  TB 2) TB  TA 3) TA  TB 4) TB  TA
7 7 7 7
5. The volume of a metal sphere increases by 0.24% when its temperature is raised by 40 oC. The
coefficient of linear expansion of the metal is .........../oC.
1) 2  105 2) 6  105 3) 18  105 4) 1.2  105
6. 1.56 × 105 J of heat is conducted through a 2 m2 wall of 12 cm thick in one hour. Temperature difference
between the two sides of the wall is 200C. The thermal conductivity of the material of the wall is (in Wm–
1
K–1)
1) 0.11 2) 0.13 3) 0.15 4) 1.2
7. A cylindrical metallic rod in thermal contact with two reservoirs of heat at its two ends conducts an
amount of heat Q in time t. The metallic rod is melted and the material is formed into a rod of half the
radius of the original rod. What is the amount of heat conducted by the new rod, when placed in
thermal contact with the two reservoirs in time t?

Q Q Q
1) 2) 3) 2Q 4)
4 16 2
8. Two spheres made of same substance have diameters in the ratio 1 : 2. Their thermal capacities are
in the ratio of
1) 1 : 2 2) 1 : 8 3) 1 : 4 4) 2 : 1
o
9. A large steel wheel is to be fitted on to a shaft of the same material. At 25 C, the diameter of the shaft
is 8.00 cm and the diameter of the hole is 7.99 cm. The shaft is cooled using ‘dry ice’. At what
temperature of the shaft does the wheel slip into the shaft. Give  for steel = 1 × 10–5 K–1.
1) 25oC 2) 125oC 3) 100oC 4) –100oC
10. A piece of ice falls from a height h so that it melts completely. Only one-quarter of the heat produced
is absorbed by the ice and all energy of ice get converted into heat during its fall. The value of h is
[Latent heat of ice is 3.4 × 105 J/kg and g = 10 N/kg] [NEET 2016]
1) 136 km 2) 68 km 3) 34 km 4) 544 km
11. An experiment takes 10 minutes to raise temperature of water from 0 C to 100oC and another 55
o

minutes to convert it totally into steam by a stabilized heater. The latent heat of vaporization comes
out to be
1) 530 cal/g 2) 540 cal/g 3) 550 cal/g 4) 560 cal/g

263
Brilliant STUDY CENTRE

12. The coefficient of volume expansion of glycerine is 49  105 o C1 . What is the fractional change in the
density for a 30oC rise in temperature
1) 0.0138 2) 1.1234 3) 2.0128 4) 5.0128
13. A glass bottle of volume 100 cmm at 0 C is filled with paraffin at 20 C. If the density of paraffin at 0oC
o o

4 o
is 0.8 g/cc,  paraffin  10 10 / C and the coefficient of linear expansion of glass is 10  10 6 / o C , the
mas of a paraffin is
1) 78.45 g 2) 78.48 g 3) 81.6 g 4) 80 g
14. If a 5 kg body falls to the ground from a height of 30 m and if all its mechanical energy is converted into
heat, the heat produced will be
1) 350 cal 2) 150 cal 3) 60 cal 4) 6 cal
15. A lead bullet at 27oC just melts when stopped by an obstacle. Assuming that 25% of heat is absorbed
by the obstacle, then the velocity of the bullet at the time of striking (M.P. of lead = 327oC, specific heat
of lead = 0.03 cal/goC, latent heat of fusion of lead = 6 cal/g and J = 4.2 joule/cal)
1) 410 m/s 2) 1230 m/s 3) 307.5 m/s 4) None of the above
16. Two identical bodies are made of a material for which the heat capacity increases with temperature.
One of these is at 100oC, while the other one is at 0oC. If the two bodies are brought into contact, then,
assuming no heat loss, the final common temperature is
1) 50oC 2) More than 50oC
3) Less than 50oC but greater than 0oC 4) 0oC
17. A cubical box made of thermocole of 30 cm side has thickness of 5.0 cm. If 4.0 kg of ice are put in the
box, estimate the amount of ice remaining after 6 hours. The outside temperature is 45oC and coefficient
of thermal conductivity of thermocole is 0.01 J/s moC. Take latent heat of fusion of ice =33.5 J/kg
1) 1.687 kg 2) 0.627 kg 3) 3.687 kg 4) 5.124 kg
18. A lead bullet strikes against a steel plate with a velocity 200 ms–1. If the impact is perfectly inelastic
and the heat produced is equally shared between the bullet and the target, then the rise in temperature
of the bullet is (specific heat capacity of lead = 125 Jkg–1K–1)
1) 80oC 2) 60oC 3) 160oC 4) 40oC
19. Steam at 100oC is passed into 20 g of water at 10oC. When water acquires a temperature of 80oC,
the mass of water present will be
[Take specific heat of water = 1 cal g–1 oC–1 and latent heat of steam = 540 cal g–1]
1) 24 g 2) 31.5 g 3) 42.5 g 4) 22.5 g
20. If a ball of 80 kg mass hits an ice cube and temperature of ball is 100oC, then how much ice converted
into water? (Specific heat of ball is 0.2 cal g–1, Latent heat of ice = 80 cal g–1)
1) 20 g 2) 200 g 3) 2 ×103 g 4) 2 × 104 g
21. Two identical long, solid cylinders are used to conduct heat from temperature T1 to temperature T2.
Originally the cylinders are connected in series and the rate of heat transfer is H. If the cylinders are
connected in parallel then the rate of heat transfer would be:

H
1) 2) 2 H 3) 4 H 4) 8 H
4

264
[Study Package - Physics - Class XI]

22. One end of a conducting rod is maintained at temperature 50oC and at the other end ice is melting
0oC. The rate of melting of ice is doubled if
1) the temperature is made 200oC and the area of cross-section of the rod is doubled
2) the temperature is made 100oC and length of the rod is made four times
3) area of cross-section of rod is halved and length of the rod is made four times
4) the temperature is made 100oC and area of cross-section of rod and length both are doubled.
23. If the temperature of the Sun were to increase from T to 2T and its radius from R to 2R. The ratio of
power, radiated by it would become
1) 64 times 2) 16 times 3) 32 times 4) 4 times
24. Two spherical bodies A (radius 6 cm) and B (radius 18 cm) are at temperatures TA and TB, respectively.
The maximum intensity in the emission spectrum of A is at 500 nm and in that of B is at 1500 nm.
Considering them to be black bodies, what will be the ratio of the rate of total energy radiated by A to
that of B?
1) 9 2) 12 3) 3 4) 6
25. Emissivity and absorptive power of a material at 2000 K is 8 and 10 respectively. The emissivity of
ideal black body is
1) 0.2 2) 0.4 3) 0.6 4) 0.8
26. A spherical black body with a radius of 12 cm radiates 450 W power at 500 K. If the radius were
halved and the temperature doubled, the power radiated in watt would be:
1) 225 2) 450 3) 900 4) 1800
27. In which of the following process convection does not take place primarily?
1) Sea and land breeze 2) Trade wind
3) Boiling of water 4) Warming of glass of bulb due to filament
28. Earth orbiting satellite has solar energy collecting panel with total area 5 m2. If solar radiations are
perpendicular and completely absorbed, the average force associated with the radiation pressure is
(solar constant = 1.4 kW m–2) [AIIMS 2019]
1) 2.33 × 10–3 N 2) 2.33 × 10–4 N 3) 2.33 × 10–5 N 4) 2.33 × 10–6 N
29. A glass full of hot milk is poured on the table. It begins to cool gradually. Which of the following is
incorrect?
1) The rate of cooling is constant till milk attains the temperature of the surroundings
2) The temperature of milk falls off exponentially with time
3) While cooling, there is a flow of heat from milk to the surroundings as well as from surroundings to
the milk but the net flow of heat is from milk to the surroundings and that is why it cools
4) All three phenomenon, conduction, convection and radiation are responsible for the loss of heat
from milk to the surroundings
30. The intensity of radiation emitted by the sun has its maximum value at a wavelength of 510 nm and
that emitted by the North Star has the maximum value at 350 nm. If these stars behave like black
bodies, then the ratio of the surface temperatures of the sun and the North Star is
1) 1.46 2) 0.69 3) 1.21 4) 0.83

265
Brilliant STUDY CENTRE

KEY WITH HINTS


LEVEL - I

C  0 F  32 183  180
1. 2  ;  F  32
100 180 100

F  329.4  32  297.4
o
2. 4 As TC   T  273  C  216.55  273  56.45o C

9 9
Also, TF  TC  32    56.45   32  101.61  32  69.61o F
5 5
3. 2 Temperature of liquid = 2/5 of distance between lower and upper fixed points = 2/5 × 100 = 40oC
On Kelvin scale, TK = 273.15 + 40 = 313.15 K.

 P P   60  50 
4. 1 t   t 0  100o C    100  25o C
 P100  P0   90  50 

I A 0.075 I B 0.045
5. 3 For 'A',  A   For ‘B’,  B  
IA T 20 100  ; IB T 20 100 

For composite rod,    A  A T   B  20   A  T

0.075 0.045
 0.060  A   20   A  ;   A  10cm
20 20
6. 3 It is given that the volume of air in the flask remains the same. This means that the expansion
in the volume of the vessel is exaclty equal to the volume expansion of mercury.

VG 3G 1000   3  9 10   150 cc


6

i.e., VG  VL or VG  G   VL  L   VL  


L 1.8  104

Mass expelled x / 100 1


7. 1  app     1.25  104 / o C
Mass remained  T x  80 8000

8. 1 Both the metals will expand, but copper will elongate more due to its greater  Cu than that of
the steel. The extra length that copper expand will be the extra length that the steel scale will
measure.

LCu  L0 1  Cu T  ; LS  L0 1  ST 

5
Extra expansion of copper LCu  LS  L 0   Cu  S  T  90 1.7  1.2   10  20  0.009 cm

Total length measured = 90 + 0.009 cm = 90.01 cm

266
[Study Package - Physics - Class XI]

FL P P
9. 1 Y  ; Y ; P  Y  2.2  108 Pa
AL L / L 
10. 2 The change in length L  LT

Total length LT  L 1  T 

Young’s modulus of the material of the rod

F LT YAL YALT YAT


Y ; F  or F 
A L LT L 1  T  1  T 
I 2r
11. 2 I  mr 2 ;    2  T
I r
12. 4 Change in length for both rods should be same

1   2  11T   2  2 T  11   2  2


13. 4 When m - gm of ice mixed with m g m of water at 10oC. The temperature will be 0oC. Whole ice
will not be melted and temperature will be 0oC.

14. 2 s = 0.91 J g–1 C–1 ; Total energy  P  t  104  2.5  60  15  105 J

1
As 50% of energy is lost,  Energy available, Q   15  105  7.5  105 J
2

Q 7.5 105
As Q  msT ; T   3
 103o C
ms 8 10  0.91
15. 3 Heat released by steam in conversion to water a 100oC is Q1 = mL = 4 × 540 = 2160 cal. Heat
required to raise temperature of water from 46oC to 100oC is

Q1
Q 2  mS  20  1 54  1080J ; Q1  Q 2 and 2
Q2

Hence all steam is not converted to water only half steam shall be converted to water.
Final mass of water = 20 + 2 = 22 gm.

m1c1T1  m 2 c 2 T2  m3c3T3
16. 2 T ; c1  c 2  c3  c
m1c1  m 2 c 2  m3 c3

m  40  m  50  3m  60 40  50  180 270
T    54o C
m  m  3m 5 5

20cal KA  T1  T2  20 cal K  2A   T1  T2 
17. 1  and  ; x  1 min
4 min 2 x min 

267
Brilliant STUDY CENTRE

Q
18. 4 As specific heat, s  ; Since boiling of water is an isothermal process, so  T  0 ;
mT
Q
s 
m0
19. 1
20. 3 When water is sprinkled over a large area, evaporation takes place. As the latent heat of
vaporisation is large, cooling takes place.
21. 4 Equivalent thermal conductivity of the composite rod in parallel combination will be,
K1A1  K 2 A 2 K1  K 2
K 
A1  A 2 2

1  2    
22. 1 By Nwton’s law of cooling  k  1 2  0 
t  2 

90  70  90  70  60  30  60  30 
 k  20  and k  20 
10  2  t  2 
20t 60
   t  36 min
10  30 25
23. 1 Let Ts be the temperature of the surroundings. According to Newton’s law of cooling.

T1  T2 T T 
 K  1 2  Ts 
t  2 

70  60  70  60  12
For first 5 minutes,   K  Ts  ; or  K  65  Ts  ..... (i)
5  2  5

60  54  60  54 
For next 5 minutes,   K  Ts  or 1.2  K  57  Ts  ...... (ii)
5  2 
2 5 65  Ts
Divide eqn. (i) by eqn. (ii), we get;  
1.2 3 57  Ts

or 285  5Ts  195  3Ts or 2Ts  90 or Ts  45o C

Q1  T 4  4R 12  e R 12
24. 1  
Q 2  T 4  4R 22  e R 22
25. 4 Rate of energy received on the Earth, u  eAT 4

4 u1 T14
For given, e,  and A, u  T  
u 2 T24

u1 T14 1
Here T2  2T1   4
 or, u 2  16u1
u 2  2T1  16

268
[Study Package - Physics - Class XI]

26. 3 According to Wein’s displacement law  m T  b ; where b is the Wein’s constant

c c
In terms of frequency, T  b or v m  T ; Hence, v -T graph is a straight line shown by
vm b m

curve C.

b 2.88 106 nmk


27. 2 According to Wien’s displacement law,  m    500 nm
T 5760k

Clearly from graph, U1  U 2  U 3


28. 1 A human body emits radiations of longer wavelength than absorbed radiations.
29. 3 For the same rate of heat flow, temperature difference must be same.
o
 o
o
Initial temperature difference = 10 C  15 C  25 C

Now room temperature = 40oC;  Outside temperature  40o C  25o C  15o C

30. 1 
4 4
By Stefan’s law, Rate of cooling, H  T  T0 
94  34 3  3  1
4 4 4 4
H  900    300  16
    ;  H  H
3  2  1
4 4 4 4 4 4
H  600    300  6 3 3
LEVEL - II
x  0 x  32
1. 1  ; x 1.8   x  32
100 180
32
0.8x  32; x   40o C
0.8
2. 2 By filling N2 gas at high pressure, the boiling point of mercury increases which extends the
range up to 500oC.
3. 1 In the given P-T diagram, Region I - Liquid; Region II - Solid; Region III - Vapour.
4. 1 Triple point of water on scale A = 200 A. Triple point of water on scale B = 350 B
273.16 273.16
200 A = 350 B = 273.16 K (given) or A K and B  K
200 350
If TA and TB represent the triple point of water on scales A and B, then
273.16 273.16 T 200 4
TA  TB or A  ; or TA  TB
200 350 TB 350 7

269
Brilliant STUDY CENTRE

 V 1  0.24  1  5
5. 1        2  10
3 3VT 3  100  40 

Q KAT
6. 2 Rate of heat flow through the wall is  ;
t L

1.56 105  12  102


K Wm 1K 1  0.13 Wm 1K 1
3600  2  20
7. 2 The amount of heat flows in time t through a cylindrical metallic rod of length L and uniform area
of cross-section A  R
2
  with its ends maintained at temperatures T1 and T2 (T1 > T2) is
given by

KA  T1  T2  t
Q ...... (i)
L
where K is the thermal conductivity of the material of the rod.
2
R R 2 A
Area of cross-section of new rod, A        ...... (ii)
2 4 4
As the volume of the rod remains unchanged
 AL  AL
where L is the length the new rod.

A
or L  L  4L (Using (ii)) ...... (iii)
A
Now, the amount of heat flows in same time t in the new rod with its ends maintained at the
same temperatures T1 and T2 is given by

KA  T1  T2  t
Q  ..... (iv)
L
Substituting the values of A' and L' from equatiosn (ii) and (iii) in the above equation, we get

K  A / 4  T1  T2  t 1 KA  T1  T2  t 1
Q    Q (Using (i))
4L 16 L 16
8. 2 Thermal capacity = Mass × Specific heat; Specific heat = constant (Because same material)
3
C1 m1 V1 r13  1  1
    
C2 m 2 V2 r23  2  8

L 0.01cm
9. 4 Decrease in temperature, T   5
 125o C
L 110  8.00 cm

 T2  T1  125  T2  125  T1  125  25  100o C

270
[Study Package - Physics - Class XI]

10. 1 Gravitational potential energy of piece of ice at a height (h) = mgh


1
Heat absorbed by the ice to melt completely Q  mgh -----(i); Q  mL -----(ii)
4
1 4L
From eqns. (i) and (ii), mL  mgh or h 
4 g

4  3.4 105
  4  34 103  136 km
10
m  1 100
11. 3 Heat given per second  cal
10  16
mL
Now, Q = mL; Heat supplied per sec to steam 
55  60
M  1 100 mL
   L  550 cal / g
10  60 55  60
12. 1 V  V 1  t   V 1  49 105  30   V 1  1470  105   1.01470V

M M M 1
Now density,   and    ; or   
V V 1.0147V 1.0147

 1 
   
 Fractional change in density    
 1.0147   1.014  1  0.0138
  1.014
13. 2 Mass = (density) (volume)

  0.8 1  10 10 4  20   100 1  3  10  106  20    80 1  0.02  0.0012  78.48g

mgh 5 10  30 
14. 1 Heat produced    350 cal
J 4.2
15. 1 Total heat required the bullet to just melt Q1  ms  mL

1
 m  0.03  327  27   m  6  15m  4.2  J ; KE 
2
 m  103  v 2 J

25% of energy is absorbed by the obstacle.


75 1 3
The energy absorbed by the bullet - Q 2   mv 2  103  mv 2  103
100 2 8
3 2
Now the bullet will melt Q 2  Q1 ; mv  103  15m  4.2
8

15  4.2  8
 v min  = 409.88 m/s  410 m/s
3 103

271
Brilliant STUDY CENTRE

16. 2 Since, heat capacity of material increases with increase in temperature. TC  50o C
2
17. 3 Surface area of all the six faces  6  side   6  0.3  0.3  0.54 m 2

KA  1  2  t
Let m mass of ice will melt  Q  mL or mL 

KA  1  2  t 0.01 0.54  45  6  60  60
or m    0.313kg
L 335 103  5  10 2
Mass of the ice left in the box  4.0  0.313  3.687 kg
2
11  v 2  200 
18. 1 msT   mv 2  ;  T    80o C
2 2  4s 4  125

19. 4 s w  1cal g 1 o C1 ; Ls  540 cal g 1


Heat lost by m g of steam at 100oC to change into water at 80oC is

Q1  mLs  ms w Tw  m  540  m 1 100  80   540m  20m  560m


Heat gained by 20 g of water to change its temperature from 10oC to 80oC is

Q 2  m w s w Tw  20 1  80  10   1400 ; According to principle of calorimetry Q1  Q 2

 560m  1400 or m  2.5g ; Total mass of water present =  20  m    20  2.5 g  22.5g


20. 4 If m is the mass of ice melted, then heat spent in melting = heat supplied by the ball.
mL  sMT ; m × 80 = 0.2 × (80 × 1000) × 100 or m = 2 × 104 g
R
21. 3 Initially effective resistance = 2R. In parallel effective resistance = . It has reduced by a factor
2
1
of so rate of heat transfer would be increased by a factor of 4, keeping other parameters
4
same.
22. 4 Rate of melting of ice  rate of heat transfer (dQ/dt)

Further dQ  temperature difference


dt   
 
 KA 

dQ  temperature difference 
or  A
dt 
dQ
If temperature difference, A and  are all doubled then and hence, rate of melting of ice will
dt
be doubled.
23. 1 Ist case, power radiated = AT4; IInd case, power radiated = 4(2)4 times = 64 times.

272
[Study Package - Physics - Class XI]

24. 1 According to Wien’s displacement law,  m T = constant.

TA   m B 1500 nm
   m A TA    m  B  TB  ; or    3 ------(i)
TB   m  A 500nm

2 4 2
E  R   T   6cm  4
E  AT 4  4R 2 T 4 ;  A  A   A     3  9
E B  R B   TB   18cm 

e 8
25. 4 From Kirchhoff’s law, Emissivity of IBB, E    0.8
a 10
26. 4 Power radiated  eAT 4  R 2 T 4 ; P = 450 × 4 = 1800 W
27. 4 In convection process heat is transferred by the bodily motion of the heated particles. It is not so
in case of warming of glass bulb due to filament heating. Infact, warming of glass bulb is due to
radiation.
28. 3 As, power = solar constant × area = (1.4 × 103) × 5 W

solar constant×area 1.4 103  5


 Force=  8
 2.33  105 N
speed of light 3  10
29. 1 The heat from hot milk spread on the table is transferred to the surroundings by conduction,
convection and radiation. By Newton’s law of cooling the temperature of milk falls exponentially
with time.
30. 2 According to Wien’s displacement law,

 m T  Constant (b). Let S = Sun, NS = North star

TS   m  NS 350
   m S TS    m  NS TNS ;     0.69
TNS   m S 510

273
Brilliant STUDY CENTRE

CHAPTER - 11
THERMODYNAMICS

Thermodynamics deals with transformation of heat energy into other forms of energy and vice versa.
Thermal Equilibrium
If two systems are at same temperature. They are at Thermal Equilibrium.
The thermodynamical equilibrium
A system is in the thermodynamical equilibrium, when it is in a state of thermal mechanical and chemical
equilibrium. Thermodynamic variables: The physical quantities like pressure, volume, temperature, energy,
entropy etc. are thermodynamic variables.
Thermodynamic process
If the state of a system changes in such way that P, V, T etc. changes the process is called
thermodynamical process.
If the system undergoes a series of changes and comes back to the initial state, then the process is
called a cyclic process.
A reversible process is one which can be reversed in such a way that all changes taking place in the
direct process are replaced when the conditions are reversed.
Zeroth law of thermodynamics
If two systems A and B are separately in thermal equilibrium with a system C, then A & B are in thermal
equilibrium with each other. When the temperature of a body is measured, the thermometer will be in
thermal equilibrium with the body.
Internal energy
Internal energy of a system is the energy possessed by the system due to molecular motion and
molecular configuration. Due to molecular motion it posses internal KE (Uk) and due to molecular configuration
it possess internal PE (Up). Then the change in energy, dU = dUk + dUp
If there is no inter molecular force dUp = 0, then dU = dUk = mCv dT
For  moles of an ideal gas, dU  C v dT ie, the internal energy in the absence of inter molecular force
is the function of temperature and state only.
First law of Thermodynamics
Ist law of thermodynamics is equivalent to law of conservation of energy.

It states that the heat energy given to a system  Q  is used for increasing the internal energy  U 

of the system and for doing the work  W  by the system against its surroundings.

Q  U  W

274
[Study Package - Physics - Class XI]

Indicator diagram
A graph representing the variation of any one of the quantity (P, V, T) against another is called an
indicator diagram. Every point on the indicator diagrams represent a unique value for (P, V or T) of gases.

P P1 A (P1, V1)

P2 B (P2, V2)

V1 V2

Work done by a gas


Let P and V represent the pressure and volume of a gas taken in a cylinder closed by a piston of area
A. Force exerted on the piston F = P × A.

P
area enclosed

Let the piston move through a small distance dx, when the gas expands. Work done during expansion
dW  F  dx  PAdx  PdV

 Work done = W   dW   PdV


The area enclosed by the curve gives work done during the process.
Thermodynamic Process
(1) Cyclic process
If the system undergoes a series of changes and comes back to the initial state then it is a cyclic
process. Let UF and UI are the final and initial internal energies.

P
In the case of cyclic process UF = UI

 change in internal energy U  U F  U I  0

Q  W ie, heat supplied is equal to the work done.

275
Brilliant STUDY CENTRE

(2) Isochoric process

P V V

T P T1 T T2

For a thermodynamic process, the volume remains constant during isochoric process. So the gas
P
cannot do any work. W  0 , P  T,  a constant
T

For isochoric process Q  CV T .

RT
 Q  U , U  Cv T     1

The energy supplied at constant volume will be increasing the internal energy of the gas and thereby
temperature increases.
Isothermal process

T
P

V P

In any thermodynamic process, the temperature remains constant during isothermal process.
Consequently the internal energy remains constant.

P1 V1 dP P
   
The equation of state for the gas is PV = a constant.
P2 V2 .  Q  0  W ;
dV V

V2 P
W  R T log  R T log 1 . Energy supplied is used to do external work.
V1 P2

Isobaric process

V
It is a process in which pressure remains constant. The equation of state for the process  constant.
T
the amount of heat transferred  Q  C P T . The slope of the P-V curve for isobaric process is zero.

Work done W  PdV  P  V2  V1   R T2  T1 



276
[Study Package - Physics - Class XI]

P dP/dV = 0 V

dQ  dU  PV

V T

Fractions of energy for internal energy changes

dV nC vdT Cv 1
  
dQ nCpdT Cp 

dW 1
Fractions of energy for work done  1
dQ 
Adiabatic Process
It is a process in which the system is thermally insulated, so that no heat enters or leaves the system.
Q  0
R  T1  T2  1
0  U  W , W   R  P1V1  P2 V2 
   1  1

dP
dV

The equation of state for adiabatic process, PV  = constant. The slope of the adiabatic curve
 dP  P
     
 dV  V
Slope of adiabatic curve =  × slope of isothermal curve
Adiabatic process is taking place if there is no exchange of heat between the system and surroundings.
For this, the system is thermally insulated from surroundings.

For an adiabatic process Q  0 .  U  W  0


dU  dW  0
C v dT  PdV  0

But for a gas PV  RT , PdV  VdP  RdT

Cv  PdV  VdP 


 PdV  0
R

277
Brilliant STUDY CENTRE

R PdV  VdP
But C v  ;   PdV  0
 1    1
dV dP
PdV  VdP  0 ,    0 ;  log e V  log e P  C
V P

log  PV    constant ,  PV  = a constant


The equation is called the equation of state for an adiabatic change.


This can be rewritten as TV  1  constant , T  P1 = constant
Slope of adiabatic is greater than isothermal.

Expansion Compression

P P adiabatic

isothermal
adiabatic
V V

stress P VP
Elasticity of gas  strain   V / V   V
 
 P
Adiabatic elasticity =  V     P
 V

 P
Isothermal elasticity  V     P
 V

P C
Ratio of adiabatic to isothermal elasticity     p 1
P Cv

Adiabtic curve for different gases

Poliatomic
diatomic
monoatomic
V

Second Law of Thermodynamics


Clausius statement
It is impossible for a self acting machine unaided by an external agency to transfer heat from a cold
body to a hot reservoir.
278
[Study Package - Physics - Class XI]

Kelvins statement
It is not possible to get a continuous supply of work from a body by cooling it to temperature lower than
that of the surroundings.
Reversible and irreversible process
A thermodynamical process taking a system from initial to final state is reversible if the process can be
turned back such that both the system and surroundings return to their original states, when the conditions
are reversed.
Any process which cannot be reversed by reversing the conditions is called an irreversible process. It
is also called the natural process because all the process occuring in nature are irreversible.
eg. current flow from high potential to low potential. Heat flow between two bodies having a temperature
gradient between them etc.
Heat engine
It is a device which converts heat into work continuously through a cyclic process. It consists of three
parts- source, working substance and sink. The working substance take heat from the source, which is at
a higher temperature T1 and converts a part of it to useful work and rejecting the rest to the sink, which at a
lower temperature. The cycle repeats and a continuous work is derived.

T 1o K Source
Q1

T1 > T2 Working
substance
Q2
Sink T 2o

The performance of a heat engine is expressed as efficiency    which is the ratio of useful work
obtained from the engine to the heat supplied to it.

work done W
   U  0  ,  Q  W ;
heat input Q1 ; For cyclic process, W = Q 1 – Q2

Q1  Q 2 Q
   1 2
Q1 Q1

for a perfect heat engine   1, W  Q1 , so that Q2 = 0

Q1 is the heat taken from source and Q2 is the heat rejected to the sink.
A perfect heat engine is the one which converts all heat supplied into work.
Refrigerator
A refrigerator is a heat engine running in reverse direction. In this the working substance (refrigerant)
takes a heat Q2 from a body at a low temperature T2, and transfers the heat Q1 to the body at a higher
temperature T1 with the help of an external agent (compressor). Thus with the help of mechanical work the
working substances transfers heat from a cold body to a hot body, so that the cold body is cooled more and
more.
279
Brilliant STUDY CENTRE

The performance of a refrigerator is expressed by coefficient of performance (K)

Heat extracted Q 2 Q2 T2
K   K
work done W Q1  Q 2 ; T1  T2

A perfect refrigerator is one which transfers heat from cold body to hot body without doing work.
ie, W = 0, so that Q 1 = Q2, hence K  
Carnot Heat Engine
1) It consists of a cylinder with perfectly non-conducting walls with a perfectly conducting base. It encloses
a perfect gas as the working substance with the help of a non-conducting frictionless piston.
2) A source of infinite thermal capacity maintained at constant higher temperature T1.
3) A sink of infinite thermal capacity maintained at constant low temperature T2,
4) A perfectly non-conducting platform (stand).

ideal gas

source sink
stand T2
T1

A carnot engine is an ideal heat engine in which the working substance passes through carnot cycle.
Working of the engine (carnot cycle)

Y
PRESSURE (P)

A(V1, P1)
Q1
T1 B(V2, P2)

D(V4, P4)

Q2 C(V3, P3)
T2

X
O K L M N
VOLUME (V)

The working substance in carnot engine passes through a reversible cycle in four steps.
1. The cylinder with ideal gas is placed on the source and the gas is allowed to expand slowly at a
constant temperature T1, during which it absorbs a heat Q1. This isothermal change is represented by
AB in the indicator diagram.

280
[Study Package - Physics - Class XI]

2. The cylinder is now placed on the platform and the gas is allowed to expand adiabatically till the
temperature falls from T1 to T2. It is represented by a adiabatic BC.
3. The cylinder is now placed on the sink and compressed at constant temperature T2 along the path CD
transferring a certain quantity of heat Q2 to the sink.
4. Finally the cylinder is placed on the platform and compressed adiabatically so that the gas returns to its
initial state along DA.
The closed path ABCDA represents the carnot cycle.
Efficiency of the engine

work done W Q1  Q 2 Q
Efficiency of the engine,    ;   1 2
heat input Q1 Q1 Q1

V 
RT2 log  3 
V2 V3 Q2
  V4 
Q1  W1  RT1 log and Q 2  W3  RT2 log
V1 V4 ; Q1 V 
RT1 log  2 
 V1 

The points B and C lie on same adiabatic.  T1V2 1  T2 V31

Also A and D lie on the same adiabatic.  T1V11  T2 V4 1

 1 1
 V2  V  V2 V3
   3  ; 
 V1   V4  V1 V4

Q 2 T2 T
    1 2
Q1 T1 T1

It is seen that efficiency of a heat engine depends only on temperatures of source and sink.
All heat engines working between the same temperatures are equally efficient.

Coefficient of refrigerator K  1  

281
Brilliant STUDY CENTRE

QUESTIONS
LEVEL - I
1. An ideal gas is taken through a cyclic thermodynmaical process through four steps. Amounts of heat
involved in these steps are Q1 = 5960 J, Q2 = 5585 J, Q3 = –2980 J and Q4 = 3645 J respectively. The
corresponding works involved are : W 1 = 2200 J, W 2 = –825 J, W 3 = –1100 J and W 4 respectively.
Value of W 4 is
1) 275 J 2) 675 J 3) 765 J 4) 1315 J
2. o o
22 gm of CO2 at 27 C is mixed with 16 gm of O2 at 37 C. If both gases are considered as ideal gases,
temperature of the mixture will be nearly
1) 34oC 2) 29oC 3) 31oC 4) 32.6oC
3. A thermodynamic system is taken through the cycle ABCD as shown in figure. Heat rejected by the
gas during the cycle

1
1) 2PV 2) 4PV 3) PV 4) PV
2
4. A thermodynamic system undergoes cyclic process ABCDA as shown in figure. The work done by the
system in the cycle is

P0 V0
1) P0V0 2) 2P0V0 3) 4) Zero
2
5. An ideal gas goes from state A to state B via three different processes as indicated in the P-V diagram.

If Q1, Q2, Q3 indicate the heat absorbed by the gas along the three processes and U1 , U 2 , U 3
indicate the change in internal energy along the three processes respectively, then
1) Q1  Q 2  Q3 and U1  U 2  U3 2) Q3  Q 2  Q1 and U1  U 2  U3

3) Q1  Q 2  Q3 and U1  U 2  U3 4) Q3  Q 2  Q1 and U1  U 2  U3

282
[Study Package - Physics - Class XI]

6. During an adiabatic process, the pressure of a gas is found to be proportional to the cube of its
Cp
temperature. The ratio of for the gas is
Cv

5 3 4
1) 2) 3) 4) 2
3 2 3
7. A gas is taken through the cycle A  B  C  A , as shown. What is the network done by the gas?

1) Zero 2) –2000 J 3) 2000 J 4) 1000 J


8. If Q, E and W denote respectively the heat added, change in internal energy and the work done in a
closed cyclic process, then
1) E = 0 2) Q = 0 3) W = 0 4) Q = W = 0
9. If U represents the increase in internal energy and W the work done by the thermodynamic system,
then
1) U   W is an isothermal process 2) U  W is an isothermal process
3) U   W is an adiabatic process 4) U  W is an adiabatic process
10. A gas undergoes a cyclic process ABCDA as shown in the figure. The part ABC of process is
semicircular. The work done by the gas is:

1) 400 J 2) 2456 J 3) 200  4) 1826 J


11. An ideal monoatomic gas undergoes the process AB as shown in the figure. If the heat supplied and
the work done in the process are Q and W respectively. The ratio Q : W is

1) 2.50 2) 1.67 3) 0.67 4) 0.40

283
Brilliant STUDY CENTRE

12. A mass of diaotmic gas    1.4  at a pressure of 2 atmospheres is compressed adiabatically so that
its temperature rises from 27oC to 927oC. The pressure of the gas in the final state is
1) 8 atm 2) 28 atm 3) 68.7 atm 4) 256 atm
13. Figure below shows two paths that may be taken by a gas to go from a state A to a state C.

In process AB, 400 J of heat is added to the system and in processs BC, 100 J of heat is added to the
system. The heat absorbed by the system in the process AC will be
1) 460 J 2) 300 J 3) 380 J 4) 500 J
14. An ideal gas at pressure P is adiabatically compressed so that its density becomes n times the initial

 CP 
value. The final pressure of the gas will be    
 CV 

1) n  P 2) n   P 3) n   1 P 4) n 1  P
15. An ideal gas A and a real gas B have their volume increased from V to 2 V under isothermal conditions.
The increase in internal energy
1) will be same in both A and B 2) will be zero in both gases
3) of B will be more than that of A 4) of A will be more than that of B
16. When heat energy of 1500 J is supplied to a gas at constant pressure, 2.1 × 105 N m–2, there was an
increase in its volume equal to 2.5 × 10–3 m3. The increase in its internal energy is
1) 450 J 2) 525 J 3) 975 J 4) 2025 J

17. Heat is supplied to a diatomic gas at constant pressure. The ratio of Q : U : W is


1) 5 : 3 : 2 2) 5 : 2 : 3 3) 7 : 5 : 2 4) 7 : 2 : 5
18. The internal energy chagne in a system that has absorbed 2 kcals of heat and done 500 J of work is
1) 6400 J 2) 5400 J 3) 7900 J 4) 8900 J
19. An engine has an efficiency of 1/6. When the temperature of sink is reduced by 62oC, its efficiency is
doubled. Temperatures of the source is
1) 37oC 2) 62oC 3) 99oC 4) 124oC
20. A Carnot engine whose sink is at 300 K has an efficiency of 40%. By how much should the temperature
of source be increased so as to increase its efficiency by 50% of original efficiency?
1) 380 K 2) 275 K 3) 325 K 4) 250 K

284
[Study Package - Physics - Class XI]

21. A diatomic ideal gas is used in a Carnot engine as the working substance. If during the adiabatic
expansion part of the cycle the volume of the gas increases from V to 32 V, the efficiency of the
engine is
1) 0.25 2) 0.5 3) 0.75 4) 0.99
22. A sample of an ideal gas is taken through a cycle a shown in figure. It absorbs 50 J of energy during
the process AB, no heat during BC, rejects 70 J during CA. 40 J of work is done on the gas during BC.
Internal energy of gas at A is 1500 J, the internal energy at C would be
P
B

C A
O V
1) 1590 J 2) 1620 J 3) 1540 J 4) 1570 J
1
23. A Carnot engine, having an efficiency of   as heat engine, is used as a refrigerator. If the work
10
done on the system is 10 J, the amount of energy absorbed from the reservoir at lower temperature is
1) 90 J 2) 1 J 3) 100 J 4) 99 J
24. A Carnot’s engine is made to work between 200 C and 0 C first and then between 0oC and –200oC.
o o

The ratio of efficiencies of the engine in the two cases is:


1) 1.73 : 1 2) 1 : 1.73 3) 1 : 1 4) 1 : 2
25. An ideal gas is taken through the cycle A  B  C  A , as shown in the figure. If the net heat
supplied to the gas in the cycle is 5 J, the work done by the gas in the process C  A is

1) – 5J 2) – 10 J 3) – 15 J 4) – 20 J
26. A sample of 0.1 g of water at 100 C and normal pressure (1.013 × 10 Nm–2) requires 54 cal of heat
o 5

energy to convert to steam at 100oC. If the volume of the steam produced is 167.1 cc, the change in
internal energy of the sample, is
1) 104.3 J 2) 208.7 J 3) 42.2 J 4) 84.5 J
27. An ideal heat engine has an efficiency  . The coefficient of performance of the engine when driven
backward will be

1 1 1  1 


1) 1    2)     3)   1 4)  
     1  
28. The temperature inside a refrigerator is t2 oC and the room temperature is t1 oC. The amount of heat
delivered to the room for each joule of electrical energy consumed ideally will be
t1 t1  273 t 2  273 t1  t 2
1) 2) 3) 4)
t1  t 2 t1  t 2 t1  t 2 t1  273

285
Brilliant STUDY CENTRE

29. The coefficient of performance of a refrigerator is 5. If the temperature inside freezer is –20oC, the
temperature of the surroundings to which it rejects heat is
1) 11oC 2) 21oC 3) 31oC 4) 41oC
30. A gas is compressed isothermally to half its initial volume. The same gas is compressed separately
through an adiabatic process unitl its volume is again reduced to half. Then
1) Compressing the gas isothermally or adiabatically will require the same amount of work.
2) Which of the case (whetehr compression through isothermal or through adiabatic process) requires
more work will depend upon the atomicity of the gas.
3) Compressing the gas isothermally will require more work to be done.
4) Compressing the gas through adiabatic process will require more work to be done.
31. An ideal refrigerator has a freezer at a temperature of –13oC. The coefficient of performance of the
engine is 5. The temperature of the air (to which heat is rejected) will be
1) 325oC 2) 325 K 3) 39oC 4) 320oC
LEVEL - II
1. 540 calories of heat convert 1 cc of water at 100oC to1671 cc of steam at 100oC under atmospheric
conditions. Increase in internal energy will be nearly
1) 300 calorie 2) 20 calorie 3) 500 calorie 4) 540 calories
2. A vessel contains 5 mole of O2. The system is given heat at constant pressure. As a result the gas
expands and does 120 joule work. Heat given to the gas is
1) 310 J 2) 390 J 3) 420 J 4) 560 J
3. One mole of an ideal gas goes from an initial state A to final state B via two processes: It first undergoes
isothermal expansion from volume V to 3V and then its volume is reduced from 3V to V at constant
pressure. The correct P-V diagram representing the two processes is

4. A system is taken from state a to state c by two paths adc and abc as shown in the figure. The internal
energy at a is Ua = 10J. Along the path adc the amount of heat, absorbed Q1  50 J and work obtained
W1  20 J whereas along the path abc the heat absorbed Q 2  36 J . The amount of work along the
path abc is

1) 10 J 2) 12 J 3) 36 J 4) 6 J

286
[Study Package - Physics - Class XI]

5. One mole of an ideal diatomic gas undergoes a transition from A to B along a path AB as shown in the
figure.

The change in internal energy of the gas during the transition is


1) 20 J 2) –12 J 3) 20 kJ 4) –20 kJ
6. In the given (V - T) diagram, what is the relation between pressures P1 and P2?

1) P2 < P1 2) Cannot be predicted 3) P2 = P1 4) P2 > P1


7. In an H2 gas process, PV2 = constant. The ratio of work done by gas to change in internal energy is

2 2
1) 2) 0.4 3) –0.4 4) 
3 3
8. A system is taken from a given initial state to a given final state along various paths represented on a
P-V diagram. The quantity that is independent of the path is
1) amount of heat transferred Q 2) amount of work done W
3) Q but not W 4) (Q – W)
9. A fixed mass of a gas is adiabatically made to expand to double its volume and then isochorically
heated to bring the gas to the original pressure. The ratio of final to initial temperature will be:
1) 1 : 1 2) 2 : 1
3) 1 : 2 4) Depends on the atomicity of the gas
10. During adiabatic process pressure P versus density  equation is:

1) P  = constant 2) P  = constant

3) P  1  = constant 4) P1/    = constant

287
Brilliant STUDY CENTRE

11. In the following P-V diagram two adiabatics cut two isothermals at temperatures T1 and T2. The value
Va
of
Vd will be

Vb Vc Vd
1) 2) 3) 4) VbVc
Vc Vb Va
12. A monoatomic gas at a pressure P, having a volume V expands isothermally to a volume 2V and then
adiabatically to a volume 16 V. The final pressure of the gas is (Take   5 / 3 )
1) 64 P 2) 32 P 3) P/64 4) 16 P
13. An ideal gas is compressed to half its initial volume by means of several processes. Which of the
process results in the maximum work done on the gas?
1) Isochroic 2) Isothermal 3) Adiabatic 4) Isobaric
14. A diatomic gas which has initial volume of 10 litre is isothermally compressed to 1/15th of its original
volume where initial pressure is 105 pascal. If temperature is 27oC then find the work done by gas
1) –2.70 × 103 J 2) 2.70 × 103 J 3) –1.35 × 103 J 4) 1.35 × 103 J
15. 4 mole of a diatomic ideal gas are heated at constant pressure from 27oC to 327oC. Internal energy of
the gas increases by (take R = 2 cal/mol-K)
1) 3000 calorie 2) 6000 calorie 3) 8400 calorie 4) 10000 calorie

1
16. A monoatomic gas at pressure P1 and volume V1 is compressed adiabatically to the of its original
8
volume. What is the final pressure of the gas?
1) 64P1 2) P1 3) 16P1 4) 32P1
17. A monoatomic ideal gas, initially at temperature T 1, is enclosed in a cylinder fitted with a frictionless
piston. The gas is allowed to expand adiatbatically to a temperature T2 by releasing the piston suddenly.
If L1 and L2 are the lengths of the gas column before and after expansion respectively, then T1/T2 is
given by
2/3 2/3
 L1  L1 L2  L2 
1)   2) 3) 4)  
 L2  L2 L1  L1 
18. n moles of an ideal gas expand adiabatically. During this process, temperature changes from an initial
value T1 to a final value T2. Work done during the process can be expressed as :

1) nCr  T1  T2  2) nC V  T1  T2  3) nR  T1  T2  4) nR  T2  T1 

288
[Study Package - Physics - Class XI]

19. During an isothermal expansion, a confined ideal gas done –150 J of work against its surroundings.
This implies that
1) 150 J of heat has been removed from the gas
2) 300 J of heat has been added to the gas
3) no heat is transferred because the process is isothermal
4) 150 J of heat has been added to the gas
20. Two Carnot engines A and B are operated in series. The first one, A, receives heat at T1 (=600 K) and
rejects to a reservoir at temperature T2. The second engine B receives heat rejected by the first
engine and, in turn, rejects to a heat reservoir at T3(=400K). Calculate the temperature T2 if the work
outputs of the two engines are equal
1) 500 K 2) 300 K 3) 400 K 4) 600 K
21. A thermodynamic process of one mole ideal monoatomic gas is shown in figure. The efficiency of
cyclic process ABCA will be:

100
1) 25% 2) 12.5% 3) 50% 4) %
13

22. An ideal gas is taken through a quasi-static process described by P  V 2 , with   5.00 atm / m 6 .
The gas is expanded to twice its original volume of 1.00 m3. How much work is done by the gas in
expanding gas in this process?
1) 1.18 MJ 2) 2.18 MJ 3) 1.28 MJ 4) 3.18 MJ
23. An ideal gas is compressed isothermally until its pressure is doubled and then allowed to expand adiabatically
to regain its original volume   1.4 and 21.4  0.38 ). The ratio of the final to initial pressure is
1) 0.76 : 1 2) 1 : 1 3) 0.66 : 1 4) 0.86 : 1
24. A motor cycle engine delivers a power of 10 kW, by consuming petrol at the rate of 2.4 kg/hour. If the
calorific value of petrol is 35.5 MJ/kg, the rate of heat rejection by the exhaust is
1) 5.5 kW 2) 13.7 kW 3) 11.2 kW 4) 9.7 kW
25. Two cylinders A and B fitted with pistons contain equal amounts of an ideal diatomic gas at 300 K. The
piston of A is free to move while that of B is held fixed. Some amount of heat is given to the gas in each
cylinder. If the rise in temperature of the gas in A is 30 K, then the rise in temperature of the gas in B is
1) 18 K 2) 42 K 3) 30 K 4) 50 K
26. A Carnot engine has an efficiency of 1/6. When the temperature of the sink is reduced by 62oC, its
efficiency is doubled. The temperature of the source and the sink are respectively
1) 124oC, 62oC 2) 99oC, 37oC 3) 37oC, 99oC 4) 62oC, 124oC

289
Brilliant STUDY CENTRE

27. Heat energy absorbed by a system in going through a cyclic process shown in figure.

1) 102 J 2) 103 J 3) 104 J 4) 107 J


28. A refrigerator works between 4oC and 30oC. It is required to remove 600 calories of heat every second
in order to keep the temperature of the refrigerated space constant. The power required is (Take 1 cal
= 4.2 Joules)
1) 236.5 W 2) 2365 W 3) 2.365 W 4) 23.65 W
29. In a mechanical refrigerator, the low temperature coils are at a temperature of –23oC and the
compressed gas in the condenser has a temperature of 27oC. The theoretical coefficient of performance
is
1) 5 2) 8 3) 6 4) 6.5
30. An ideal gas is taken from state 1 to state 2 through optional path A, B, C and D as shown in the PV
diagram. Let Q, W and U represent the heat supplied, work done and change in internal energy of the
gas respectively. Then,

1) Q A  Q D  WA  WD 2) Q B  WB  QC  WC

3) WA  WB  WC  WD 4) Q A  Q B  QC  Q D

290
[Study Package - Physics - Class XI]

KEY WITH HINTS


LEVEL - I
1. 3 For cyclic process U  0

 W4  Q1  Q2  Q3  Q 4  W1  W2  W3  765 J
2. 3 Let temperature of mixture be T then

n1C v1 T1  n 2C v2 T2
n1Cv1  T  T1   n 2Cv2  T2  T  ;  T   31o C
n1C v1  n 2C v2

3. 1 In a cyclic process, U  0 .
In a cyclic process work done is equal to the area under the cycle.
 W  –Area of rectangle ABCD = –P(2V) = – 2PV

According to first law of thermodynamics, for cyclic process, Q  W,  u  0 

So heat absorbed,  Q  W  2PV ;  Heat rejected by the gas = 2 PV


4. 4 Work done is positive if the cycle is clockwise and negative if anticlockwise.

1 1
WAEDA   area of AED   P0 V0 ; WBCEB   Area of BCE   P0 V0
2 2

1 1
The net work done by the system is Wnet   P0 V0  P0 V0  zero
2 2
5. 1 Change in internal energy is path independent and depends only on the initial and final states.

Therfore, U1  U 2  U 3
Workdone, W = Area under P-V graph, area under curve 1 > curve 2 > curve 3

 W1  W2  W3 ;  Q1  Q 2  Q3 ,  U same 
3
6. 2 P  T3 ; PT  a constant

r
For an adiabatic process, PT  /1 = constant ----- (ii);  3
1 r

3 Cp 3
  3  3 ; 2  3 or   ;  
2 Cv 2
7. 4 For cyclic process, the network done by the gas is W = Area of the cycle ABCA

1
  5  103  4  105  10  10 2 J  1000J
2

291
Brilliant STUDY CENTRE

8. 1 Internal energy depends only on the initial and final states of temperature and not on the path. In
a cyclic process, as initial and final states are the same, the internal energy change is zero.
9. 3 According to first law of thermodynamics Q  U  W
In an adiabatic process, Q  0 ;  0  U  W or U   W
2
R 2   20 
10. 3 W  Area ABCDA    200  joule
2 2
Q nC P T C P 5
11. 1 (P = constant);   
W nRT R 2

T  Tf  1
12. 4 For an adiabatic process 1 = constant; Pf  Pi   ------ (i)
P  Ti 
1.4
 1200 1.4 1
T1  300K, Tf  1200K ; Pi  2atm,   1.4 ; Pf   2   
 300 
1.4/ 0.4 7/ 2 7
  2  4   2  22    2  2   28  256 atm

13. 1 As initial and final points are same so U ABC  U AC

AB is isochoric process. Q AB  U AB  400J (since WAB  0 )

BC is isobaric process. Q BC  U BC  WBC

100  U BC  6 10 4  4 103  2 103  ; ABC  100  120  20 J

As, U ABC  U AC ; U AB  U BC  Q AC  WAC

 1 
400  20  Q AC   2  104  2  103   2  103  4  104  ; Q AC  380  80  460 J
 2 
14. 1 In adiabatic compression, PV  = const.

m   
 P    const ;  P  d  = const.;  P1  d1   P2  d 2 
d
 
d   nd 
 P2  P1   2   P    ;  P  Pn 
 d1   d  2

15. 3 In ideal gas there is no interactions between particles, no intermolecular forces. So pressure
changes at constant temperature does not change internal energy U  0 .
But real gases have interactions, i.e., attraction between molecules at low pressure and repulsion
at high pressure. Thus even if temperature is constant, change in pressure, changes internal
energy U  0 .

292
[Study Package - Physics - Class XI]

Q  1500J ; W  PV   2.1 10 Nm  2.5  10 m   5.25  10 N m  525J


5 2 3 3 2
16. 3

Q  W  U ;  U  Q  W  1500J  525J  975J

7  7  5  5 
17. 3 Q  C P T  RT  CP  R  ; U  C V T  RT  CV  R 
2  2  2  2 
and W  Q  U  RT  Q : U : W  7 : 5 : 2
18. 3 Heat energy given Q = dU + dW where dU is the change in internal energy and dW is the work
done.
Given dQ = 2 kcals = 2000 × 4.2 J and dW = 500 J
 dQ  2000  4.2  dU  500 ; dU = 7900 J

T2
19. 3 Efficiency of an engine,   1 
T1
where T1 is is the temperature of the source and T2 is the temperature of the sink.

1 T T 5
  1  2 or 2 
6 T1 T1 6
When the temperature of the sink is decreased by 62oC (or 62K), efficiency becomes double.
Since, the temperature of the source remain unchanged

1  T  62  1  T  62 
2  1 2 or  1  2 ; 2T1  3T2  186
6 T1 3 T1

5  T
   2  Ti  186 or 1  186 ; or T1  372K  99 o C
2  2

T2 T2 40 3
20. 4  1  1  ; T1  5 T2  500 K
T1 ; or,
T1 100 5 3

50 3
New efficiency,         60%
100 2

T2 60 2
  1  ; T   5  300  750 K ; Increase in temperature 
T1 100 5 1 T1  T1  250 K
2

21. 3 For an adiabatic process ; TV  1 = constant;  T1V11  T2 V2  1


1
V   1 7
T1  T2  2   T2  32  ;  ; T1   32 
(7/5) 1
; T2  4T2
 V1  5

T2  1  3
Efficiency of the engine,   1   1   0.75
T1  4  ; 4

293
Brilliant STUDY CENTRE

22. 1 WAB  0 as V = constant.  Q AB  U AB  50J (Given)

U A  1500J  U B  1500  50  J  1550J ; WBC  U BC  40J (Given)

 U BC  40J  U C  1550  40  J  1590J

T1 1 T T 9
23. 1 For Carnot engine, Efficiency,   1  ;  1 1 ; 1 
T2 10 T2 T2 10

Q 2  Q1  W  Q1  10

Q 2 T2
For refrigerator,  
Q1 T1

Q1  10 10 10 10 1
 ;   1  ; Q = 90 J;  Heat absorbed = 90 J
Q1 9 Q1 9 9 1

T2  273  0   200
24. 2 In the first case 1  1   1
T1  273  200  473 ;

 273  200   200 ; 1 1


In second case, 2  1     1:1.73
 273  0  273 2  473 
 
 273 

25. 1 Q  W  U  Wcycle  Q  U  5  0  5 J

But, Wcycle  WAB  WBC  WCA ; So, 5  10  2  1  0  WCA ;  WCA  5 J

26. 2 Change in heat  Q   54cal  225.72J ; W  P  v  v w   16.97 J

Since, Q  U  w ;  U  225.72  16.917 ;  U  208.8J

 1
T2

T2

T2 / T1

1    1  1
27. 3 T1 and T1  T2 1   T2 / T1   

Heat given to high temperature  Q1  T1


28. 2 For refrigerator, 
Heat taken from lower temperature  Q 2  T2

Q1 Q1 t  273 W t 2  273 W t t
  1 ; 1  ;  1 2
Q 2 Q1  W t 2  273 Q1 t1  273 Q1 t1  273

t1  273
Heat delivered to the room for each joule of electrical energy (W = 1 J) Q1 
t1  t 2

294
[Study Package - Physics - Class XI]

T2
29. 3 The coefficient of performance of a refrigerator is  
T1  T2 ; T2 = 253 K

253K
5  5T1  253K  5  253K   6  253K 
T1  253K ;

6
T1   253K   303.6K  30.6 o C  31 o C
5
30. 4 V1 = V, V2 = V/2

On P-V diaram, Area under adiabatic curve > Area under isothermal curve.
So compressing the gas through adiabatic process will require more work to be done.

T2  273  13  260


31. 3 Coefficient of performance K  5
T1  T2 T1   273  13 T1  260

 5T1  1300  260  5T1  1560 ;  T1  312 K  39 o C

LEVEL - II

105 1670  106


1. 3 U  Q  W  540  P V  540   540  40  500cal
4.2

7  7 7
2. 3 W  nRT  120 ; Q  nCp T  n  2 R  T  2 W  2  120  420 J

3. 4

295
Brilliant STUDY CENTRE

4. 4 According to first law of thermodynamics Q  U  W


Along the path adc; Change in internal energy, U1  Q1  W1  50J  20J  30J

Along the path abc; Change in internal energy U 2  Q 2  W2  36J  W2
As change in internal energy is path independent.
 U1  U 2 ;  30J  36J  W2 ; W2  36J  30J  6J

 5R  5nR  PB VB PA VA 
5 4 We know, U  nC v T  n    TB  TA     
 2  2  nR nR 

5 5 5
  PB VB  PA VA    2 103  6  5 103  4    8 103   20 kJ
2 2 2
nRT
6. 1 According to ideal gas equation PV = nRT or V 
P
For an isobaric process, P = constant and V  T
Therefore, V - T graph is a straight line passing through origin. Slope of this line is inversely
proportional to P. In the given figure,  Slope  2   Slope 1 ;  P2  P1

T2 273 273 2  1000  5 


7. 4    ; Mass of ice melting per second    g
T1  T2 293  273 20 3600 9
To prevent the melting of ice, heat to be driven out per second.

5 40 2 Q Q 186
i.e., Q 2     80  cal  186J ; As,   W , W    273 / 20  13.6J
9 9  
Power of the motor = 13.6 W
8. 4 The only quantity (Q – W) which itself is the internal energy of the system is independent of the
path.
9. 2 PV/T = constant. Evidently, the pressure is the same but volume gets doubled and so, T also
get doubled.
10. 2 In adibatic process, PV  = constant .......(1)

m
Density   ; or   V 1 ; Hence, equation (1) can be written as P  = constant
V
11. 1 For adiabatic process TV  1 = constant
1
1 1 T V 
For bc curve, T1V b  T2 Vc  2  b 
T1  Vc 

1
1 1 T V  Vb Va
For cd curve, T1V a  T2 Vd  2  a  ;  
T1  Vd  Vc Vd

296
[Study Package - Physics - Class XI]

P
12. 3 First, isothermal expansion PV = P'(2V) (PV = a constant); P 
2
 
Then, adiabatic expansion P  2V   Pf 16V  ( PV  = constant)

5/3 5/3 5/3


P
 2V 5/3  Pf 16V 5/3 ; Pf  P  2V  P1
  
P 1 
  3 
P 1  P
 
2 2  16V  28 22  2  25  64

V0
13. 3 The P-V diagram of an ideal gas compressed from its initial volume V0 to by several
2
processes is shown in the figure.
Work done on the gas = Area under P-V curve.
As area under the P-V curve is maximum for adiabatic process, so work done on the gas is
maximum for adiabatic process.

V  V  1
14. 1 W  nRT ln  2   P0 V0 ln  2   105  10  103 ln    2.70  103 J
 V1   V1   15 

5 
15. 2 U  nC V T  4  R   327  27   3000R  6000 cal
2 

16. 4 For an adiabatic process, PV  = constant;  P1V1  P2 V2


5/3
5 5/3  V1  5
For a monoatomic gas,   ;  P1V1  P2   or P2  P1   2   32P1
3  8 
17. 4 For an adiabatic process, TV  1 = constant. V1  AL1 , V2  AL2

5 2/3
1
T  V 3 5 T1  L 2 
 1   2  , as   for monoatomic gas.; or  
T2  V1  3 T2  L1 

18. 2 W  U  nC V  T1  T2 
19. 4

WA  T2  WB  T3 
20. 1 For Carnot engine A,  1    1  
Q1  T1  ; For Carnot engine B,
Q 2  T2 

 T   T  T1  T2  T  Q1 T1 
WA  WB ; Q1 1  2   Q2  1  3  ;  1   1 3
T2  T1  T2   
 T1   T2   Q 2 T2 

T1 T T  T 600  400
 1  1  3 ; T2  1 3   500K
T2 T2 2 2

297
Brilliant STUDY CENTRE

1 1 13
21. 4 W P0 V0  RT0 ; Heat supplied  Q AB  Q BC  C V T0  C P 2T0  RT0
2 2 2

1 1
P0 V0
 Efficiency  13 13    100  7.7%
 2 100  P0 V0  RT0  13
13  2 2 
P0 V0
2
f f
1
Work done by the gas W   PdV ; W   V dV    Vf  Vi 
2 3 3
22. 1
i i 3

Vf  2Vi  2 1.00m 3   2.00m 3

1 3 3
W   5.00 atm / m 6 1.013 105 Pa / atm     2.00m 3   1.00m 3    1.18MJ
3 
 
23. 1 Let V be the original volume of the gas.; For an isothermal process, PV = constant

P   P  V
 Pi Vi  Pf Vf or Vf  Vi  i  or Vf  V  i  
 Pf   2Pi  2
For an adiabatic process, PV   constant.


V Pf  1.4 0.76

According to question  2Pi     Pf V or  2  2   2  2   2  0.38  
2 Pi 1
24. 2 Heat received by burning of petrol in one hour will be = 2.4 × 3.5 × 106 = 85.2 × 106 J/hour

85.2  106 J 4
 The rate at which heat is received   3600s   2.37  10 J / s  23.7 kW

Evidently, the rate of heat rejection = rate at which heat is produced – rate at which work is
obtained = 23.7 kW – 10 kW = 13.7 kW

25. 2 Q A  nCP  T  A and Q B  nC V  T B ; According to question Q A  Q B

CP
  T B   T  A  42K
CV

1 T TC  TC   62   T  62  1 1 62
26. 3  1 C ;  1   1 C  ;  
6 TH TH TH TH TH 3 6 TH

62 1
 ;  TH  372K  99o C ; 1  1  TC ;  TC  5 ;
TH 6 6 372 372 6

5  372
TC   310K  37 o C
6

298
[Study Package - Physics - Class XI]

 30  10  30  10  2
27. 1 Q  W  U  W  0       100J  10 J
 2  2 

28. 1 Given, T2  4o C  277 K, T1  30 o C  303K ; Q2 = 600 cal per second

T2 277 277
Coefficient of performacne,    
T1  T2 303  277 26 .

Q2
Also,   ;  Work to be done per second = power required
W

Q2 26
W   600 cal per second
 277

26
  600  4.2 J per second = 236.5 W
277

T2  273  23 250 250


29. 1 Coefficient of performance K  ;   5
T1  T2  273  27    273  23 300  250 50
30. 1 Work done by gas in all four process is positive and in order

WA  WB  WC  WD ;  (3) is false.
The change in initial energy U is same for all process.

 Q A  U  WA ------(1); Q B  U  WB -------(2)

QC  U  WC ------(3); Q D  U  WD ---------(4)

Hence, Q A  Q B  Q C  Q D ;  (4) is false.

From equations (1) and (4) Q A  Q D  WA  WD ;  1 is true

From equations (2) and (3) Q B  Q C  WB  WC ;   2  is false.

299
Brilliant STUDY CENTRE

CHAPTER - 12
KINETIC THEORY

Ideal gas
A gas which follows all gas laws and gas equation at every possible temperature and pressure is known as
an ideal or perfect gas.
Kinetic Theory of Gases
Postulates
• A gas consists of small identical, particles called molecules.
• The molecules are spherical, rigid and elastic point masses.
• The size of the molecule is negligible in comparison with the intermolecular distance.
• Molecules travel in all directions with all possible velocities.
• The molecules during their travel make collisions among themselves and also with the walls of the
container.
• Between molecules there is no attractive or repulsive force.
• When the molecules collides with the walls of the container, their momentum changes and due to this,
pressure is applied on the walls.
• The time of collision is negligible as compared to the interval between collisions.
• The average distance travelled by a molecule between two successive collisions is called its mean
free path.
Pressure Exerted by a gas
Consider an ideal gas of N molecules contained in a cubical box of side a.
y

v
a
a
x
a
 
Consider a molecule travelling with a velocity v from the origin strikes a wall, the velocity v can be
  
resolved into its components v x , v y and v z along x, y and z axis. After collision the molecule retraces the
same path.

300
[Study Package - Physics - Class XI]

 Change in momentum along x -direction, p  mv x   mv x   2mv x

2a
The interval between successive collision on the same wall is, t 
vx .

vx
The number of collision made by a single molecule in one sec on the same face is, n  .
2a

 vx  mv x2
 Change in momentum per sec on one face due to a single molecule     2mv x  
 2a  a

m
Change in momentum due to all the molecules striking a face, Fx   v 2x
a

F Fx m 2 m 2
Pressure exerted on the wall Px    v x  v x  V  a3
A A a3 V

m
 Total pressure exerted will be  Px  Py  Pz    v x2  v 2y  v z2 
V

2 2 2 2 m 2
Let Px  Py  Pz  P and v x  v y  v z  v ; 3p  v
V

2
If  v  is the mean square velocity, then  v 
2

v2
1  v22  ......v 2N 

v2  3P 
m
N  v2 
N N V

1 m  2 1 2 mN
P N v  P  v  = P is density of gas 
3V 3 V

2 v12  v 22  .......v 2N
The root mean square speed of the gas v rms   v 
N

3P v  1
Also v rms  ; rms
  when P is constant.

When pressure remaining a constant.

1 2
Kinetic energy per unit volume E   v
2

For a given gas v rms  T

When temperature remains constant.

301
Brilliant STUDY CENTRE

1 3RT 3kT
v rms  ;  v rms  
M M m

m  N 0 M ; M - molecualr weight, m - mass of a molecule.

 v1  v2  v3  ......
Average speed v av 
N

8RT
For continuous distribution v av 
M
Most probable velocity in the velocity possessed by the maximum fraction of gaseous molecules at a
particular time.

2RT
v mp  ; M - molecular mass.
M

2RT 8RT 3RT


v mp : v ay : v rms  : :
M M M

8
v mp : vavg : v rms  2 : : 3

v mp  vav  v rms

Mean free path   

Average distance travelled by a gas molecular between collissions is called mean free path.

1

N
2d 2
v

N
n is the no. of molecules per unit volume.
v
d - is the diameter of the molecule.

1

2d 2 n
Degrees of freedom
The no. of independent ways in which a molecule or an atom can exhibit motion are called degrees of
freedom. The motion can be translational, rotational or vibrational or a combination of them.
The number of independent co-ordinates required to specify the dynamical state of the system is called
degrees of freedom.
302
[Study Package - Physics - Class XI]

A body moving in space have 3 translational degrees of freedom.


A diatomic gas molecule (H2, O2) can have 5 degree of freedom (3-translational and 2 rotational). In
addition to this a diatomic molecule can have two vibrational degrees of freedom. Such a molecule has 7
degrees of freedom (At high temperatures)
A polyatomic molecule such as CO2, H2O have three translational and 3 rotational degrees of freedom,
so it have six degrees of freedom.
Law of equipartition of energy
The energy of a gas molecule is equally distributed among its various degrees of freedom. Associated
with each degree of freedom, there is an energy of ½ kT, where T is the absolute temperature.
For one molecule of a gas
Let f is the no. of degree of freedom of a molecule.

f
Energy related with all degree of freedom  kT
2

3
According to the equipartition theorem the average energy of an ideal gas molecule will be kT . Since
2
5
it has three degrees of freedom. For diatomic and polyatomic gases the average energy will be   kT and
2
6
  kT respectively..
2

Volume coefficient of expansion (  ) and pressure coefficient of expansion (  ) is same for an ideal gas and
1 o 1 o
is equal to / C . ie,     / C
273 273
Equation of state for an ideal gas
A relation connecting pressure, temperature and volume of a gas describes the state of the gas. That
is physical condition of the system is called equation of state. The equation of state for an ideal gas is given
by

PV  RT

m N
where  = no. of moles of gas.   
M N A , where m is the mass of the gas. M is the mass of a
molecules. N is no. of molecules in the gas. NA is the Avogadros number.

N  R 
PV  RT    NT  NkT , k is Boltzmanns constant.
NA  NA 

For 1 mole of a gas   1 ,  PV  RT

303
Brilliant STUDY CENTRE

N
P = nkT n no. of molecules / unit volume.
V
This is the gas equation for an ideal gas in terms of number of molecules per unit volume.
Gas laws
1) Boyles’ law
For a given mass of an ideal gas at constant temperature, the volume of a gas is inversely proportional
to its pressure.
1
V , where T = a constant and m = a constant
P
Charle’s law
For a given mass of an ideal gas,
Volume of a gas is directly proportional to its absolute temperature, when the pressure is constant.

V  T , when P is a constant, m is a constant.


Gay - Lussac’s law
For a given mass of ideal gas at constant volume, pressure of a gas is directly proportional to its
absolute temperature.

P  T , when m and V are constant.


Avogadro’s Law
Equal volumes of all gases at the same temperature and pressure contains equal number of molecules.
N1 = N2 when P, V and T are the same for two gases.
Dalton’s Law
Pressure exerted by a gaseous mixture is equal to the sum of partial pressure of each component
gases in the mixture.
ie, P = P1 + P2 + ...........
Specific heat capacity
When a gas is heated it can change its volume or pressure. So a gas will be having two specific heat
capacities. Specific heat at constant volume and specific heat at constant pressure.
Specific heat at constant volume (Cv) is defined as the quantity of heat required to raise the temperature
 Q v
of unit mass of gas through 1 K when its volume is kept constant C v 
mT
Specific heat at constant pressure (Cp) is the quantity of heat required to raise the temperature of unit
 Q p
mass of gas through 1 K, keeping the pressure constant. Cp =
mT
At constant volume heat supplied.

 Q v  U  Cv T U is the increase in internal energy..

304
[Study Package - Physics - Class XI]

At constant pressure  Q p  C p T

PV
Cp  Cv 
T
for an ideal gas PV  RT
Cp – Cv = R. This is called the Mayers Relation.
 Molar heat capacity at constant pressure is greater than that of constant volume.
U 1
Cv   Rf
T 2

1  f
Cp  Cv  R  Rf  R  R 1  
2  2

Cp
The ratio of specific heat capacity is  
Cv

Cp 2
  1 ; f = 3n – r; n - no of atom/molecule; r - no. of possible relations/restrictions.
Cv f
for monoatomic n = 1, r = 0; f = 3n – r = 3 (translational)

Cp 5
For monoatomic gas   
Cv 3
For diatomic n = 2, r = 1; f = 3 × 2 – 1 = 5; 3-translational + 2 rotational.

7
R
cp
2 7
For diatomic gas    
cv 5 R 5
2

R R
Cv  ; Cp 
 1  1

R 3 R 5
For monoatomic; C v   R ; for diatomic C v  7  R
5
1 2 1 2
3 5

8
R cp
4
For polyatomic gas   2 
cv 6 R 3
2
 decreases with number of atoms in a molecule.

305
Brilliant STUDY CENTRE

Specific heat of solids

1
Average energy associated with an atom due to its oscillations in one dimension  2  kT  kT
2
In three dimension, average energy  U  3kT

For 1 gm mole of solid U  3kT  N A  3RT

From Ist law of thermodynamics

Q  W  U  U  PV
For solid V is negligible

 Q  U

Q U 3RT
C    3R
T T T
Specific heat capacity of water
Consider water like a solid made up of 3 atoms (2 hydrogen + 1 oxygen)

Total energy of 1 mole of water U  3  3kT  N A  9RT

U 9RT
C   9R
T T
When n1 molecules of an ideal gas is mixed with n2 moles of another gas.

n1M1  n 2 M 2
M mixutres 
n1  n 2

n1T1  n 2 T2
Tmixture 
n1  n 2

n1C v1  n 2 C v2
Cv mixture 
n1  n 2

n1Cp1  n 2 C p2
C pmixture 
n1  n 2

C pmixture
rmixture 
C vmixture

306
[Study Package - Physics - Class XI]

QUESTIONS
LEVEL - I
1. An ideal gas at 17oC has a pressure of 760 mm of Hg. The gas is compressed isothermally until its
volume is halved. The final pressure of the gas will be
1) 2010 mm of Hg 2) 1890 mm of Hg
3) 1520 mm of Hg 4) 1650 mm of Hg
2. A thin balloon filled with air at 47oC has a volume of 3.0 litre. If on placing it in a cooled room its volume
becomes 2.7 litre, the temperature of the room is
1) 42oC 2) 30oC 3) 15oC 4) 0oC
3. The volume V of an enclosure contains a mixture of three gases, 16 g of oxygen, 28 g of nitrogen and
44 g of carbon dioxide at absolute temperature T. Consider R as universal gas constant. The pressure
of the mixture of gases is

4RT 3RT 88RT 5 RT


1) 2) 3) 4)
V V V 2 V
4. 8 g of O2, 14 g of N2 and 22 g of CO2 is mixed in a container of 10 L capacity at 27oC. The pressure
exerted by the mixture in terms of atmospheric pressure is (R = 0.082 L atm K–1 mol–1)
1) 1.4 atm 2) 2.5 atm 3) 3.7 atm 4) 8.7 atm

5. The equation of state corresponding to 8 g of O2 is

RT RT
1) PV = 8RT 2) PV  3) PV = RT 4) PV 
4 2
6. The equation of state for 5 g of oxygen at a pressure P and temperature T, when occupying a volume
V, will be (R is the gas constant)
1) PV = (5/32)RT 2) PV = 5RT 3) PV = (5/2) RT 4) PV = (5/16) RT
7. At what temperature the molecules of nitrogen will have the same rms velocity as the molecules of
oxygen at 120oC?
1) 70oC 2) 350oC 3) 273oC 4) 457oC
8. A sample of gas is at 0oC; to what temperature must it be raised in order to double the rms speed of
its molecules?
1) 103oC 2) 273oC 3) 819oC 4) 1092oC
9. At room temperature the rms speed of the molecules of a certain diatomic gas is found to be 1930 m/s.
The gas is
1) H2 2) F2 3) O2 4) Cl2
10. At what temperature is the r.m.s. velocity of a hydrogen molecule equal to that of an oxygen molecule
at 47oC?
1) 80 K 2) –73 K 3) 3 K 4) 20 K

307
Brilliant STUDY CENTRE

11. Figure shows plots of certain processes with a given amount of ideal gas, then

1) T1 > T2 2) T1 < T2 3) T1 = T2 4) None of these


12. Under which of the following conditions is the law PV = RT obeyed most closely by a real gas?
1) High pressure and high temperature 2) Low pressure and low temperature
3) Low pressure and high temperature 4) High pressure and low temperature
13. The root mean square velocity of the molecules of a gas is 1260 m/s. The most probable speed of the
molecules is
1) 1029 m/s 2) 1161 m/s 3) 1671 m/s 4) 917 m/s
14. The root mean square velocity of the molecules of a gas is 1260 m/s. The average speed of the
molecules is:
1) 1029 ms–1 2) 1161 ms–1 3) 1671 ms–1 4) 917 ms–1
15. In a jar having a mixture of H2 and He.
1) hydrogen has more mean KE 2) helium has more mean KE
3) both have same mean KE 4) both have same internal energy
16. The work of 146 kJ is performed in order to compress one kilomole of gas adiabatically and in this
process the temperature of the gas increases by 7oC. The gas is (R = 8.3 J mol–1 K–1)
1) diatomic 2) triatomic
3) a mixture of monatomic and diatomic 4) monatomic
17. A sealed container with negligible thermal coefficient of expansion contains helium (a monoatomic
gas). When it is heated from 300 K to 600 K, the average kinetic energy of helium atom is:
1) halved 2) left unchanged

3) doubled 4) increases by a factor of 2


18. The ratio of the total energy of all the molecules of one mole of O2 to the total energy of all the
molecules of two moles of He at the same temperature is:
1) 2 : 1 2) 1 : 2 3) 5 : 4 4) 5 : 6
19. The temperature of argon, kept in a vessel, is raised by 1oC at constant volume. Part of total heat
supplied to the gas may be taken as (i) translational and (ii) rotational energies. Their respective
shares are
1) 60%, 40% 2) 100%, 0% 3) 0%, 10% 4) 40%, 60%
20. 310 J of heat is required to raise the temperature of 2 moles of an ideal gas at constant pressure from
25oC to 35oC. The amount of heat required to raise the temperature of the gas through the same
range at constant volume is
1) 384 J 2) 144 J 3) 276 J 4) 452 J

308
[Study Package - Physics - Class XI]

21. A spherical balloon of volume 4.00 × 103 cm3 contains helium at pressure of 1.20 × 105 Pa. How
many moles of helium are in the balloon if the average kinetic energy of the helium atoms is 3.60 ×
10–22 J?

1) 3.32 mol 2) 2.16 mol 3) 4.12 mol 4) 2.8 mo


22. An ideal gas occupies a volume of 2 m3 at a pressure of 3 × 106 Pa. The energy of the gas is

1) 9 106 J 2) 3  10 2 J 3) 6  104 J 4) 108 J

23. A container has N molecules at absolute temperature T. If the number of molecules is doubled but
kinetic energy in the box remains the same as before, the absolute temperature of the gas is:
1) T 2) T/2 3) 2T 4) zero

24. An ideal gas is enclosed in a cylinder at pressure of 2 atm and temperature, 300 K. The mean time
between two successive collisions is 6  108 s . If the pressure is doubled and temperature is increased
to 500 K, the mean time between two successive collisions will be close to

1) 4  108 s 2) 3  106 s 3) 0.5  108 s 4) 2  107 s

25. For a gas C P  C V  R in a state P and C P  C V  1.10 R in a state Q, TP and TQ are the temperature
in two different states P and Q respectively. Then

1) TP  TQ 2) TP  TQ 3) TP  TQ 4) TP  0.9TQ

26. If 70 cals of heat is required to raise the temperature of 2 moles of an ideal gas at constant pressure
from 30oC to 35oC, then the amount of heat required to raise the temperature of same gas through
same range at constant volume is

1) 50 cals 2) 70 cals 3) 60 cals 4) 65 cals

27. Three moles of oxygen are mixed with two moles of helium. What will be the ratio of specific heats at
constant pressure and constant volume for the mixture?
1) 2.5 2) 3.5 3) 1.5 4) 1

28. A gaseous mixture enclosed in a vessel contains 1 g mole of a gas A (with  = 5/3) and another gas
B (with   7 / 5 ) at a temperature T. The gases A and B do not react with each other and assumed to
be ideal. The number of gram moles of B, if  for the gaseous mixture is 19/13 is

1) 2 2) 12 3) 16 4) 8

29. A spherical balloon of volume 4.00 × 103 cm3 contains helium at pressure of 1.20 × 105 Pa. How
many moles of helium are in the balloon if the average kinetic energy of the helium atoms is 3.60 ×
10–22 J?
1) 3.32 mol 2) 2.16 mol 3) 4.12 mol 4) 2.8 mo

30. 200 gram of a solid ball at 20oC is dropped in an equal amount of water at 80oC. The resulting
temperature is 60oC. This means that specific heat of solid is

1) one fourth of water 2) one half of water 3) twice of water 4) four times of water

309
Brilliant STUDY CENTRE

LEVEL - II
1. In a vessel, the gas is at pressure P. If the mass of all the molecules is halved and their speed is
doubled, then the resultant pressure will be
1) 2 P 2) P 3) P/2 4) P
2. A vessel of volume 4 L contains a mixture of 8 g of oxygen, 14 g of nitrogen and 22 g of carbon dioxide
at 27oC. The pressure exerted by the mixture is
1) 5.79  105 Nm 2 2) 6.79  105 Nm 2 3) 7.79  103 Nm 2 4) 7.79  105 Nm 2
3. The volume of a gas at constant pressure of 103 N/m2 expands by 0.25 m3. The work done is equal
to
1) 2.3 erg 2) 250 J 3) 250 watt 4) 250 N
4. When 2 g of a gas is introduced into an evacuated flask kept at 25oC the pressure is found to be one
atmosphere. If 3 g of another gas is added to the same flask the pressure becomes 1.5 atmospheres.
The ratio of the molecular weights of these gases will be
1) 1 : 3 2) 3 : 1 3) 2 : 3 4) 3 : 2
5. The root mean square velocity of the molecules in a sample of helium is 5/7th that of the molecules
in a sample of hydrogen. If the temperature of the hydrogen as is 0oC, that of helium sample is about
1) 0oC 2) 4 K 3) 273 oC 4) 100 oC
6. At room temperature (27oC) the rms speed of the molecules of a certain diatomic gas is found to be
1920 ms 1 . The gas is
1) Cl2 2) O2 3) N2 4) H2
7.. The temperature of an ideal gas is increased from 27oC to 127oC, then percentage increase in vrms is
1) 37% 2) 11% 3) 33% 4) 15.5%
8. At the top of a mountain a thermometer reads 7oC and a barometer reads 70 cm of mercury. At the
foot of the mountain, they read 27oC and 76 cm of mercury respectively. The ratio of density of air at
the top to that at the bottom of the mountain is
1) 0.885 2) 0.987 3) 0.75 4) 1.0
9. For V versus T curves at constant pressures P1 and P2 for an ideal gas shown in figure.

1) P1 > P2 2) P1 < P2 3) P1 = P2 4) P1  P2
10. Vessel has 6 g of oxygen at pressure P and temperature 400 K. A small hole is made in it so that
P
oxygen leaks out. How much oxygen leaks out if the final pressure is and temperature 300 K?
2
1) 5 g 2) 4 g 3) 2 g 4) 3 g

310
[Study Package - Physics - Class XI]

11. A flask contains argon and chlorine in the ratio 3 : 1 by mass. The temperature of the mixture is 300 K.
If atomic mass of argon = 39.9 u and molecular mass of chlorine = 70.9 u, then the ratio of average
kinetic energy per molecule of argon to chlorine gas is
1) 1 : 1 2) 3 : 1 3) 1 : 3 4) 39.9 : 70.9

o
12. In a gas at 1 atm pressure and 27oC a molecule has diameter of 5 A find mean free path of molecules

1) 3.8  108 m 2) 2  108 m 3) 8.3  103 m 4) 3.2  10 6 m

13. An ideal gas is found to obey an additional law VP2 = constant. The gas is initially at temperature T
and volume V. Then it expands to a volume 2 V, the temperature becomes

T 2T
1) 2) 2T 3) 4) 4T
2 2
14. The average energy and the rms speed of molecules in a sample of oxygen gas at 400 K are
1
7.211021 J and 524 ms respectively. The corresponding values at 800 K are nearly

1) 14.42  1021 J, 1048 ms 1 2) 10.18  1021 J, 741 ms 1

3) 7.21 10 21 J,1048 ms 1 4) 14.42  1021 J, 741 ms 1

15. The internal energy of one gram of helium at 100 K and one atmospheric pressure is
1) 100 J 2) 1200 J 3) 300 J 4) 500 J
16. The average energy per molecule of a triatomic gas at room temperature T is

1 3 5
1) 3 kT 2) kT 3) kT 4) kT
2 2 2
17. When x amount of heat is given to a gas at constant pressure, it performs x/3 amount of work. The
average number of degrees of freedom per molecule of the gas is
1) 3 2) 4 3) 5 4) 6
18. One mole of an ideal gas is taken through an adiabatic process where the temperature rises from
27oC to 37oC. If the ideal gas is composed of polyatomic molecule that has 4 vibrational modes,
which of the following is true? [R = 8.314 J mol–1 K–1]
1) Work done on the gas is close to 332 J 2) Work done by the gas is close to 332 J
3) Work done on the gas is close to 593 J 4) Work done by the gas is close to 593 J
19. The average energy and the rms speed of molecules in a sample of oxygen gas at 400 K are
1
7.211021 J and 524 ms respectively. The corresponding values at 800 K are nearly

1) 14.42  1021 J, 1048 ms 1 2) 10.18  1021 J, 741 ms 1

3) 7.21 10 21 J,1048 ms 1 4) 14.42  1021 J, 741 ms 1

311
Brilliant STUDY CENTRE

20. An insulated container containing monatomic gas of molar mas m moving with a velocity v0. If the
container is suddenly stopped. The change in temeprature is

mv 20 mv 20 R 3mv02
1) 2) 3) 4)
2R 3R mv 20 2R
21. A cylinder with fixed capacity of 67.2 litre contains helium gas at STP. The amount of heat needed to
raise the temperature of the gas by 20oC is [Given that R = 8.31 J mol–1 K–1]
1) 350 J 2) 700 J 3) 748 J 4) 374 J
22. An ideal gas undergoes a quasi static, reversible process in which its molar heat capacity C remains
constant. If during this process the relation of pressure P and volume V is given by PVn = constant,
then n is given by (Here CP and Cv are molar specifici heat at constant pressure and constant volume,
respectively)

C  Cp Cp  C CC v Cp
1) n  2) n  3) n  C  C 4) n 
C  Cv C  Cv p Cv
23. If CP and CV denoted the specific heats of unit mass of nitrogen at constant pressure and volume
respectively, then

R R R
1) C P  C V  2) C P  C V  3) CP  C V  4) CP  CV  R
28 7 14
24. The temperature of the mixture of one mole of helium and one mole of hydrogen is increased from
0oC to 100oC at constant pressure. The amount of heat delivered will be
1) 600 cal 2) 1200 cal 3) 1800 cal 4) 3600 cal
25. To raise the temperature of a certain mass of gas by50oC
at a constant pressure, 160 calories of heat
is required. When the same mass of gas is cooled by 100oC at constant volume, 240 calories of heat
is released. How many degrees of freedom does each molecule of this gas have (assume gas to be
ideal)?
1) 5 2) 6 3) 3 4) 7
26. The average degree of freedom per molecule for a gas is 6. The gas performs 25 J of work when it
expands at constant pressure. The heat absorbed by the gas is
1) 75 J 2) 100 J 3) 150 J 4) 125 J

312
[Study Package - Physics - Class XI]

KEY WITH HINTS


LEVEL - I

V   V 
1. 3 P1V1  P2 V2  P2  P1  1   760    1520 mm of Hg
 V2  V/2

2. 3 Here P = constant so V  T

V2 T2 2.7 T2
Therefore,     T2  288 K  15o C
V1 T1 3 320

16 28
3. 4 Number of moles of O2, n1   0.5 mole , for N 2 , n 2   1mole
32 28

44 5
For CO2, n 3   1mole , Total number of moles, n  n1  n 2  n 3  moles
44 2

P
 nRT    5  RT 
Now, PV = nRT;   
V  2  V 

n1RT  n 2 RT  n 3 RT RT  8 14 22  0.082  300


4. 3 As, p    n1  n 2  n 3        3.69 atm
V V  16 28 44  10

RT
5. 2 8 g of oxygen is equivalent to (1/4) mole;  PV  RT 
4

m 5  5 
6. 1 As PV = NRT; N    PV    RT
molecular mass 32  32 

T vO TO / M O TO M O M
7. 1 v rms  ;  ;   TN  T0  N ; TN  70o C
M vN TN / M N TN M N MO

T
8. 3 v rms  T ;  v rms  273 and 2v rms  T ;  2  or T  4  273
273

 T  1092 K  1092  273  819o C

25
3  300
3RT 3
9. 1 v rms   1930   M w  2  103 kg  2g
Mw Mw

313
Brilliant STUDY CENTRE

RT 273  47 T
10. 4 v rms  ;   v rms  O   v rms  H ; or   T  20 K
M 2 2
32 2
11. 1 At constant volume V1

as P1  P2 so T1  T2
12. 3 At low pressure and high temperature, the molecules are farther apart so that molecular size is
negligible as compared to the size of the vessel and also molecular forces do not come in.

3kT 3kT 8kT


13. 1 v rms  , v most probable  and vavg 
m m m

8kT 3kT v av 8/ 
14. 2 v av  , vrms  ;   ;
m m v rms 3

v av 8 8
or  ;  v av  1260   1161 ms 1
1260 3 3
15. 1 As H2 has five degrees of freedom while He 3, the mean energy of H2 will be greater than He.
However, mean translational K.E. for both will be same = (3/2) kT.

nRT 1000  8.3  7 58.1 58.1


16. 1 W   146000  ; or 1        1  1.4
1  1  146 146
Hence, the gas is diatomic

3 3
17. 3 KE  kT ; E1 = kT ; E2 = E1 × 2
2 2

3 3 3 E O2 3 / 2 kT 1
18. 2 E  N kT; E O2  kT; E He  2  kT;  
2 2 2 E He 2  3 / 2 kT 2
19. 2 Argon is monoatomic. Its atoms have no rotational motion. Therefore, entire energy is
translational.

 Q  310
20. 2 Cp      15.5 J mol1K 1
 nT  P constant 2 10 

At constant volume, heat required  nC v T  2 15.5  8.3  10   144J

314
[Study Package - Physics - Class XI]

1 2 
1 2 3 2  2 m0 v 
21. 1 We know kinetic energy of a molecule is given by m0 v  k B T  T   
2 2 3  kB 
 

2  3.60 1022 
 T    17.4 K
3  1.38 1023 

PV 1.20  105  4.00  103 


Now, PV  nRT  n  ; n  3.32 mol
RT  8.314 17.4 
2
22. 1 We know that PV  E
3

3PV 3 106
E  3  2  9 106 J
2 2

3 3 3 KT
23. 2 KE  kT ; KE  N kT; KE  2n
2 2 2 2
On doubling the number of molecules and keeping total KE same, average KE ( KE ) becomes
half, resulting in half the temperature.
24. 1 Mean time betwen two successive collisions

1 volume
 
velocity  number of particles per unit volume velocity

1 T T P T1
   ;  1  2 2
T P P P1 T2

6 108 500
 2   15 10 8  4 108 s
2 300

25. 3 In state P, C P  C V  R ; In state Q, C P  C V  1.10R

For the ideal gas, the relation CP  CV  R holds.

We know that, gas behaves as ideal gas at high temperatures so, TP  TQ

70
26. 1 From Q  mCp  T  ; 70  2  C p   35  30  ;  Cp   7 cals / mole / o C
10

C V  C P  R  7  2  5 cals / mole / oC ; Q  mCV  T   2  5  5  50 cals.

315
Brilliant STUDY CENTRE

5 7
27. 3 For a monoatomic He;  He  ; For a diatomic O2  O2 
3 5

7 5
3   O2  2   He 3  2 
  mixture   5 3  113  1.5
3  2 5 15  5
28. 1 Let the mixture contain n moles of gas B

Cp R
As C p  C v  R and   ;  Cv 
Cv  1

R 3 R 5
For gas A, C v   R ; For gas B, C v   R
5 7
1 2 1 2
3 5

R 13
For the mixture, C v   R
19
1 6
13
By conservation of energy, molar specific heat of the mixture is

3 5
n A  C v A  n B  C v  B 1 R  n  R
Cv  13
;  R 2 2   3  5n  R ;  n  2
n A  nB 6 1 n 2 1  n 

1 2 
1 2 3 2  2 m0 v 
29. 1 We know kinetic energy of a molecule is given by m0 v  k B T  T   
2 2 3  kB 
 

2  3.60 1022 
 T    17.4 K
3  1.38 1023 

PV 1.20  105  4.00  103 


Now, PV  nRT  n  ; n  3.32 mol
RT  8.314 17.4 
30. 2 Heat gained by solid ball = Heat lost by water

m1C1    1   m 2C2  2    ; 200  C1  60  20   200  1 80  60 

200  20 1 1
C1   ; As C2 for water is cal/g/oC, therefore, C1 
200  40 2 2
i.e., specific heat of solid is half the specific heat of water.

316
[Study Package - Physics - Class XI]

LEVEL - II

1 mN 2
1. 1 As P  vrms ---- (i)
3 V
where m is the mass of each molecule, N is the total number of molecules, V is the volume of
the gas.
When mass of all the molecules is halved and their speed is doubled, then the pressure will be

1 m  N 2 2 mN 2
V       2v rms   v rms  2P
3 2  V 3 V

 nRT   nRT   nRT 


2. 4 As, p1  p1  p 2  p3   V    V    V 
  O2   N2  CO2

 0.25  0.5  0.5 8.31  300  7.79 10 Nm



 n O2  n N2  n CO2  RT
V

4 10 3
5 2

3. 2 W  PV  103  0.25   250J

2
1 M w1 M w1 1
M   
4. 1 Here V and T are constants so P  n or P  2 3
M w ; Therefore, 1.5  Mw2 3
M w1 M w 2

7 Vrms He THe M H2 THe 2


5. 1     
5 Vrms H 2 M He TH2 4 273 ;

25 THe 25  273  2
  THe   273 K or 0o C
49 273  2 49

3RT 3RT 3  8.31 300


6. 4 As, c ms  ; or
M   3
1920   2 10 kg  2g
2 2
M c rms

Since, M = 2 for the hydrogen molecule. Hence, the gas is hydrogen.

3RT2 3RT1

3RT v  M M  100%
7. 4 v rms  ; % increase in rms 3RT1
M
M

T2  T1 400  300
  100%   100%  20  17.32  100%  15.5%
T1 300 17.32

317
Brilliant STUDY CENTRE

PM PM d1  0.987
8. 2 PV = RT or  RT , i.e., d  ;
d RT d 2

V nR
9. 1 PV  nRT    P1  P2
T P

m
10. 3 PV  RT (from m grams of gas) ------(i)
M

m
PV  RT ------(ii) ; Dividing equation (ii) by (i) we get
M

P  m T  P T  P / 2  400
 ; m    m     6  4g
P m T P T  P  300

 Mass of oxygen leaked, m  m  m  6  4  2g

3
11. 1 Average KE  kT ; So, ratio will be 1 : 1
2

1 o
12. 1  ; Here d  5 A
2d 2 n

P  N A 105  6.023 10 23


n  ; n  2.415  10 25
RT 8.314  300

1
 ; 8
2  3.14  25  1020  2.415  1025   3.8  10 m

13. 1 As P 2 V = a constant, therefore, when V becomes 2V, p2 becomes half.

1 1 T
2
i.e., p  or p  times . As p = a constant, so T  p ; Thus, T becomes
2 2 T 2

800 E 800
14. 4 Average energy E  T ;    2 ; or E  2  E  2  7.21 10 21
 14.42  1021 J
E 400 400 800 400

800
RMS velocity, c  T ;  C800  C 400  524  2  741 ms1
400

318
[Study Package - Physics - Class XI]

3
15. 3 The helium molecule is monoatomic and hence its internal energy per molecule is k BT
2
3
(wher kB is the Botlzmann constant). The internal energy per mole is therefore is RT . One
2
1 3
gram of helium is one fourth mole and hence its internal energy is  R  100  300J , taking
4 2
the value of R to be approximately 8 J mol–1 K–1.
16. 1 A triatomic (non-linear) gas molecule has 6 degrees of freedom (3 translational, 3 rotational and
non vibrational) at room temperature.
According to the law of equipartition of energy, the average energy per molecule of a triatomic
gas at room temperature T is
 1 1
E  f kT   6  kT  3kT
2 2

W P V nRT x / 3 f 
17. 2    ;  C p  3R    1 R  f  4
Q nCp T nCp T x 2 
18. 3 Number of moles, n = 1; T1 = 300 K; T2 = 310 K; R = 8.314 J/mol K
Each vibratinoal mode corresponds to 2 degrees of freedom.
so, f = 3(translational) + 3(rotational) + 8(vibrational) = 14

2 2 8
  1  1 
f 14 7

nRT 1 8.314  10


Work done WorkW   
 1 8
1
7
W   593J ; Work is negative, so work is done on the gas.

800E 800
19. 4 Average energy E  T ;    2 ; or E  2  E  2  7.21 10 21
 14.42  1021 J
E 400 400 800 400

800
RMS velocity, c  T ;  C800  C 400  524  2  741 ms1
400

1
20. 2 If the container is suddenly stopped loss in kinetic energy of gas   mn  v02 , where n is number
2
of moles of gas.
Let T is the fall in temperature of gas.

3  1 2 mv 20
n
Then,  R T   mnv 0 , T 
2  2 3R

319
Brilliant STUDY CENTRE

67.2 litre
21. 3 Number of moles of gas, n   3mol
22.4 litre

3 3
Q  nCV T  3  R  T  3   8.31 20  747.9  748 J
2 2
R Cp  Cv Cp  Cv
22. 1 Since C  Cv  ; C  Cv   1 n 
1 n 1 n C  Cv
Cp  Cv C  Cp
n  1 
C  Cv C  Cv
23. 1 Molar heat capacity = Molar mass × specific heat capacity.
Molar heat capacities of nitrogen at constant pressure and constant volume will be 28Cp and
28 Cv respectively.
R
 28Cp  28Cv  R or Cp  Cv 
28
1C p1   2 C p 2  5 7 
24. 2 C 
p mix 
1   2
;  C p1  He  
2
R and C p 2  H 2   R 
2 

5 7
1 R  1 R
C 
p mix  2 2  3R  3  2  6cal / mol. oC
1 1

 Amount of heat needed to raise the temperature from 0o C to 100o C

 Q p  C p T  2  6  100  1200 cal


25. 2 H  160 cal, T  50o C

H  240cal, T  100o C
At constant pressure, Q  nC P T

160  nCP  50

And at constant volume, 240  nC v  100

240 2C V C 4 4 2 2 4 1
  P   ;  1   1   f  6
160 CP CV 3 3 f f 3 3

2 2 4 W  Q  U  1  U  1  nCV T  1  1  1  3  1  C P   
26. 2   1  1  ;  
f 6 3 Q Q Q nC P T  4 4  CV 
 Q  4W  100 J

320
[Study Package - Physics - Class XI]

CHAPTER - 13
OSCILLATIONS

SIMPLE HARMONIC MOTION

Periodic Motion
When a body or moving particle repeats its motion along a definite path after a regular interval of time,
the motion is said to be periodic motion. The constant interval of time after which the motion is repeated is
called time period (T). The particle may travel along a linear or curved path.
eg: (1) Motion of pendulum of a clock. (2) Motion of planets around the sun
Oscillatory Motion
When a body moves back and forth (to and fro) about a fixed point after a regular interval of time, the
motion is said to be oscillatory. The fixed point about which the body oscillates is called equilibrium position
or mean position.
eg. (1) The movement of a simple pendulum. (2) The vertical movement of a mass suspended on a spring
balance.
The force or torque directed towards mean position in oscillatory motion is called restoring force or
torque.
Every oscillatory motion is periodic. But all periodic motions are not oscillatory.
Simple Harmonic Motion (SHM)
An oscillatory motion is said to be simple harmonic if the restoring force acting on the body in oscillatory
motion is directly proportional to the displacement of the body, and is directed towards the equilibrium
position.
SHM can be divided into two.
(1) Linear SHM - When a particle moves to and fro about the mean position, along a straight line then
the motion is called linear SHM. eg. Motion of a mass connected to a spring.
O
A B
A and B are extreme positions and O is the mean position.
(2) Angular SHM - When a system oscillates angularly with respect to a fixed axis, then its motion is
called angular SHM. eg: Motion of the bob of a simple pendulum.
Characteristics of SHM
Displacement of a particle in SHM is the distance of the particle from the mean position at that instant,
Displacement y = A sin  t   
Amplitude: The maximum value of displacement of the particle from equilibrium position is called
amplitude.
321
Brilliant STUDY CENTRE

y   A when sin  t      1 ---------(1)


Period of SHM (T)
The smallest time interval after which the oscillatory motion gets repeated is called Time period.
2
T ---------(2)

Frequency (v): The number of vibrations completed in unit time interval is called frequency of oscillation,
1 
  , unit-Hz. (Hertz)
T 2
2
Angular frequency ():    2 , unit - rad/sec.
T
Phase: The state of a vibrating particle is described by the phase of the particle at an instant
y  A sin  t    ,  t    is the phase. At t = 0,  is called epoch or initial phase.
SHM as a projection of circular motion
y
aQ
P

D
R P aC
aR
A
y
 
C x Q x
O B

E
Suppose a particle is moving along the circumference of a circle of radius A with angular speed  . The
points Q and R are foot of the perpendiculars drawn from P to X axis and Y axis.
As the particle P completes one revolution in the anticlockwise direction, the projection R and Q moves
in the X axis and Y axis respectively completing one oscillation. In circular motion the oscillation of the
projection in any axis represents the simple harmonic motion.
Let at t = 0, particle P is on the X axis. After a time t, the point P makes an angle  with the X-axis, then

x  A cos t 
 ---------(3)
y  A sin t 
Here, x & y are the displacements (OQ and OR from the centre) at time t; They represent displacement
equations of SHM.
So when P is under uniform circular motion, Q and R perform SHM about O, with the same angular
speed  as that of P..

The acceleration of P  a c  2 A towards centre.

a c can be resolved into two components; a R  A2 sin t  2 y and a Q  A2 cos t  2 x
aR and aQ are the accelerations corresponding to R and Q respectively.
Velocity (v)
Time rate of change of displacement of a particle at an instant is defined as the velocity in SHM.

322
[Study Package - Physics - Class XI]

dy d
v   A sin t   A cos t
dt dt

y2
v   A 1  sin t   A 1 
2
2
   A 2  y2 ;  y  A sin t 
A

v 2  2  A 2  y 2 

At mean position y = 0, v   A ; At y   A , then the velocity of particle executing SHM is zero.


The direction of velocity is towards or away from mean position.
Acceleration in SHM
Time rate of change of velocity of a particle at an instant is called acceleration.
dv d
a   A cos t   2 A sin t  2 y
dt dt
The acceleration of the particle executing SHM is always directed towards the mean position. Acceleration
2
 
is maximum at extreme position  A and minimum at the mean position (0).

Force law for simple harmonic motion

Using Newton’s second law of motion, and the expression for acceleration  a  2 y  of a particle
undergoing SHM, the force acting on a particle of mass m in SHM is

F = ma   m2 y

i.e., F = – ky ------ (4)

where k  m2 ------- (5)

k
or   ------ (6)
m
Like acceleration, force is always directed towards the mean position - hence it is sometimes called
the restoring force in SHM.
Note that the force in Eq. is linearly proportional to x. A particle oscillating under such a force is,
therefore, calling a linear harmonic oscillator. In the real world, the force may contain small additional terms
proportional to y2, y3, etc. These then are called non-linear oscillators.
Energy of particle in SHM
Potential Energy

dU
F   dU    Fdy ; F = –ky for SHM   dU      ky dy
dx

1 2 1 2
U ky  C ; At y = 0, U = U0, therefore C = U0.  U ky  U 0
2 2

1 2
U0 is the PE at equilibrium position. When U0 = 0, U  kx
2
323
Brilliant STUDY CENTRE

Also y  A sin  t   

1 2 k 2 1
U ky   A sin  t      kA 2 sin 2  t   
2 2 2

1 1
If   0 , U  kA 2 sin 2 t  m2 A 2 sin 2 t
2 2

PE is minimum at mean position and maximum at extreme position   A 

U Umax = ½ kA
2

–A A

T 3T T 3T 5T
PE is a periodic function of time. It is minimum at t  0, , T, etc. and maximum at , and .
2 2 4 4 4
Kinetic Energy
For a particle in SHM m & v are mass and velocity, then KE at any instant is given by

1 1 1
mv 2  m2  A 2  y 2   k  A 2  y 2  ------------(7)
2 2 2

Also v  A cos  t   

KE max. KE = ½ m2A2

–A A

1
KE  m2 A 2 cos 2  t   
2

1
If initial phase  is zero.; KE  m2 A 2 cos 2 t ------------(8)
2
2 1 
KE is maximum at mean position  kA  and minimum (0) at extreme position.
2 
T 3T 5T
KE is a periodic function of time. It is minimum at t  , , .........
4 4 4
T 3T
It is maximum at 0, , T, ........
2 2

324
[Study Package - Physics - Class XI]

Total Energy

1 2 1 1
E = PE + KE  ky  k  A 2  y 2   kA 2  a constant ------------(9)
2 2 2
1 2 2 1 1
Also E  kA sin t  kA 2 cos 2 t  kA 2
2 2 2
Some Sytems executing Simple Harmonic Motion
Spring System
When a spring is given a small displacement by stretching or compressing, a restoring force is developed
in it. According to Hook’s law. F   x or F = – kx [k is spring constant]
Spring constant depends on length (l), radius of the wire used and the material used.
For any spring, kl = a constant where l is the natural length of spring.
When a spring is stretched or compressed, work done on it is stored as elastic potential energy.
When the force F is applied on a spring produces a compression or stretching by a distance x,
1 2
then W  Fdx  kx dx ; U  W 
  kx
2
U

Parabola

O x

1
When the spring is stretched from a length l1 to l2 then work done  W  k  l22  l12  --------(10)
2
Here the spring is considered massless.
Spring Mass System
When a small mass is suspended from a massless spring, then this arrangement is known as spring
mass system. For small displacement the motion of the spring is SHM.

d2 x
F  kx  m  kx
dt 2
d2x k k
2
  x  2 x , 2 
dt m m

2 m 1 k
Time period T   2 , Frequency  
 k 2 m

325
Brilliant STUDY CENTRE

 1 
 
It is seen that T  m and  T 

.
k


If two masses m1 & m2 are connected by a spring and made to oscillate, then time period T  2
k

m1m 2
where,  
m1  m 2 is called the reduced mass.

Combination of springs

Series combination

In series combination of springs same restoring force is acting on all the springs but extension depends
on the force constant.

Total displacement in springs x = x1 + x2.

F F F F
Force acting on both springs F   k1 x1   k 2 x 2 ; x1   , x2   ; x    
k1 k2  k1 k 2 

Let ks is the equivalent force constant, then

F F F 1 1 1 k1 k 2
F  –k s x    , or   ; ks 
k s k1 k 2 k s k1 k 2 k1  k 2 ---------(11)

m m  k1  k 2 
Time period T  2  2 ---------(12)
ks k1 k 2

1 k1 k 2
Frequency   2 m  k1  k 2  ---------(13)

ks k1 k 2
Angular frequency    ---------(14)
m m  k1  k 2 

326
[Study Package - Physics - Class XI]

Parallel combination of springs

When springs are connected in parallel, the deformation on each spring is the same but restoring force
will be different. kp is the effective spring constant when they are connected in parallel.
F  F1  F2 , F1 = –k1x and F2 = –k2x F   k 1x  k 2 x
F   k p x   k 1x  k 2 x  k p  k1  k 2 ---------(15)

m m
Period T  2 k  2 k  k ---------(16)
p 1 2

1 k1  k 2 k1  k 2
Frequency n  ---------(17);Angular frequency   ---------(18)
2 m m
Note:
Period and frequency of a spring mass system is independent of the acceleration due to gravity whenever
they are making horizontal or vertical oscillations.
Simple Pendulum
An arrangement in which a heavy point mass, suspended by a weightless inextensible string from a
rigid support, is called a simple pendulum.

When a mass m is suspended from a rigid support S by a massless string and the mass is displaced
OA y
through an angle  , then restoring force acting on the pendulum, F   mg sin  . sin   
SA l
y g  y
 ma   mg sin    mg ; a
l l
When y is small, acceleration is proportional to displacement and it is directed towards mean position.
So the motion is SHM.

g 2 l
a  2 y, 2  ; T  2 ---------(19)
l  g

327
Brilliant STUDY CENTRE

Second’s Pendulum
A pendulum whose time period is 2s is called a second’s pendulum. For second’s pendulum
l
T 2  4  4 2 .
g

g
l 1 m .
2
ie, the length of second’s pendulum on the surface of earth is = 1 m
Free or undamped vibrations
In the absence of any dissipative force if a body executes SHM, the period of motion is independant of
the amplitude of the vibrating body then the body vibrates indefinitely with constant amplitude. Such oscillations
are called free or undamped oscillations. It will be having a single natural frequency.
Vibrations of a tuning fork, oscillation of a pendulum are examples. Such vibrations can be represented
d2y
by m  ky , where y  a sin  t   
dt 2
In addition to the linear restoring force if some dissipative forces are acting on the body executing SHM,
the motion of the body gets modified. Such vibrations can be classified into damped vibrations, maintained
vibration, forced vibrations or resonant vibrations.
1. Damped vibrations
When dissipative forces like friction or viscosity acts on a body executing SHM, the forces offer resistance
to the motion and the mechanical energy gradually decreases. Due to this the amplitude decreases gradually
and finally comes to rest. This type of motion is called damped motion.

The motion of a pendulum in a liquid, the motion of the coil of a ballistic galvanometer are examples. In
case of damped vibration, if we assume that the dissipative force is non-periodic and varies linearly with
velocity, the motion can be represented by

d2y dy dy
m 2
 ky    - is the damping constant. v 
dt dt dt


Take b  , the solution can be written as
2m
2
 bt 2 2   
2
y  Ae sin  1t    where 1    b    
0 0  ---------(20)
 2m 

328
[Study Package - Physics - Class XI]

1 2
Ae–bt is the amplitude. For undamped oscillator the mechanical energy is kA ; for damped oscillator,,
2
the amplitude decreases.

1 2 m t
So mechanical energy is Et  kA e ---------(21)
2
Under Damping

 k 
If  ie,  2 . It is heavily damped.
2m m m
Here the oscillation dies out in a short time. Such a motion is called dead beat.

Critical damping
Over damping

Under damping

Critical damping

 k 
When  , or  20 .
2m m m

Here damping rate is much faster as compared to heavily damped. It is used in situations where we
desire a high decay without oscillation. Eg. In pointer type galvanometer, the pointer move immediately to
the correct position without any oscillation immediately.
Over damping

 k 
When  , or  20 slow damping takes place.
2m m m

Over damped cannot oscillate and instead slowly decays towards equilibrium.
The oscillation dies out after a long time.
Quality factor or Q value in SHM
The rate at which the energy decays in the damped oscillatory systems is measured by the Q value of

329
Brilliant STUDY CENTRE

1 k 2 
the system. It is given by, Q  ; where 1   2 and
b
2b m 4m 2m

m k 2 m k
Q  2
 ---------(22)
 m 4m  m
Quality factor is defined as the ratio of energy stored in the system in one cycle to the average energy
lost in one cycle.
If energy is supplied to overcome the energy lost due to dissipative forces, the oscillations are maintained
and they vibrate with natural frequency.
Forced vibrations
When a system is subjected to an external periodic force and if the system vibrates according to the
frequency of the force applied it is called forced vibrations.
eg. If we press the stem of a vibrating tuning fork against a table, the table vibrates with the frequency of the
tuning fork.
If a periodic force F  F0 sin t is applied to a body of natural frequency 0 in presence of small

d2y dy
damping, the differential equation of motion can be written as m 2
 ky    F0 sin t
dt dt
On solving y  A sin  t   

F0
A
 2 1/ 2 ---------(23)
2    
m  20  2     
  m  

  
  tan 1   ---------(24)
 m  0    
2 2

Resonant Vibrations

When the frequency of the external periodic force is equal to the natural frequency of the body,    0 
the amplitude of vibration increases at each step and becomes very large. Such vibrations are called resonant
vibrations and the phenomenon is called resonance.

When there is no damping the peak is higher and narrower, called sharp resonance.

330
[Study Package - Physics - Class XI]

QUESTIONS
LEVEL - I
1. A particle executes SHM of amplitude A and time period T. the distance travelled by the particle in the
 5
duration its phase changes from to .
12 12

1 3 2 2
1) A 2) A 3) A 4) A
2 2 3 3
2. A mass m is performing linear simple harmonic motion, then the correct graph for the acceleration a
and the corresponding linear velocity v is

v2 v2 v2 v2
1) 2) 3) 4)
2 2 2 2
a a a a

3. For a particle executing S.H.M., the kinetic energy K is given by K  K 0 cos 2 t . The maximum value
of potential energy is :
1) K0 2) Zero 3) K0/2 4) Not obtainable
4. An object is attached to the bottom of a light vertical spring and set vibrating. The maximum speed of
the object is 15 cm/sec and the period is 628 milli seconds, the amplitude in centimeters is :
1) 3.0 2) 2.0 3) 1.5 4) 1.0
5. A force of 6.4 N stretches a vertical spring by 0.1 m. The mass that must be suspended from the
spring so that it oscillates with a time period of  / 4 second is

 4
1) kg 2) kg 3) 1 kg 4) 10 kg
4 
6. The equation of a simple harmonic wave is given by


y  3sin (50 t  x),
2
where x and y are in metres and t is in seconds. The ratio of maximum particle velocity to the wave
velocity is

3 2
1) 2  2)  3) 3  4) 
2 3
7. Average velocity of a particle executing SHM in one complete vibration is :

A A2
1) Zero 2) 3) A 4)
2 2

331
Brilliant STUDY CENTRE

8. A particle of mass 1 kg is undergoing S.H.M. for which the graph between force and displacement
(from mean position) is as shown. It’s time period, in seconds, is

 2  
1) 2) 3) 4)
3 3 6 4
9. A large horizontal surface moves up and down in SHM with an amplitude of 1 cm. If a mass of 10 kg
(which is placed on the surface) is to remain continually in contact with it, the maximum frequency of
S.H.M. will be:
1) 0.5 Hz 2) 1/5 Hz 3) 5 Hz 4) 10 Hz
10. For a body of mass m attached to the spring, the spring factor is given by (  , the angular frequency)
m
1) 2) m2 3) m 2  4) m 2 2
2
11. A loaded vertical spring executes SHM with a time period of 4s. The difference in time between the
maximum KE and max. PE of this system varies with period of
1) 1 s 2) 2 s 3) 4 s 4) 8 s
12. A weightless spring which as a force constant k oscillates with frequency n when a mass m is
suspended from it. The spring is cut into two equal halves and a mass 2 m is suspended from one
part of the spring. The frequency of oscillation will now become.
n 1/ 2
1) n 2) 2n 3) 4) n  2 
2

13. The displacement of two particles executing SHM are represented by equations y1  2sin 10t    ,

y 2  3cos10t . The phase difference between the velocity of these particles is:
 
1)  2)  3)   3)  
2 2
14. Two particles are executing S.H.M. of same amplitude and frequency along the same straight line
path. They pass each other when going in opposite directions, each time their displacement is half of
their amplitude. What is the phase difference between them?
5 2  
1) 2) 3) 4)
6 3 3 6
15. Masses m and 3m are attached to the two ends of a spring of constant k, if the system vibrates freely,
the period of oscillation will be

m m 3m 3m
1)  2) 2 3)  4) 2
k k k k

332
[Study Package - Physics - Class XI]

16. Two particles are executing SHM of same amplitude and frequency along the same straight line path.

3
They pass each other when going in opposite directions, each time their displacement is times
2
of their amplitude. What is the phase difference between them?

  2 5
1) 2) 3) 4)
6 3 3 6

17. The potential energy of a simple harmonic oscillator of mass 2 kg in its mean position is 5 J. If its total
energy is 9 J and its amplitude is 0.01 m, its time period would be

   
1) sec 2) sec 3) sec 4) sec
10 20 50 100

18. The equation of motion of a particle executing simple harmonic motion is a  16 2 x  0 . In this
equation, a is the linear acceleration in m/s2 of the particle at a displacement x in meter. The time
period in simple harmonic motion is

3 1
1) sec 2) sec 3) 1 sec 4) 2 sec
4 2

19. For a particle executing simple harmonic motion, determine the ratio of average acceleration of particle
from extreme position to equilibrium position to the maximum acceleration.

4 2 1 1
1) 2) 3) 4)
   2

20. The maximum acceleration of a particle in SHM is made two times keeping the maximum speed to
be constant. It is possible when

1) amplitude of oscillation is doubled while frequency remains constant

2) amplitude is doubled while frequency is halved

3) frequency is doubled while amplitude is halved

4) frequency is doubled while amplitude remains constant

21. A particle executes SHM on a straight line path. The amplitude of oscillation is 2 cm. When the
displacement of the particle from the mean position is 1 cm, the numerical value of magnitude of
acceleration is equal to the numerical value of magnitude of velocity. Then find out the frequency of
SHM

3 3 3 3
1) 2) 3) 4)
2 2 2 

333
Brilliant STUDY CENTRE

22. What will be time period of the displaced body of mass m?

m 3m 3m 3m
1) 2 2) 2 3) 2 4) 
2k k 2k k
23. In figure S1 and S2 are identical springs. The oscillation frequency of the mass m is f. If one spring is
removed, the frequency become

f
1) f 2) 2f 3) f 2 4)
2
24. A body at the end of a spring executes S.H.M. with a period t1, while the corresponding period for
another spring is t2. If the period of oscillation with the two springs in series is T, then
1 1 1 1 1 1
1) T  t1  t 2 2) T 2  t12  t 22 3)   4) 2
 2 2
T t1 t 2 T t1 t 2
25. A particle starts oscillating simple harmonically from its equilibrium position then, the ratio of kinetic
energy and potential energy of the particle at the time T/12 is: (T = Time period)
1) 2 : 1 2) 3 : 1
3) 4 : 1 4) 1 : 4

26. The velocity of a body executing S.H.M. at displacements a and b are b and a respectively. The
amplitude of S.H.M. wil be

1) a 2  b2 2) a 2  b2 3) (a + b) 4) (a – b)

27. A body executes simple harmonic motion. The potential energy (PE), kinetic energy (KE) and total
energy (TE) are measured as a function of displacement x. Which of the following statement is true?

1) TE is zero when x = 0 2) PE is maximum when x = 0

3) KE is maximum when x=0 4) KE is maximum when x is maximum

25
28. An object of mass 0.2 kg executes simple harmonic along X-axis with frequency of Hz. At the

position x = 0.04 m, the object has kinetic energy of 0.5 J and potential energy of 0.4 J
amplitude of oscillation in meter is equal to :

1) 0.05 2) 0.06 3) 0.01 4) None of these

334
[Study Package - Physics - Class XI]

29. A body is executing SHM under the action of a force whose maximum magnitude is 50 N. The magnitude
of force acting on the particle at the time when its energy is half kinetic and half-potential is (Assume
potential energy to be zero at mean position).

1) 12.5 2 N 2) 12.5 N

3) 25 N 4) 25 2 N
30. A mass of 0.98 kg attached to a spring of constant K = 100 Nm–1 is hit by a bullet of 20 gm moving with
a velocity 30 ms–1 horizontally. The bullet gets embedded and the system oscillates with the mass on
horizontal friction less surface. The amplitude of oscillations will be :
1) 0.6 cm 2) 6 cm 3) 1.2 cm 4) 12 cm
LEVEL - II
1. The radius of circle, the period of revolution, initial position and sense of revolution are indicated in the
figure.

y-projection of the radius vector of rotating particle P is :

 t 
1) y(t)  3cos   , where y in m 2) y(t)   3cos 2 t , where y in m
2

 t   3t 
3) y(t)  4 sin   , where y in m 4) y(t)  3cos   , where y in m
2  2 
2. The length of a simple pendulum is increased by 45%. What is the percentage increase in its time
period?

1) 44% 2) 44% 3) 10% 4) 20%


3. A coin is placed on a horizontal platform, which undergoes horizontal SHM about a mean position O.
The coin placed on the platform does not slip, when angular frequency of the SHM is  . The coefficient
of friction between the coin and the platform is  . The amplitude of oscillation is gradually increased.
The coir will begin to slip on the platform for the first time
1) at the mean position
2) at any position of oscillations
g
3) for an amplitude of
2
g
4) for an amplitude of
2

335
Brilliant STUDY CENTRE

4. If a simple pendulum of length  has maximum angular displacement  , then the maximum kinetic
energy of bob of mass m is:

1  1 mg
1) m  2) 
2 g 2 
1
3) mg  1  cos   4)  mg sin 
2
a a
5. A particle executes SHM of type x  a sin t . It takes time t1, from x = 0 to x  and t2 from x  to
2 2
x = a. The ratio t1 : t2 will be
1) 1 : 1 2) 1 : 2
3) 1 : 3 4) 2 : 1

6.
2
 
A particle of mass 10 gm is placed in a potential field given by V  50x  100 J / kg . The frequency
of oscillation in cycle/sec is:
10 5 100 50
1) 2) 3) 4)
   
7. Three masses 700 g, 500 g, and 400 g are suspended at the end of a spring as shown and are in
equilibrium. When the 700 g mass is removed, the system oscillates with a period of 3 seconds,
when the 500 gm mass is also removed, it will oscillate with a period of

12
1) 1 s 2) 2 s 3) 3 s 4) s
5
8. A mass M is suspended from a spring of negligible mass. The spring is pulled a little and then released,
so that the mass executes SHM of time period T. If the mass is increased by m, the time period
becomes 5T/3. Then ratio of m/M is
9 25 16 4
1) 2) 3) 4)
16 9 9 3
9. Find the ratio of time periods of two identical springs if they are first joined in series and then in parallel
and a mass m is suspended from them
1) 4 2) 2 3) 1 4) 3
10. A horizontal platform with an object placed on it is executing S.H.M. in the vertical direction. The
amplitude of oscillation is 4.0 × 10 –3 m. What must be the least period of these oscillations, so that
the object is not detached from the platform? (Take g = 10 m/s2)
   
1) sec 2) sec 3) sec 4) sec
25 18 14 20
336
[Study Package - Physics - Class XI]

11. The frequency of oscillation of the springs shown in the figure will be

k1

k2

1 k1  k 2 1  k1  k 2  m k1  k 2 1 k1 k 2
1) 2) 3) 2 4) 2 m  k1  k 2 
2 m 2 k1 k 2 m
12. A mass at the end of a spring executes harmonic motion about an equilibrium position with an amplitude
A. Its speed as it passes through the equilibrium position is V. If extended to 2A and released, the
speed of the mass passing throug the equilibrium positions will be
v v
1) 2v 2) 4v 3) 4)
2 4

 2 
13. A 4 kg particle is moving along the x-axis under the action of the force F    xN. At t = 2 sec, the
 16 
particle passes through the origin and at t = 10 sec its speed is 4 2 m / s. The amplitude of the motion
is :

32 2 16 4 16 2
1) m 2) m 3) m 4) m
   
14. A particle is executing linear SHM. The average kinetic energy and average potential energy, over a
period of oscillation, respectively are K av and Uav. Then,

U av K av K av
1) K av  2) U av  3) K av  U av 4) U av 
2 2 3
15. A body of mass 0.01 kg executes simple harmonic motion (S.H.M.) about x = 0 under the influence of
a force shown below. The period of the S.H.M. is :

1) 1.05 s 2) 0.52 s 3) 0.25 s 4) 0.30 s


16. A particle is performing simple harmonic motion along x-axis with amplitude 4 cm and time period 1.2
sec. The minimum time taken by the particle to move from x = 2 cm to x = +4 cm and back again is
given by
1) 0.6 sec 2) 0.4 sec 3) 0.3 sec 4) 0.2 sec

337
Brilliant STUDY CENTRE

17. Two simple pendulum, first of bob mass M1 and length L1, second of bob mass M2 and length L2.
M1 = M2 and L1 = 2 L2. If the vibrational energies of both are the same. Then which is correct?
1) Amplitude of B is greater than A 2) Amplitude of B is smaller than A
3) Amplitude will be same 4) Amplitude of B is double that of A

18. The displacement function of a S.H.M. is given by y  A cos  t     . If at t = 0, the displacement is

y  1cm and velocity is  cm-s 1 . The value of amplitude (A in cm) is

1) 2 2) 1 3) 2 3) 1/ 2

19. Two simple harmonic motions are represented by y1  10 sin 2t  3 cos 2t  and

 
y 2  5sin  2t   . The ratio of amplitudes is:
 4

1) 1 : 1 2) 4 : 1 3) 1 : 3 4) 3 :1
20. The amplitude of a damped oscillator becomes half in one minute. The amplitude after 3 minute will be
1
times the original, where X is :
X
1) 2 × 3 2) 23 3) 32 4) 3 × 22
21. A simple pendulum with a solid metal bob has a period T. The metal bob is now immersed in a liquid
of density one-tenth that of the bob. The liquid is non-viscous. Now the period of the same pendulum
with its bob remaining all the time in the liquid will be

9  10   9 
1) T 2) 3)  T 4)  T
10T  9   10 
22. A simple pendulum of length  is vibrating with an amplitude A. Its energy is E. Consider the following
situations:
(i) Amplitude is doubled keeping length the same.
(ii) Length is doubled, keeping amplitude the same.
The energy of the pendulum in the two situations will be:

E E
1) 4E, 2) 2E, 3) E, E 4) 2E, 2E
2 2
23. Two pendulum of lengths 1.69 m and 1.44 m start swinging together. After how many vibrational will
they again start swinging together?
1) 8 vibrations 2) 10 vibrations
3) 12 vibrations 4) 14 vibrations
24. If the length of a second’s pendulum is increased by 2%, how many seconds will it lose per day?
1) 3927 s 2) 3727 s
3) 3427 s 4) 864 s
338
[Study Package - Physics - Class XI]

25. A block of mass M is performing SHM of amplitude A between two springs. If a mass m is placed on M
at its equilibrium position, then new amplitude will be:

mM  M 
1)  A 2)  A
 M   mM 

 m   mM 
3)  A 4)  A
mM  M 

26. When two displacements represented by y1  a sin  t  and y 2  b cos  t  are superimposed, the
motion is

a
1) simple harmonic with amplitude
b

2) simple harmonic with amplitude a 2  b2

a  b 
3) simple harmonic with amplitude
2
4) not a simple harmonic
27. Potential energy of a particle of mass 0.1 kg moving along x-axis is given as U = 5x (x – 4) J, here x is
in meters, then which of the following is correct
(i) particle is acted upon by a constant force
(ii) speed of particle is maximum at x = 2 m
(iii) particle executes SHM
(iv) period is  / 5 sec
1) (i), (ii), (iii) 2) (ii), (iii), (iv)
3) (i), (ii), (iv) 4) (i), (ii), (iii) and (iv)
28. Values of the acceleration x of a particle moving in simple harmonic motion as a function of itss
displacement x are given in the table below. The period of the motion is


x(mm / s 2 ) 16 8 0 8 16
x  mm  4 2 0 2 4

1 2
1) s 2) s
 

3) s 4) s
2

339
Brilliant STUDY CENTRE

KEY WITH HINTS


LEVEL - I
5  A
1. 1 Distance travelled by the particle = A sin  A sin 
12 12 2

2. 2 v 2  2  A 2  x 2   2 x 2  2 A 2

a 2  2 x 2
 v 2   a 2  2 A 2 straight line with a negative slope

3. 1 In S.H.M., if rest energy is zero max P.E. = max K.E. = K 0 (cos 2 t) max  K 0 1  K 0

 2   max T 15  628  10 3
4. 3 
Maximum velocity,  max  a or max  a   ; a    1.5cm
 T  2 2  3.14

6.4 m  m
5. 3 Spring constant K   64 N / m. Now T  2 or  2  m 1kg
0.1 k 4 64

6. 2 We know that  max  a and   n

 max a a(2n) 2a 2a  3


      ka   3 
 n n  2 / k 2 2
7. 1 A displacement in one complete vibration is zero, therefore average velocity is zero
8. 2 Slope of F-x curve gives K

13.5
Slope 
1.5
F   Kx  K  9

K
2  9
m
2
  3, T 
3
9. 3 Since, amax = g

 2 A  g  (2f )2 A  g

10
 f2  2
 25  f  5 Hz
4  0.01

m 4 2
10. 2 T  2 or k  2 m  2 m
k T

340
[Study Package - Physics - Class XI]

11. 1 The KE is a maximum at the mean position and PE is maximum at the extreme position. The
T 4
time difference between maximum KE and maximum PE is   1 s
4 4

1 k 1 k 1 2k
12. 1 n
2 m
; n  
2 2m 2 2m
 k  2k  ;  n  n

dy1  
13. 4 v1   2  10 cos 10t    ; v 2  3  10sin10t  30 cos 10t  
dt  2

  
 Phase difference  10t     10t  2    
  2

r 1  5
14. 2 y  r sin  t    ; when y  , then sin  t      sin or sin
2 2 6 6

 5   2 
Hence, phase difference    
 6  6 3

3m
m1m 2 m  3m 3m 
Reduced mass of the system,     4   3m
15. 3
m1  m 2 m  3m 4 ; T  2 k  2  k k

3a 3  2
16. 2 y  a sin  t    ; , then  a sin  t    ; or sin  t      sin or sin
2 2 3 3
 2 2  
So the phase of the two particles is and radians, phase difference   
3 3 3 3 3
17. 4 Total Mechanical Energy = Total Energy at Mechanical Position + Total oscillation energy
9 = 5 + Total oscillation energy
1 2
Total oscillation energy = Ka  4
2
M 
 K  8 104  T  2 T sec
K 100
2 2 1
18. 2 a  16 2 x ; for SHM : A  2 x ;   4 ; T    4  2 sec
19. 2 Let the equation of simple harmonic motion be, x  Asin t
T /4
2
  A sin t dt
0 2
Then, the required ratio is 
T 
 2 A
4

341
Brilliant STUDY CENTRE

Ai
20. 3 2A i i2  A f f2 ; Ai i  A f f ; If f  2i and A f  . The condition will be satisfied.
2
21. 1 The speed of particle at x = 1, v   A 2  x 2   22  12  3  ------(1)

 3
at x = 1 , a  2 x  2 -------(2) ; from (1) and (2)    3 ; or f  
2 2
k1k 2 k  2k 2k
22. 3 Equivalent force constant of three springs, k eq   
k1  k 2 3k 3

m m 3m
T  2  2  T  2
k eq 2k / 3 ; 2k
1 2K 1 k
23. 4 Ef fective f orce constant K eff  2K ; f ; for one spring f   ;
2 a 2 m
f 1 f
 ; f 
f 2 2

m m m K1K 2
T1  2 , T2  2 , T  2 K eq  2 2 2
24. 2 K1 K2 K eq ;
K1  K 2 ;  T  t 1  t 2

T   A
25. 2   , x  A sin 
12 6 6 2
1 2 1 KA 2 A 3
P.E.  Kx  ; v  A cos  
2 2 4 2
1 1 3  KE
KE  mv 2  K  A 2    3 :1
2 2 4  PE

26. 1 Velocity of harmonic oscillator

v   a2  x2
Here, b   A 2  a 2 and b   A 2  b 2
1/ 2
b  A2  a2 
   A  a 2  b2
a  A 2  b2 

342
[Study Package - Physics - Class XI]

1 1 1
27. 3 y = x; PE  m2 y 2 ; KE  m2 (a 2  y 2 ) ; TE = PE + KE = m2 a 2
2 2 2
Since PE is maximum at x = a and KE is maximum at x = 0, therefore TE remains constant
throughout the motion.

1 1 1 2E
28. 2 E  m2 A 2  E  m (2 f ) 2 A 2 ’  A
2 2 2f m

1 2  (0.5  0.4)
Putting values we obtain, A   A  0.06 m
 25  0.2
2  
 

29. 4 Fmax  m2 A  50 N

1 1  1  2 m2 A 50
2
Given kx  2
kA   x  A / 2 ; F  m x    25 2 N
2 
2 2 2 2

1
30. 2 Conservation of momentum : (0.98 + 0.02) v = 0.02 × 30    0.6 ms

1 1
Conservating energy in oscillation we get (0.98  0.02) 2  KA 2
2 2

2  2 (0.6)2 0.6
 A    A  0.06 m  6cm
K 100 10
LEVEL - II

2   t 
1. 1   ; For y-projection, y = A cos (t) ;  y  3cos  
4 2 2

 45
2. 4 As T  2 or T   ;       1.45
g 100

T  1.45
   1.2 ;
T  

 T 
% increase in time period    1100  1.2  1 100  20%
T 

343
Brilliant STUDY CENTRE

3. 3 Let O be the mean position and x be the distance of coin from O. The coin will slip if restoring
force on coin just becomes equal to force of friction i.e., mx2  mg
R
F
2 g Fc
The coin will slip if x = A;  mA  g or A  O
2 mg
x

4. 3 In figure, AC   cos  ;  OC  OA  AC     cos    1  cos  

Maximum KE of bob at O = Maximum PE of bob at B  mg  OC  mg 1  cos  


A

C B
h
O
T/4
T T t1 t2
5. 2 Given t1  t 2  ; or t 2   t1 ------(i)
4 4 O a/2 a
a a   T  2
at time t  t1 , x  ;   a sin t1 or t 1  or t 1  ;  t2    ; t1 : t 2  1: 2
2 2 6 6 4 6 6

dU
6. 2
2

2
Potential energy U = mv;  U  50x  100 10 ; F    dx
  100x 102

 m2 x   100  10 2  x; 10  10 3 2 x  100  10 2 x

 10 5
 2  100;    10 ;  f   
2 2 
7. 2 When mass 700 gm is removed, the left out mass (500 + 400) gm oscillates with a period of
3 sec

 3  t  2
 500  400 
---------(i)
k
When 500 gm mass is also removed, the left out mass is 400 gm.

400 3 900 3 3
 t   2 ------(ii);      t   2s
k t 400 t 2

5T
8. 3 m1  M, T1  T ; m 2  M  m, T2 
3
2 2
T1 2 m1 / k m1 M M  m  T2   5T / 3  25 m 16
        
T2 2 m 2 / k m2 Mm ; or
M  T1   T 
;
9 M 9

344
[Study Package - Physics - Class XI]

2m m K2 K T1
9. 2 T1  2 , T2  2 n
; Series K eq   , parallel K qn  2K ; 2
K 2K 2K 2 T2
10. 1 By drawing the free body diagram of object during the downward motion at extreme position, for
equilibrium of mass

mg  R  mA (A = Acceleration); For critical condition R = 0; so mg  mA  ma2

10  2 
  g/a  3
 50 ;  T    sec
4.0 10  50 25

1 1 1 kk
11. 4 As springs are connected in series, effective force constant.   k 1 2
k k1 k 2 k1  k 2

1 k 1 k1 k 2
Hence, frequency of oscillation is n  2 
m 2  k1  k 2  m
12. 1 v1  A ........ (i); v 2  2A ..... (ii)

v1 1 v 1
By eq. (i) and (ii);     v  2v
v2 2 v 2

 2  2  2
13. 1 a     x    ;  T 16 sec
 64  64 8 
There is a time difference of T/2 between t = 2 sec to t = 10 sec. Hence particle is again passing
through the mean position of SHM where its speed is maximum.

4 2 32 2
i.e., Vmax  A 4 2 ;  A  m
/8 

1 1
14. 3 K  mv 2  mA 2 2 cos 2 t ; Average value of cos2 t is 1 over one cycle
2 2 2
1 1 1
 K av  m2 A 2 --------(i); U  m2 x 2  m2 A 2 sin 2 t
4 2 2

2 1
Average value of sin t is over one cycle.
2

1
 U av  m2 A 2 -----(ii); From Eqs. (i) and (ii), we get Kav = Uav
4

345
Brilliant STUDY CENTRE

F 8 m 0.01
15. 4 Slope (k)    4 ; T  2  T  2  0.3sec
x 2 k 4
T 1.2
16. 2 Time taken by the particle to move from x = 0 to x = 4    0.3s .
4 4
Let t be the time taken by the particle to move from x = 0 to x = 2 cm.
2 1 2
y  a sin t  2  4 sin t   sin t
T 2 1.2
 2
  t  t  0.1 s . Hence time to move from x = 2 to x = 4 will be equal to 0.3 – 0.1 = 0.2 s.
6 1.2
Hence total time to move from x = 2 to x = 4 and back again = 2 × 0.2 = 0.4 sec.

1 g  n1   2  L 2 1
17. 2 n ; ;   n 2  2 n1 ;  n  n
2  n2 1 2L 2 2 2 1

1 a 2 m n2
Energy E  m2 a 2  22 mn 2 a 2 ;  12  2 22
2 a 2 m1n1
since m1 = m2 ;  a1  a 2

18. 3 y  a cos   t     A cos  t    ; v  A sin  t   


y0 v
At t  0, y 0  A cos    cos  -----(1); v 0  A sin    0  sin  -----(2)
A A
2 2 2
y0 v0 2 v0 2
Squaring and adding (1) and (2), we get 2  2 2  1  y 0  2  A
A A 
2 2
1
At y 0  1cm , v 0   cms ; 1  2
 A 2 or A  2 cm

1 3 
19. 2 y1  10 sin 2t  3 cos 2t   20  sin 2t  cos t 
2 2 
       
 20 cos sin 2t  sin cos 2t   20 sin  2t   
 3 3    3 

  A1 20 4
 A1  20 and y 2  5sin  2t   ; A 2  5 ; Hence,  
 4 A2 5 1
 b 
 t
 2m 
b
20. 2 Amplitude of damped oscillator - A(t)  A 0 e ; = constant
2m
b b b
A0 1
For t =1 min,  A 0 e 2m  e 2m   e 2m  2
2 2
 b 
  3 A0 A0
For t = 3 min, A  A 0 e  2m 
 ;  A X  23
 b/2m 3 23
e 
346
[Study Package - Physics - Class XI]

 L  
21. 3 Net force on bob in liquid = Mg – Bf  Mg  1   
 B 

  
L  1 9
Net acceleration  g  1     g  1  10   10 g
 B   

1 T g 10
T    T  T
g T 9 9
g
10

1
22. 1 E m2 A 2
2

1
(i) A  2.A  E  4E ; (ii) T    T  2T, E   
2
;
T2

E
 E 
2

1.69 1.44
23. 3 T1  2 ; T2  2
g g
Let the shorter pendulum makes (n + 1) vibrations then the longer makes n vibrations to come
in phase again then

 n  1 T2  nT1 ;

n  1 T1 1.69
   n  12
n T2 1.44

T 1 
24. 4 T    ;
T 2 

1 2 
 T     T  0.01T
 2 100 
Loss of time per day = 0.01 × 24 × 60 × 60 = 864 sec
25. 2 From conservation of linear momentum, MA   M  m  A

 M
where  ;
 mM

 M 
So, A   A
 mM 

347
Brilliant STUDY CENTRE

 
26. 2 y1  a sin t and y 2  b cos t  b sin  t  
 2
Since the frequencies for both SHMs are same, resultant motion will be SHM. Now

Amplitude A  A12  A 22  2A1A 2 cos 


Here, A1  a, A 2  b sin   ; So, A  a 2  b 2
2

A  a 2  b2
dU
27. 2 U  5x  x  4   5x 2  20x ; F   10x  20  10  x  2
dx
 SHM mean position at x = 2 m

2 2 10
Here m  10     100    10
0.1
2 2 
Time period, T    s
 10 5
28. 4 In SHM a  2 x ; So, 16  2  4     2

2 2
 Time period T   
 2

348
[Study Package - Physics - Class XI]

CHAPTER - 14
WAVES

Wave Motion
A wave is a mode of transference of energy from point to point in the direction of propagation of the
wave. The waves are periodic disturbances transmitted through a medium from a point of origin called the
source.The disturbances from the source are handed over to the particles in the medium. The particles in
the medium behave as if they are connected by some elastic forces. When a particle in the medium is set
into vibration, the adjacent particles acquire a similar kind of vibration.
A wave motion is a form of disturbance which travels through the medium due to the repeated periodic
motion of the particles of the medium about their equilibrium positions, the disturbance being handed over
from particle to particle.
A wave is a disturbance which propagate energy from one place to another without transport of matter.
1. Characteristics of wave motion
(i) It is a sort of disturbance which travels through a medium.
(ii) Material medium is essential for the propagation of mechanical waves.
(iii) When a wave motion passes through a medium, particles of the medium only vibrate simple
harmonically about their mean position.
(iv) There is a continuous phase difference amongst successive particles of the medium ie, particle 2
starts vibrating slightly later than particle 1 and so on.
(v) The velocity of the particle during their vibration is different at different position.
(vi) The velocity of wave motion through a particular medium is constant. It depends only on the nature
of the medium and not on the frequency, wavelength or intensity.
(vii) Energy is propagated along with the wave motion without any net transport of the medium.
2. Mechanical waves: The waves which require a medium for their propagation are called mechanical
waves.
Example: Waves on string and spring, waves on water surface, sound waves, seismic waves.
3. Non-mechanical waves: The waves which do not require a medium for their propagation are called
non-mechanical or electromagnetic waves.
Examples: Light, heat (Infrared), radio waves,   rays , X-rays etc.
4 Transverse waves: Particles of the medium execute simple harmonic motion about their mean position
in a direction perpendicular to the direction of propagation of wave motion.
(i) It travels in the form of crests and troughs.
(ii) A crest is a portion of the medium which is raised temporarily above the normal position of rest of
the particles of the medium when a transverse wave passes through it.

349
Brilliant STUDY CENTRE

(iii) A trough is a portion of the medium which is depressed temporarily below the normal position of
rest of the particles of the medium, when transverse wave passes through it.
(iv) Examples of transverse wave motion : Movement of string of a sitar or violin, movement of the
membrane of a Tabla or Dholak, movement of kink on a rope, waves set-up on the surface of water.
(v) Transverse waves can be transmitted through solids, they can be setup on the surface of liquids.
But they can not be transmitted into liquids and gases.
5. Longitudinal waves : If the particles of a medium vibrate in the direction of wave motion, the wave is
called longitudinal.
(i) It travels in the form of compressions and rarefactions.
(ii) A compression (C) is a region of the medium in which particles are compressed (high density).
(iii) A rarefaction (R) is a region of the medium in which particles are rarefied (low density).

(iv) Sound waves travel through air in the form of longitudinal waves. Vibration of air column in organ
pipes , Vibration of air column above the surface of water in the tube of a resonance apparatus are
longitudinal waves
Wavelength : Wavelength is the distance between any two nearest particles of the medium, vibrating in the
same phase. Distance between two consecutive crests or troughs is a wavelength. Distance between
consecutive compressions or rarefactions is a wavelength.

Frequency : (i) Frequency of vibration of a particle is defined as the number of vibrations completed by
particle in one second.
(ii) It is the number of complete wavelengths traversed by the wave in one second.
(iii) Units of frequency are hertz (Hz) and per second.
Time period : (i) Time period of vibration of particle is defined as the time taken by the particle to complete
one vibration about its mean position.
(ii) It is the time taken by the wave to travel a distance equal to one wavelength.
1
Relation between frequency and time period : Time period = 1/Frequency  T  ---------(1)

350
[Study Package - Physics - Class XI]

Relation between velocity, frequency and wavelength of the wave:

distance s
By definition, velocity   =
time t
In one complete vibration of the particle, distance travelled, s =  and time taken, t  T

v    1  ,  - wavelength, v - velocity,,  -frequency..


T T
Using (1), we get v   -------------(2)
Hence velocity of a wave is the product of frequency and wavelength of the wave. This relation holds for
transverse as well as longitudinal waves.
Some terms related to progressive waves
(i) Wave number (  ) : The number of waves present in unit length is defined as the wave number

1
  -------------(3); Unit = meter–1 ; Dimension = [L–1].

 Phase difference between particles


(ii) Propagation constant (k) : k  
x Distance between them

 Angular velocity 2
k  and k   2 ---------(4)
v Wave velocity 
(iii) Wave velocity ( υ ) : The velocity with which the crests and troughs or compression and rarefaction
  
travel in a medium, is defined as wave velocity v     
k 2 T
(iv) Phase and Phase difference : Phase of the wave is given by the argument of sine or cosine in the
2
equation of wave. It is represented by   x, t    vt  x  ,

(v) At a given position (for fixed value of x) phase changes with time (t)

d 2v 2 2 2
   d  . dt  Phase difference   Time difference.
dt  T T T
(vi) At a given time (for fixed value of t) phase changes with position (x).
d 2 2 2
  d   dx  Phase difference  × Path difference.---------(5)
dx   
T
 Time difference   Path difference ---------(6)

One dimensional Travelling wave
To completely describe a wave on a string, we need a function that gives the shape of the wave.
Consider a sinusoidal wave travelling to the right with constant speed, transverse displacement of the
particles of the string is measured with the coordinate y. Figure (a) represents the shape and position of the

351
Brilliant STUDY CENTRE

pulse at t = 0. At this time, the shape of the pulse, whatever it may be, can be represented as y = f(x), that is
y is some definite function of x. The maximum displacement ym is called the amplitude of the wave. The
pulse travel to the right a distance vt in a time t, then the transverse displacement y of a string element at
position x at time t is given by y = f(x – vt)
Similarly, if the wave pulse travel to the left, the displacement y is given by y = f(x + vt).

The speed of travelling wave


Consider a wave travelling in the direction of increasing x, the entire wave pattern is moving a distance
x in that direction during the interval t . The ratio x / t is the wave speed v..
v = dx / dt

A snapshot of the travelling wave, at t = 0 and a later time t  t . During the time interval t , the
entire curve shifts a distance x to the right.
(1) Speed of transverse wave motion :

T
(i) On a stretched string : v  ------(7); T = Tension in the string; m = Linear density of the string.
m


(ii) In a solid body : v  ---------(8)  = Modulus of rigidity;  = Density of the material.

(2) Speed of longitudinal wave motion :

4
k 
(i) In a solid medium v  3 ---------(9) k = Bulk modulus;  = Modulus of rigidity;  = Density

Y
When the solid is in the form of long bar v  -----(10) Y = Young’s modulus of material of rod

352
[Study Package - Physics - Class XI]

k
(ii) In a liquid medium v  ---------(11)

k
(iii) In gases v  ---------(12)

Reflection and transmission of waves

Consider a travelling pulse of a string whose one end is fixed to a rigid support. When the pulse reached
the fixed wall, it will be reflected.
When the pulse reaches at the end of the string that is fixed at the support, the string produces an
upward force on the support. By Newton’s third law, the support must then exert and equal and opposite
reaction force on the string, which causes the pulse to invert upon reflection.
Let us now suppose that the right end of the string is attached to a light frictionless ring which can freely
move on a vertical rod. Now when the pulse reaches at right end, there is no corresponding restoring force
from the right as the rod does not exert a vertical force on the ring. In this case pulse will be reflected, but this
time its displacement is not inverted. The amplitude is the same as that of the incoming pulse.
Now consider a situation in which the end point is neither completely fixed nor completely free to move.
For example, consider a light string attached to a heavier string as shown in figure. If a wave pulse is
produced on the light string (figure (c)) moving towards the junction, a part of the wave is reflected and a part
is transmitted on the heavier string. The reflected wave is inverted to the original one. If the wave is produced
on the heavier string (figure (d)), which moves towards the junction, a part will be reflected and a part is
transmitted but the reflected wave is no more inverted.
Progressive Wave
(1) These waves propagate in the forward direction of medium with a finite velocity.
(2) Energy and momentum are transmitted in the direction of propagation of waves without actual
transportation of matter.
(3) In progressive waves, equal changes in pressure and density occurs at all points of medium.
(4) Progressive wave can be represented as.

353
Brilliant STUDY CENTRE

Where y = displacement, A = Amplitude,  = Angular


y  A sin  t  kx or  frequency, = Frequency, k = Propogation constant,

 2        (13) T= Period. = Wave length, v = Wave velocity, t = Time,
y  A sin  t  x
    x = Position of the particle from the origin

Standing Waves
When two waves of identical frequency (of similar kind) travelling from opposite directions meet, the
resultant wave obtained is known as standing waves. Standing waves can either be pure standing waves or
partial standing waves.
Pure Standing Waves
Here the amplitude of the two superposing waves need to be identical too.
Let the two travelling waves be

y1  x, t   A sin  t  kx  ; y 2  x, t   A sin  t  kx 

Due to their superposition, the resultant wave is

y R  x, t   y1  x, t   y 2  x, t   A sin  t  kx   sin  t  kx     2A cos kx  sin t -------- (14)

where (2A cos kx) = Ax is clearly a position dependent function (cosine, hence periodic too) and is a constant
for a particular location and is known as amplitude of oscillation at that location.

While the second term, sin  t  indicates SHM of the particle at position x with amplitude Ax. Therefore,

sin  t  term is common for all particles, hence each particle has same frequency of oscillation.

The amplitude, Ax = 2Acos(kx) as a function of x can be plotted as below.

There is no phase difference between oscillations of different elements of the wave. The string as a
whole vibrates in phase with different amplitudes at different points. The wave pattern is neither moving to
the right nor to the left. The points at which the amplitude is zero (where there is no motion at all) are nodes;
the points at which the amplitude is the largest are called antinodes.
The positions of nodes are given by sin kx = 0

kx  n ; n = 0, 1, 2, 3, ......

2 n
k  x ; n = 0, 1, 2, 3, .....
 2

354
[Study Package - Physics - Class XI]


The distance between any two successive nodes is .
2
In the same way positions of antinodes are given by the largest value of sin kx.
| sin kx |  1

 1
kx   n    ; n = 0, 1, 2, 3, ....
 2

2  1
Since k  ; We get x   n   ; n = 0, 1, 2, .....
  2 2


The distance between any consecutive antinodes is .
2
Standing waves in a string
Consider a string of length l, tension T and linear density  stretched between two fixed points

The smallest frequency with which standing waves can be set up in any system is said to be fundamental
frequency (  0 ). Frequency  0 , clearly means having largest wavelength so that

v   = constant,
since velocity of wave is only medium dependent. Since both the ends are fixed, they will necessarily
form nodes.
Therefore, to have largest wavelength we will have to insert one antinode between these two nodes.

v T/
 v   0 0   0  2l  ;  0   -----------(15)
2l 2l
Overtones are the higher frequencies, with respect to the fundamental frequency with which standing
waves can be set up. For example, 1st overtone is the immediate next higher frequency (hence immediate
next lower wavelength). Therefore we insert one more antinode here.

355
Brilliant STUDY CENTRE

v
 v  11  1    2  20
 2l 

 1st overtone = 2  0 = 2nd harmonic.

(Note: All integral multiples of fundamental frequency are said to be harmonics of  0 .

Similarly for finding 2nd overtone.

3v
 v  2 2  2  2  30
2l
 2nd overtone = 3rd harmonic.  Generalising, we have, nth overtone

th v
n   n  1 harmonic   n  1 0   n  1
2l
If the string oscillates in m loops, the frequency of oscillation is

v T/
m  m 0  m m
2l 2l
We can easily change the frequency of oscillation by changing T (by increasing/decreasing tension), 
(thin/thick wire) or l (long/short wire). This is the basic of sound production of different frequency in guitar
wires. By the movement of the fingers, on the string one controls the location of nodes whereas by plucking
it with the other hand, we control the location of antinodes.
If the string’s tension is increased/decreased slighly (less than 5%), we can apply error formula to find
the percentage change in frequency of string.

 1 T
ie,  -----------(16)
 2 T
Laws of Transverse Vibrations of String - Sonometer wire

1  L2
 so 1 
(a) Law of length
L 2 L1 ; if T &  are constant.-----------(17)

356
[Study Package - Physics - Class XI]

1 T
(b) Law of tension   T so  1 ; if L &  are constant.-----------(18)
2 T2

1  2
(c) Law of mass  so 1 
 2 1 ; if T & L are constant.-----------(19)

SOUND WAVES

Sound Waves
Sound waves are the most common example of longitudinal waves. They travel through any material
medium with a speed that depends on the properties of the medium. As the waves travel through air, the
elements of air vibrate to produce changes in density and pressure along the direction of motion of the
wave. If the source of the sound waves vibrates sinusoidally, the pressure variations are also sinusoidal.
The mathematical description of sinusoidal sound waves is very similar to that of sinusoidal string
waves.
Sound waves are divided into three categories that cover different frequency ranges. (1) Audible waves
lie within the range of sensitivity of the human ear. They can be generated in a variety of ways, such as by
musical instruments, human voices, or loudspeakers. (2) Infrasonic waves have frequencies below the
audible range. Elephants can use infrasonic waves to communicate with each other, even when separated
by many kilometers. (3) Ultrasonic waves have frequencies above the audible range. Ultrasonic waves are
also used in medical imaging.
Velocity of Sound in Elastic Medium.
When a sound wave travels through a medium such as air, water or steel, it will set particles of medium
into vibration as it passes through it. For this to happen the medium must possess both inertia (i.e. mass
density so that kinetic energy may be stored) and elasticity (so that PE may be stored). These two properties
of matter determine the velocity of sound.
i.e. velocity of sound is the characteristic of the medium in which wave propagate.
E
v ----------(1) (E = Elasticity of the medium;  = Density of the medium)

Important points
(1) As solids are most elastic while gases least i.e. ES > EL > EG . So the velocity of sound is maximum in
solids and minimum in gases
v steel  v water  v air
Lesser the velocity, denser is said to be the medium and vice-versa.
(2) Newton’s formula : He assumed that when sound propagates through air temperature remains constant

K P
.(i.e. the process is isothermal) v air   -----------(2)
 

As K = E   P ; E  = Isothermal elasticity; P = Pressure.

357
Brilliant STUDY CENTRE

By calculation v air = 279 m/sec.

However the experimental value of sound in air is 332 m/sec which is greater than (52 m/s = 15.6%)
that given by Newton’s formula.
(3) Laplace correction : He modified Newton’s formula assuming that propagation of sound in air as
adiabatic process.

K E
v  ----------(3) (As K  E   P  Adiabatic elasticity)
 

v  1.41  279  331.3 m / s   Air  1.41

P 1
(4) Effect of density : v  v
 

P RT
(5) Effect of pressure : v   . -------------(4)
 M
Velocity of sound is independent of the pressure of gas provided the temperature remains constant. (
P   when T = constant)

RT
(6) Effect of temperature : v  v T (T in K)
M

When the temperature change is small then v t  v 0 1  t 

v 0 = velocity of sound at 0°C, v t = velocity of sound at t°C ,  = temp-coefficient of velocity of sound.

m/s
Value of   0.608 o
 0.61  Approx 
C
Temperature coefficient of velocity of sound is defined as the change in the velocity of sound, when
temperature changes by 1°C.
(7) Effect of humidity : With increase in humidity, density of air decreases. So with rise in humidity, velocity
of sound increases.
This is why sound travels faster in humid air (rainy season) than in dry air (summer) at the same
temperature.
(8) Effect of wind velocity : Because wind drifts the medium (air) along its direction of motion therefore
the velocity of sound in a particular direction is the algebraic sum of the velocity of sound and the
component of wind velocity in that direction. Resultant velocity of sound along SL  v  w cos 

358
[Study Package - Physics - Class XI]

(9) Sound of any frequency or wavelength travels through a given medium with the same velocity.
(v = constant) For a given medium velocity remains constant. All other factors like phase, loudness,
pitch, quality etc. have practically no effect on sound velocity.
(10)Relation between velocity of sound and root mean square velocity.

RT 3RT v rms 3 1/ 2


v sound  and v rms  , so  or v sound    / 3 v rms ----------(5)
M M vsound 

(11) There is no atmosphere on moon, therefore propagation of sound is not possible there. To do conversation
on moon, the astronaut uses an instrument which can transmit and detect electromagnetic waves.
Standing Waves in a Closed Organ Pipe.
Organ pipes are the musical instruments which are used for producing musical sound by blowing air
into the pipe. Longitudinal stationary waves are formed on account of superimposition of incident and reflected
longitudinal waves.

2vt 2x
Equation of standing wave y  2a cos sin
 
4L
General formula for wavelength   2n  1
 
4L v
(1) First normal mode of vibration : n  1  1   4L 1 
 2  1 4L

This is called fundamental frequency. The note so produced is called fundamental note or first harmonic.

v 3v
(2) Second normal mode of vibration : (n = 2) 2    31
 2 4L

This is called third harmonic or first overtone.

5v
(3) Third normal mode of vibration : (n = 3) 3   51
4L
This is called fifth harmonic or second overtone.
Standing Waves in Open Organ Pipes
General formula for wavelength

2L
 where n = 1, 2, 3 ............
n

359
Brilliant STUDY CENTRE

v v
(1) First normal mode of vibration : n =1, 1  2L  1    2L
1

This is called fundamental frequency and the note so produced is called fundamental note or first
harmonic.
v v  v 
(2) Second normal mode of vibration (n = 2)  2    2   21   2  21
2 L  2L 
This is called second harmonic or first overtone.
v 3v
(3) Third normal mode of vibration (n = 3) 3   ,  3  31
 3 2L
This is called third harmonic or second overtone.

In organ pipe an antinode is not formed exactly at the open end rather it is formed a little distance away
from the open end outside it. The distance of anitnode from the open end of the pipe is konwn as end
correction.
Comparative study of stretched strings, open organ pipe and closed organ pipe.
S.No. Parameter Stretched Open organ pipe Closed organ pipe
string
1 Fundamental frequency or v v v
1st harmonic 1  1  1 
2l 2l 4l
2 Frequency of 2nd harmonic  2  21  2  21 Missing
3 Frequency of 3rd harmonic  3  31  3  31 3  31
4 Frequency ratio of 2 : 3 : 4 …… 2 : 3 : 4 ….. 3 : 5 : 7 …..
overtones
5 Frequency ratio of 1 : 2 : 3 : 4 …. 1 : 2 : 3 : 4 …. 1 : 3 : 5 : 7 ….
harmonics
6 Nature of waves Transverse Longitudinal Longitudinal
stationary stationary stationary

360
[Study Package - Physics - Class XI]

BEATS
When two sound waves of slightly different frequencies, travelling in a medium along the same direction,
superimpose on each other, the intensity of the resultant sound at a particular position rises and falls
regularly with time. This phenomenon of regular variation in the intensity of sound with time at a particular
position, when two sound waves of nearly equal frequencies superimpose on each other is called beats.
Beat period : The time interval between two successive maxima or minima of sound is called the beat
period.
Beat frequency : The number of beats produced per second is called beat frequency.
Persistence of hearing : The impression of sound heard by our ears persist in our mind for 1/10th of
a second. If another sound is heard before 1/10 second is over, the impression of the two sound mix up
and our mind cannot distinguish between the two.
So for the formation of distinct beats, frequencies of two sources of sound should be nearly equal
(difference of frequencies should be less than 10)
FORMATION OF BEATS
(a) Graphical method
Suppose we have two tuning forks A and B. Fig. below shows superimposition of the two waves from
forks A and B and in Fig.(d), we have represented the resultant wave according to the principle of superposition.
t=0 t = T/4 t = T/2 t = 3T/4 t=T

(a)

(c)

P T (d)
Q R S

DOPPLER EFFECT
The phenomenon of the apparent change in the frequency of sound produced by a source as heard by
a listener whenever there is relative motion between the source and the listener is called Doppler effect.
General expression for apparent frequency
Consider a source S producing sound of frequency  . Let V be the velocity of sound in the medium and
 the wave­length of sound when the source and the listener are at rest. Then the frequency of sound heard
by the listener is

 V   V
 

361
Brilliant STUDY CENTRE

+ve
V
S L
VS VL
Let the source and the listener be moving with velocities Vs and Vl in the direction of propagation of sound
from source to listener. The direction S to L is taken positive.
Sign convention : All velocities along the direction S to L are taken as positive and all velocities along the
direction L to S are taken as negative.

The relative velocity of sound wave with respect to the source  V  Vs

1 V  Vs
Apparent wavelength of sound  

Since the listener is moving with a velocity Vl , the relative velocity of sound with respect to the listener
V1 = V – Vl
Apparent frequency of sound as heard by the listener is given by,

Relative velocity of sound w.r.t.the listener V1


ν1   1
Apparent wavelength 

 V  Vl 
1      -----------(6)
 V  Vs 
Note:
Velocities in the direction in which the sound travels in order to reach the listener are taken positive;
while those in the opposite direction are taken negative.
Case (1) Listener at rest and source in motion
(a) When the source moves towards the stationary listener, Vs is positive and Vl= 0

 
1   V   
 V  Vs 
(b) When the source moves away from the stationary listener, Vs is negative and Vl= 0

 
1   V   
 V  Vs 
Case (2) Listener in motion and source at rest
(a) When the listener moves towards the stationary source, Vl is negative and Vs = 0

 V  Vl 
1   
 V 
(b) When the listener moves away from the stationary source, Vl is positive and Vs = 0

 V  Vl 
1   
 V 

362
[Study Package - Physics - Class XI]

Case(3) When both the source and the listener are in motion

(a) When the source and the listener move towards each other, Vs is positive and Vl is negative

 V  Vl 
1   
 V  Vs 

(b) When the source and the listener move away from each other. Vs is negative and Vl is positive

 V  Vl 
1   
 V  Vs 

(c) When the source moves away from the listener and the listener moves towards the source, Vs is
negative and Vl is negative

 V  Vl 
1   
 V  Vs 

(d) When the listener moves away from the source and the source moves towards the listener, Vl is
positive and vs is positive

 V  Vl 
1   
 V  Vs 

Effect of motion of medium

When a wind is blowing with a velocity W in the direction of propagation of sound, the resultant velocity
of sound will be (V + W).

 V  W  Vl 
 1      ----------(7)
 V  W  Vs 

If the wind is blowing in a direction opposite the direction of propagation of sound, the resultant velocity of
sound will be ( V — W).

Limitation of Doppler effect

Doppler effect is applicable as long as the relative velocity between the source and the listener is less
than the velocity of sound. The principle is not applicable if the source moves towards the listener with
supersonic velocity.

Applications of Doppler effect

1 To estimate the speed of a submarine (Sonar)

Ultrasonic waves are transmitted from the ship and their reflected waves obtained by reflection from
the submarine are observed. By finding the difference in frequency of the transmitted wave and the
reflected wave, we can calculate the velocity of the submarine.

363
Brilliant STUDY CENTRE

2 To estimate the speed of aeroplane, automobile etc.

Short radio waves are emitted from an observation centre. These waves are reflected from the plane
(or automobile) and received by the centre. By measuring the change in frequency of the transmitted
and the reflected waves, the velocity and the direction of motion of the plane (or the automobile) can be
calculated.

3 To track artificial satellites

Doppler effect provides a convenient method for tracking an earth satellite. The earth satellite emits
radio signals of constant frequency  . The apparent frequency 1 of the signal as received by the
tracking station on the earth is noted. From this the velocity of the earth satellite can be obtained.

4 To estimate the velocity of rotation of the sun

By the study of Doppler shift from the light received from the western and eastern edges of the sun, it
has been found that the shift is due to a velocity of 2 kms -1. Since no such shift is observed from light
received from north and south edges, it is concluded that sun rotates about north-south axis with a
velocity 2 kms-1.

364
[Study Package - Physics - Class XI]

QUESTIONS
LEVEL - I

 2
1. A progressive wave moving along x axis is represented by y  A sin   vt  x  . The wavelength at
 
which the maximum particle velocity is 3 times the wave velocity is

A 2A 3 2
1) 2) 3) A 4) A
3 3 4 3
2. Equation of a transverse wave is given by y = 20 sin (t + 0.2 x). The wavelength of wave is :
1) 0.6  2) 0.3 3) 15  4) 10 
3. A 5.5 metre length of string has a mass of 0.035 kg. If the tension in the string is 77 N, the velocity of
the wave on the string is :
1) 210 ms–1 2) 40 ms–1 3) 110 ms–1 4) 55 ms–1
4. Two sinusoidal waves of intensity I having same frequency and same amplitude interferes constructively
at a point. The resultant intensity at that point will be [AIIMS 2012]
1) I 2) 2 I 3) 4 I 4) 8 I
5. Identify the correct statement about a stationary wave
1) Stationary wave is formed by superposition of two waves of different frequencies
2) The energy of oscillation is minimum at the antinode
3) Amplitude at the antinode is the same as that at the node
4) The pressure change is least at the antinode
6. If we study the vibration of a pipe open at both ends, then the following statement is not true
[NEET 2013]
1) All harmonics of the fundamental frequency will be generated
2) Pressure change will be maximum at both ends.
3) Open end will be antinode
4) Odd harmonics of the fundamental frequency will be generated
7. In the case of a travelling wave, the reflection at a rigid boundary will take place with a phase change
of

  
1) radian 2) radian 3)  radian 4) radian
2 4 6
8. A wave of wavelength 2m is reflected from a surface. If a node is formed at 3 m from the surface, then
at what distance from the surface another node will be formed?
1) 1 m 2) 2 m 3) 3 m 4) 4 m
9. A standing wave is produced in a string fixed at both ends. In this case
1) All particles vibrate in phase
2) All antinodes vibrate in phase
3) All alternate antinodes vibrate in phase
4) All particles between two consecutive antinodes vibrate in phase

365
Brilliant STUDY CENTRE

 x
10. A longitudinal wave is represented by x  x 0 sin 2  t   . The maximum particle velocity will be
 
four times the wave velocity if

x 0 x 0
1)   2)   2x 0 3)   4)   4x 0
4 2
11. A string of mass 2.5 kg is under a tension of 200 N. The length of the stretched string is 20 m. If a
transverse jerk is struck at one end of the string, how long does the disturbance take to reach the
other end?
1) 0.3 s 2) 0.5 s 3) 0.7 s 4) 0.9 s

 
12. A transverse harmonic wave on a string is described by y  3sin  36t  0.018x   where x and y
 4
are in cm and t in seconds. The least distance between two successive crests in the wave is
1) 2.5 m 2) 3.5 m 3) 4.5 m 4) 5 m
13. A pipe closed at one end produces a fundamental note of 412 Hz. It is cut into two pieces of equal
lengths. The fundamental notes produced by the two pieces are
1) 200 Hz, 412 Hz 2) 206 Hz, 824 Hz 3) 412 Hz, 824 Hz 4) 824 Hz, 1648 Hz


14. A string of length  is elongated by the time taken by the transverse wave to cover the string is t1.
3

If the string is elongated by a distance , the time taken by the transverse wave to cover the string is
2
t1
t2. Then, 
t 2 (Assume that the string obey’s Hook’s law)

4 4 4 2
1) 2) 3) 4)
3 3 3 3 3
15. The speed of sound in hydrogen at NTP is 1270 m/s. Then the speed in a mixture of hydrogen and
oxygen in the ratio 4 : 1 by volume will be
1) 317 m/s 2) 635 m/s
3) 830 m/s 4) 950 m/s
16. Distance between successive compressions and rarefactions is 1 m and velocity of sound is
360 ms–1. The frequency is.
1) 180 Hz 2) 45 Hz 3) 120 Hz 4) 90 Hz
17. A person is observing two trains one coming towards him and the other leaving with the same velocity
4 m/s. If the whistling frequencies are 240 Hz each, then the number of beats per second. [Vsound =
320 m/s]
1) 3 2) 6 3) 9 4) Zero

366
[Study Package - Physics - Class XI]

18. In order to double the frequency of the fundamental note emitted by a stretched string, the length is
reduced to 3/4th of the original length and the tension is changed. The factor by which the tension is
to be changed is

3 2 8 9
1) 2) 3) 4)
8 3 9 4
19. 5 beats/s are produced on blowing together two closed organ pipes of the same diameter but of
different lengths. If the shorter pipe is 10 cm length and speed of sound in air is 300 m/s, length of the
other pipe is
1) 10.06 cm 2) 11.22 cm 3) 16 cm 4) 14 cm
20. A uniform rope of length L and mass m1 hangs vertically from a rigid support. A block of mass m2 is
attached to the free end of the rope. A transverse pulse of wavelength 1 is produced at the lower end
of the rope. The wavelength of the pulse when it reaches the top of the rope is  2 . The ratio  2 / 1
is:

m1 m1  m 2 m2 m1  m 2
1) 2) 3) 4)
m2 m2 m1 m1
21. A copper wire is held at the two ends by rigid supports. At 30oC the wire is just taut, with negligible
tension. The speed of transverse waves in this wire at 10 oC is: (   1.7  105 / o C ,
3 3
Y  1.3  1011 N / m 2 , d  9  10 kg / m )
1) 80 m/s 2) 90 m/s
3) 100 m/s 4) 70 m/s
22. A string of length 1 m and linear mass density 0.01 kg m–1 is stretched to a tension of 100 N. When
both ends of the string are fixed, the three lowest frequencies for standing wave are f1, f2 and f3, when
only one end of the string is fixed, the three lowest frequencies for standing wave are n1, n2 and n3.
Then

1) n 3  5n1  f3  125Hz 2) f3  5f1  n 3  125Hz

f1  f 2
3) f3  n 2  3f1  150Hz 4) n 2   75Hz
2

 2 
23. The equation of a stationary wave is given by y  0.5sin  x  cos (120 t) where x, y are in m and t
 3 
in s. Which of the following statements is wrong?
1) The frequency of the wave is 60 Hz 2) The speed of the wave is 180 ms–1
3) The wavelength of the wave is 3 m 4) The speed of the wave is 90 ms–1
24. A car moving at a velocity of 17 ms–1 towards an approaching bus that blows a horn at a frequency of
640 Hz on a straight track. The frequency of this horn appears to be 680 Hz to the car driver. If the
velocity of sound in air is 340 ms–1, then the velocity of the approaching bus is :
1) 2 ms–1 2) 4 ms–1 3) 8 ms–1 4) 10 ms–1

367
Brilliant STUDY CENTRE

LEVEL - II

  
1. If y  5sin  30t  x  30o  y  mm, t  second, x  m . For given progressive wave equation,
 7 
phase difference between two vibrating particles having path difference 3.5 m would be

  
1) 2)  3) 4)
4 3 2
2. In a plane progressive harmonic wave, particle speed is always less than the wave speed if
1) amplitude of wave   / 2  2) amplitude of wave     

3) amplitude of wave   4) amplitude of wave   / 


3. A sonometer wire resonates with a given tuning fork forming standing waves with five antinodes
between the two bridges when a mass of 9 kg is suspended from the wire. When this mass is
replaced by a mass M, the wire resonates with the same tuning fork forming three antinodes for the
same positions of the bridges. The value of M is
1) 25 kg 2) 5 kg 3) 12.5 kg 4) 1/25 kg
4. A string of mass 0.2 kg/m has length  = 0.6 m. It is fixed at both ends and stretched such that it has
a tension of 80 N. The string vibrates in three segments with amplitude = 0.5 cm. The amplitude of
transverse particle velocity is
1) 9.42 m/s 2) 3.14 m/s 3) 1.57 m/s 4) 6.28 m/s
5. A uniform wire of length L, diameter D and density  is stretched under a tension T. The correct
relation between its fundamental frequency ‘f’, the length L and the diameter D is

1 1 1 1
1) f  2) f  3) f  4) f 
LD L D D2 LD 2
6. The speed of a wave on a string is 150 m/s when the tension is 120 N. The percentage increase in the
tension in order to raise the wave speed by 20% is
1) 44% 2) 40% 3) 20% 4) 10%

7. A progressive wave is represented by y  12sin  5t  4x  cm . On this wave, how far away are the
two points having difference of 90o?

   
1) cm 2) cm 3) cm 4) cm
2 4 8 16
8. Sound waves of frequency 660 Hz fall normally on a perfectly reflecting wall. The shortest distance
from the wall at which the air particle has maximum amplitude of vibration is (velocity of sound in air is
330 m/s)
1) 0.125 m 2) 0.5 m 3) 0.25 m 4) 2 m
–3
9. The mass of 1 mole of air is 29 × 10 kg. The speed of sound in air at standard temperature and
 7
pressure is  Take   
 5
1) 270 ms–1 2) 290 ms–1 3) 330 ms–1 4) 350 ms–1

368
[Study Package - Physics - Class XI]

10. A pipe 30 cm long is open at both ends. If the nth harmonic mode of the pipe resonates with a 1100 Hz
source, for what harmonic will the resonance be observed if one end of the pipe is closed? Velocity of
sound v = 330 ms–1
1) 2 2) 3 3) 4 4) does not exist
11. A siren placed at a railway platform is emitting sound of frequency 5 kHz. A passenger sitting in a
moving train A records a frequency of 5.5 kHz while the train approaches the siren. During his return
journey in a different train B he records a frequency of 6.0 kHz while approaching the same siren.
The ratio of the velocity of the train B to that of train A is

242 5 11
1) 2) 2 3) 4)
252 6 6

12. The vibrations of 4 air columns are as shown in the figure. The ratio of the frequencies  p ,  q ,  r ,  s
is

1) 12 : 6 : 3 : 4 2) 1 : 2 : 4 : 3
3) 4 : 2 : 3 : 1 4) 6 : 2 : 3 : 4
13. The length of the pipe open at both ends is 48 cm and its fundamental frequency is 320 Hz. If the
speed of sound is 320 m/s what is the diameter of the pipe?
1) 0.02 m 2) 3.33 cm 3) 0.33 m 4) 3.33 mm
o
14. By how much must the temperature of air near 0 C be changed to cause the speed of sound in it to
change by 1%?
1) 5.5oC 2) 11oC 3) 1oC 4) 2oC
15. An open pipe is in resonance in its 2nd harmonic with a tuning fork of frequency f1. Now it is closed at
one end. If the frequency of tuning fork is increased slowly from f1 then again a resonance is obtained
with a frequency f2. In this case, the pipe vibrates in nth harmonic, then

3 5 3 9
1) n  5, f 2  f1 2) n  3, f 2  f1 3) n  3, f 2  f1 4) n  5, f 2  f1
4 4 4 4
16. The velocity of sound in air at 200C and 1 atm pressure is 344.2 m/s. At 400C and 2 atm pressure, the
velocity of sound in air is approximately
1) 350 m/s 2) 356 m/s
3) 363 m/s 4) 370 m/s
17. Two solid bars are having same Young’s modulus but made of different materials such that the ratio
of the mass densities is  1 / 2   4 . The ratio of the speeds of the longitudinal waves in the bars,
i.e., (v2/v1) is:
1) 1 2) 2 3) 3 4) 4

369
Brilliant STUDY CENTRE

18. The fundamental frequency in an open pipe is equal to the third harmonic of a closed organ pipe. If the
length of the closed organ pipe is 20 cm, the length of the open organ pipe is
1) 13.3 cm 2) 8 cm 3) 12.5 cm 4) 16 cm
19. The frequencies of two tuning forks A and B are respectively 1.5% more and 2.5% less than that of
the tuning fork C. When A and B are sounded together, 12 beats are produced in 1 sec. The frequency
of the tuning fork C is
1) 200 Hz 2) 240 Hz 3) 360 Hz 4) 300 Hz
20. Two closed pipes produce 10 beats per second when emitting their fundamental notes. If their lengths
are in ratio of 25 : 26, then their fundamental frequencies in Hz, are
1) 270, 280 2) 260, 270 3) 260, 250 4) 260, 280
21. Two factories are sounding their sirens and 400 Hz each. A man walks from one factory towards the
other at a speed of 2 ms–1. The speed of sound is 320 ms–1. The number of beats heard per second
by the man is
1) 6 2) 5 3) 2.5 4) 7.5
22. At which temperature the speed of sound in hydrogen will be same as speed of sound in oxygen at
100oC?
1) –148oC 2) –212.5oC 3) –317.5oC 4) –249.7oC
23. A student is performing the experiment of Resonance Column. The diameter of the column tube is 4
cm. The frequency of the tuning fork is 512 Hz. The air temperature is 38oC in which the speed of
sound is 336 m/s. The zero of the metre scale coincides with the top end of the Resonance Column
tube. When the first resonance occurs, the reading of the water level in the column is
1) 14.0 cm 2) 15.2 cm 3) 16.4 cm 4) 17.6 cm

370
[Study Package - Physics - Class XI]

KEY WITH HINTS


LEVEL - I
dy 2
1. 4 Vparticle   Av
dt 

2 2
Vmax  3v  Av ;  A
 3
2. 4 As y = 20 sin (t + 0.2 x); Compare it with standard equation of wave form

 2 2  2 2
y  A sin  t  x   0.2 or    10 
T    0.2

0.035 77
3. 3 Here, m  , T  77 N ;   T / m   110 ms 1
5.5 0.035 / 5.5

4. 3 The resultant intensity is given by I R  I1  I 2  2 I1I 2 cos 

For constructive interference, cos   1 ;  I R  I1  I 2  2 I1I 2 or I R  4I (Given I1  I 2  I)

5. 4 1) Stationary wave is formed by superposition of two waves of same frequencies


2) The energy of oscillation is maximum at the antinode
3) Amplitude at the antinode is different from as that at the node. It is maximum at the antinode
and zero at the node
4) The pressure change is least at the antinode
6. 2 Pressure change will be minimum at both ends
7. 3 In case of a travelling wave, the reflection at a rigid boundary will take place with a phase reversal
or with a phase change of  or 1800

  2m
8. 1 Distance between two consecutive nodes is ;    1m
2 2 2
9. 3 All particles between one pair or consecutive nodal points are in phase which is opposite to that
of the particles between the preceding or succeeding pair. Thus all alternate antinodes vibrate in
phase

x  x
10. 3 Wave speed =  ; Particle speed   2x 0 cos 2  t  
t  

 x   x 
 t   2x 0 ; If  t   4 (Wave speed)
  max   max

x 0
2x 0  4      
2

371
Brilliant STUDY CENTRE

T 200 L 20m
11. 2 v   40 ms 1 ; t    0.5s
  2.5  / 20 v 40 ms 1

 2
12. 2 k or is the coefficient of x in the phase angle   0.018 cm 1 ;
 
2
  349cm  3.5 m
0.018

v v  v 
13. 4 Here,  412 Hz ; For one part, closed at one end, 1   2    2  412  824Hz
4 4  / 2  4 

v v v
For second part, open at both ends,  2  2  / 2    4  4  4  412  1648 Hz
 
K / 3
14. 4 V1  M  K  4 ;
/3 3  3M



t1  3  4  4  3  3M  2. M
V1 3V1 3 K  4 K


K
V2  2  K  3
M 2  2M
/2


 t1 M K 2
t2  2  3  3  2  2M  3M ; 2  
V2 2V2 2 K  3 K t2 K 3M 3

RT v Hydrogen  M Mixture


15. 2 v ; v
M Mixture M Hydrogen ; MMixture = of 4 parts of hydrogen and 1 part of oxygen by

4  2   1 32 
volume  8;
5

M Hydrogen 2 1270
 v mixture  v Hydrogen  1270 m / s  .   635 m / s
M Mixture 8 2
16. 1 Distance between successive comperssions and rarefractions is  / 2 or
1   / 2 or   2 m
v 360 ms1
Frequency   , where v is the velocity of sound;    180 Hz
 2m

372
[Study Package - Physics - Class XI]

17. 2 When source is moving towards observer

320
n   240 ; n' = 243 Hz
320  4
When source is moving away from observer.

320
n   240 ; n  237Hz ; Beats heard = 243 – 237 = 6
320  4

1 T 1 T
18. 4 From n  --------(i); 2n  --------(ii)
2 m 2 m

3 1 T T  3 3 T 9
where    ; Divide (ii) by (i): 2  ;   2  2  ; 
4  T T  4 2 T 4

v1 1 
19. 1 We know that, m  n1  n 2    
4  1  2  ; v = 300 m/s = 30000 cm/s

1 1 20 1 1 1 1 149 1500
        2   10.06 cm
1  2 300  100 1500 ;  2 10 1500 1500 149

T1
20. 2 Wavelength of pulse at the lower end, 1  velocity  v1  

T2
Similarly,  2  v 2 


2

T2

 m1  m 2  g 
m1  m 2
1 T1 m2g m2

F/A F
21. 4 Y  ;
 /  A  T
 F  YA  T
m m m
Now,    ; m  A ;  A  
v A 

T F YA  T
Now, v   
  A
v = 70.07 m/s

373
Brilliant STUDY CENTRE

100 n
22. 4 v  100 m / s ; f v
0.01 2
f1  50 z, f 2  100 Hz, f3  150 Hz

 2n  1 v
For open f  ; n1  25Hz, n 2  75 Hz, n 3  125 Hz
4
 2 
23. 4 The given equation of a stationary wave is y  0.5 sin  x  cos(120  t) ----(1)
 3 
The standard equation of a stationary wave is y  2a sin (kx) cos (t) ----(2)
Comparing (i) and (ii) we get,
2 2 2
k or  or   3m ;   120  or 2  120  or   60 Hz
3  3
Speed of the wave,     180 ms 1  option (4) is wrong.

   0   340  17 
24. 2 Using,  '     ;  680  640  ; s  4 ms 1
    s   340   s 

LEVEL - II
  
1. 4 The given equation is y  5sin  30t  x  30o  ;   ?; x  3.5 m
 7 
 2 2 
Compare the given equation with its standard form y  r sin  t x   ;
 T  
2  2 2 
 ;   14 m ;    x    3.5 
 7  14 2
2. 1 Let the equation of a plane progressive harmonic wave be y  A sin  t  kx  ;

wave speed v  ; and max particle speed v p  A
k
1 1 
As v P  v  A   / k or A  ; or A  2 /  , i.e., A  2
k  
p T
3. 1 When a string vibrates in p loops, frequency n 
2 m

5 9g 3 Mg
 n1  ; n2 
2 m 2 m

5 9g 3 Mg
As n1  n 2 , therefore  ; Hence M = 25 kg
2 m 2 m

374
[Study Package - Physics - Class XI]

3
4. 3 As the string is vibrating in three segments, therefore,   or
2
2 2   0.6 
   0.4 m
3 3

T 80 v 20
As v  ; v  20 m / s ; n   50 Hz
m 0.2  0.4
Amplitude of particle velocity,
 dy 
     a max    a max  2n    0.5 102   2 50  1.57 m / s
 dt  max

1 T
5. 1 From f  ; Now m = mass of unit length of wire
2L m

D2 1 T 1 T
  1  ;  f  2 ;f 
4 2L D / 4   LD 

1
Hence, f 
LD
6. 1 v1 = 150 m/s, T1 = 120 N;

20
v 2  150   150  180 m / s, T2  ?
100

T2 v 2 180 6 T2 36
   ; 
T1 v1 150 5 T1 25

 T2  T1 
 100  
36  25 100
Percentage increase in tension   44%
T1 25
7. 3 Given, y = 12 sin (5t – 4x) cm. Let x1 and x2 be the two points at a time t having phase difference

of 90o or rad. Phase of wave at point x1 , 1  5t  4x1 ; Phase of wave at point x 2,
2
2  5t  4x 2

 
 2  1  ;  5t  4x 2  5t  4x1 
2 2

 
or 4  x1  x 2   or x1  x 2  cm
2 8

375
Brilliant STUDY CENTRE

330 1  1
8. 1   ;   ;  0.125 m
600 2 4 8

P Mass of 1 mole of air 29  10 3 kg


9. 3 Speed  ;    ;   1.3 kg / m3
 Volume of 1 mole at STP 22.4  103 m3

7  1.01 105
PStandard 5 2
 1.01 10 Nm ;  Speed   330 ms 1
5  1.3

 2n  1 v   2n  1 330 
10. 4 For closed pipe,    2n  1 275
4L 4  0.3
1100
For resonance,  2n  1 275  1100 ;  2n  1  4
275
3
n is not a natural number so the resonance is not possible.
2

 v  vA  v    v  vB  v  
11. 2 1     0 or A   1  1 ; Similarly,  2     0 or B   2  1
 v  v  0   v  v  0 

 1 
  1
v A   0   1   0   5.5  5  kHz vA vB
   
v B   2    2   0   6  5  kHz ; v  0.5 or v  2
  1 B A
 0 

 v v
12. 2 For figure, p,  L or   L ;   P  
4  4L

 v  v
For fig. q, = L or   2L   q  ; For fig. r,, 2    L  r 
2 2L 2 L

3 v 3v 1 1 3
For fig. s,  L;  s   ;   p :  q :  r :  s  : :1:  1: 2 : 4 : 3
4  4L 4 2 4

376
[Study Package - Physics - Class XI]

v 320 m / s
13. 2 Fundamental frequency of open pipe   2   0.6D or 320 Hz  2 48  0.6D 102 m
   
1 1
 48  0.6 D   102 or 0.6 D  2 ;  D  3.33 cm
2 0.3
1/ 2
1/ 2  t  
1/ 2 1/ 2  273 1    1
14. 1 v T 
 273  t    273 
1 ;  273   1
1/ 2 
 273 100 2731/ 2
100
t 1 546
or 1  1  ; or t   5.46o C  5.5o C
546 100 100
v v
15. 2 f1 = frequency of 2nd harmonic (of open pipe)  2   -------(i)
2 
v
f2 = frequency of nth harmonic (of closed pipe)   2n  1 ---------(ii)
4
v 3v 5v
If n = 1, f 2  ; If n = 2; f 2   f1 ; If n = 3, f 2  which is greater than f1
4 4 4
5
 n  3, f 2  f1
4

RT 1 T
16. 2 Velocity of sound in air,   ; For a given medium,   T ;  1
M 2 T2

T2 313
v 2  v1  344.2   356 ms 1
T1 293

Y
17. 2 v

v1 2 1 1 v2
    2
v2 1 4 2 v1

3v
18. 1 For closed organ pipe, third harmonic is ; For open organ pipe, fundamental frequency is
4

2 
Given, third harmonic for closed organ pipe = fundamental frequency for open organ pipe.
3v v 4 2
      ;
4 2  3 2 3
where,  and  are the lengths of closed and open organ pipes respectively..
2  20
    13.33 cm
3

377
Brilliant STUDY CENTRE

19. 4 Let the frequencies of tuning forks A, B and C are  A ,  B and  C respectively..

1.5 2.5
According to given problem,  A   C   C ------(i) and  B   C   C ------(ii)
100 100
 1.5   2.5 
Also,  A   B = 12 Hz;   C  C    C   C   12 (Using (i) and (ii))
 100   100 
4 C 12  100
 12   C   300 Hz
100 4
1 25 1 n 2 1 25
20. 3 m = 10 beats/sec.,  ; n1 = ?; n2 = ?; As n  , therefore  
 2 26  n1  2 26 --------(i)

25 n
Now, n1  n 2  m  10 ; n1  n1  10 ; 1  10, n1  260 Hz
26 26
25 25
n2  n1   260  250Hz
26 26
 320  2   320  2 
21. 2 f    400 ; f     400
 320   320 
 322 318  4
f  f   f   400     400  5
 320 320  320

273  t 273  100


22. 4  vH t C   vO 100 C ;  vH 0
o o o
273
  v O  0o
273

vO 273  t H 2 1 1
     ; or 273  t  1
vH 373 O 32 16 4 373 16

373  4368 3995


4368 + 16t = 373; t    249.7 o C
16 16

23. 2 First resonance,    1  0.6r 
4
v  v
Also,   1  0.6r     
4  
Here, r is internal radius of resonance column and 1 is the length of water level in column.

4
Given, v = 336 m/s,   512 Hz and r  cm
2
336 10 2 4
 1    0.6    1  15.2 cm
4  512 2

378

You might also like